SSC CGL (Tier-2) Main Examination 2017 Held on March 9, 2018 Quantitative Aptitude Paper-1 Question Paper With Answer Key

SSC CGL (Tier-2) Main Examination 2017 Held on March 9, 2018 Quantitative Aptitude Paper-1
SSC CGL (Tier-2) Main Examination 2017 Held on March 9, 2018 Quantitative Aptitude Paper-1 Question Paper With Answer Key

SSC CGL (Tier-2) Main Examination 2017 Held on March 9, 2018

Quantitative Aptitude Paper-1

1. If the unit digit of 433 × 456 × 43N is (N + 2), then what is the value of N?

(a)  1

(b)  8

(c)  3

(d)  6

Answer: (d)

2. If N = (12345)2 + 12345 + 12346, then what is the value of √N?

(a)  12346

(b)  12345

(c)  12344

(d)  12347

Answer: (a)

3. Which of the following statement(s) is/are true?

(I) (0.03/0.2) + (0.003/0.02) + (0.0003/0.002) + (0.00003/0.0002) = 0.6

(II) (0.01) + (0.01)2 + (0.001)2 = 0.010101

(a)  Only I

(b)  Only II

(c)  Neither I nor II

(d)  Both I and II

Answer: (d)

4. What is the value of 1/(0.1)2 + 1/(0.01)2 +1/(0.5)2 + 1/(0.05)2?

(a)  10504

(b)  10404

(c)  10004

(d)  11400

Answer: (a)

5. Which of the following statement(s) is/are true?

(a)  Only I

(b)  Only II

(c)  Neither I nor II

(d)  Both I and II

Answer: (d)

6. Which of the following statement(s) is/are true?

(a)  Only I

(b)  Only II

(c)  Neither I nor II

(d)  Both I and II

Answer: (a)

7. If  then what is the value of f(1) + f(2) + f(3) + … f(10)?

(a)  9/10

(b)  10/11

(c)  11/12

(d)  12/13

Answer: (b)

8. If N = 411 + 412 + 413 + 414, then how many positive factors of N are there?

(a)  92

(b)  48

(c)  50

(d)  51

Answer: (a)

9. If N = 99, then Nis divisible by how many positive perfect cubes?

(a)  6

(b)  7

(c)  4

(d)  5

Answer: (b)

10. If N = 314 + 313 – 12, then what is the largest prime factor of N?

(a)  11

(b)  79

(c)  13

(d)  73

Answer: (d)

11. Which of the following statement(s) is/are true?

(a)  Only I

(b)  Only II

(c)  Neither I nor II

(d)  Both I and II

Answer: (d)

12. What is the value of 

(a)  2

(b)  4

(c)  8

(d)  6

Answer: (d)

13. The sum of two positive numbers is 14 and difference between their squares is 56. What is the sum of their squares?

(a)  106

(b)  196

(c)  53

(d)  68

Answer: (a)

14. What is the value of 10062 – 1007 × 1005 + 1008 × 1004 – 1009 × 1003?

(a)  6

(b)  3

(c)  12

(d)  24

Answer: (a)

15. If a2 + b2 = 4b + 6a – 13, then what is the value of a + b?

(a)  3

(b)  2

(c)  5

(d)  10

Answer: (c)

16. x and y are positive integers, if x4 + y4 + x2y2 = 481 and xy = 12, then what is the value of x2 – xy + y2?

(a)  16

(b)  13

(c)  11

(d)  15

Answer: (b)

17. If A = 1 + 2P and B = 1 + 2P, then what is the value of B?

(a)  (A + 1)/(A – 1)

(b)  (A + 2)/(A + 1)

(c)  A/(A – 1)

(d)  (A – 2)/(A + 1)

Answer: (c)

18. If a and b are roots of the equation ax3 + bx + c = 0, then which equation will have roots (ab + a + b) and (ab – a – b)?

(a)  a2x2 + 2acx + c2 + b2 = 0

(b)  a2x2 + 2acx + c2 – b2 = 0

(c)  a2x2 – 2acx + c2 + b2 = 0

(d)  a2x2 + 2acx + c2 – b2 = 0

Answer: (*)

19. If  then what is the value of 

(a)  2

(b)  √2

(c)  1

(d)  None of these

Answer: (b)

20. If (a + b)2 – 2(a + b) = 80 and ab = 16, then what can be the value of 3a – 19b?

(a)  −16

(b)  −14

(c)  −18

(d)  −20

Answer: (b)

21. If xy + z = 1, yx + z = 1024 and zx + y = 729 (x, y and z are natural numbers), then what is the value of (z + 1)y + x + 1?

(a)  6561

(b)  10000

(c)  4096

(d)  14641

Answer: (b)

22. If x + y + z = 1, x2 + y2 + z2 = 2 and x3 + y3 + z3 = 3, then what is the value of xyz?

(a)  1/3

(b)  1/6

(c)  1/2

(d)  1/4

Answer: (b)

23. In triangle PQR, the internal bisector of ∠Q and ∠R meets at O, If ∠QPR = 70°, then what is the value (in degrees) of ∠QOR?

(a)  45

(b)  125

(c)  115

(d)  10

Answer: (b)

24. PQR is a triangle such that PQ = PR and QT are the median to the sides PQ and PR respectively. If the medians RS and QR intersect at right, angle, then what is the value of (PQ/QR)2?

(a)  3/2

(b)  5/2

(c)  2

(d)  None of these

Answer: (b)

25. PQR is a triangle. S and T are the midpoints of the sides PQ and PR respectively. Which of the following is True?

(1) Triangle PST is similar to triangle PQR

(2) ST = 1/2(QR)

(3) ST is parallel to QR.

(a)  I and II

(b)  II and III

(c)  I and III

(d)  All of these

Answer: (d)

26. ABC is a triangle in which ∠ABC = 90°, BD is perpendicular to AC, which of the following is true?

(I) Triangle BAD is similar to triangle CBD.

(II) Triangle BAD is similar to triangle CAB.

(III) Triangle CBD is similar to triangle CAB.

(a)  Only I

(b)  II and III

(c)  I and III

(d)  All of these

Answer: (d)

27. Two parallel chords are one the one side of the centre of a circle. The length of the two chords is 24 cm and 32 cm. If the distance between the two chords is 8 cm, then what is the area of the circle?

(a)  724.14 cm2

(b)  832.86 cm2

(c)  924.12 cm2

(d)  988.32 cm2

Answer: (b)

28. Two circles of radius 4 cm and 6 cm touch each other Internally. What is the length of the longest chord of the outer circle, which is also a tangent to inner circle?

(a)  12√2 cm

(b)  8√2 cm

(c)  6√2 cm

(d)  4√2 cm

Answer: (b)

29. In the given figure, PT is a common tangent to three circles at points A, B and C respectively. The radius of the small, medium and large circles is 4 cm, 6 cm and 9 cm. O1, O2 and O3 are the centre of the three circles. What is the value of PC?

(a)  18√6 cm

(b)  9√6 cm

(c)  24√6 cm

(d)  15√6 cm

Answer: (a)

30. PQRS is a cyclic quadrilateral, PR and QS intersect at T. If ∠SPR = 40° and ∠PQS = 80°, then what is the value of ∠PSR?

(a)  60°

(b)  40°

(c)  80°

(d)  100°

Answer: (a)

31. In the given figure, ∠PSR = 105° and PQ is the diameter of the circle. What is the value of ∠QPR?

(a)  75°

(b)  15°

(c)  30°

(d)  45°

Answer: (b)

32. There are two identical circles of radius 10 cm each. If the length of the direct common tangent is 26 cm, then what is the length of transverse common tangent?

(a)  2√69 cm

(b)  4√23 cm

(c)  4√46 cm

(d)  3√46 cm

Answer: (a)

33. PQRS is a rectangle in which side of PQ = 24 cm and OR = 16 cm. T is a point on RS. What is the area of the triangle PTQ?

(a)  192 cm2

(b)  162 cm2

(c)  148 cm2

(d)  Cannot be determined

Answer: (a)

34. In the given figure, ABCD and BEFG are squares of sides 8 cm and 6 cm respectively. What is the area (in cm2) of the shaded region?

(a)  14 cm2

(b)  12 cm2

(c)  8 cm2

(d)  16 cm2

Answer: (b)

35. PQRS is a parallelogram and its area is 300 cm2. Side PQ is extended to X such that PQ =QX. If XS intersects QR at Y, then what is the area of triangle SYR?

(a)  75 cm2

(b)  50 cm2

(c)  120 cm2

(d)  100 cm2

Answer: (a)

36. PQRST is a regular pentagon. If PR and QT intersects each other at X, then what is the value of ∠TXR?

(a)  98°

(b)  90°

(c)  72°

(d)  108°

Answer: (d)

37. In the given figure, ABCDEF is a regular hexagon whose side is 12 cm. What is the shaded area?

(a)  54√3 cm2

(b)  36√3 cm2

(c)  48√3 cm2

(d)  52√3 cm2

Answer: (a)

38. ABCD passes through the centres of the three circles as shown in the figure. AB = 2 cm and CD = 1. If the area of middle circle is the average of the areas of the other two circles, then what is the length of BC?

(a)  (√6) – 1 cm

(b)  (√6) + 1 cm

(c)  (√6) – 3 cm

(d)  (√6) + 3 cm

Answer: (a)

39. A = Area of the largest circle drawn Inside a square of side 1 cm.

B = Sum of areas of 4 identical (largest possible) circles drawn inside a square of side 1 cm.

C = Sum of areas of 9 identical circle (largest possible) drawn inside a square of side 1 cm.

D = Sum of area of 16 identical circles (largest possible) drawn inside a square of side 1 cm.

Which of the following is true about A, B, C and D?

(a)  A > B > C > D

(b)  A < B < C < D

(c)  A > B = C > D

(d)  No option is correct

Answer: ()

40. A prism has a square base whose side is 8 cm. The height of prism is 80 cm. The prism is cut into 10 identical parts by 9 cuts which area parallel to base of prism, what is the total surface area of all the 10 parts together?

(a)  4260 cm2

(b)  2560 cm2

(c)  3840 cm2

(d)  3220 cm2

Answer: (c)

41. A cone of radius 90 cm and height 120 cm stands on its base. It is cut into 3 parts by 2 cuts parallel to its base such that the height of the three parts (from top to bottom) are in ratio of 1 : 2 : 3. What is the total surface area of the middle part?

(a)  14600 cm2

(b)  16500 cm2

(c)  17800 cm2

(d)  18500 cm2

Answer: (b)

42. The curved surface area of cylinder is 594 cm2 and its volume is 1336.5 cm3. What is the height of the cylinder?

(a)  14 cm

(b)  21 cm

(c)  24.5 cm

(d)  10.5 cm

Answer: (b)

43. A hollow cylinder is made up of metal. The difference between outer and inner curved surface area of this cylinder is 352 cm2. Height of the cylinder is 28 cm. If the total surface area of this hollow cylinder is 2640 cm2, then what are the inner and outer radius (in cm)?

(a)  4, 6

(b)  10, 12

(c)  8, 10

(d)  6, 8

Answer: (d)

44. A solid metal sphere has a radius 14 cm. It is melted to form small cones of radius 1.75 cm and height 3.5 cm. How many small cones will be obtained from the sphere?

(a)  512

(b)  256

(c)  1024

(d)  2048

Answer: (c)

45. A metallic hemispherical bowl is made up of steel. The total steel used in making the bowl is 342n cm3, The bowl can hold 144n cm3 What is the thickness (in cm) of bowl and the curved surface area (in cm2) of outer side?

(a)  6, 162n

(b)  3, 162n

(c)  6, 81n

(d)  3, 81n

Answer: (b)

46. There is a box of cuboid shape. The smallest side of the box is 20 cm and largest side is 40 cm. Which of the following can be volume of the box?

(a)  18000 cm3

(b)  12000 cm3

(c)  36000 cm3

(d)  42000 cm3

Answer: (a)

47. A pyramid has a square base, whose side is 8 cm. If the height of pyramid is 16 cm, then what is the total surface area of the pyramid?

(a)  64(√17 + 1) cm3

(b)  32(√13 + 1) cm3

(c)  64(√3 + 1) cm3

(d)  32(√5 + 1) cm3

Answer: (a)

48. What is the value of 

(a)  sin 2θ

(b)  sin2 θ

(c)  cos2 θ

(d)  cos 2θ

Answer: (d)

49. What is the value of 

(a)  2 cot A

(b)  2tan A

(c)  cot A

(d)  tan A

Answer: (d)

50. What is the value of cos15° − cos 165°?

(a) 

(b) 

(c) 

(d) 

Answer: (c)

51. If P + Q + R = 60°, then what is the value of cos Q cos R (cos P – sin P) + sin Q sin R (sin P – cos P)

(a)  1/2

(b)  √3/2

(c)  1/√2

(d)  √2

Answer: (a)

52. What is the value of 

(a)  − sin 2θ

(b)  − cos 2θ

(c)  cos 2θ

(d)  sin 2θ

Answer: (a)

53. What is the value of 

(a)  cos x + sin x

(b)  sin x – cos x

(c)  sec x + tan x

(d)  sec x – tan x

Answer: (d)

54. What is the value of sin(180 – θ) sin(90 – θ) – [cot(90 – θ) /1+ tan2θ]?

(a)  cos2 θ sin θ

(b) 

(c) 

(d)  sin2 θ cos θ

Answer: (*)

55. A pole is sanding on the top of a house. Height of house is 25 m. The angle of elevation of the top of house from point P is 45° and the angle of elevation of the top of pole from P is 60°. Point P is on the ground level. What is the height of pole?

(a)  10(√3 + 1)m

(b)  15(√3 + 1)m

(c)  25(√3 − 1)m

(d)  20(√3 − 1)m

Answer: (c)

56. A ladder is placed against a wall such that it just reaches the top of the wall. The foot of the ladder is at a distance of 5 m from the wall. The angle of elevation of the top of the wall from the base of the ladder is 45°. What is the length of the ladder?

(a)  5√6 – 5√3 m

(b)  5√6 – 5√2 m

(c)  5√2 – 1m

(d)  5√3 + 5√2 m

Answer: (b)

57. An aeroplane is flying horizontally at a height of 1.8 km above the ground. The angle of elevation of plane from point X is 60° and after 20 seconds, its angle of elevation from X is become 30°. If point X is on ground, then what is the speed of aeroplane?

(a)  216√3 km/h

(b)  105√3 km/h

(c)  201√3 km/h

(d)  305√3 km/h

Answer: (a)

Directions (Q. Nos. 58-62) The table given below shows the production of maize by 5 different States a percentage of total production. Each State produces only maize and rice. There are three types of rice – R1, R2 and R3. The table also shows the R1 type of rice produced as a percentage of total rice production and the ratio of R2 and R3 type of rice. Total production by each State is 625000.

58. What is the difference between then R1 type of rice produced by State X and the R2 type of rice produced by State H?

(a)  115000

(b)  120000

(c)  55000

(d)  65000

Answer: (b)

59. What is the sum of the total production of maize by State X and T and total production of R2 type of Rice by State S and R?

(a)  868500

(b)  1025000

(c)  925000

(d)  892500

Answer: (d)

60. Production of R3 type of rice by State X is what percentage of production of R1 type of rice by State S?

(a)  45.45

(b)  52.52

(c)  42.5

(d)  39.5

Answer: (a)

61. A = Average of the R3 type of rice produced by State H, R, S and X together.

B = Difference between the R2 type of rice produced by State T and R1 type of rice produced by State R.

What is the value of B – A?

(a)  54750

(b)  56750

(c)  57500

(d)  57000

Answer: (c)

62. F = Total production of R2 types of rice by all the States.

K = Average of the total production of R1 type of rice by all the States.

What is the value of K/F?

(a)  0.875

(b)  0.802

(c)  0.08

(d)  0.702

Answer: (b)

63. If x beakers of 100 ml containing 1 : 4 acid-water solution are mixed with y beakers of 200 ml containing 3 : 17 acid-water solution then the ratio of acid to water in the resulting mixture becomes 19 : 91. Find x : y.

(a)  5 : 3

(b)  3 : 5

(c)  7 : 13

(d)  13 : 7

Answer: (a)

64. In what ratio should 20% ethanol solution be mixed with 40% ethanol solution to obtain a 28% ethanol solution?

(a)  2 : 3

(b)  8 : 5

(c)  3 : 2

(d)  5 : 8

Answer: (c)

65. A and B start a business by investing equal amounts. Four months later, C joins them by investing Rs 3.5 lakh. By withdrawing his investment in the business B leaves the business 4 months after C joined. At the end of the year the business makes Rs 62400 profit out of which A collects Rs 24000 as his share of profit. How much should be paid to C as his share of profit?

(a)  Rs 16000

(b)  Rs 32000

(c)  Rs 22400

(d)  Rs 27800

Answer: (c)

66. A and B invest in a business in the ratio 3 : 7. The business makes a profit of Rs 60000 in 1 year. They decide to distribute the profit remaining after reinvesting 40% of the profit. How much will A get?

(a)  Rs 25200

(b)  Rs 15600

(c)  Rs 10800

(d)  Rs 20400

Answer: (c)

67. A can do a work in 72 days and B in 90 days. If they work on it together for 10 days, then what fraction of work is left?

(a)  3/4

(b)  1/4

(c)  4/5

(d)  5/6

Answer: (a)

68. A is thrice as a good a workman as B. C alone takes 48 days to paint a house. All three A, B and C working together take 16 days to paint the house. It will take how many days for B alone to paint the house?

(a)  32

(b)  64

(c)  96

(d)  72

Answer: (c)

69. C is 5 times as productive as B, A takes 60 days to complete a task. If A, B and C work together they can complete the task in 12 days. In how many days can B complete the task if the worked alone?

(a)  18

(b)  27

(c)  90

(d)  72

Answer: (c)

70. A can complete 50% of a job in 9 days and B can complete 25% of the job in 9 days if they worked alone. If they worked together how much of the job can they complete in 9 days?

(a)  80%

(b)  90%

(c)  75%

(d)  100%

Answer: (c)

71. Giving two successive discounts of 60% is equal to giving one discount of

(a)  90%

(b)  72%

(c)  96%

(d)  84%

Answer: (d)

72. If an item marked at Rs 480 is being sold at Rs 400, then what is the effective discount on the item?

(a)  20%

(b)  16.67%

(c)  25%

(d)  15%

Answer: (b)

73. On an item there is cash 5% discount on the marked price of Rs 25000. After giving an additional season’s discount the item is sold at Rs 20900. How much was the season’s discount?

(a)  11%

(b)  10%

(c)  12%

(d)  9%

Answer: (c)

74. A retailer marks up his goods by 20% and then offers 25% discount. What will be the selling price on an item that he sells if its cost price is Rs 2500?

(a)  Rs 2400

(b)  Rs 3000

(c)  Rs 2750

(d)  Rs 2250

Answer: (d)

75. Find two numbers such that their mean proportional is 18 and the third proportional to them is 144.

(a)  6 and 42

(b)  9 and 36

(c)  3 and 18

(d)  6 and 12

Answer: (b)

76. If 6A = 4B = 9C; find A : B : C

(a)  6 : 4 : 9

(b)  6 : 9 : 4

(c)  4 : 9 : 6

(d)  9 : 6 : 4

Answer: (b)

77. Find the third proportional to 10 and 25.

(a)  2.5

(b)  62.5

(c)  40

(d)  100

Answer: (b)

78. A purse has Rs 34.5 in the form of Rs 1, 50 paise and 10 paise coins in the ratio 6 : 9 : 10. Find the number of 10 paise coins.

(a)  10

(b)  30

(c)  20

(d)  40

Answer: (b)

79. What number should be added to each of numbers 103, 135, 110 and 144 so, that the resulting numbers are in proportion?

(a)  12

(b)  15

(c)  9

(d)  6

Answer: (c)

80. When ticket prices to a water park are increased in the ratio 11 : 12 then the number of daily visitors to the park fall in the ratio 8 : 7. If the dally revenues before the increase in ticket price was Rs 176000 then find the daily revenues after the increase in ticket price.

(a)  Rs 264000

(b)  Rs 112000

(c)  Rs 192000

(d)  Rs 168000

Answer: (d)

81. The average weight of X, Y and Z is 745 kg. In the average weight of X and Y be 68 kg and that and Z be 78 kg, then the weight of Y is.

(a)  72 kg

(b)  70 kg

(c)  68 kg

(d)  66 kg

Answer: (b)

82. Of the 3 numbers whose average is 126, the first is 2/11 times the sum of other two. The first number is

(a)  16

(b)  13

(c)  11

(d)  12

Answer: (d)

83. The average weight of a class of 50 students is 48.6 kg. If the average weight of the 20 boys is 54 kg, then find the average weight of the girls in the class.

(a)  40 kg

(b)  46 kg

(c)  45 kg

(d)  42 kg

Answer: (c)

84. The average of all odd numbers from 113 to 159 to

(a)  135

(b)  134

(c)  133

(d)  136

Answer: (d)

85. A trader buys jowar at Rs 30 per kg. 20% of the grain gets wasted. He plans to sell the remaining grain to that he makes 40% overall profit. At what price should he shell the grain?

(a)  Rs 48 per kg

(b)  Rs 50 per kg

(c)  Rs 52.5 per kg

(d)  Rs 47.5 per kg

Answer: (c)

86. If a vendor sells a watermelon at Rs 69 he makes 8% loss. If he wants to make 16% profit then at what price should he sell?

(a)  Rs 91

(b)  Rs 83

(c)  Rs 87

(d)  Rs 79

Answer: (c)

87. The cost of 25 items is the same as the revenue earned by selling x items. Find x, if the profit made in the transaction is 25%.

(a)  25

(b)  16.67

(c)  20

(d)  32

Answer: (c)

88. An item is sold for Rs 7130 making a 15% profit. What is the cost price of this item?

(a)  Rs 6000

(b)  Rs 6125

(c)  Rs 6250

(d)  Rs 6200

Answer: (d)

89. 0.02% of 150% of 600 is

(a)  0.18

(b)  1.8

(c)  18

(d)  0.018

Answer: (a)

90.  When a number is increased by 40, it becomes 125% of itself, what is the number?

(a)  200

(b)  60

(c)  160

(d)  100

Answer: (c)

91. In an exam of 300 marks a students gets 75 marks. If she had scored 6 more she would have attained passing percentage. What is the passing percentage?

(a)  25%

(b)  30%

(c)  35%

(d)  27%

Answer: (d)

92. A man’s annual income has increased by Rs 7 lakh, but the tax on income that he has to pay has reduced from 20% to 16%. He now pays the same amount of tax as before. What is his increased income?

(a)  Rs 8 lakh

(b)  Rs 10 lakh

(c)  Rs 12 lakh

(d)  Rs 6 lakh

Answer: (d)

93. A car covers 630 km in 20 hours. Calculate its average speed in meters/second?

(a)  8.25

(b)  7.25

(c)  8.75

(d)  7.25

Answer: (c)

94. A jet ski goes upstream at a speed of 48 km/h and comes back the same distance at 80 km/h. Find the average speed for the total journey.

(a)  64 km/h

(b)  62 km/h

(c)  66 km/h

(d)  60 km/h

Answer: (d)

95. A bullet fired from a rifle travels at an average speed of 2520 km/hr. It hits the target after 0.2 seconds. How far is the target from the rifle?

(a)  70 m

(b)  140 m

(c)  100 m

(d)  200 m

Answer: (b)

96. Train A and B start at the same time. Train A travels at 55 km/h from station X to station Y and train B travels at 80 km/h from station Y to station X. They cross each other after 1 hour and 36 minutes. What is the distance between station X and Y?

(a)  196 km

(b)  232 km

(c)  240 km

(d)  216 km

Answer: (d)

97. If in 2 years at simple interest the principal increases by 16%, what will be the compound interest earned on Rs 25000 in 2 years at the same rate?

(a)  Rs 4000

(b)  Rs 2160

(c)  Rs 2000

(d)  Rs 4160

Answer: (d)

98. If compound interest received on a certain amount in the 2nd year is Rs 250. What will be the compound interest for the 3rd year on the same amount at 12% rate of interest?

(a)  Rs 250

(b)  Rs 300

(c)  Rs 280

(d)  Rs 270

Answer: (c)

99. What is the difference between the compound interest on Rs 12500 for 1 year at 8% per annum compounded yearly and half-yearly?

(a)  Rs 16

(b)  Rs 25

(c)  Rs 20

(d)  Rs 40

Answer: (c)

100. The amount received at 8% per annum compound interest after 2 years is Rs 72900. What was the principle?

(a)  Rs 65000

(b)  Rs 67500

(c)  Rs 60000

(d)  Rs 62500

Answer: (d)

SSC CGL (Tier 1) Exam 2018 Held on March 19, 2018 Question Paper With Answer Key

SSC CGL (Tier 1) Exam 2018 Held on March 19, 2018
SSC CGL (Tier 1) Exam 2018 Held on March 19, 2018 Question Paper With Answer Key

SSC CGL (Tier 1) Exam 2018 Held on March 19, 2018

Directions (Q. Nos. 1-2) In the following questions, some part of the sentence may have errors. Find out which part of the sentence has an error and select the appropriate option. If a sentence is free error, select ‘No Error’.

1. What happen (a)/ at your (b)/ college today? (c) No error (d)

(a)  1

(b)  2

(c)  3

(d)  4

Answer: (a)

2. Some girls are taking prior permission to (a)/sport a loosely tied plait on the day (b)/ they had washed their hair. (c) No error (d)

(a)  1

(b)  2

(c)  3

(d)  4

Answer: (a)

Directions (Q. Nos. 3-4) In the following questions, the sentence given will blank to be filled in with an appropriate word. Select the correct alternative out of the four and indicate it by selecting the appropriate option.

3. Her trekking was met …….. obstacles.

(a)  with

(b)  from

(c)  by

(d)  of

Answer: (a)

4. Aryabhatta was the first astronomer to State ……… that the Earth is round and it rotates on its axis.

(a)  sympathetically

(b)  automatically

(c)  emphatically

(d)  accidentally

Answer: (c)

Directions (Q. Nos. 5-6) In the following questions, out of the given four alternative, select the one which best expresses the meaning of the given word.

5. Barbarous

(a)  Refined

(b)  Polite

(c)  Crude

(d)  Civilized

Answer: (c)

6. Pejorative

(a)  Right

(b)  Discussion

(c)  Derogatory

(d)  Complimentary

Answer: (c)

Directions (Q. Nos. 7-8) In the following questions, out of the given four alternatives, select the one which is opposite in meaning of the given word.

7. Decimate

(a)  Preserve

(b)  Destroy

(c)  Execute

(d)  Abate

Answer: (a)

8. Adjure

(a)  Order

(b)  Require

(c)  Advise

(d)  Disclaim

Answer: (d)

9. Rearrange the parts of the sentence in correct order.

Cities have

P : intellectual stimulus

Q : always provided

R : and educational leadership

(a)  PQR

(b)  RQP

(c)  QPR

(d)  PRQ

Answer: (c)

10. A sentence has been given in Active/Passive Voice. Out of the four given alternatives, select the one which best expresses the same sentence in Passive/Active Voice.

The watch man opened the gate.

(a)  The gate opened the watch man.

(b)  The gate is being opened by the watchman.

(c)  The gate was opened by the watchman.

(d)  The gate is opened by the watchman.

Answer: (c)

11. A sentence has been given in Direct/Indirect Speech. Out of the four given alternatives, select the one which best expresses the same sentence in Indirect/Direct Speech.

He said, “I am hungry.”

(a)  He inform about his hunger.

(b)  He says he was hungry.

(c)  He was felling hungry.

(d)  He said that he was hungry.

Answer: (d)

12. In the following question, a word has been written in four different ways out of which only one is correctly spelt. Select the correctly spelt word.

(a)  Apeareance

(b)  Appearrance

(c)  Appearence

(d)  Appearance

Answer: (d)

Directions ( Q. Nos. 13-17) In the following passage, some of the words have been left out. Read the Passage carefully and select the correct answer for the given blank out of the four alternatives.

Many a travelogue I read nowadays are about how the traveler went on a road less travelled ………. some of-the-track trek or to an unknown destination or simply into the wild. How no internet …… is what you need to reconnect with yourself! How leaving your watch and phone ……. and going into the world of no check-ins and no updates would bring you solitude! These write-ups are laced with adjectives describing the geography of the region and filled with ………… details of the author’s days in that remote place. And, ironically, they all end with a strange advice to travel and tell no one. I beg to …… .

13. road less travelled ……. some off-the-track trek

(a)  but

(b)  or

(c)  but also

(d)  also

Answer: (b)

14. How no internet ……. is what you need to

(a)  connectivity

(b)  to connect

(c)  connects

(d)  connecting

Answer: (a)

15. watch and phone ………… and going into

(a)  under

(b)  below

(c)  backside

(d)  behind

Answer: (d)

16. and filled with …….. details of the author’s

(a)  hour

(b)  second

(c)  time

(d)  minute

Answer: (d)

17. and tell no I beg to ……..

(a)  differ

(b)  different

(c)  difference

(d)  differentiation

Answer: (a)

Directions (Q. Nos. 18-19) In the following questions, out of the four alternative, select the alternative which best expresses the meaning of the idiom/phrase.

18. A bad patch

(a)  A potholed road

(b)  A medical bandage

(c)  A period of difficulty

(d)  Shoddy repair work

Answer: (c)

19. A dime a dozen

(a)  A very expensive proposition.

(b)  Something which appears cheap but which will prove expensive in the long run.

(c)  Something which appears attractive but has zero value.

(d)  Very common and of no particular value.

Answer: (d)

Directions (Q. Nos. 20-21) In the following questions, out of the four alternatives, select the alternative which is the best substitute of the words/sentence.

20. Wrap or cover for warmth

(a)  Encourage

(b)  Muffle

(c)  Divulge

(d)  Expose

Answer: (b)

21. Keen interest or enthusiasm

(a)  Lassitude

(b)  Lethargy

(c)  Avidity

(d)  Soporific

Answer: (c)

Directions (Q. Nos. 22-23) In the following questions, out of the four alternatives, select the alternative which will improve the bracketed part of the sentence. In case no improvement is needed, select ‘no improvement.’

22. The realization came when we noticed that our children (had becoming) too dependent on us.

(a)  had become

(b)  has became

(c)  has become

(d)  No improvement

Answer: (a)

23. The technical aspect of English that deals with ironies (has always intrigue) my inquisitive mind.

(a)  is always intrigued

(b)  will always intrigued

(c)  has always intrigued

(d)  No improvement

Answer: (c)

24. The question below consists of a set of labelled sentences. Out of the four options given, select the most logical order of the sentence to form a coherent paragraph.

It is ethics and

(A) values that have inherent

(B) of motivation into inspiration

(C) potential to transform the spirit

(a)  ABC

(b)  ACB

(c)  CBA

(d)  CAB

Answer: (b)

25. In the following question, four words are given out of which one word is correctly spelt. Select the correctly spelt word.

(a)  anotate

(b)  annotate

(c)  annottate

(d)  anottate

Answer: (b)

26. In the following question, select the related word pair from the given alternatives.

Road : Asphalt : : ? : ?

(a)  Sugarcane : Sugar

(b)  Cloth : Shirt

(c)  Pen : Pencil

(d)  Metal : Ore

Answer: (d)

27. In the following question, select the related number from the given alternatives. 697 : 976 : : 532 : ?

(a)  320

(b)  354

(c)  237

(d)  325

Answer: (d)

28. In the following question, select the related letter/letters from the given alternatives.

RTU : SUW : : CEF : ?

(a)  FGI

(b)  DGH

(c)  EGF

(d)  DFH

Answer: (d)

29. In the following question. select the odd word pair from the given alternatives.

(a)  Oxygen-Gas

(b)  Metal-Platinum

(c)  Liquid-Water

(d)  Solid-Iron

Answer: (a)

30. In the following question, four number pairs are given. The number of left side of (-) is related to the number on the right side of (-) with some Logic/Rule/Relation. Three are similar on basis of same Logic/Rule/Relation. Select the odd one out from the

(a)  27-32

(b)  31-36

(c)  33-38

(d)  23-27

Answer: (d)

31. In the following question, select the odd letter/letters from the given alternatives.

(a)  SPMJ

(b)  XURO

(c)  NKHE

(d)  ZWSP

Answer: (d)

32. Arrange the given words in the sequence in which they occur in the dictionary.

(1) Tinned     (2) Timber

(3) Tinkle      (4) Thunderstorm

(4) Thursday

(a)  5, 3, 2, 1, 4

(b)  2, 1, 3, 4, 5

(c)  4, 5, 2, 3, 1

(d)  1, 2, 4, 3, 5

Answer: (c)

33. In the following question, select the missing number from the given series.

17, 20, 23, 26, 29, ?

(a)  31

(b)  34

(c)  30

(d)  32

Answer: (d)

34. A series is given with one term missing. Select the correct alternative from the given ones that will complete the series.

FT, JX, NB, RF, ?

(a)  TX

(b)  LJ

(c)  VJ

(d)  YJ

Answer: (c)

35. Present age of Amit is twice of Baman’s present age. After 5 years Baman’s age will be 4 times of the Chetan’s present age. If Chetan celebrated his sixth birthday 7 years ago, then what will be the present age of Amit?

(a)  98 yr

(b)  110 yr

(c)  94 yr

(d)  92 yr

Answer: (c)

36. From the given alternative, select the word which cannot be formed using the letters of the given word.

Compassionate

(a)  Pass

(b)  Moss

(c)  Passion

(d)  Comply

Answer: (d)

37. In a certain code language, ‘RAPID’ is written as ‘GLSDU’. How is ‘WATER’ written in that code language?

(a)  DZHKM

(b)  UHWDZ

(c)  VHKDZ

(d)  MKHDZ

Answer: (b)

38. In a certain code language, ‘−’ represents ‘×’, ‘÷’ represents ‘−’. Find out the answer to the following questions.

18 ÷ 12 – 5 + 30 × 6 = ?

(a)  19

(b)  15

(c)  41

(d)  14

Answer: (d)

39. The following equation is incorrect. Which two signs should be interchanged to correct the equation?

20 + 14 ÷ 35 – 10 × 12 = 10

(a)  + and ×

(b)  + and ÷

(c)  − and +

(d)  ÷ and ×

Answer: (a)

40. If 16#2 = 16, 12#6 = 36 and 6#1 = 3, then find the value of 4 # 4 = ?

(a)  2

(b)  4

(c)  10

(d)  8

Answer: (d)

41. Which of the following terms follows the trend of the given list?

ABCABABAB, ABABCABAB, ABABABCAB, ABABABABC, ACBABABAB, ……….

(a)  ABABCABAB

(b)  ABCABABAB

(c)  ABACBABAB

(d)  ABABACBAB

Answer: (c)

42. A village woman walks 2.5 km South starting from her home is search of water. She then turns West and walks 1.5 km, then the she turns South and walks 0.5 km, then she turns to her left and walks 1.5 km. Where is she now with respect to her home?

(a)  2 km South

(b)  3 km North

(c)  3 km South

(d)  2 km North

Answer: (c)

43. In the question two statements are given, followed by two conclusions, I and II. You have to consider the statements to be true even if it seems to be at variance from commonly known facts. You have to decide which of the given conclusions, if any, follows from the given statements.

Statements (I) All tickets are coupons

(II) All cards are tickets

Conclusions (I0 Some coupons are cards

(II) Some cards are coupons

(a)  Only conclusion I follows

(b)  Only conclusion II follows

(c)  Both conclusions I and II follow

(d)  Neither conclusion I nor conclusion II follows

Answer: (c)

44. In the following figure, rectangle represents opticians, circle represents illustrators, triangle represents Photographers and square represents Football players. Which set of letters represents Football players who are both Photographers as well as Opticians?

(a)  D

(b)  ED

(c)  DF

(d)  EDF

Answer: (a)

45. A series is given with one term missing. Select the correct alternative from the given ones that will complete the series.

HHQ, HEN, EBK, ? YVE

(a)  ZWF

(b)  AXG

(c)  CZI

(d)  BYH

Answer: (d)

46. In the following question, select the missing number from the given series.

6, 9, 13, 18, 24, ?

(a)  31

(b)  34

(c)  29

(d)  28

Answer: (a)

47. In the following question, four groups of three numbers are given. In each group the second and third number are related to the first number by a Logic/Rule/Relation. Three are similar on basis of same Logic/Rule/Relation. Select the odd one out from the given alternatives.

(a)  (5, 16, 22)

(b)  (6, 19, 25)

(c)  (4, 13, 17)

(d)  (9, 28, 37)

Answer: (a)

48. If a mirror is placed on the line MN, then which of the answer figures is the right image of the given figure?

Answer: (b)

49. Which of the following cube in the answer figure cannot be made based on the unfolded cube in the question figure?

Answer: (a)

50. A word is represented by only one set of numbers as given in any one of the alternatives. The sets of numbers given in the alternatives are represented by two classes of alphabets as shown in the given two matrices. The columns and rows of Matrix-I are numbered from 0 to 4 and that of Matrix-II are numbered from 5 to 9. A letter from these matrices can be represented first by its row and next by its column, for example ‘D’ can be represented by 24, 41 etc. and ‘Y’ can be represented by 57, 98 etc. Similarly, you have to identify for the word ‘BREW’.

(a)  36, 68, 44, 95

(b)  43,78, 23, 59

(c)  30, 66, 42, 96

(d)  23, 68, 22, 66

Answer: (b)

51. Calculate the value of x if

(a)  64

(b)  44

(c)  36

(d)  54

Answer: (a)

52. Find the unit place digit in the given expression.

(153)144 – (115)123 – (111)510 + (216)25

(a)  1

(b)  5

(c)  6

(d)  3

Answer: (a)

53. Which of the following value of ‘x’ satisfies the equation 4x – 1 ∙ 92x – 6 = 5184?

(a)  2

(b)  3

(c)  4

(d)  6

Answer: (c)

54. If , then find the value of 

(a)  14

(b)  16

(c)  20

(d)  24

Answer: (a)

55. The tangent at any point of the circle is ……. to the radius through the point of contact.

(a)  parallel

(b)  intersecting

(c)  perpendicular

(d)  equal

Answer: (c)

56. Calculate the value of LED from the figure shown below

(a)  30°

(b)  40°

(c)  60°

(d)  80°

Answer: (b)

57. The income of X is 60% more than Y’s income and the income of Y is 40% more than Z’s income. X’s income is how much percentage more than Z’s income?

(a)  134%

(b)  224%

(c)  100%

(d)  124%

Answer: (d)

58. If x : y = 1 : 3, then what will be the ratio of (3y2 + 3x2) : (2y2 – 8x2)?

(a)  4 : 1

(b)  2 : 1

(c)  3 : 1

(d)  5 : 1

Answer: (c)

59. 5 L of honey contains 80% sugar. 3 L of water is added to this honey. What is the percentage of sugar in the new mixture?

(a)  40%

(b)  50%

(c)  60%

(d)  75%

Answer: (b)

60. Average of the marks of 132 students of a college is 40. If the average of the marks of the passed students is 42 and the average of the marks of the failed students is 20, then what will be the respective ratio of the total marks of passed students and the total marks of failed students?

(a)  21 : 1

(b)  23 : 2

(c)  19 : 11

(d)  9 ; 2

Answer: (a)

61. A sum of Rs 8000 becomes Rs 12500 in 2 years at a certain rate of compound interest. What will be the sum after 3 years?

(a)  Rs 13175

(b)  Rs 14225

(c)  Rs 12575

(d)  Rs 15625

Answer: (d)

62. If the selling price of an article is doubled, then its loss percentage gets converted into the equal profit percentage. What is the loss percentage?

(a)  16.66%

(b)  50%

(c)  33.33%

(d)  37.5%

Answer: (c)

63. The marked price of a pencil is 35% more than its cost price. What maximum discount percentage can be offered by the shopkeeper to sell his pencil at no profit or no loss?

(a)  34.24%

(b)  44.44%

(c)  25.92%

(d)  18.18%

Answer: (c)

64. What is the least number that should be added to the product 11 × 12 × 13 × 14 to make it a perfect square?

(a)  1

(b)  311

(c)  5

(d)  0

Answer: (a)

65. If 8 men can do a piece of work in 14 days, then in how much time will 7 men do the same piece of work?

(a)  16 days

(b)  15 days

(c)  17 days

(d)  18 days

Answer: (a)

66. Two buses are moving towards each other with speed of 20 km/h respectively. What will be the distance between the two buses one second before they collide?

(a)  21 m

(b)  18 m

(c)  12 m

(d)  15 m

Answer: (d)

Directions (Q. Nos. 67-70) The bar graph shows how many visitors participated in which adventure activity during their stay in an adventure resort. Study the diagram and answer the following questions.

67. Which adventure activity had the second lowest number of participants?

(a)  F

(b)  A

(c)  D

(d)  G

Answer: (c)

68. What is the ratio of participants of activity G to participants of activity B?

(a)  9 : 5

(b)  12  7

(c)  5 : 9

(d)  7 : 12

Answer: (c)

69. Participants of activity A were lesser than that of F by ……..%.

(a)  25

(b)  30

(c)  20

(d)  35

Answer: (c)

70. If 30% of participants of activity F had chosen to participate in activity G instead then the number of participants of G would have increased by?

(a)  90%

(b)  55%

(c)  45%

(d)  60%

Answer: (c)

71. The perimeter and the breadth of a rectangle are 82 cm and 20 cm respectively. Calculate the length of its diagonal.

(a)  58 cm

(b)  21 cm

(c)  42 cm

(d)  29 cm

Answer: (d)

72. Find the circumference of a circle of radius 7 cm.

(a)  56 cm

(b)  44 cm

(c)  16 cm

(d)  32 cm

Answer: (b)

73. The volume of a hemisphere is 89.83 cm3. Find its diameter.

(a)  3.5 cm

(b)  7 cm

(c)  14 cm

(d)  10.5 cm

Answer: (b)

74. ∆ABC is right angled at B. If m∠C = 45°, then find the value of (cosec A – √3).

(a) 

(b) 

(c) 

(d) 

Answer: (b)

75. In ∆PQR measure of angle Q is 90°. If  and PQ = 1 cm, then what is the length (in cm) of side QR?

(a)  2.6

(b)  3

(c)  2.4

(d)  4

Answer: (c)

76. Brent Index is associates with which of the followings?

(a)  crude oil prices

(b)  copper future prices

(c)  gold future prices

(d)  shipping rate index

Answer: (a)

77. Which of the following is not correctly matched?

(a)  First Five Year Plan : 1951-56

(b)  Second Five Year Plan : 156-61

(c)  Third Five Year Plan : 1961-66

(d)  Fourth Five Year Plan : 1966-71

Answer: (d)

78. Where is the Southern Alps mountain range located?

(a)  Australia

(b)  New Zealand

(c)  South Africa

(d)  Indonesia

Answer: (b)

79. By what name is the combined water stream of Ganga and Brahmaputra known?

(a)  Padma

(b)  Jamuna

(c)  Meghna

(d)  Tsangpo

Answer: (c)

80. Who was the exponent of Yog Darshan?

(a)  Patanjali

(b)  Gautam

(c)  Jaimini

(d)  Shankaracharya

Answer: (a)

81. During the Maratha period, who was of officer responsible for the village administration?

(a)  Havaladar

(b)  Patil

(c)  Chaudhary

(d)  Sardeshmukh

Answer: (b)

82. Govind Swami Pillai is associated with which musical instrument?

(a)  Mridangam

(b)  Tabla

(c)  Veena

(d)  Violin

Answer: (d)

83. Which country’s Parliament has passed a law that will limit the powers of its police?

(a)  Israel

(b)  France

(c)  Britain

(d)  China

Answer: (a)

84. Who among the following has been awarded the Blue Planet Prize 2017 in October, 2017?

(a)  Hans Joachim Schellnhuber

(b)  JK Anderson

(c)  M. Suzanchuk

(d)  Garry Bentzone

Answer: (a)

85. When was the national emblem of Bangladesh adopted?

(a)  1971

(b)  1973

(c)  1975

(d)  1977

Answer: (a)

86. Mass number is always equal to

(a)  number of protons

(b)  number of neutrons

(c)  sum of number of protons and number of neutron

(d)  sum number of protons and number of neutron

Answer: (d)

87. Which of the following statements are correct?

(I) Ethanol is a good solvent for organic compounds.

(II) Boiling and melting points of ethanol are low with respect to water.

(III) Ethanol is used tincture iodine.

(a)  I and II

(b)  I and III

(c)  II and III

(d)  All of these

Answer: (d)

88. Along with ‘Right to Life’ in Article-21 of Indian Constitution, it also includes

(a)  Right of Health

(b)  Right of Food

(c)  Right of Water

(d)  All options are correct.

Answer: (d)

89. Which Constitutional Amendment Act gave constitutional recognition to municipalities?

(a)  72nd Constitutional Amendment Act

(b)  73rd Constitutional Amendment Act

(c)  74th Constitutional Amendment Act

(d)  75th Constitutional Amendment Act

Answer: (*)

90. Which of the following is not a waste material removed from blood in the kidney?

(a)  Ammonia

(b)  Carbon dioxide

(c)  Urea

(d)  Uric acid

Answer: (b)

91. The movement of water molecules through the plasma membrane is called

(a)  Osmosis

(b)  Sedimentation

(c)  Precipitation

(d)  Transpiration

Answer: (a)

92. In November, 2017 second phase of ‘Bharat Net Project’ was launched. How much its approximate outlay?

(a)  Rs 55000

(b)  Rs 10000

(c)  Rs 20000

(d)  Rs 34000

Answer: (d)

93. NASA successfully tested supersonic landing parachute in November, 2017 for its approximate ……….. mission.

(a)  Zuma 2020

(b)  GOLD

(c)  ICON

(d)  Mars 2020

Answer: (d)

94. In October 2017, who launched India’s 3rd biggest ‘Initial Public Offering (IPO)’ of Rs 11370 crore?

(a)  Reliance Industries Limited

(b)  GIC Re

(c)  Coal India Limited

(d)  Infosys

Answer: (b)

95. On January 5, 2018 Lok Sabha passed High Court and Supreme Court Judges (Salaries and Conditions of Service) Amendment Bill, 2017. The Bill proposes to hike the salary of Chief Justice of India (CJI) to nearly

(a)  Rs 2.80 lakh per month

(b)  Rs 3.90 lakh per month

(c)  Rs 2 lakh per month

(d)  Rs 5 lakh per month

Answer: (a)

96. What is the resistance (in Ω) of an electrical component if a current of 0.4 A passes through it on application of 12 V of potential difference across it?

(a)  4.8

(b)  60

(c)  9.6

(d)  30

Answer: (d)

97. If the acceleration due to gravity on the surface of Earth is g, then the acceleration due to gravity on the surface of a planet who mass is half of that of Earth and radius is same as that of Earth is

(a)  g/2

(b)  4g

(c)  2g

(d)  g/4

Answer: (a)

98. An earthquake of magnitude …….. on Richter scale has thousands times more destructive energy than an earthquake of magnitude 4.

(a)  5

(b)  7

(c)  6

(d)  8

Answer: (c)

99. Which of the statements given below are correct?

(A) In 2017, Giannis Antetokounmpo played for the NBA team Houston Rockets.

(B) In 2018 IPL auctions, Kings XI Punjab retained Axar Patel.

(C) In 2017, David Warner captained the IPL team Royal Challengers Bangalore.

(a)  Only B

(b)  A and B

(c)  All of these

(d)  None of these

Answer: (a)

100. ……….. provides network backbone and e-governance support to government bodies in India.

(a)  NIC

(b)  NASSCOM

(c)  Department of Information Technology

(d)  State Bank of India

Answer: (a)

SSC Combined Graduate Level (Tier-I) Examination Held on 10-8-2017 Question Paper With Answer Key

SSC Combined Graduate Level (Tier-I) Examination Held on 10-8-2017
SSC Combined Graduate Level (Tier-I) Examination Held on 10-8-2017 Question Paper With Answer Key

SSC Combined Graduate Level (Tier-I) Examination Held on 10-8-2017

 

1. In the following question, select the related word from the given alternatives,

Pressure : Barometer : : ? : Odometer

(A) Humidity

(B) Distance

(C) Thickness

(D) Wind

Answer: (B)

2. In the following question, select the related letters from the given alternatives.

AEDM : ZQRN : : FLMO : ?

(A) BZYS

(B) CZYS

(C) SZYB

(D) YZBC

Answer: (A)

3. In the following question, select the related number from the given alternatives.

243 : 578 : : 163 : ?

(A) 291

(B) 326

(C) 347

(D) 443

Answer: (B)

4. In the following question, select the odd word from the given alternatives.

(A) Flower

(B) Fruit

(C) Leaves

(D) Root

Answer: (D)

5. In the following question, select the odd letters from the given alternatives.

(A) CEAC

(B) FHDF

(C) PRMP

(D) TVRT

Answer: (C)

6. In the following question, select the odd number pair from the given alternatives.

(A) 2132 – 161

(B) 2678 – 672

(C) 4325 – 120

(D) 6931 – 162

Answer: (A)

7. Arrange the given words in the sequence in which they occur in the dictionary.

(1) Dillydallying

(2) Dillydallied

(3) Dillydally

(4) Dilled

(5) Dillydallies

(A) 42351

(B) 42531

(C) 45312

(D) 45321

Answer: (B)

8. In the following question, which one set of letter when sequentially placed at the gaps in the given letter series shall complete it?

1 m_o_n m_l_n_ _n m l

(A) molnon

(B) nolmoo

(C) nomloo

(D) noolmm

Answer: (B)

9. In the following question, select the missing number from the given series.

21, 25, 52, 68, 193, ?

(A) 229

(B) 242

(C) 257

(D) 409

Answer: (A)

10. Kamal starts walking from his home facing west direction. After walking 10 km he takes a right turn and walks another 10 km, He takes another right turn and walks 10 km to reach his school. How far (in km) and in which direction is he from his home?

(A) 10 km, North

(B) 10 km, South

(C) 20 km, North-East

(D) 20 km, South-West

Answer: (A)

11. In a class, P has more marks than Q and R has more marks than P, who among them will have the least marks?

(A) P

(B) Q

(C) S

(D) T

Answer: (B)

12. In the following question, from the given alternative works, select the word which cannot be formed using the letters of the given word :

‘RECIPROCATE’

(A) PROCEED

(B) RACE

(C) REPEAT

(D) TEAR

Answer: (A)

13. In a certain code language, ‘CASIO’ is written as ‘3119915’. How is ‘CITIZEN’ written in that code language?

(A) 295629134

(B) 3192295614

(C) 3912659214

(D) 3920926514

Answer: (D)

14. In the following question, correct the equation by interchanging the two signs.

6 ÷ 17 × 51 + 6 – 12 = 4

(A) × and ÷

(B) + and ÷

(C) + and −

(D) − and ÷

Answer: (A)

15. If 6 * 9 = 58 and 3 * 9 – 7 = 34, then in the expression A * 4 – 9 = 91, what is the value of ‘A’?

(A) 6.5

(B) 17.5

(C) 20.5

(D) 30.5

Answer: (C)

16. In the following question, select the number which can be placed at the sign of question mark (?) from the given alternatives.

(A) 4

(B) 6

(C) 8

(D) 16

Answer: (A)

17. How many triangles are there in the given figure?

(A) 4

(B) 5

(C) 6

(D) 7

Answer: (C)

18. In the following question below are given some statements followed by some statements to be true even if they seem to b e at variance from commonly known facts, read all the conclusions and then decide which of the given conclusion logically follows the given statements.

Statements :

(I) Some staplers are pins.

(II) All pins are markers.

Conclusions :

(I) Some staplers are markers.

(II) All markers are pins.

(A) Only conclusion I follows

(B) Only conclusion II follows

(C) Neither conclusion I nor conclusion II follows

(D) Both conclusions follow

Answer: (A)

19. Two positions of a cube are shown below. What will come opposite to face containing ‘4’?

(A) 1

(B) 2

(C) 3

(D) 6

Answer: (A)

20. Identify the diagram that best represent the relationship among the given classes.

Green, Mango, Fruits

Answer: (B)

21. Which answer figure will complete the pattern in the question figure?

Answer: (D)

22. From the given answer figures, select the one in which the question figure is hidden/embedded.

Answer: (B)

23. A piece of paper is folded and punched as shown below in the question figures. From the given answer figures, indicate how it will appear when opened?

Answer: (B)

24. If a mirror in place on the line AB, then which of the answer figure is right image of the given figure?

Answer: (C)

25. A word is represented by only one set of numbers as given in any one of the alternatives. The sets of numbers given in the alternatives are represented by two classes of alphabets as shown in the given two matrices. The columns and rows of Matrix-I are numbered from 0 to 4 and that of Matrix-II are numbered from 5 to 9. A letter from these matrices can be represented first by its row and next by its column, for example, ‘A’ can be represented by 20, 43, , and ‘U’ can be represented by 68, 87 etc. Similarly, you have to identify the set for the word ‘GUIDE’.

(A) 00, 68, 95, 58, 04

(B) 14, 75, 88, 87, 40

(C) 23, 99, 76, 78, 31

(D) 41, 87, 57, 66, 12

Answer: (C)

26. In which market form, a market or industry is dominated by a few firms?

(A) Perfect Competition

(B) Monopoly

(C) Oligopoly

(D) Monopolistic

Answer: (C)

27. Which amongst the following is not a component of monetary policy in India?

(A) Repo rate

(B) Moral suasion

(C) Credit Rationing

(D) Public Debt

Answer: (D)

28. Who among the following is not a member of any of two houses of our country?

(A) Prime Minister

(B) Finance Minister

(C) President

(D) Railway Minister

Answer: (C)

29. Which article of Indian Constitution has the provision for National Emergency?

(A) Article 350

(B) Article 352

(C) Article 312

(D) Article 280

Answer: (B)

30. Who led the Bardoli Satyagraha movement?

(A) Mahatma Gandhi

(B) Rabindra Nath Tagore

(C) Sardar Vallabhbhai Patel

(D) Chittaranjan Das

Answer: (C)

31. Who is known as the ‘Father of Indian Unrest’?

(A) Ananth Singh

(B) Bal Gangadhar Tilak

(C) Bhagat Singh

(D) Dadabhai Naoroji

Answer: (B)

32. Which of the following region is covered by tropical evergreen forest?

(A) Eastern Ghat

(B) Vindhyanchal

(C) Aravalli

(D) Western Ghat

Answer: (D)

33. The final boundary between the Earth and the outer space is called-

(A) magnetosphere

(B) ionosphere

(C) mesopause

(D) magnetopause

Answer: (D)

34. What is the name of the hormone produced by thymus gland?

(A) Thyroxine

(B) Auxins

(C) Cytokinis

(D) Thymosin

Answer: (D)

35. Photosynthesis takes place in the presence of chlorophyll and-

(A) water

(B) nutrients

(C) carbon-dioxide

(D) sunlight

Answer: (D)

36. Which blood group is universal acceptor?

(A) O+

(B) O

(C) AB

(D) AB+

Answer: (D)

37. If objects appear enlarged and inverted in a rear view mirror, then which type of mirror is used?

(A) Concave

(B) Convex

(C) Cylindrical

(D) Plane

Answer: (A)

38. Soap bubble attains spherical shape due to –

(A) inertia

(B) pressure

(C) surface tension

(D) viscosity

Answer: (C)

39. CAD stands for-

(A) Common Aided Design

(B) Computer Aided Design

(C) Complex Aided Design

(D) Communication Aided Design

Answer: (B)

40. Which of the following is a characteristic of an exothermic reaction?

(A) Release of heat

(B) Absorption of heat

(C) Doesn’t involve any change in temperature

(D) None of the option is correct

Answer: (A)

41. What is the chemical formula for Sodium Chloride (Salt)?

(A) NaCl2

(B) NaCl

(C) Na2Cl

(D) Na2C

Answer: (B)

42. Which of the following gas contributes the maximum to the phenomena of global warming?

(A) Methane

(B) Chloroflurcarbon (CFC)

(C) Nitrozen dioxide

(D) Carbon dioxide

Answer: (D)

43. Who selects the Social Audit Committee under MGNREGA scheme?

(A) Chief Minister

(B) Gram Sabha

(C) Mayor

(D) B.D.O

Answer: (B)

44. Which of the following was invented by Sir Humphry Davy?

(A) Safety Pin

(B) Steam Engine

(C) Safety Lamp

(D) X-Rays

Answer: (C)

45. Who has won the 2016 Men’s Single title at US Open?

(A) Novak Dojokovic

(B) Rafael Nadal

(C) Stan Wawrinka

(D) Andy Murray

Answer: (C)

46. Match the following.

Dancer

(1) Radha Reddy

(2) Padma Subrahmanyam

(3) Sitara Devi

Dance

(a) Bharatnatyam

(b) Kathak

(c) Kuchipudi

(A) 1-b, 2-a, 3-c

(B) 1-c, 2-b, 3-a

(C) 1-c, 2-a, 3-b

(D) 1-a, 2-c, 3-b

Answer: (C)

47. Who has been recently awarded with Nobel Prize for peace in 2016?

(A) Juan Manuel Santos

(B) Henry Dunant

(C) Kailash Satyarthi

(D) Malala Yousefzai

Answer: (A)

48. Which of the following is a book written by Shashi Tharoor?

(A) It’s Not About You

(B) Invisible People

(C) An Era of Darkness

(D) Democrats and Dissenters

Answer: (C)

49. With which country India has recently decided to partner with for strategic storage of crude oil in southern India?

(A) Iran

(B) Iraq

(C) United Arab Emirates

(D) United States of America

Answer: (C)

50. Which neighbouring country of India is also reffered as ‘Druk Yul’?

(A) Myanmar

(B) Maldives

(C) Bhutan

(D) Afghanistan

Answer: (C)

51. How many numbers are there between 1 to 200 which are divisible by 3 but not by 7?

(A) 38

(B) 45

(C) 57

(D) 66

Answer: (C)

52. 10 women can do a piece of work in 6 days, 6 men can do same work in 5 days and 8 children can do it in 10 days. What is the ratio of the efficiency of woman, a man and a child respectively?

(A) 4 : 6 : 3

(B) 4 : 5 : 3

(C) 2 : 4 : 3

(D) 4 : 8 : 3

Answer: (D)

53. The ratio of the volume of two cylinders is 7 : 3 and the ratio of their heights is 7 : 9. If the area base of the second cylinder is 154 cm2, then what will be the radius (in cm) of the first cylinder?

(A) 6√2

(B) 6√3

(C) 7√2

(D) 7√3

Answer: (D)

54. Kanchan bought a clock with 25% discount on marked price. She sold it with 75% gain on the price she bought. What was her profit percentage on the marked price?

(A) 31.25

(B) 50

(C) 56.25

(D) 60

Answer: (A)

55. A, B and C received an amount of Rs 8400 and distributed among themselves in the ratio of 6 : 8 : 7 respectively. If they save in the ratio of 3 : 2 : 4 respectively and B saves Rs 400, then what is the ratio of the expenditures of A, B and C respectively?

(A) 6 : 8 : 7

(B) 8 : 6 : 7

(C) 9 : 14 : 10

(D) 12 : 7 : 9

Answer: (C)

56. The average age of 24 students is 12 years. It was observed that while calculating the average age, the age of a student was taken as 14 years instead of 8 years. What will be the correct average age (in years)?

(A) 11.25

(B) 11.5

(C) 11.75

(D) 12.25

Answer: (C)

57. 70% of the cost price of a article is equal to the 40% of its selling price. What is the profit or loss percentage?

(A) 63% loss

(B) 70% loss

(C) 75% profit

(D) 80% profit

Answer: (C)

58. a% of b + b% of a = ……

(A) 2a% of b

(B) 2a% of 2b

(C) 2a% of 2a

(D) 2b% of 2b

Answer: (A)

59. If I walk at 7/6 of my usual speed, then I reach my office 15 minutes early. What is the usual time taken (in minutes) by me to reach the office?

(A) 60

(B) 75

(C) 90

(D) 105

Answer: (D)

60. A person lent Rs 10000 to B for 3 years and 6000 to C for 4 years on simple interest at same rate of interest and received Rs 5400 in all from both of them as interest. What is the rate of interest (in %) ?

(A) 10

(B) 12.5

(C) 15

(D) 20

Answer: (A)

61. If x3 + 2x2 – 5x + k is divisible by x + 1, then what is the value of k?

(A) −6

(B) −1

(C) 0

(D) 6

Answer: (A)

62. If 3x + [1/(5x)] = 7, then what is the value of 5x/(15x2 + 15x + 1)?

(A) 1/5

(B) 1/10

(C) 2/5

(D) 10

Answer: (B)

63. If x + [1/4x)] = 5/2, then what is the value of (64x6 + 1)/8x3?

(A) 110

(B) 115

(C) 125

(D) 140

Answer: (A)

64. If x2 + x = 19, then what is the value of (x + 5)2 + [1/(x + 5)2]?

(A) 77

(B) 79

(C) 81

(D) 83

Answer: (B)

65. In triangle ABC, AD, BE and CF are the medians intersecting at point G and area of triangle ABC is 156 cm2. What is the area (in cm2) of triangle FGE?

(A) 13

(B) 26

(C) 39

(D) 52

Answer: (A)

66. In triangle ABC, ∠ABC = 15°. D is a point on BC such that AD = BD. What is the measure of ∠ADC (in degrees)?

(A) 15

(B) 30

(C) 45

(D) 60

Answer: (B)

67. The length of diagonal of a square is 9√2 cm. The square is reshaped to form a triangle. What is the area (in cm2) of largest incircle that can be formed in that triangle?

(A) 6n

(B) 9n

(C) 12n

(D) 15n

Answer: (C)

68. The length of the common chord of two intersecting circles is 12 cm. If the diameters of the circles are15 cm and 13 cm, then what is the distance (in cm) between their centers?

(A) 7/2

(B) 7

(C) 7√2

(D) 14

Answer: (B)

69. What is the simplified value of sec4θ – sec2 θ tan2 θ?

(A) cosec2 θ

(B) sec2 θ

(C) cot2 θ

(D) sec θ tan θ

Answer: (B)

70. What is the simplified value of (sin A – cosec A) (sec A – cos A) (tan A + cot A) ?

(A) 1

(B) −1

(C) 0

(D) 2

Answer: (B)

71. If (1/cos θ) – (1/cot θ) = 1/P, then what is the value of cos θ?

(A) (P + 1)/(P – 1)

(B) (P2 + 1) / 2P

(C) 2P/(P2 + 1)/P

(D) 2P/(P2 + 1)

Answer: (D)

Directions- (Q. 72-75) The table given below represents the production and sales of wheat in 4 different countries A, B, C and D over a period of 4 years. All the end of year 2010 A, B,  C and D had a stock of 5200, 3500, 7835 and 1956 (in ‘000 quintals) of wheat respectively. For any given year, the stock of wheat is calculated as :

    Stock of year (n + 1) = stock at end of year (n) + production in year (n + 1) – sales in year (n + 1)

    And, Surplus of year (n) = production in year (n) – sales in year (n)

72. What is the surplus (in ‘000 quintals) of country A of years 2013 and 2014 taken together?

(A) 122

(B) 131

(C) 143

(D) 158

Answer: (B)

73. What is the stock (in ‘100 quintals) of country C at end of the 4 years period?

(A) 5926

(B) 6213

(C) 6826

(D) 8844

Answer: (C)

74. What is the difference (in ‘000 quintals) in average production and average sales respectively of country C in the given four years?

(A) −252.25

(B) −415.50

(C) 350.75

(D) 275.25

Answer: (A)

75. What can be said about total surplus of country B and country D over the 4 years?

(A) Surplus of B = Surplus of D

(B) Surplus of D > Surplus of B

(C) Surplus of B > Surplus of D

(D) No relation is threre

Answer: (B)

Directions – (Q. 76-77) In the following question, some part of the sentence may have errors. Find out which part of the sentence has an error and select the appropriate option. If a sentence is free from error, select ‘No error’.

76. Inspite of the doctor’s stern warming (1)/ Latika continued taking (2)/ sugars in her milk. (3)/ No error (4)-

(A) 1

(B) 2

(C) 3

(D) 4

Answer: (C)

77. Myself and Roshni (1)/ will take care of (2)/ the event on Sunday. (3)/ No Error (4)-

(A) 1

(B) 2

(C) 3

(D) 4

Answer: (A)

Directions – (Q. 78-79) In the following question, the sentence given with blank to b e filled in with an appropriate word. Select the correct alternative out of the four and indicate it by selecting the appropriate option.

78. …………. the rain stopped, the concert had to be suspended.

(A) Until

(B) Unless

(C) Till

(D) While

Answer: (A)

79. The elephant stampeded and tore …… the streets.

(A) on

(B) out

(C) off

(D) down

Answer: (D)

Directions – (Q. 80-83) In the following question, out of the four alternatives, select the word similar in meaning to the word given.

80. Scuttle

(A) Solitary

(B) Superficial

(C) Soothing

(D) Brazier

Answer: (D)

81. Loquacious

(A) Talkative

(B) Foolishness

(C) Graceful

(D) Entertainer

Answer: (A)

82. Obfuscate

(A) Envelop

(B) Puzzle

(C) Haze

(D) Clarify

Answer: (D)

83. Triumph

(A) Establish

(B) Sorrow

(C) Disdain

(D) Elation

Answer: (B)

Directions – (Q. 84-85) In the following question, out of four alternatives, select the alternative which best expresses the meaning of the  idiom/phrase.

84. Chicken-hearted

(A) Coward

(B) Short tempered

(C) Composed

(D) Bold

Answer: (A)

85. Red letter day

(A) Starting day

(B) Holiday

(C) Significant day

(D) Ending day

Answer: (C)

Directions (Q. 86-87) Improve the bracketed part of the sentence.

86. He jumped off the train while it (had been running).

(A) has been running

(B) ran

(C) was running

(D) No improvement

Answer: (C)

87. I (didn’t see) him since we met two years ago.

(A) am not seeing

(B) have not seen

(C) had not seen

(D) No improvement

Answer: (B)

Directions (Q. 88-89) In the following question, out of the four alternatives, select the alternative which is the best substitute of the phrase.

88. One who is new to a profession.

(A) Nuance

(B) Pun

(C) Tyro

(D) Vandal

Answer: (C)

89. A speech or a presentation made without previous preparation.

(A) Euphemism

(B) Obituary

(C) Extempore

(D) Soliloquy

Answer: (C)

Directions (Q. 90-91) In the following question, four words are given out of which one word is incorrectly. Spelt find the incorrectly spelt word.

90.

(A) Millionaire

(B) Omission

(C) Foreign

(D) Proprietry

Answer: (D)

91.

(A) Acquaintence

(B) Appeasement

(C) Abnormality

(D) Accentuate

Answer: (A)

Directions- (Q. 92-93) The question below consist of a set of labeled sentences. Out of the four option given, select the most logical order of the sentences to form a coherent paragraph.

92. P : And the victims are likely to be the poorest of the poor as well the very sources of water-rivers, wetlands and aquifers.

Q : In India, water conflicts are likely to worsen before they begin to be resolved.

R : Till then they pose a significant threat to economic growth, security and health of the ecosystem.

S : Water is radically altering and affecting political boundaries all over world, between as well as within countries.

(A) SQPR

(B) PRQS

(C) QRPS

(D) PSQR

Answer: (A)

93. P : For one, very few entrepreneurs are willing to take on a new outsource, unless it comes with a guarantee of a certain level of sales.

Q : This invariably acts as an incentive for outsources to be lax in developing the business.

R ; Despite being the dominant partner in the relationship, the outsourcer doesn’t always have all the advantages.

S : The trade refers it as the minimum guarantee clause, which means that if a outsource is unable to reach an anticipated sales level, he will be compensated for the balance amount.

(A) PRQS

(B) SPQR

(C) QSPR

(D) RPSQ

Answer: (D)

Directions – (Q. 94) In the following question, a sentence has been given in Active/Passive voice. Out of four alternatives suggested, select the one which best expresses the same sentence in Passive/Active voice.

94. Somebody told me that there had been robbery in the jewellery exhibition.

(A) I was informed that there was  robbery in the jewellery exhibition.

(B) I was told by somebody that there has been a robbery in the jewellery exhibition.

(C) I was told by somebody about a robbery in the jewellery exhibition.

(D) I was told about a robbery in the jewellery exhibition.

Answer: (B)

Directions – (Q. 95) In the following question, a sentence has been given in Direct/Indirect speech. Out of the four alternatives suggested, select the one which best expresses the same sentence in Indirect/Direct speech.

95. Rohan said, “Where shall I be this time next month”.

(A) Rohan contemplated where shall he be that time the following month.

(B) Rohan asked that where should be that time next month.

(C) Rohan wondered where he should be that time the next month.

(D) Rohan wondered where he would be that time the following month.

Answer: (D)

Directions – (Q. 96-100) In the following passage some of the words have been left out. Read the passage carefully and select the correct answer for the given blank out of the four alternatives.

    The modes of action are …………. in science and religion. Science relies on experiment, whereas religion is based on experience. Any religious …….. whether it is Christ’s or Ramkrishna’s is personal and  ……… Science, on the other hand is marked by objectivity. Theory has to corroborated by …….. proof providing material comforts. The frontiers of science do not end in knowledge but are ……… to the formation of appliances for actual use.

96. The modes of action are ….. in science and religion.

(A) similar

(B) different

(C) equal

(D) relevant

Answer: (B)

97. Whereas religion is based on experience. Any religious ……….. whether.

(A) experience

(B) thought

(C) festival

(D) activity

Answer: (A)

98. It is Christ’s or Ramakrishna’s is personal and-

(A) significant

(B) irrelevant

(C) subjective

(D) objective

Answer: (C)

99. Theory has to be corroborated by ………… proof providing material comforts.

(A) intangible

(B) transparent

(C) tangible

(D) unique

Answer: (C)

100. The frontiers of science do not end in knowledge but are …….. to the formation of appliances for actual use.

(A) implied

(B) associated

(C) designated

(D) extended

Answer: (D)

Staff Selection Commission (SSC) Delhi Police Constable (Executive) Recruitment Examination Held on December 7, 2017 Question Paper With Answer Key

Staff Selection Commission (SSC) Delhi Police Constable (Executive) Recruitment Examination Held on December 7, 2017
Staff Selection Commission (SSC) Delhi Police Constable (Executive) Recruitment Examination Held on December 7, 2017 Question Paper With Answer Key

Staff Selection Commission (SSC) Delhi Police Constable (Executive) Recruitment Examination Held on December 7, 2017

Part A Reasoning

1. In the following question, select the related word from the given alternatives. Shirt : Garment : : Necklace : ?

(a)   Neck

(b)   Ornament

(c)   Beads

(d)   Round

Answer: (b)

2. In the following question, select the related letters from the given alternatives

TULIP : ZAROV : SCALP : ?

(a)   HRIGV

(b)   YIGRV

(c)   PRIHV

(d)   VHPRG

Answer: (b)

3. In the following question, select the related number from the given alternatives.

5 : 25 : : 10 : ?

(a)   96

(b)   50

(c)   42

(d)   121

Answer: (b)

4. In the following question, select the odd word from the given alternatives.

(a)   paragraph

(b)   word

(c)   sentence

(d)   circle

Answer: (d)

5. In the following question, select the odd letters from the given alternatives.

(a)   ZUPK

(b)   TOJD

(c)   WRMH

(d)   VQLG

Answer: (b)

6. In the following question, select the odd number pair from the given alternatives.

(a)   10-100

(b)   12-144

(c)   13-171

(d)   15-225

Answer: (c)

7. In the following question, select the odd number pair from the given alternatives.

(a)   6361-16

(b)   5921-16

(c)   4361-14

(d)   2963-20

Answer: (b)

8. How many triangles are there in the given figure?

(a)   3

(b)   4

(c)   5

(d)   6

Answer: (c)

9. How many rectangles are there in the given figure?

(a)   11

(b)   12

(c)   10

(d)   13

Answer: (d)

10. How many surface are there in the given figure?

(a)   6

(b)   7

(c)   8

(d)   5

Answer: (b)

11. Three positions of a cube are shown below. What will come opposite to face containing ‘α’?

(a)   β

(b)   θ

(c)   P

(d)   A

Answer: (c)

12. From the given answer figures, select the one in which the question figure is hidden/embedded.

Answer: (b)

13. Which answer figure will complete the pattern in the question figure?

Answer: (b)

14. If a mirror is placed on the line AB, then which of the answer figures is the right mirror image of the given

figure?

Answer: (d)

15. In the following question, select the number which can be placed at the sign of question mark from the given alternatives.

(a)   31

(b)   33

(c)   37

(d)   43

Answer: (a)

16. In the given figure, how many books are either old or historic but not both?

(a)   195

(b)   131

(c)   149

(d)   96

Answer: (c)

17. Arrange the given words in the sequence in which they occur in the dictionary.

(1) Doom      (2) Down    (3) Drone    (4) Drape    (5) Ding

(a)   51243

(b)   51234

(c)   52143

(d)   52134

Answer: (a)

18. Identify the diagram that best represents the relationship among the given classes.

Answer: (b)

19. In the following question, below are given some statements followed by some conclusions. Taking the given statements to be true even if they seem to be at variance from commonly known facts. Read all the conclusions and then decide which of the given conclusion logically follows the given statements.

Statement I. All leaves are green.

II. Some leaves are hard.

Conclusions I. Some hard are green.

II. All green are leaves

(a)   Only conclusion I follows

(b)   Only conclusion II follows

(c)   Both conclusion follow

(d)   Neither conclusion I nor conclusion II follows

Answer: (a)

20. In the following question, below are given some statements followed by some conclusions. Taking the given statements to be true even if they seem to be at variance from commonly known facts. Read all the conclusions and then decide which of the given conclusion logically follows the given statements.

Statements I. All pink are doors.

II. All male are doors.

Conclusions I. Some males are pink.

II. All doors are pink.

III. All pink are males

(a)   Only conclusion I follows

(b)   Only conclusion II follows

(c)   Both conclusions II and III follow

(d)   Neither conclusion follows

Answer: (d)

21. Pointing towards a man, a lady said, ‘He is the son of my husband’s brother. ‘How is the lady’s husband related to the man?

(a)   Son

(b)   Uncle

(c)   Brother

(d)   Husband

Answer: (b)

22. F is the mother of T. T is the sister of W who is the only son of K. J is the brother of K. How is the mother of W relate to K?

(a)   Mother

(b)   Sister

(c)   Wife

(d)   Niece

Answer: (c)

23. On a certain sum at the rate of 10% per annum, the difference between simple interest for 2 years and the simple interest for 3 years is Rs 54. What is the value of sum?

(a)   Rs 540

(b)   Rs 1080

(c)   Rs 720

(d)   Rs 480

Answer: (a)

24. P, Q and R can complete a piece of work in 45, 15 and 5 days respectively. In how many days all three together can complete the same work?

(a)   45/14 days

(b)   45/13 days

(c)   9/2 days

(d)   11/2 days

Answer: (b)

Directions (Q. Nos. 25-27) In the given questions, there are four figures given out of which three are similar in some manner and one is not like the others. Select the figure which is odd one out.

25. 

Answer: (b)

26. 

Answer: (b)

27. 

Answer: (b)

28. In the following question, select the missing number from the given series.

4, 5, 7, 10, 11, 13, 16, ?

(a)   14

(b)   17

(c)   19

(d)   23

Answer: (b)

29. In the following question, select the missing number from the given series.

5, 3, 8, 11, 19, ?

(a)   43

(b)   37

(c)   25

(d)   30

Answer: (d)

Directions (Q. Nos. 30-32) In the following questions, select the figure which can be placed at the sign of question mark from the given alternatives.

30. 

Answer: (d)

31. 

Answer: (a)

32. 

Answer: (c)

33. In a certain code language, ‘FEARS’ is written as ‘HHCUU’. How is ‘STAIR’ written in that code language?

(a)   VWLTC

(b)   CLTUW

(c)   UWCLT

(d)   WCTLX

Answer: (c)

34. In a certain code language, ‘ROADS’ is written as ‘57’ and ‘HORN’ is written as ‘55’. How is ‘BLOW’ written in that code language?

(a)   46

(b)   48

(c)   47

(d)   52

Answer: (d)

35. In a certain code language, ‘TUNES’ is written as ‘16’ and ‘FEEZE’ is written as ‘11’. How is ‘CLIMB’ written in that code language?

(a)   14

(b)   10

(c)   12

(d)   16

Answer: (c)

Part B General Knowledge/Current Affairs

36. In India, inflation is measured on the basis of

(a)   Wholesale Price Index

(b)   Producer Price Index

(c)   Consumer Price Index

(d)   No option is correct.

Answer: (c)

37. The workforce population includes people aged from …….. years to …….. years.

(a)   10, 69

(b)   15, 64

(c)   21, 55

(d)   18, 65

Answer: (b)

38. In which type of economy, there is neither exports nor imports?

(a)   Closed

(b)   Open

(c)   Mixed

(d)   Global

Answer: (a)

39. Which price is declared by the government every year before the sowing season to provide incentives to the farmers for raising the production of their crops?

(a)   Maximum support price

(b)   Minimum support price

(c)   Maximum stock price

(d)   Minimum stock price

Answer: (a)

40. What is an association of workers for the purpose of maintaining or improving the conditions of their employment called as?

(a)   Federation

(b)   Trade Union

(c)   Accord

(d)   Colony

Answer: (b)

41. ……… is the right of a person. party or nation to stop a certain decision or law.

(a)   Strike

(b)   Veto

(c)   Both Strike and Veto

(d)   Neither Strike nor Veto

Answer: (b)

42. Which part of the Indian Constitution deals with the Municipalities?

(a)   Part IV

(b)   Part V

(c)   Part IX A

(d)   Part X

Answer: (c)

43. Anti-defection law is given in which Schedule of Indian Constitution?

(a)   Seventh

(b)   Eighth

(c)   Ninth

(d)   Tenth

Answer: (d)

44. Which among the following is the most frequently mentioned tax in the inscription of Cholas who ruled in Tamil Nadu?

(a)   Vetti

(b)   Kadami

(c)   Thari

(d)   Manai

Answer: (a)

45. Chahamanas who ruled over the region around Delhi and Ajmer were later known as

(a)   Chalukyas

(b)   Chauhans

(c)   Cholas

(d)   Chandelas

Answer: (b)

46. In which session, Sarojini Naidu became the first Indian women President of Indian National Congress?

(a)   1922, Gaya

(b)   1925, Kanpur

(c)   1928, Calcutta

(d)   1931, Karachi

Answer: (b)

47. The period from 1915-47 in India’s freedom struggle was termed as

(a)   Extremist phase

(b)   Moderate phase

(c)   Gandhian phase

(d)   No option is correct

Answer: (c)

48. What is the name of the Arabic book by Al-Biruni?

(a)   Kitab-ul-Hind

(b)   Hindustan-nama

(c)   Tarikh-e-Hindustan

(d)   Fatawa-e-Hindustani

Answer: (a)

49. Who started the Shuddhi Movement in India in the 1920’s?

(a)   Subhash Chandra Bose

(b)   Swami Dayanand Saraswati

(c)   Chandrashekhar Azad

(d)   Raja Ram Mohan Roy

Answer: (b)

50. The Tropic of Cancer passes through how many States of India?

(a)   6

(b)   7

(c)   8

(d)   9

Answer: (c)

51. What is the approximate time lag from Gujarat to Arunachal Pradesh?

(a)   1 h

(b)   1.5 h

(c)   2 h

(d)   2.5 h

Answer: (b)

52. Which is not a part of the Deccan plateau in the Northeast India?

(a)   The Meghalaya plateau

(b)   Karbi-Anglong plateau

(c)   North Cachar hills

(d)   The Kaimur hills

Answer: (d)

53. Which vegetation covers the deltas of the Ganga, the Mahanadi, the Krishna, the Godavari and the Kaveri?

(a)   Montane forest

(b)   Mangrove forests

(c)   Thorn forests

(d)   Tropical deciduous forest

Answer: (b)

54. Anamudi is the highest peak of

(a)   Anaimalai hills

(b)   Nilgiri hills

(c)   Aravalli hills

(d)   Jaintia hills

Answer: (a)

55. Which State is largest producer of jute in India?

(a)   Bihar

(b)   Assam

(c)   Odisha

(d)   West Bengal

Answer: (d)

56. Each centimeter has ten equal divisions called

(a)   Metre

(b)   Micrometre

(c)   Decimetre

(d)   Millimetre

Answer: (d)

57. Translucent objects are the objects through which we

(a)   cannot see at all

(b)   can see clearly

(c)   can see but not very clearly

(d)   No option is correct

Answer: (c)

58. A thin wire that gives off light from the bulb is called

(a)   Terminal

(b)   Tip

(c)   Source

(d)   Filament

Answer: (d)

59. Which among the following is the reproductive part of a plant?

(a)   Root

(b)   Stem

(c)   Flower

(d)   Fruit

Answer: (c)

60. When a disease-carrying microbe enters our body, the body produces………. to fight the microbe.

(a)   Antigen

(b)   Antibodies

(c)   Antibiotics

(d)   Anti Allergenic

Answer: (c)

61. Which man-made fibre is obtained from wood pulp?

(a)   Nylon

(b)   Rayon

(c)   Silk

(d)   Polyster

Answer: (b)

62. Which chemical is used to test whether substance is acidic or basic?

(a)   Indicator

(b)   Promoter

(c)   Catalyst

(d)   Neutralizer

Answer: (a)

63. Presence of which among the following essential for rusting?

(I) Water (or water vapour)

(II) Oxygen

(a)   Only I

(b)   Only II

(c)   Both I and II

(d)   Neither I nor II

Answer: (c)

64. Which is not an example of one-sided symbiotic relationship?

(a)   Cattle egrets and cattle

(b)   A hermit crab and an empty seashells

(c)   A spider on a tree

(d)   Tapeworm in host’s stomach

Answer: (d)

65. What does the sum total of the populations of the same kind of organisms called?

(a)   Kingdom

(b)   Class

(c)   Phylum

(d)   Species

Answer: (d)

66. Which is new scheme named ‘Saubhagya’ to carry out electrification of all households in India?

(a)   Pradhan Mantri Safal Ujjwal Bhagya Yojana

(b)   Pradhan Mantri Sahaj Bijli Har Ghar Yojana

(c)   Pradhan Mantri Samridh Ujala Bhagyashree Yojana

(d)   No option is correct

Answer: (b)

67. The government of which state will merge it existing seven health schemes to launch a unified scheme-Arogya Bhagya?

(a)   Karnataka

(b)   Maharashtra

(c)   Meghalaya

(d)   Odisha

Answer: (a)

68. Which country’s scientists have successfully created the world’s shortest X-ray laser pulse?

(a)   Sweden

(b)   Bulgaria

(c)   Norway

(d)   Switzerland

Answer: (b)

69. Which space agency has launched Joint Polar Satellite System-I into space on November 18, 2017?

(a)   Indian Space Research Organization

(b)   China National Space Administration

(c)   National Aeronautics and Space Administration

(d)   Russia State Corporation for Space Activities

Answer: (c)

70. India’s Sundar Singh Gurjar won Gold medal in which event at World Para Athletics, 2017?

(a)   Javelin throw

(b)   Long jump

(c)   Discus throw

(d)   High Jump

Answer: (a)

71. India’s Mithali Raj became first women cricketer to score…… career runs in Women’s One Day International Cricket.

(a)   5000

(b)   6000

(c)   7000

(d)   8000

Answer: (b)

72. In which country India has designated Zorinpui Land Check post as an authorized immigration check post?

(a)   Myanmar

(b)   Bangladesh

(c)   Bhutan

(d)   Nepal

Answer: (c)

73. What is the name of India-Nepal joint military exercise whose 12th edition was held from September 3-16, 2017 in Nepal?

(a)   Sampriti

(b)   Surya-kiran

(c)   Indra

(d)   Prabal

Answer: (b)

74. Author of the novel named ‘Lincoln in the Bardo’ has won which literary award in 2017?

(a)   Pulitzer Prize

(b)   Man Booker Prize

(c)   European Union Prize

(d)   Giller Prize

Answer: (b)

75. I which field Richard H. Thaler has won Nobel Prize 2017?

(a)   Economic Sciences

(b)   Literature

(c)   Physics

(d)   Chemistry

Answer: (a)

76. What is the name of Google’s digital payment app launched on September 18, 2017?

(a)   Tez

(b)   Veg

(c)   Kranti

(d)   Bhim

Answer: (a)

77. Who is the head of the Economic Advisory Council to the Prime Minister Constituted on September 25, 2017?

(a)   Ashima Goyal

(b)   Ratan Watal

(c)   Bibek Debroy

(d)   Surjit Bhalla

Answer: (c)

78. Which is the 15th country to ratify the Solar Alliance Framework Agreement?

(a)   Somalia

(b)   Tuvalu

(c)   Fiji

(d)   Guinea

Answer: (d)

79. Which is the last country to join the Paris Climate Agreement in November, 2017?

(a)   Fiji

(b)   Syria

(c)   Afghanistan

(d)   Ghana

Answer: (b)

80. Which Metro system is the world’s first completely green Metro system?

(a)   Hong Kong Mass Transit Railway

(b)   Singapore Mass Rapid Transit

(c)   Delhi Metro Rail Corporation

(d)   London Underground

Answer: (c)

81. On November 1, 2017, the Energy Efficiency Services Limited (EESL) under the Ministry of Power, launched “Creating and Sustaining Markets for Energy Efficiency project” in partnership with

(a)   World Wildlife Fund (WWF)

(b)   Global Environment Facility (GEF)

(c)   Green Climate Fund (GCF)

(d)   World Resources Group (WRG)

Answer: (c)

82. Who among the following has been re-elected on November 21, 2017 as a member of International Court of Justice?

(a)   Dalveer Bhandari

(b)   Hishashi Owada

(c)   Peter Tomka

(d)   Ronny Abraham

Answer: (a)

83. On November 10, 2017 Aurdey Azoulay was appointed as the new Director General of which United Nations body?

(a)   WHO

(b)   UNICEF

(c)   UNDP

(d)   UNESCO

Answer: (d)

84. The ninth BRICS summit 2017 was organized in which city of China?

(a)   Suzhou

(b)   Xian

(c)   Hong Kong

(d)   Xiamen

Answer: (d)

85. Which country is at the top of the eighth edition of the World Giving Index compiled in October, 2017 by the ‘Charities Aid Foundation’?

(a)   United States of America

(b)   Australia

(c)   Myanmar

(d)   China

Answer: (c)

Part C Numerical Ability

86. The sum of a positive number and its cube is 10. What is the value of number?

(a)   1

(b)   2

(c)   3

(d)   4

Answer: (b)

87. What is the value of 13 + 23 + ….93?

(a)   2025

(b)   3075

(c)   1575

(d)   1975

Answer: (477)

88. What is the value of 

(a)   10/11

(b)   7/11

(c)   21/11

(d)   12/11

Answer: (c)

89. What is the Highest Common Factor of 14, 42 and 84?

(a)   2

(b)   7

(c)   14

(d)   1

Answer: (c)

90. What is the value of 

(a)   0.22

(b)   2.22

(c)   2.32

(d)   2.42

Answer: (c)

91. If 80% of of P = 2/5 of Q, then what is P : Q?

(a)   2 : 1

(b)   1 : 2

(c)   1 : 4

(d)   4 : 1

Answer: (b)

92. The annual income of Rahul and Mohit are in the ratio 17 : 12 and the ratio of their expenditure is 5 : 3. If each of them saves Rs 9000 yearly, then what will be the annual income of Mohit?

(a)   Rs 24000

(b)   Rs 21000

(c)   Rs 22400

(d)   Rs 34000

Answer: (a)

93. 12 persons have Rs 49 on an average. If a person with Rs 55.5 join them, then what will be the new average?

(a)   Rs 49.5

(b)   Rs 50

(c)   Rs 49.75

(d)   Rs 50.25

Answer: (a)

94. What will be the simple interest on a sum of Rs 4000 for 6 years at the rate of 8% per annum?

(a)   Rs 2020

(b)   Rs 1920

(c)   Rs 2080

(d)   Rs 1860

Answer: (b)

95. Vyom purchased 30 dozen pens at the rate Rs 48 per dozen. He sold 10 dozen at 20% profit and the remaining 20 dozen at 10% profit. What is his profit percentage?

(a)   13.33%

(b)   15.17%

(c)   25%

(d)   20%

Answer: (a)

96. The cost price of an article is 64% of the marked price. What will be the gain percentage after allowing a discount of 12% on the marked price?

(a)   25%

(b)   37.5%

(c)   33.33%

(d)   40%

Answer: (b)

97. The ratio of the outer and the inner perimeter of a circular path is 14 : 11. If path is 15 m wide, then what is the radius of the inner circle?

(a)   70 m

(b)   33 m

(c)   35 m

(d)   55 m

Answer: (d)

98. A boat covers 20 km upstream and 40 km downstream distance in 4 hours. while it covers 70 km upstream and 60 km downstream distance in 10 hours. What is the speed of the current?

(a)   10 km/h

(b)   15 km/h

(c)   5 km/h

(d)   12.5 km/h

Answer: (c)

99. P started a business by investing Rs 60000. After 4 months Q joined the business by investing Rs 80000. After 4 more months, R joined the business by investing Rs 100000. What will be the ratio of profits of P, Q and R at end of 2 years?

(a)   9 : 11 : 10

(b)   9 : 12 : 10

(c)   12 : 14 : 15

(d)   9 : 10 : 10

Answer: (d)

100. S and V can do a work in 12 days. M and S can do the same work in 16 days. M and V can do the same work in 24 days. In how many days S. V and M together can complete the same work?

(a)   32/3 days

(b)   10 days

(c)   16/3 days

(d)   11 days

Answer: (a)

Staff Selection Commission (SSC) Multi-Tasking Examination Held on October 29, 2017 Question Paper With Answer Key

Staff Selection Commission (SSC) Multi-Tasking Examination Held on October 29, 2017
Staff Selection Commission (SSC) Multi-Tasking Examination Held on October 29, 2017 Question Paper With Answer Key

Staff Selection Commission (SSC) Multi-Tasking Examination Held on October 29, 2017

 

1. In the following question, select the number which can be placed at the sign of question mark (?) from the given alternatives.

(a)  121

(b)  100

(c)  89

(d)  113

Answer: (a)

2. How many triangles are there in the given figure?

(a)  5

(b)  7

(c)  8

(d)  9

Answer: (b)

3. A word is represented by only one set o numbers as given in anyone of the alternatives. The sets of numbers given in the alternatives are represented by two classes of alphabets as shown in the given two matrices. The columns and rows of Matrix-I are numbered from 0 to 4 and that of Matrix-II are numbered from 5 to 9. A letter from these matrices can be represented first by its row and next by its column, for example, ‘D’ can be represented by 02, 79, etc., and ‘Z’ can be represented by 00, 77, etc. Similarly, you have to identify the set for the word MAKE.

(a)  04, 11, 23, 86

(b)  11, 40, 45, 23

(c)  43, 88, 12, 41

(d)  66, 55, 67, 34

Answer: (a)

4. From the given answer figures, select the one in which the question figure is hidden/embedded.

Answer: (a)

5. Which answer figure will complete the pattern in the question figure?

Answer: (b)

6. If a mirror is placed on the line AB, then which of the answer figures is the right image of the given figure?

Answer: (c)

7. A piece of paper is folded and punched as shown below in the question figures.

From the given answer figures, indicate how it will appear when opened?

Answer: (c)

8. By interchanging which two signs the equation will be correct?

4 + 9 ÷ 18 × 11 – 6 = 7

(a)  × and ÷

(b)  + and ×

(c)  ÷ and −

(d)  − and +

Answer: (b)

9. If 4 # 3 @ 1 = 13 and 5 # 2 @ 2 = 12, then 6 @ 1 # 4 = ?

(a)  11

(b)  10

(c)  12

(d)  8

Answer: (b)

10. Arrange the given words in the sequence in which they occur in the dictionary

(1) Modest    (2) Monkey (3) Modern (4) Modular          (5) Monger

(a)  3 4 1 2 5

(b)  3 1 4 2 5

(c)  3 4 1 5 2

(d)  3 1 4 5 2

Answer: (d)

11. In the following question, select the missing number from t he given series.

3, 3, 6, 18, 72, ?

(a)  320

(b)  360

(c)  480

(d)  540

Answer: (b)

12. A series is given with one term missing. Select the correct alternative from the given ones that will complete the series.

E, H, K, N, Q ?

(a)  V

(b)  T

(c)  U

(d)  S

Answer: (b)

13. In the following question, select the odd word from the given alternatives

(a)  Bus

(b)  Car

(c)  Truck

(d)  Aeroplane

Answer: (d)

14. In the following question, select the odd number pair from the given alternatives.

(a)  6-12

(b)  7-14

(c)  3-5

(d)  4-8

Answer: (c)

15. In the following question, select the odd letters from the given alternatives

(a)  FE

(b)  NM

(c)  DC

(d)  QR

Answer: (d)

16. Identify the diagram that best represents the relationship among the given classes.

Answer: (c)

17. In the following question, select the related word from the given alternatives.

Summer : Hot :: Winter : ?

(a)  Cold

(b)  Ice

(c)  Cotton

(d)  Jacket

Answer: (a)

18. In the following question, select the related number from the given alternatives

12 : 145 : : 16 : ?

(a)  255

(b)  257

(c)  349

(d)  423

Answer: (b)

19. In the following question, select the related letters from the given alternatives.

MARKET : TEKRAM : : SATURN : ?

(a)  NRUTAS

(b)  NRTUAS

(c)  NRUATS

(d)  NURTAS

Answer: (a)

20. In the following question, select the word which cannot be formed using the letters of the given word.

POSSIBILITIES

(a)  POSTER

(b)  BOSS

(c)  BILE

(d)  SOIL

Answer: (a)

21. From directional perspective, If North is called West, North-West is called South-West and South-West is called South-East, then what will be North-East called as?

(a)  West

(b)  South-Eat

(c)  North

(d)  North-West

Answer: (d)

22. Pointing to a lady, Vikas said “She is the daughter of the wife of my father”. How is lady related to Vikas?

(a)  Aunt

(b)  Sister

(c)  Daughter

(d)  Wife

Answer: (b)

23. In the following question below are given some statements followed by some conclusions. Taking the given statements to be true even if they seem to be at variance from commonly known facts, read all the conclusions and then decide which of the given conclusion logically follows the given statements?

Statements All short are fat.

Some short are fair.

Conclusions I. Some fair are fat.

II. Come fat are fair.

(a)  Only conclusion I follows

(b)  Only conclusion II follows

(c)  Both conclusions follow

(d)  Neither conclusion I nor conclusion II follows

Answer: (b)

24. In a certain code language, ‘TIMBER’ is written as ‘UKPFJX’. How is ‘TRAIL’ written in that code language?

(a)  VTMEF

(b)  UTDMQ

(c)  QTUMD

(d)  FMUQT

Answer: (b)

25. In a certain code language, ‘TIN’ is written as ‘47’ and ‘TAX’ is written as ‘49’. How is ‘TOPS’ written in that code language?

(a)  72

(b)  69

(c)  73

(d)  74

Answer: (d)

26. If A = 45 × 73 and B = 72 × 46, then what is the value of A × B?

(a)  411 × 75

(b)  410 × 718

(c)  430 × 76

(d)  47 × 79

Answer: (a)

27. What is the value of 21 + 24 + 27 … + 51?

(a)  324

(b)  396

(c)  416

(d)  288

Answer: (b)

28. A number M is divisible by 25. If (M + 5) (M + 1) is divided by 25, then what will be the remainder?

(a)  5

(b)  6

(c)  1

(d)  3

Answer: (a)

29. Which of the following statement(s) is/are true?

(I) (7)450 > (9)300      (ii) (2)900 > (7)450

(a)  Only I

(b)  Only II

(c)  Both I and II

(d)  Neither I nor II

Answer: (a)

30. What is the value of (4.6 + 3.1)2 – (4.6 – 3.1)2?

(a)  54.68

(b)  58.86

(c)  53.32

(d)  57.04

Answer: (d)

31. Which of the following number is an odd and prime number?

(a)  61

(b)  87

(c)  81

(d)  69

Answer: (a)

32. M alone can do a work in 50 days. M is 10% less efficient that N. In how many days can N alone do the same work?

(a)  60 days

(b)  45 days

(c)  40 days

(d)  43 days

Answer: (b)

33. Three taps R, S and T can fill a tank in 90, 100 and 180 respectively. If all the taps are opened together, then in how many minutes will the tank be filled?

(a)  120 min

(b)  37.5 min

(c)  45 min

(d)  90 min

Answer: (b)

34. What is the area (in cm2) of the rhombus having side as 20 cm and one of the diagonal as 24 cm?

(a)  768

(b)  384

(c)  480

(d)  240

Answer: (b)

35. A shopkeeper offers 35% discount on shirts which have been marked 60% above the cost price. Ajay bought the shirt for Rs 728. What was the cost price of the shirt?

(a)  Rs 520

(b)  Rs 700

(c)  Rs 480

(d)  Rs 680

Answer: (b)

36. The cost price of an article is 90% of the marked price. What is the gain percentage after allowing discount of 1% on the marked price?

(a)  12.5%

(b)  20%

(c)  10%

(d)  8.33%

Answer: (c)

37. In what ratio a man must mix pulses at Rs 11.10 per kg with Rs 15.20 per kg pulse so as to make a mixture worth Rs 13.20 per kg?

(a)  4 : 3

(b)  16 : 27

(c)  4 : 7

(d)  20 : 21

Answer: (d)

38. Rs 1050 is divided among M, N and P in the ratio 3 : 5 : 7 respectively. What is the difference between the shares of M and N?

(a)  Rs 140

(b)  Rs 210

(c)  Rs 155

(d)  Rs 315

Answer: (a)

39. Average of 12 numbers is 15. If a number 41 is also included, then what will be the average of these 13 numbers?

(a)  16

(b)  18

(c)  19

(d)  17

Answer: (d)

40. By selling 96 L of milk, a person’s gain is equal to the cost price of 24 L of milk. What is his gain percent?

(a)  30%

(b)  18.18%

(c)  25%

(d)  16.67%

Answer: (c)

41. A man sold 280 chairs and had a gain equal to selling price of 35 chairs. What is his profit percentage?

(a)  20%

(b)  14.28%

(c)  15%

(d)  16.67%

Answer: (b)

42. A man saves 64% of his income in 8 months. If he wants to save the same amount in 4 months, then by how much percentage should he increase his monthly savings?

(a)  80%

(b)  75%

(c)  100%

(d)  95%

Answer: (c)

43. A man spends 65% of his income and saves the rest. If his income increases by 25% and spending increases by 20%, then what is the percentage change in his savings?

(a)  31.67% increase

(b)  34.28% increase

(c)  41.66% decrease

(d)  29.87% decrease

Answer: (b)

44. A train running with a speed of 18 km/h, crosses an electric pole. If the length of train is 540 m, then what is the time taken by the train to cross the pole?

(a)  110 sec

(b)  100 sec

(c)  108 sec

(d)  105 sec

Answer: (c)

45. A 100 m long train crosses a 800 m long bridge. If the speed of train 30 km/h, then what is the time taken to cross the bridge?

(a)  105 sec

(b)  108 sec

(c)  118 sec

(d)  120 sec

Answer: (b)

46. What is the sum of money which will become Rs 201840 at the rate of 16% per annum at compound interest in two years?

(a)  Rs 160000

(b)  Rs 150000

(c)  Rs 180000

(d)  Rs 200000

Answer: (b)

Directions (Q. Nos. 47-50) The table given below shows the number of mobile covers sold (in thousands( by 2 companies VAPT and ABC from year 2011 to 15.

47. Companies VAPT and ABC show the same difference in the number of mobile covers sold for which of the following years?

(a)  2012 and 2013

(b)  2011 and 2012

(c)  2012 and 2014

(d)  None of these

Answer: (b)

48. What is the percentage change in the number of mobile covers sold by company VAPT from year 2011-13?

(a)  68

(b)  84

(c)  78

(d)  34

Answer: (a)

49. What is the average number (in thousands) of mobile covers sold by company VAPT over the 5 given years?

(a)  80

(b)  81.8

(c)  84

(d)  92

Answer: (b)

50. What is the percentage change in the number of mobile covers sold by company ABC from year 2012 to year 2015?

(a)  36%

(b)  66%

(c)  60%

(d)  90%

Answer: (c)

Directions (Q. Nos. 51-52) In the following questions, some part of the sentence may have errors. Find out which part of the sentence has an error and select the appropriate option. If a sentence is free from error, select ‘No Error’.

51. The traffic police department has put up huge notice boards at all the (1)/ major junctions of the city warning drivers for refrain from using (2)/ cell phones while driving or else their licences will be impounded. (3) No error (4).

(a)  1

(b)  2

(c)  3

(d)  4

Answer: (b)

52. Contrary to popular perception, the inclusion segment is not a singular impoverished (1)/ undifferentiated mass and it is important to navigate its diversity (2)/ this identify the right target customers for various programs. (3) No error (4).

(a)  1

(b)  2

(c)  3

(d)  4

Answer: (c)

Directions (Q. Nos. 53-54) In the following questions, the sentence given with blank to be filled in with an appropriate word. Select the correct alternative out of the four and Indicate it by selecting the appropriate option.

53. Have you ……….. your pen?

(a)  find

(b)  found

(c)  finds

(d)  finded

Answer: (b)

54. Yukimura should be ……… as the Chairman

(a)  appointed

(b)  smiled

(c)  coughed

(d)  smirked

Answer: (a)

Directions (Q. Nos. 55-57) In the following questions, out of the given four alternatives, select the one which best expresses the meaning of the given word.

55. Prompt

(a)  Immediate

(b)  Late

(c)  Tardy

(d)  Slow

Answer: (a)

56. Quake

(a)  Tremble

(b)  Still

(c)  Order

(d)  Tranquil

Answer: (a)

57. Rip

(a)  Tear

(b)  Closure

(c)  Sew

(d)  Value

Answer: (a)

Directions (Q. Nos. 58-60) In the following questions, out of the given four alternatives, select the one which is opposite in meaning of the given word.

58. Dwell

(a)  Abide

(b)  Bunk

(c)  Hole up

(d)  Move on

Answer: (d)

59. Encroach

(a)  Keep away

(b)  Barge in

(c)  Crash

(d)  Horn in

Answer: (a)

60. Endow

(a)  Deprive

(b)  Award

(c)  Back

(d)  Bestow

Answer: (a)

Directions (Q. Nos. 61-62) In the following questions, out of the four given alternatives, select the alternative which best expresses the meaning of the Idiom/Phrase.

61. Second to none

(a)  As good as or better than all others

(b)  Creating warm all around

(c)  To keep a secret of other

(d)  Most reliable person

Answer: (a)

62. Keep a low profile

(a)  To avoid attracting attention to yourself

(b)  Observe a threshold

(c)  Take the help of

(d)  Being very miser

Answer: (a)

Directions (Q. Nos. 63-64) Improve the bracketed part of the sentence.

63. All over the world, it is a niche programme and not a mass programme, where (each off) can get into it,

(a)  everybody

(b)  each one

(c)  all me

(d)  No improvement

Answer: (a)

64. There has been a substantial drop in number of people (opted) for new telephone connection from the public sector telephone company in the recent months.

(a)  optioning

(b)  opting

(c)  opt

(d)  No improvement

Answer: (b)

Directions (Q. Nos. 65-66) In the following questions, out of the four given alternatives, select the alternative which is the best substitute of the phrase.

65. Time when night and day are of equal length.

(a)  Equinox

(b)  Equivocal

(c)  Equate

(d)  Equidistant

Answer: (a)

66. Fertile spot in desert.

(a)  Oasis

(b)  Tributary

(c)  Well

(d)  Peninsula

Answer: (a)

Directions (Q. Nos. 67-68) In the following questions, a word has been written in four different ways out of which only one is correctly spelt. Select the correctly spelt word.

67.

(a)  Circate

(b)  Circuit

(c)  Sircuit

(d)  Circiut

Answer: (b)

68.

(a)  Concurence

(b)  Concurreeme

(c)  Concurrence

(d)  Concureence

Answer: (c)

Directions (Q. Nos. 69-70) Rearrange the parts of the sentence in correct order.

69. Cleanliness is a greatest

P : responsibility to enhance the standard of life.

Q : virtue which should be followed

R : by everyone as a great

(a)  QRP

(b)  PQR

(c)  RQR

(d)  PRQ

Answer: (a)

70. National integration is of

P : the individual level development in this

Q : country and make it a strong country.

R : great importance in India to enhance.

(a)  QRP

(b)  RPQ

(c)  PQR

(d)  PRQ

Answer: (b)

Directions (Q. Nos. 71-75) In the following passage some of the words have been left out. Read the passage carefully and select the correct answer for the given blanks out of the four alternatives.

Changes in the climatic conditions are also having a ………. impact on the water system. It has ………. in the ……….. of glaciers and erratic rainfall patterns that in …………. are leading to environmental imbalance. It is important to take the climate change issue seriously and control human activities that are contributing this change.

71. also having a ……. impact

(a)  indifferent

(b)  positive

(c)  neutral

(d)  negative

Answer: (d)

72. It has ……..

(a)  affected

(b)  resulted

(c)  incorporated

(d)  decorated

Answer: (b)

73. in the ………. of glaciers

(a)  freezing

(b)  heating

(c)  melting

(d)  creating

Answer: (c)

74. patterns that in ……. are leading

(a)  turn

(b)  advance

(c)  rational

(d)  cult

Answer: (a)

75. that are contributing ……… this change,

(a)  above

(b)  on

(c)  of

(d)  towards

Answer: (d)

76. Which one of the following is not considered as fixed capital?

(a)  Tools

(b)  Machines

(c)  Money

(d)  Building

Answer: (c)

77. ‘Disguised unemployment’ is prominently found in …….

(a)  urban areas

(b)  semi-urban areas

(c)  rural areas

(d)  No option is correct

Answer: (c)

78. Who among the following was the first Chie Electron Commissioner of India?

(a)  KVK Sundaram

(b)  Sukumar Sen

(c)  Dr. Nagendra Singh

(d)  T. Swaminathan

Answer: (b)

79. Who of the following administers oath of the office and secrecy to the President of India?

(a)  Vice-President

(b)  Prime Minister

(c)  Chief Justice of the Supreme Court

(d)  Lok Sabha Speaker

Answer: (c)

80. Which one of the following places of Indus Valley Civilization was situated on the bank of Ravi River?

(a)  Lothal

(b)  Mohenjodaro

(c)  Chanhudaro

(d)  Harappa

Answer: (d)

81. Tipu Sultan was defeated by the British forces in the ………..Anglo-Mysore war.

(a)  First

(b)  Second

(c)  Third

(d)  Fifth

Answer: (c)

82. Which one of the following landscapes is not formed by the running water or river?

(a)  V-shaped Valley

(b)  Meander

(c)  Pothole

(d)  Moraine

Answer: (d)

83. Which of these is the major source of fresh water in India?

(a)  Ground water

(b)  Ocean water

(c)  Tanks

(d)  Water falls

Answer: (a)

84. From which part of the organ does the digestion of food in the human body start?

(a)  Mouth

(b)  Stomach

(c)  Small Intestine

(d)  Large Intestine

Answer: (a)

85. Which one of the following has the capability of cell division in plants?

(a)  Parenchyma

(b)  Scelerenchyma

(c)  Xylem

(d)  Apical Meristem

Answer: (d)

86. Which one of the following is a true fish?

(a)  Jellyfish

(b)  Starfish

(c)  Dogfish

(d)  Silverfish

Answer: (c)

87. When a car starts suddenly from rest, the people sitting inside the car more backwards. This event is an example of :

(a)  Newton’s First Law of Motion

(b)  Newton’s Second Law of Motion

(c)  Newton’s Third Law of Motion

(d)  No option is correct

Answer: (a)

88. Which of the following wave types is an example of the waves produced by a moving motorboat on the surface of water?

(a)  Longitudinal only

(b)  Transverse waves only

(c)  Electromagnetic waves

(d)  Both longitudinal and transversal

Answer: (d)

89. Which of the following keys is used in computer to change the lower case letter mode to uppercase?

(a)  Alt

(b)  Ctrl

(c)  Caps Lock

(d)  Enter

Answer: (c)

90. Who discovered Proton, one of the fundamental particle of an atom?

(a)  Chadwick

(b)  JJ Thomas

(c)  Rutherford

(d)  Newton

Answer: (c)

91. During summer, water kept in an earthen pot becomes cool because of the phenomenon of

(a)  osmosis

(b)  evaporation

(c)  diffusion

(d)  transpiration

Answer: (b)

92. Which one of the following is not a human made environment?

(a)  Swimming pool

(b)  Zoo

(c)  Lake

(d)  Garden

Answer: (c)

93. In which of the following years was the Pradhan Mantri Gram Sadak Yojana (PMGSY) launched?

(a)  1988

(b)  2000

(c)  2002

(d)  2004

Answer: (b)

94. Who among the following scientist discovered the Law of Floatation?

(a)  Newton

(b)  Thomas Edison

(c)  Archimedes

(d)  John Dalton

Answer: (c)

95. Who is the first Indian woman gymnast to qualify for the Olympics?

(a)  Karnam Malleswari

(b)  PT Usha

(c)  Dipa Karmakar

(d)  Gita Fogat

Answer: (c)

96. Which of the following folk dance forms is related to Odisha?

(a)  Chhau

(b)  Tamasha

(c)  Mallakhamb

(d)  No option is correct

Answer: (a)

97. Which of the following is the second highest gallantry award in India?

(a)  Veer Chakra

(b)  Mahavir Chakra

(c)  Ashok Chakra

(d)  Shaurya Chakra

Answer: (b)

98. Who was written the book ‘Harry Potter and the Cursed Child’?

(a)  Patrick Suskind

(b)  Anne Frank

(c)  JK Rowling

(d)  Elena Ferrante

Answer: (c)

99. Which of the following countries is committed to strengthen bilateral relations with India in the field of Agriculture in January, 2017?

(a)  France

(b)  Russia

(c)  Israel

(d)  The UK

Answer: (c)

100. Which of the following countries does not share the land boundary with Myanmar?

(a)  Bangladesh

(b)  India

(c)  Cambodia

(d)  Laos

Answer: (c)

Staff Selection Commission (SSC) Combined Higher Secondary (10+2) Recruitment Examination-2015 Held on November 1, 2015 Question Paper With Answer Key

Staff Selection Commission (SSC) Combined Higher Secondary (10+2) Recruitment Examination-2015 Held on November 1, 2015
Staff Selection Commission (SSC) Combined Higher Secondary (10+2) Recruitment Examination-2015 Held on November 1, 2015 Question Paper With Answer Key

Staff Selection Commission (SSC) Combined Higher Secondary (10+2) Recruitment Examination-2015 Held on November 1, 2015

Part I General Intelligence

1. If in a certain code ONE is coded as 231, FIVE is coded as 9641, then how will be NINE coded?

(a)   3316

(b)   3361

(c)   3613

(d)   3631

Answer: (d)

Directions (Q. Nos. 2-10) In the following questions, find out the odd word/letters/number/number pair from the given alternatives

2. Find out the odd pair.

(a)   Crayon-Chart

(b)   Pencil-Lead

(c)   Pen-Ink

(d)   Brush-Paint

Answer: (a)

3. Find out the odd number pair.

(a)   (54, 216)

(b)   (61, 244)

(c)   (24, 96)

(d)   (27, 135)

Answer: (d)

4. Find out the odd word.

(a)   Rin

(b)   Hamam.

(c)   Pears

(d)   Cinthol

Answer: (a)

5. Find out the odd letters

(a)   PET

(b)   CAP

(c)   RFD

(d)   TIW

Answer: (c)

6. Find out the odd letters.

(a)   RQPA

(b)   STUA

(c)   MLKA

(d)   HGFA

Answer: (b)

7. Find out the odd number.

(a)   368127

(b)   876321

(c)   742956

(d)   564327

Answer: (c)

8. Find out the odd word.

(a)   Humming bird

(b)   Kingfisher

(c)   Crane

(d)   Sparrow

Answer: (c)

9. Find out the odd number pair.

(a)   57-75

(b)   34-43

(c)   15-41

(d)   12-21

Answer: (c)

10. Find out the odd letters.

(a)   LORU

(b)   IMQU

(c)   WZCF

(d)   ADGJ

Answer: (b)

Directions (Q. Nos. 11-12) In the following questions, which one set of letters when sequentially placed at the gaps in the given letter series shall complete it?

11. b_ac_cc_cb_ab_ac

(a)   cbaba

(b)   abbbc

(c)   bbaac

(d)   aabba

Answer: (d)

12. _zy_zxy_yxzx_zyx_xy

(a)   yxzyz

(b)   zxyzy

(c)   xyzzy

(d)   yzxyx

Answer: (a)

Directions (Q. Nos. 13-21) In the following questions, select the related word/letters/number from the given alternatives.

13. Governor : State :: President : ?

(a)   Country

(b)   Organization

(c)   Constituency

(d)   District

Answer: (a)

14. Car :Garrage :: ? : Hangar

(a)   Truck

(b)   Bus

(c)   Train

(d)   Aeroplane

Answer: (d)

15. HIJK : GFED :: NOPQ : ?

(a)   FDEC

(b)   MLKJ

(c)   EFGH

(d)   BCDE

Answer: (b)

16. Heart : Cardiologist : : Brain : ?

(a)   Ophthalmologist

(b)   Nephrologist

(c)   Neurologist

(d)   Gyanecologist

Answer: (c)

17. 235 : 587 : : 435 : ?

(a)   789

(b)   788

(c)   989

(d)   988

Answer: (a)

18. 17 : 102: : 23 : ?

(a)   216

(b)   413

(c)   112

(d)   138

Answer: (d)

19.  GFEH : RQPS : : LKJM : ?

(a)   ONMP

(b)   VUWX

(c)   LKNM

(d)   IHJG

Answer: (a)

20. CEIM : DGLQ : : FGIO : ?

(a)   GLIS

(b)   GILS

(c)   GMIS

(d)   GMSI

Answer: (b)

21. 4 : 36 :: ? : 49

(a)   6

(b)   5

(c)   7

(d)   8

Answer: (b)

Directions (Q. Nos. 22-24) In the following questions, arrange the following words as per order in the dictionary.

22. (1) P late (2) Plane

(3) Plaster     (4) Plasma

(5) Plastic

(a)   2, 5, 3, 4, 1

(b)   2, 1, 5, 3, 4

(c)   1, 2, 3, 4, 5

(d)   2, 4, 3, 5, 1

Answer: (d)

23. (1) Intervention (2) Intertwine

(3) Interview     (4) Intervene

(a)   3, 4, 1, 2

(b)   2, 3, 4, 1

(c)   4, 3, 2, 1

(d)   2, 4, 1, 3

Answer: (d)

24. (1) Smoulder (2) Smother

(3) Smuggle   (4) Smudge

(a)   1, 4, 3, 2

(b)   1, 2, 3, 4

(c)   2, 1, 4, 3

(d)   4, 3, 2, 1

Answer: (c)

25. Raghu jogged 2 km southwards, then the turned right and walked 5 km. He again turned right and walked and jogged 8 km. In which direction was he seen moving last?

(a)   West

(b)   South

(c)   East

(d)   North

Answer: (d)

26. Identify the diagram that best represents the relationship among classes given below :

Answer: (d)

Directions (Q. Nos. 27-29) In the following questions, a series is given, with one term missing. Choose the correct alternative from the given ones that will complete the series.

27. 1, 16, 81, 256, 625, ?

(a)   1225

(b)   4163

(c)   2225

(d)   1296

Answer: (d)

28. 1, 5, 13, 25, 41, 61, ?

(a)   85

(b)   81

(c)   77

(d)   91

Answer: (a)

29. EZ FY XG ?

(a)   WH

(b)   GW

(c)   WG

(d)   GW

Answer: (a)

30. If a means +, b means ×, c means ÷ d means −, then 20a10b45c5d12 = ?

(a)   68

(b)   88

(c)   74

(d)   98

Answer: (d)

31. If a mirror is placed on the line MN, then which of the answer figures is the right image of the given figure?

Answer: (d)

Directions (Q. Nos. 32-35) In the following questions, select the missing number from the given responses.

32. 

(a)   50

(b)   25

(c)   75

(d)   100

Answer: (a)

33. 

(a)   6

(b)   8

(c)   1

(d)   12

Answer: (d)

34. 

(a)   8

(b)   19

(c)   12

(d)   5

Answer: (c)

35. 

(a)   5

(b)   1

(c)   10

(d)   12

Answer: (d)

36. Select the set of symbols which can be fitted correctly in the equation.

5_3_8_4_2 = 21

(a)   −, ×, + ÷

(b)   +, ×, −, ÷

(c)   ×, +, −, ÷

(d)   +, ×, +, ÷

Answer: (c)

37. Identify the answer figure from which pieces given in the question figure have been cut.

Answer: (b)

38. A watch reads 4 : 30. If the minute hand points East in which direction will the hour hand point?

(a)   South

(b)   South-West

(c)   North-East

(d)   North

Answer: (c)

39. If PNLJ : 2468, then QOKL : ?

(a)   1276

(b)   1367

(c)   1376

(d)   3591

Answer: (c)

40. In five years, Rs 5000 amounts to Rs 9000. In what time at the same rate will Rs 600 amounts to Rs 900?

(a)   2 yr

(b)   3yr

(c)   6yr

(d)   5yr

Answer: (b)

41. In the question, two statements are given. You have to answer considering the statements to be true, even if they seem to be at variance from commonly known facts.

Statements

(I) Plants grow from seeds.

(II) Seeds are  produced by flowers.

Conclusions

(a)   Flowers and seeds are produced by plants.

(b)   Seeds are produced by plants.

(c)   Plants grow from flowers.

(d)   Flowers are produced by plants.

Answer: (c)

42. A word is represented by only one set of numbers as given in any one of the alternatives. The sets of numbers given in the alternatives are represented by two classes of alphabets as in two matrices given below. The columns and rows of Matrix I are numbered from 0 to 4 and that of Matrix II are numbered from 5 to 9. A letter from these matrices can be represented first by its row and next by its column, e.g., M can be represented by 01, 10 etc., and A can be represented by 56, 65 etc. Similarly, you have to identify the set for the word ‘ROD’.

(a)   58, 66, 78

(b)   56, 66, 86

(c)   67, 96, 57

(d)   58, 69, 65

Answer: (c)

43. Five students are standing one behind the other in the playground facing the instructor, Malini is behind Anjana, but in front of Gayathri. Meena is in front of Shena, but behind Gayathri. What is the position of Meena?

(a)   Extreme last

(b)   Extreme first

(c)   Second from last

(d)   Second from first

Answer: (c)

44. Find out the incorrect term in the series.

2, 5, 10, 3, 6, 18, 4, 7, 30

(a)   10

(b)   30

(c)   6

(d)   7

Answer: (b)

45. A piece of paper is folded and punched as shown below in the question figures. From the given answer figures, indicate how it will appear when opened.

Answer: (b)

46. Which answer figure will complete the pattern in the question figure?

Answer: (a)

Directions (Q. Nos. 47-48) In the following questions, unscramble the letters to form a meaningful word. Then find out the correct numerical position of the letters.

47. N I  T  I  F  E  I  N

    2  4  5  4  3  6  4  2

(a)   3 2 4 3 5 2 4 6

(b)   4 2 3 4 2 4 5 6

(c)   2 4 3 2 4 2 5 6

(d)   3 2 5 3 5 2 4 6

Answer: (b)

48. H N  R  C  A  B

     1   2   3  4   5  6

(a)   6 5 3 4 1 2

(b)   4 1 5 6 2 3

(c)   6 3 5 2 4 1

(d)   3 5 6 4 1 2

Answer: (c)

49. In the question, two statements are given followed by two conclusions I and II. You have to consider the two statements to be true even if they seem to be at variance from commonly known facts. You have to decide which of the given conclusions, if any, follow from the given statements.

Statements

Creative  persons are learned people.

Some doctors are creative persons.

Conclusions

(I) Some creative persons are learned people.

(II) Some learned people are doctors.

(a)   Only II

(b)   Neither I nor II

(c)   Only I

(d)   I and II

Answer: (c)

50. Four positions of a dice are given below :

Find the number on the face opposite to the face showing 4.

(a)   5

(b)   1

(c)   6

(d)   3

Answer: (a)

Part II English Language

Directions (Q. Nos. 51-54) In the following questions, choose the word opposite in meaning to the given words.

51. Mammoth

(a)   Stupendous

(b)   Prodigious

(c)   Minute

(d)   Monumental

Answer: (c)

52. Abhor

(a)   Miss

(b)   Hate

(c)   Mad

(d)   Love

Answer: (d)

53. Cautious

(a)   Thoughtful

(b)   Watchful

(c)   Careless

(d)   Alert

Answer: (c)

54. Dubious

(a)   Assured

(b)   Double

(c)   Easy

(d)   Demise

Answer: (a)

Directions (Q. Nos. 55-58) In the following questions, four alternatives are given for the idiom/phrase underlined. Choose the alternative which best expresses the meaning of the idiom/ phrase.

55. A wild goose chase

(a)   A desperate effort

(b)   A useless effort

(c)   A baseless effort

(d)   A good effort

Answer: (b)

56. Men like Vidyasagar and Bhagat Singh are the salt of the Earth.

(a)   famous men

(b)   makers of the Earth

(c)   hardworking men

(d)   ideal men

Answer: (d)

57. There was a definite element of fabrication in the story, but he believed the account hook, line and sinker.

(a)   not wholly

(b)   a little bit

(c)   completely

(d)   hardly

Answer: (c)

58. To eat humble pie.

(a)   To pretend to be

(b)   To starve humble

(c)   To act apologetically

(d)   To have a low estimate of oneself

Answer: (c)

Directions (Q. Nos. 59-60) In the following questions, a sentence has been given in Direct/Indirect speech. Out of the four alternatives suggested, select the one which best expresses the same sentence in Indirect/Direct Speech.

59. The maid confessed to the police that it was she who had taken the diamond ring.

(a)   The police were told, “I have taken the diamond ring.”

(b)   The maid said, “It is I who took the diamond ring.”

(c)   The maid said, “I have stolen the diamond ring.”

(d)   The maid told the police. “It is I who have taken the diamond ring.”

Answer: (d)

60. My father said to me, “Go now”.

(a)   My father ordered me to go later.

(b)   My father ordered me to go then.

(c)   My father ordered me to go now.

(d)   My father ordered to go now.

Answer: (b)

Directions (Q. Nos. 61-70) In the questions given below the in the following passage, some of the words have been left out. Read the passage carefully and choose the correct answer to each question out of the four alternatives.

Did the Earth always (i) as it does today? No, (ii) its origin about 4.8 billion years ago, our planet has (iii) a lot of changes. The initial ball of fire (iv) down and the surface slowly (v) solid. Today we (vi) that the Earth’s crust is divided (vii) several plates, (viii) move continuously. Entire continents (ix) been displaced and mountains, islands (x) new seas have been formed or have disappeared.

61. (v)

(a)   turned

(b)   turning

(c)   has turned

(d)   turn

Answer: (c)

62. (ix)

(a)   are

(b)   had

(c)   has

(d)   have

Answer: (b)

63. (ii)

(a)   from

(b)   since

(c)   before

(d)   after

Answer: (b)

64. (x)

(a)   but

(b)   so

(c)   or

(d)   and

Answer: (c)

65. (i)

(a)   looking

(b)   look

(c)   looked

(d)   looks

Answer: (a)

66. (viii)

(a)   whose

(b)   who

(c)   whom

(d)   which

Answer: (c)

67. (vii)

(a)   on

(b)   to

(c)   in

(d)   into

Answer: (d)

68. (iii)

(a)   undergo

(b)   undergone

(c)   undergoes

(d)   underwent

Answer: (d)

69. (vi)

(a)   had known

(b)   have known

(c)   know

(d)   knew

Answer: (d)

70. (iv)

(a)   cools

(b)   cool

(c)   cooled

(d)   cooling

Answer: (d)

Directions (Q. Nos. 71-74) In the following questions, sentences are given with blanks to be filled in with on appropriate word(s). Four alternatives are suggested for each question. Choose the correct alternative out of the four and indicate the appropriate answer.

71. The doctor ……………. The woman that her son would recover.

(a)   assumed

(b)   insured

(c)   assured

(d)   ensured

Answer: (c)

72. I have no ………… riches.

(a)   desirous of

(b)   desire of

(c)   desire to

(d)   desire for

Answer: (d)

73. A warm reception was …………. Him.

(a)   allied to

(b)   accorded to

(c)   given for

(d)   allotted to

Answer: (b)

74. He is only an ……… director in the organization but he does hid word sincerely.

(a)   honorific

(b)   honour

(c)   honourable

(d)   honorary

Answer: (d)

Directions (Q. Nos. 75-78) In the following questions, out of the four alternatives, choose the one which best expresses the meaning of the given words.

75. Luxuriant

(a)   Lush

(b)   Lovely

(c)   Luxurious

(d)   Class

Answer: (a)

76. Malady

(a)   Lady

(b)   Disease

(c)   Parody

(d)   Melody

Answer: (b)

77. Despair

(a)   Discourage

(b)   Distress

(c)   Disrupt

(d)   Hopeless

Answer: (d)

78. Languid

(a)   Spirited

(b)   Lethargic

(c)   Lively

(d)   Luxurious

Answer: (b)

Directions (Q. Nos. 79-80) In the following questions, a sentence has been given in Active/Passive voice. Out of the four alternatives suggested, select the one which best expresses the same sentence in Passive/Active voice.

79. Was the question answered?

(a)   Did the question get answered?

(b)   Did they answer the question?

(c)   Was the answer to the question given?

(d)   Were they answering the question?

Answer: (b)

80. He may have left a message for me.

(a)   A message may have been left for me.

(b)   He might have left a message for me.

(c)   A message may have been left for me by him.

(d)   A message might be left for me by him.

Answer: (c)

Directions (Q. Nos. 81-84) In the following questions, out of the four alternatives, choose the one which can be substituted for the given words/sentences.

81. A place where arms and weapons are stored.

(a)   Tannery

(b)   Arsenal

(c)   Kiln

(d)   Granary

Answer: (b)

82. A partner in crime

(a)   Assistant

(b)   Helper

(c)   Accomplice

(d)   Alibi

Answer: (c)

83. A general act of forgiveness or pardon.

(a)   Benediction

(b)   Emancipation

(c)   Amnesty

(d)   Gratification

Answer: (c)

84. One who sells articles at public sales.

(a)   Auctioneer

(b)   Juggler

(c)   Teller

(d)   Milliner

Answer: (a)

Directions (Q. Nos. 85-88) In the following questions, a sentence/a part of the sentence is underlined. Below are given alternatives to the underlined part which may improve the sentence. Choose the correct alternative. In case no improvement is needed, choose the ‘No improvement’ option.

85. Let you and I go together.

(a)   me

(b)   my

(c)   No improvement

(d)   mine

Answer: (a)

86. He summarily dismissed what his son told him in a fit of rage.

(a)   He summarily dismissed in a fit of rage what his son told him.

(b)   No improvement

(c)   What his son told in a fit of rage he summarily dismissed.

(d)   In a fit of rage he summarily dismissed what his son told him.

Answer: (d)

87. Each and everyone of you are guilty of some crime or the other.

(a)   No improvement

(b)   Each and each of you are guilty

(c)   Everyone are guilty

(d)   Each and everyone of you is guilty

Answer: (d)

88. There were times in his life when he struggled for one square meal a day.

(a)   he struggled with one square meal a day.

(b)   he keeps struggling for meals.

(c)   he struggled one day for food.

(d)   No improvement

Answer: (d)

Directions (Q. Nos. 89-92) In the following four questions, some parts of the sentences have errors and some are correct. Find out which part of a sentence has an error. If a sentence is free from error, choose the ‘No error’ option.

89. The flower and the vine are growing well.

(a)   are growing well

(b)   The flower

(c)   and the vine

(d)   No error

Answer: (d)

90. Forty minutes later James return to his family bearing an important piece of news.

(a)   Forty minutes later

(b)   James return to his family

(c)   bearing an important piece of news

(d)   No error

Answer: (b)

91. Tutankhamun’s death mask is still untarnished after thirty four centuries.

(a)   Tutankhamun’s death mask

(b)   after thirty four centuries

(c)   No error

(d)   is still untarnished

Answer: (c)

92. I jumped with the opportunity of going to Kashmir.

(a)   No error

(b)   with the opportunity

(c)   I jumped

(d)   of going to Kashmir

Answer: (b)

Directions (Q. Nos. 93-96) In the following questions, the first and the last part of a sentence are numbered 1 and 6. The rest of the sentence is split into four parts and named P, Q, R and S. These four parts are not given in their proper order. Read the sentence and find out which of the four combinations is correct.

93. (1) A social network is

(P) provides a clear way of analyzing

(Q) a set of actors and the

(R) a social structure made up of

(S) dyadic ties between the se actors

(6) the structure of whole social entities.

(a)   PQRS

(b)   QSPR

(c)   RQSP

(d)   SPRQ

Answer: (c)

94. (1) The pilgrimage to Mecca

(P) giving alms

(Q) recognizing Allah as the only God

(R) is one of the five religious duties of each Muslim

(S) along with praying five times a day

(6) and fasting four weeks once a year.

(a)   RQPS

(b)   RSQP

(c)   SQRP

(d)   SPQR

Answer: (b)

95. (1) Some people seem to imagine

(P) and that when you have filled it with anything

(Q) and it becomes as empty

(R) that the human mind is like a bottle\

(S) you pour it out again

(6) as it was before

(a)   PQRS

(b)   RPSQ

(c)   RSPQ

(d)   PRSQ

Answer: (b)

96. (1) England’s Cath Webb

(P) with her resolution\

(Q) to make sponge cake very day for a year

(R) hit the headlines in January

(S) and give it to a friend, family member or stranger

(6) just to make then smile.

(a)   RQSP

(b)   RPQS

(c)   SPRQ

(d)   SRPQ

Answer: (a)

Directions (Q. Nos. 97-100) In the following questions, four words are given in each question, out of which only one word is correctly spelt. Find the correctly spelt word.

97.

(a)   Occurrence

(b)   Occurance

(c)   Ocurrance

(d)   Occurrance

Answer: (a)

98.

(a)   Bouganvella

(b)   Bougainvillea

(c)   Bougenvillea

(d)   Bougainvilla

Answer: (b)

99.

(a)   Courteus

(b)   Courtoeus

(c)   Corutious

(d)   Courteous

Answer: (d)

100.

(a)   Dimenutive

(b)   Diminutiv

(c)   Diminutive

(d)   Diminuitive

Answer: (d)

Part III Quantitative Aptitude

101. A man spends his three months income in four months time. If his monthly income is Rs 1000, then his annual saving is

(a)   Rs 3000

(b)   Rs 9000

(c)   Rs 6000

(d)   Rs 4000

Answer: (a)

102. The length of two parallel chords of a circle of radius 5 cm are 6 cm and 8 cm in the same side of the centre.

(a)   1.5 cm

(b)   1 cm

(c)   3 cm

(d)   2 cm

Answer: (b)

103. After deducting a commission of 5%, a TV set costs Rs 9595. Its marked price is

(a)   Rs 10000

(b)   Rs 10074.75

(c)   Rs 12000

(d)   Rs 10100

Answer: (d)

104. One-fifth of half of a number is 20. Then 20% of that n umber is

(a)   60

(b)   20

(c)   40

(d)   80

Answer: (c)

105. The sum of  is

(a) 

(b) 

(c) 

(d) 

Answer: (d)

Directions (Q. Nos. 106-109) Study the following histogram and answer the questions.

106. The total number of students on whom this survey was made is

(a)   220

(b)   190

(c)   200

(d)   210

Answer: (d)

107. The number of students securing marks in the range 90-100, is

(a)   40

(b)   30

(c)   10

(d)   20

Answer: (c)

108. The percentage of students securing marks less than 50, is

(a) 

(b) 

(c) 

(d) 

Answer: (b)

109. The range of marks obtained by maximum number of students, is

(a)   60-70

(b)   80-90

(c)   50-60

(d)   30-40

Answer: (c)

110. The weight of a container completely filled with water is 2.25 kg. The container weights 0.77 kg when its 0.2 part is filled with water. The weight of the container when 0.4 part of it is filled with water, is

(a)   0.74 kg

(b)   1.14 kg

(c)   1.88 kg

(d)   0.40 kg

Answer: (b)

111. BE, CF are two medians of ∆ABC and G is their point of intersection. EF cuts AG at O. Ratio of AO : OG is equal to

(a)   1 : 3

(b)   2 : 3

(c)   3 : 1

(d)   1 : 2

Answer: (c)

Directions (Q. Nos. 112-116) Read it carefully and answer the questions.

Total cost of production of a firm is Rs 250 lakh. Following pie-charts shows the percentage of costs of production in different purposes.

Letters A, B, C, D represent the following :

A = Cost of raw materials

B = Cost of packing materials

C = Cost of labour

D = Maintenance cost

112. If the production increases to five times of the present cost, then the percentage increase of the cost is

(a)   50%

(b)   500%

(c)   300%

(d)   400%

Answer: (d)

113. If the total maintenance cost increases from Rs 12.5 lakh to Rs 50 lakh, then the percentage increase of the maintenance cost is

(a)   400%

(b)   25%

(c)   75%

(d)   300%

Answer: (d)

114. Cost of packing materials and raw materials together amounts to

(a)   Rs 137.5 lakh

(b)   Rs 162.5 lakh

(c)   Rs 87.5 lakh

(d)   Rs 175 lakh

Answer: (b)

115. If the cost of production doubles in a period of three years, then the corresponding maintenance cost will be

(a)   Rs 25 lakh

(b)   Rs 125 lakh

(c)   Rs 12.5 lakh

(d)   Rs 75 lakh

Answer: (a)

116. If the packing cost increases by 2%, then the new packing cost will be

(a)   Rs 5 lakh

(b)   Rs 62.5 lakh

(c)   Rs 1.25 lakh

(d)   Rs 63.75 lakh

Answer: (d)

117. A cand do half of a piece of work in 1 day, whereas B can do full. B can do half the work as C in 1 day. The ratio of their efficiencies of work, is

(a)   2 : 1 : 4

(b)   4 : 2 : 1

(c)   2 : 4 : 1

(d)   1 : 2 : 4

Answer: (d)

118. A bag contains coins of Rs 1.50 paise and 25 paise in the ratio 2 : 3 : 5. If the total value of these coins is Rs 228, then the number of 50 paise coins in that bag was

(a)   96

(b)   112

(c)   124

(d)   144

Answer: (d)

119. The value of (cos 0° + sin 45° + sin 30°) (sin 90° − cos 45° + cos 60°) is

(a)   3/2

(b)   7/4

(c)   5/4

(d)   3/4

Answer: (b)

120. If x = a sec θ + b tan θ and y = a tan θ + b sec θ (a ≠ b), then the value of  is

(a)   1

(b)   0

(c)   2

(d)   1/2

Answer: (a)

121. The value of sec4A(1− sin4A) – 2tan2 A is

(a)   1/2

(b)   1

(c)   0

(d)   2

Answer: (b)

122. AB is a diameter of a circle having centre at O. P is a point on the circumference of the circle. If ∠POA = 120°, then measure of ∠PBO is

(a)   68°

(b)   60°

(c)   75°

(d)   70°

Answer: (b)

123. If 5 tan A = 4, then the value of  is

(a)   9/14

(b)   5/6

(c)   7/9

(d)   5/14

Answer: (d)

124. A low land, 48 m long and 31.5 m broad is raised to 6.5 dm. For this, Earth is remove from a cuboidal hole, 27 m long and 18.2 m broad, dug by the side of the land. The depth of the hole will be

(a)   3 m

(b)   2.2 m

(c)   2.5 m

(d)   2 m

Answer: (d)

125. Base of a prism of height 10 cm is square. Total surface area of the prism is 192 sq cm. The volume of the prism is

(a)   640 cm3

(b)   160 cm3

(c)   90 cm3

(d)   120 cm3

Answer: (b)

126. Arvind deposited a sum of money with a bank on January 1, 2012 at 8% simple interest per annum. He received an amount Rs 3144 on August 7, 2012. The money he deposited with the bank was

(a)   Rs 3100

(b)   Rs 3000

(c)   Rs 3080

(d)   Rs 2500

Answer: (b)

127. 15 men can finish a work in 20 days, however it takes 24 women to finish it in 20 days. If 10 men and 8 women undertake to complete the work, then they will take

(a)   15 days

(b)   30 days

(c)   10 days

(d)   20 days

Answer: (d)

128. Simplify 

(a)   1.25

(b)   1/5

(c)   0

(d)   2.4

Answer: (c)

129. Divide 27 into two parts so that 5 times the first and 11 times the second together equal to 195. Then ratio of the first and second parts is

(a)   17 : 10

(b)   3 : 2

(c)   2 : 7

(d)   5 : 4

Answer: (a)

130. The time required for a sum of money of amount of four times itself at 15% simple interest per annum will be

(a)   30 yr

(b)   20 yr

(c)   10 yr

(d)   15 yr

Answer: (b)

131. In 2 kg mixture of copper and aluminium, 30% is copper. How much aluminium powder should be added to the mixture so that the proportion of copper becomes 20%?

(a)   800 gm

(b)   1200 gm

(c)   1000 gm

(d)   900 gm

Answer: (c)

132. A vendor purchased 40 dozen bananas for Rs 250. Out of these 30 bananas were rotten and could not be sold. At what rate per dozen should he sell the remaining bananas to make a profit of 20%?

(a)   Rs 10

(b)   Rs 6

(c)   Rs 12

(d)   Rs 8

Answer: (d)

133. A discount series of 10%, 20% and 40% is equal to single discount of

(a)   60.28%

(b)   56.8%

(c)   60%

(d)   50%

Answer: (b)

134. If  then the value of (1 – a +a2) (1 + a – a2)

(a)   4

(b)   0

(c)   1

(d)   −4

Answer: (b)

135. Which one of the following is true?

(a) 

(b) 

(c) 

(d) 

Answer: (a)

136. The average marks of 14 students was 71. It was later found that the marks of one of the students has been wrongly entered as 42 instead of 56 and another as 74 instead of 32. What is the correct average?

(a)   67

(b)   68

(c)   69

(d)   71

Answer: (c)

137. The radii of a sphere and a right circular cylinder are 3 cm each. If their volumes are equal, then curved surface area of the cylinder is (Assume π = 22/7)

(a) 

(b) 

(c) 

(d) 

Answer: (d)

138. A man rows 12 km in 5 hours against the stream, the speed of current being 4 km/h. What time will be taken by him to row 15 km with the stream?

(a) 

(b) 

(c) 

(d) 

Answer: (a)

  1. If x = 5, then the value of the expression x2 – 2 + 1/x2 is

(a)   24/5

(b)   625/24

(c)   24/25

(d)   576/25

Answer: (d)

140. A right prism has triangular base. If v be the number of vertices, e be the number of edges and f be the number of a faces of the prism. The value of  is

(a)   10

(b)   4

(c)   5

(d)   2

Answer: (c)

141. A man rows to a place 60 km distant and back in 13 hours 30 minutes. He finds that he can row 5 km with the stream in the same time as he can row 4 km against the stream. Find the rate of the stream.

(a)   8 km/h

(b)   1/2 km/h

(c)   1 km/h

(d)   10 km/h

Answer: (c)

142. If x = 2, y = 1 and z = −3, then x3 + y3 + z3 – 3xyz is equal to

(a)   6

(b)   8

(c)   2

(d)   0

Answer: (d)

143. The LCM of two numbers is 12 times their HCF. The sum of the HCF and LCM is 403. If one of the number is 93, then the other is

(a)   120

(b)   124

(c)   116

(d)   112

Answer: (b)

144. If 2 sin2θ – 3 sin θ + 1 = 0, θ being positive acute angle, then the values of θ are

(a)   45°, 50°

(b)   60°, 55°

(c)   60°, 45°

(d)   30°, 90°

Answer: (d)

145. Ram, Rahim and Robert started a partnership business investing Rs 30000, Rs 50000 and Rs 40000 respectively. If they made an annual profit of Rs 18504, then the share of Rahim is

(a)   Rs 7510

(b)   Rs 7710

(c)   Rs 7170

(d)   Rs 7010

Answer: (b)

146. ABC is a triangle in which ∠A = 90°. Let P be any point on side AC. If BC = 10 cm, AC = 8 cm and BP = 9 c, then AP =

(a)   2√3 cm

(b)   3√5 cm

(c)   3√3 cm

(d)   2√5 cm

Answer: (b)

147. Average age of seven persons in a group is 30 years, the average age of five persons of this group is 31 years. What is the average age of the other two persons in the group?

(a)   55 yr

(b)   15 yr

(c)   26 yr

(d)   None of these

Answer: (d)

148. ABCD is a cyclic quadrilateral, AB is the diameter of the circle. If ∠ACD = 50°, the measure of ∠BAD is

(a)   40°

(b)   140°

(c)   130°

(d)   50°

Answer: (a)

149. (x3 + y6) (x3 – y6) is equal to

(a)   x6 – y12

(b)   x9 + y36

(c)   x9 – y36

(d)   x6 + y12

Answer: (a)

150. If the angles of a triangle are in the ratio 2 : 3 : 5, then the measure of the least angle of the triangle is

(a)   18°

(b)   20°

(c)   90°

(d)   36°

Answer: (d)

Part IV General Awareness

151. According to which Article, State Legislative Council can be created or abolished?

(a)   Article-168

(b)   Article-170

(c)   Article-167

(d)   Article-169

Answer: (d)

152. Television was invented by

(a)   JL Baird

(b)   Louis Braille

(c)   Lawrence

(d)   RA Millikan

Answer: (a)

153. Breath analyzers used by police to test drunken drivers works on the chemical basis of

(a)   complexation reactions

(b)   acid-base reactions

(c)   precipitation reactions

(d)   redox reactions

Answer: (b)

154. Amjad Ali Khan is associated with which of the following musical instruments?

(a)   Sarod

(b)   Veena

(c)   Sitar

(d)   Violin

Answer: (a)

155. Which scientist wrote a book called “A brief History of Time”?

(a)   Stephen Hawking

(b)   Edward Jenner

(c)   JL Baird

(d)   Pasteur

Answer: (a)

156. Which country earned the title “The Queen of Seas”?

(a)   Italy

(b)   Britain

(c)   Germany

(d)   France

Answer: (d)

157. Mercury thermometer was invented by

(a)   Fahrenheit

(b)   Priestley

(c)   Galileo

(d)   Newton

Answer: (a)

158. Which gas is responsible for depletion of ozone layer around Earth which protect us from harmful ultra-violet rays?

(a)   Nitrogen

(b)   Nitrogen oxide

(c)   Chlorofluro carbons

(d)   Oxygen

Answer: (c)

159. Who is the founder of Homeopathy?

(a)   Domagk

(b)   Hahnemann

(c)   Wakesman

(d)   Lainnec

Answer: (b)

160. The Earth is nearest to the Sun on

(a)   September 23

(b)   March 21

(c)   January 3

(d)   July 4

Answer: (c)

161. Which of the following is the best indicator of SO2 pollution?

(a)   Lichen

(b)   Pteridophyte

(c)   Bryophyte

(d)   Algae

Answer: (a)

162. The first national Park of India is

(a)   Simlipal National Park

(b)   Corbett National Park

(c)   Dachigam Wildlife Sanctuary

(d)   Hazaribagh National Park

Answer: (b)

163. The hard enamel layer of teeth is

(a)   calcium phosphate

(b)   calcium oxide

(c)   calcium hydroxy apatite

(d)   calcium hydroxide

Answer: (a)

164. Centre for Ecological Sciences is situated at

(a)   New Delhi

(b)   Bengaluru

(c)   Allahabad

(d)   Karnal

Answer: (b)

165. ‘ICMP’ is used for

(a)   forwarding

(b)   error reporting

(c)   addressing

(d)   multicasting

Answer: (b)

166. The two components of an ecosystem are

(a)   biotic and abiotic

(b)   plants and animals

(c)   weeds and microorganisms

(d)   plants and light

Answer: (a)

167. The last dynasty of the Delhi Sultanate was the

(a)   Khilji dynasty

(b)   Slave dynasty

(c)   Lodi dynasty

(d)   Syed dynasty

Answer: (c)

168. There are only two metals that are non-silver in colour they are

(a)   copper and gold

(b)   nickel and zinc

(c)   sodium and magnesium

(d)   palladium and platinum

Answer: (a)

169. The lowest temperature is recorded by

(a)   alcohol thermometer

(b)   maximum reading thermometer

(c)   mercurial thermometer

(d)   minimum reading thermometer

Answer: (a)

170. To whom did Allauddin Khilji entrust the mission to conquer South?

(a)   Malik Kafur

(b)   Shaji Malik

(c)   Khizra Khan

(d)   Ulugha Khan

Answer: (a)

171. A plane glass slab is kept over coloured letters, the letter which appears least raised is

(a)   green

(b)   violet

(c)   red

(d)   blue

Answer: (b)

172. Who was the first Delhi Sultan to break the power of the Turkish nobles known as the Chahalgani or the Forty?

(a)   Raziya

(b)   Iltutmish

(c)   Qutb-ud-din Aibak

(d)   Balban

Answer: (d)

173. Which is the final Appellate Court of Justice?

(a)   Civil Court

(b)   High Court

(c)   Supreme Court

(d)   District Court

Answer: (c)

174. Where is Brabourne stadium located?

(a)   Cuttack

(b)   Kolkata

(c)   Jamshedpur

(d)   Mumbai

Answer: (d)

175. When did India become a fully Sovereign Democratic Republic?

(a)   January 26, 1949

(b)   November 26, 1951

(c)   November 26, 1930

(d)   November 26, 1949

Answer: (*)

176. Of the following rapeseed belongs to

(a)   pepper

(b)   coffee

(c)   linseed

(d)   mustard

Answer: (c)

177. When is Human Rights Day celebrated?

(a)   October 10

(b)   December 9

(c)   December 10

(d)   December 12

Answer: (c)

178. Which of the following is observed as Sports Day every year?

(a)   October 2

(b)   July 26

(c)   August 29

(d)   April 22

Answer: (c)

179. A defect of vision in which the points in one plane of an object appear in focus while those in another plane are out of focus is called

(a)   Myopia

(b)   Astigmatism

(c)   Distortion

(d)   Hypermetropia

Answer: (b)

180. Who was the first Deputy Prime Minister of India?

(a)   Sardar Vallabhbhai Patel

(b)   Gulzarilal Nanda

(c)   Maulana Abul Kalam Azad

(d)   Jawaharlal Nehru

Answer: (a)

181. An electronic path, that sends signals from one part of computer to another, is

(a)   Serial Port

(b)   Bus

(c)   Logic Gate

(d)   Modem

Answer: (b)

182. What is Hawala?

(a)   Full details of a subject

(b)   Tax evasion

(c)   Illegal transactions of foreign exchange

(d)   Illegal trading of shares

Answer: (c)

183. Which of the following regulates the working of share market in India?

(a)   MRTP Act

(b)   BIFR

(c)   FERA

(d)   SEBI

Answer: (d)

184. Mass number is the sum of

(a)   Electrons and neutrons

(b)   Only protons

(c)   Electrons and protons

(d)   Protons and neutrons

Answer: (d)

185. Reserve Bank of India was nationalized in

(a)   1945

(b)   1949

(c)   1947

(d)   1948

Answer: (b)

186. The Great Bath was found at

(a)   Lothal

(b)   Chanhudaro

(c)   Harappa

(d)   Mohenjodaro

Answer: (d)

187. Which of the following species is locally extinct in India?

(a)   Siberian Crane

(b)   White Bellied Heron

(c)   They Gyps vulture

(d)   Forest Owlet

Answer: (b)

188. India’s first Hi-speed Rural Broadband Network has been commissioned in a district of

(a)   Karnataka

(b)   Andhra Pradesh

(c)   Kerala

(d)   Telangana

Answer: (c)

189. Vitamin which provides immunity, is

(a)   C

(b)   E

(c)   A

(d)   K

Answer: (a)

190. The resistance of an ideal voltmeter is

(a)   zero

(b)   infinite

(c)   high

(d)   low

Answer: (b)

191. Who was the first to use the term ‘Micro’ and ‘Macro’ in Economics in 1933?

(a)   Ragnar Frisch

(b)   I. Fischer

(c)   James Tobin

(d)   Gurly

Answer: (a)

192. Which of the following works on the basis of conservation of linear momentum?

(a)   Helicopter

(b)   Aeroplane

(c)   Rocket

(d)   Jet

Answer: (d)

193. Penicillin was discovered by

(a)   Alexander Fleming

(b)   Ian Fleming

(c)   Edward Jenner

(d)   Louis Pasteur

Answer: (a)

194. A sufficiently large scale map indicating the detailed surface features of an area including relief is called

(a)   wall map

(b)   topographical map

(c)   chorographical map

(d)   relief map

Answer: (b)

195. In marine whales, the limbs are modified as

(a)   flappers

(b)   flippers

(c)   slippers

(d)   grippers

Answer: (a)

196. The ‘World Environment Day’ is celebrated on

(a)   June 16

(b)   June 5

(c)   August 6

(d)   April 7

Answer: (b)

197. The most stable measure of central tendency is

(a)   mean

(b)   mode

(c)   range

(d)   median

Answer: (a)

198. The system in which the few govern many is known as

(a)   oligarchy

(b)   autocracy

(c)   plutocracy

(d)   monarchy

Answer: (a)

199. Respiration is controlled by …………. part of brain.

(a)   olfactory lobes

(b)   hypothalamus

(c)   medulla oblongata

(d)   cerebellum

Answer: (c)

200. Aggregate Monetary Resources is

(a)   M1

(b)   M2

(c)   M3

(d)   M4

Answer: (c)

Staff Selection Commission (SSC) Combined Higher Secondary (10+2) Recruitment Examination-2015 Held on November 22, 2015 Question Paper With Answer Key

Staff Selection Commission (SSC) Combined Higher Secondary (10+2) Recruitment Examination-2015 Held on November 22, 2015
Staff Selection Commission (SSC) Combined Higher Secondary (10+2) Recruitment Examination-2015 Held on November 22, 2015 Question Paper With Answer Key

Staff Selection Commission (SSC) Combined Higher Secondary (10+2) Recruitment Examination-2015 Held on November 22, 2015

Part I General Intelligence

Directions (Q. Nos. 1-9) Find the odd word/ letters / number from the given alternatives.

1.

(a)   Caprice

(b)   Foible

(c)   Quirk

(d)   Whim

Answer: (b)

2.

(a)   15

(b)   17

(c)   6

(d)   12

Answer: (b)

3.

(a)   532

(b)   734

(c)   853

(d)   751

Answer: (d)

4.

(a)   Solid

(b)   Plump

(c)   Thick

(d)   Fathom

Answer: (d)

5.

(a)   HGFE

(b)   DCBA

(c)   PONM

(d)   IJKL

Answer: (d)

6.

(a)   FUEV

(b)   VUXW

(c)   QPSR

(d)   YXWV

Answer: (a)

7.

(a)   3463

(b)   8948

(c)   6514

(d)   5725

Answer: (c)

8.

(a)   NPR

(b)   RQP

(c)   HGF

(d)   DCB

Answer: (a)

9.

(a)   qqstuu

(b)   aacdff

(c)   mmopqq

(d)   ggijkk

Answer: (b)

Directions (Q. Nos. 10-18) Select the related word / letters / number from the given alternatives.

10. Eyes : Tears : : ___ : ____

(a)   Hunger : Bread

(b)   Volcano : Lava

(c)   Heart : Artery

(d)   Sea : Water

Answer: (b)

11. 196 : 256 : : ? : 400

(a)   324

(b)   204

(c)   452

(d)   144

Answer: (a)

12. 32 : 28 : : 160 : ?

(a)   80

(b)   120

(c)   110

(d)   140

Answer: (d)

13. 414 : 636 : : 325 : ?

(a)   222

(b)   636

(c)   547

(d)   414

Answer: (c)

14. Perch : Fresh water : : ? : Salt water

(a)   Frog

(b)   Cod

(c)   Snake

(d)   Crocodile

Answer: (b)

15. SNAKE : VQDNH : : CRADLE : : ?

(a)   EVFGOF

(b)   FUDGOH

(c)   EUDGOH

(d)   FVDGPH

Answer: (b)

16. Vacation : Holiday : : Vocation : ?

(a)   Money

(b)   Pleasure

(c)   Degree

(d)   Career

Answer: (d)

17. 

(a)   22

(b)   12

(c)   21

(d)   24

Answer: (c)

18. FE : HG : : ML : ?

(a)   QP

(b)   PO

(c)   JI

(d)   ON

Answer: (c)

19. Arrange the following words as per the reverse order in a dictionary.

(1) Multitude          (2) Multinational

(3) Multiplier          (4) Multinomial

(5) Multilingual

(a)   1 3 2 4 5

(b)   4 5 3 1 2

(c)   5 4 3 2 1

(d)   1 3 4 2 5

Answer: (d)

20. Insert the arithmetical operations in the following numerical figure :

4 _ 3 _ 4 = 48

(a)   × ×

(b)   + +

(c)   × −

(d)   + −

Answer: (a)

21. Which answer figure will complete the pattern in the question figure?

 

Answer: (a)

Directions (Q. Nos. 22-25) Select the missing number from the given responses.

22. 

(a)   54

(b)   34

(c)   78

(d)   24

Answer: (a)

23. 

(a)   4

(b)   32

(c)   2

(d)   16

Answer: (d)

24. 

(a)   11

(b)   7

(c)   4

(d)   10

Answer: (b)

25. 

(a)   20

(b)   11

(c)   13

(d)   15

Answer: (c)

26. Karan has a brother Prem and a sister Neesha.. Karan’s wife is Naj and has a daughter Naksha. Naksha got married with Neesha’s son Akbar and has a baby girl Riya. What is relation between Naksha and Neesha?

(a)   Mother and daughter

(b)   Sister

(c)   Niece and Aunt

(d)   Mother and grand-daughter

Answer: (c)

Directions (Q. Nos. 27-28) From the given alternative words, select the word which cannot  be formed using the letters of the given word.

27. Calculate

(a)   Cat

(b)   Team

(c)   Tea

(d)   Late

Answer: (b)

28. Correspondence

(a)   Condense

(b)   Respond

(c)   Correspond

(d)   Respondent

Answer: (d)

Directions (Q. Nos. 29-31) Which one set of letters when sequentially placed at the gaps in the given letter series shall complete it?

29. a_baa_baa-ba

(a)   bba

(b)   bbb

(c)   aab

(d)   bab

Answer: (b)

30. _ _ babbba _ a _ _

(a)   babbb

(b)   ababb

(c)   bbaba

(d)   baaab

Answer: (a)

31. oopqop _ qoo _ qo _ oqo _ pq

(a)   oppo

(b)   popo

(c)   poop

(d)   opop

Answer: (a)

32. A piece of paper is folded and cut as shown below in the question figures. From the given answer figures, indicate how it will appear when opened.

Answer: (c)

33. Which conclusion is true with respect to the given statements?

Statements

(1) All squares are rectangles.

(2) All rectangles are polygons.

Conclusions

(a)   Square is a polygon.

(b)   Square is a rectangle and polygon.

(c)   Square is not a polygon.

(d)   Square is not a rectangle.

Answer: (b)

34. From the given answer figures, select the one in which the question figure is hidden/embedded.

Answer: (c)

35. If Sita walks 10 km towards West, then turned toward South and walked 10 km, then turned East walked 10 km, and turned North walked 10 km. How far she is from starting point?

(a)   40 km

(b)   20 km

(c)   0 km

(d)   10 km

Answer: (c)

36. A word is represented by only one set of numbers as given in any one of the alternatives. The sets of numbers given in the alternatives are represented by two classes of alphabets as in two matrices given below. The columns and rows of Matrix-I are numbered from 0 to 4 and that of Matrix-2 are numbered from 5 to 9. A letter from these matrices can be represented first by its row and next by its column, e.g., ‘M’ can be represented by 01, 10 etc. and ‘R’ can be represented by 58, 85 etc. Similarly, you have to identify the set for the word NOW.

(a)   55, 78, 11

(b)   86, 58, 11

(c)   95, 55, 34

(d)   95, 67, 02

Answer: (c)

Directions (Q. Nos. 37-39) A series is given, with one term missing. Choose the correct alternative from the given ones that will complete the series.

37. 361, ?, 169, 121, 49, 25

(a)   196

(b)   324

(c)   256

(d)   289

Answer: (d)

38. 127, 131, 139, ?, 151, 157, 163, 167

(a)   143

(b)   147

(c)   141

(d)   149

Answer: (b)

39. 1, 1, 2, 3, 5, ?, 13, 21

(a)   8

(b)   6

(c)   7

(d)   9

Answer: (a)

40. Identify the diagram that best represents the relationship among classes given below :

Social Science, History and Geography

Answer: (c)

41. If ‘Stress’ is coded as Rtress. Then ‘Pulse’ will be coded as

(a)   Oulse

(b)   Qulse

(c)   Rulse

(d)   Fulse

Answer: (a)

42. Which one of the given responses would be a meaningful order of the following in ascending order?

(1) Atom       (2) Matter

(3) Molecule  (3) Electron

(a)   4 1 3 2

(b)   3 4 1 2

(c)   1 2 3 4

(d)   3 1 4 2

Answer: (a)

43. If ARMS equal 1234 then MARS will equal to

(a)   1243

(b)   4321

(c)   3124

(d)   4213

Answer: (c)

44. If + stands for division; × stands for addition; − stands for multiplication, ÷ stands for subtraction, then which of the following is correct?

(1) 25 × 3 – 7 ÷ 8 + 12 = 18     (2) 25 + 3 × 7 – 8 ÷ 12 = 10.89

(3) 25 – 3 ÷ 7 × 8 + 12 = 132   (4) 25 ÷ 3 × 7 – 8 + 12 = 19.3

(a)   1

(b)   2

(c)   4

(d)   3

Answer: (c)

45. Arrange the following words as per order in the dictionary.

(1) Silt (2) Silicon   (3) Silicate  (4) Silken

(a)   1, 4, 3, 2

(b)   2, 1, 4, 3

(c)   4, 1, 3, 2

(d)   3, 2, 4, 1

Answer: (d)

46. Six faces of the dice are A, B, C, D E and F. A is adjacent to B. B is adjacent to D but not C. E is adjacent to D and F. What is the side opposite to A?

(a)   E

(b)   D

(c)   C

(d)   F

Answer: (a)

47. Ali had Rs 320. He spent 3/4 of it to buy a watch. Of the remainder, he used 1/8 of it to buy a pen. How much money is left?

(a)   Rs 120

(b)   Rs 90

(c)   Rs 70

(d)   Rs 100

Answer: (c)

48. In the question, two statements are given each followed by two conclusions I and II. You have to consider the statements to be true even if they seem to be at variance from commonly known facts. you have to decide which of the given conclusions, if any, follows from the given statements.

Statements Some clerks are poor. A is poor.

Conclusions I. A is clerk. II. A has a large family.

(a)   Neither conclusion I nor II follows

(b)   Both conclusions I and II follow

(c)   Only conclusion II follows.

(d)   Only conclusion I follows.

Answer: (a)

49. Laxmi went 10 km to the West from my house, then turned left and walked 20 km. She then turned East and walked 25 km and finally turning left covered 20 km. How far was she from my house?

(a)   5 km

(b)   15 km

(c)   40 km

(d)   10 km

Answer: (b)

50. If a mirror is placed on the line RS, then which of the answer figures is the right image of the given figure?

Answer: (a)

Part II English Language

Directions (Q. Nos. 51-54) Choose the word opposite in meaning to the given word.

51. Desist

(a)   Hope

(b)   Continue

(c)   Request

(d)   Assign

Answer: (b)

52. Defile

(a)   Yield

(b)   Repair

(c)   Describe

(d)   Purify

Answer: (d)

53. Sterile

(a)   Dense

(b)   Fertile

(c)   Barren

(d)   Infertile

Answer: (b)

54. Prudent

(a)   Careless

(b)   Miserly

(c)   Foolish

(d)   Strange

Answer: (c)

Directions (Q. Nos. 55-58) Out of the four alternatives, choose the one which best expresses the meaning of the given word.

55. Jealous

(a)   Proud

(b)   Envious

(c)   Greedy

(d)   Lustful

Answer: (b)

56. Quest

(a)   Trial

(b)   Test

(c)   Search

(d)   Decision

Answer: (c)

57. Diligent

(a)   Modest

(b)   Energetic

(c)   Industrious

(d)   Intelligent

Answer: (c)

58. Desultory

(a)   Diminish

(b)   Random

(c)   Forsake

(d)   Frugal

Answer: (b)

Directions (Q. Nos. 59-62) Out of the four alternatives, Choose the one which can be substituted for the given words/sentences.

59. Introductory part or lines to a discourse or play

(a)   Blurb

(b)   Monologue

(c)   Epilogue

(d)   Prologue

Answer: (d)

60. A cinema show held in the afternoon

(a)   Play

(b)   Premiere

(c)   Entertainment

(d)   Matinee

Answer: (d)

61. Intentional damage to arrest production

(a)   Sangfroid

(b)   Modus opernadi

(c)   Sabotage

(d)   Nemesis

Answer: (c)

62. A handsome man

(a)   Tycoon

(b)   Debonair

(c)   Adonis

(d)   Cavalier

Answer: (c)

Directions (Q. Nos. 63-64) A sentence has been given in Direct/Indirect. Out of the four alternatives suggested, select the one which best expresses the ‘same sentence’ in Indirect/Direct.

63. The peon said, to his officer, “Please forgive me”.

(a)   The peon requested his officer that he forgive him.

(b)   The peon requested his officer to forgive  him.

(c)   The peon said to  his officer that he should forgive him.

(d)   The peon told his officer please forgive him.

Answer: (b)

64. My brother told me that he would buy me a notepad the next day.

(a)   My brother said to me, “I will be buying you a notepad tomorrow.”

(b)   My brother said to me, “I would buy you a notepad tomorrow.”

(c)   My brother said to me, “I shall buy you a notepad tomorrow.

(d)   My brother “I am going to buy you a notepad tomorrow.”

Answer: (c)

Directions (Q. Nos. 65-74) In the following passage some of the words have been left out. Read the passage carefully and choose the correct answer to each question out of the four alternatives and fill in the blanks

Past I us in understanding our present world in a II better way. If we know III our ancestors lived or IV the wars were waged, we can understand the today’s world V and get ideas for a VI future. What appears to VII as wrong today could have been right VIII. It is also interesting to know how IX in the past X the life of humans.

65.

(a)   (I)-helps

(b)   (I)-helped

(c)   (I)-help

(d)   (I)-is helping

Answer: (a)

66.

(a)   (II)- most

(b)   (II)- just

(c)   (II)- more

(d)   (II)- much

Answer: (d)

67.

(a)   (III)-when

(b)   (III)-why

(c)   (III)-where

(d)   (III)-how

Answer: (d)

68.

(a)   (IV)-how

(b)   (IV)-which

(c)   (IV)-when

(d)   (IV)-why

Answer: (d)

69.

(a)   (V)-best

(b)   (V)-well

(c)   (V)-betterment

(d)   (V)-better

Answer: (d)

70.

(a)   (VI)-fairy

(b)   (VI)-fairer

(c)   (VI)-fair

(d)   (VI)-fairest

Answer: (b)

71.

(a)   (VII)-him

(b)   (VII)-them

(c)   (VII)-us

(d)   (VII)-they

Answer: (c)

72.

(a)   (VIII)-earliest

(b)   (VIII)-early

(c)   (VIII)-earlier

(d)   (VIII)-later

Answer: (c)

73.

(a)   (IX)-adventures

(b)   (IX)-inventors

(c)   (IX)-inventing

(d)   (IX)-inventions

Answer: (d)

74.

(a)   (X)-improvement

(b)   (X)-improve

(c)   (X)-improves

(d)   (X)-improved

Answer: (d)

Directions (Q. Nos. 75-78) Sentences are given with blanks to be filled in with an appropriate word(s). Four alternatives are suggested for each question. Choose the correct alternative out the four.

75. We can ……… right and wrong.

(a)   distinguish between

(b)   distinguish by

(c)   distinguished for

(d)   distinguished from

Answer: (a)

76. You take a decision. The ball is in …………. now.

(a)   your court

(b)   your pocket

(c)   your garden

(d)   your net

Answer: (a)

77. The cold breath of autumn had ……….. the ivy leaves from the vine and the branches remained almost bare.

(a)   striking

(b)   strike

(c)   stricken

(d)   strucked

Answer: (c)

78. He is a man of ……. simplicity.

(a)   childless

(b)   childish

(c)   child

(d)   childlike

Answer: (d)

Directions (Q. Nos. 79-82) The first and the last part of the sentence/passage are numbered 1 and 6. The rest of the sentence/passage is split into four parts and named P, Q, R and S. These four parts are not given in their proper order. Read the sentence/passage and find out which of the four combinations is correct.

79. (1) Though the government

(P) the growth of population

(Q) has undertaken a series of plans

(R) and for raising the standard of living of the people

(S) for economic development

(6) has upset all the plans.

(a)   QSRP

(b)   QPRS

(c)   SPQR

(d)   SRPQ

Answer: (a)

80. (1) Glorious tributes were paid

(P) who, after fighting a battle with cancer,

(Q) the original superstar of Bollywood

(R) passed away last month

(S) to the legendary Rajesh Khanna,

(6) at  his residence in Mumbai.

(a)   SRPQ

(b)   SQRP

(c)   SQPR

(d)   QSPR

Answer: (c)

81. (1) A volcano is an opening or rupture in a planet’s surface.

(P) Erupting volcanoes can pose many hazards.

(Q) This opening allows magma, ash and gases to escape from below the surface.

(R) Volcanic ash can be a threat to aircraft.

(S) Volcanoes are generally found where tectonic plates are diverging or converging.

(6) Historically, so-called volcanic winters have caused catastrophic famines.

(a)   SPQR

(b)   RSPQ

(c)   SQRP

(d)   QSPR

Answer: (d)

82. (1) What gives some persons

(P) torturing physical pain

(Q) after experiencing

(R) after the loss of a precious loved one

(S) the power to fight on

(6) day after day?

(a)   PSQR

(b)   PQRS

(c)   SRQP

(d)   PQSR

Answer: (c)

Directions (Q. Nos. 83-86) Some parts of the sentences have errors and some are correct. Find out which part of a sentence has an error. If a sentence is free from error, you answer is ‘No error’.

83. Whoever assumes his statement true is foolish.

(a)   No error

(b)   is foolish

(c)   his statement true

(d)   Whoever assumes

Answer: (b)

84. Iodine deficiency is an easy and inexpensive nutrient disorder to prevent.

(a)   Iodine deficiency

(b)   No error

(c)   is an easy and inexpensive

(d)   nutrient disorder to prevent

Answer: (d)

85. The people gathered at the funeral to pay respect.

(a)   at the funeral

(b)   No error

(c)   The people gathered

(d)   to pay respect

Answer: (d)

86. When the workers threatened to go on a strike, the mill owner declared a lay off on his mill.

(a)   No error

(b)   declared a lay off on his mill.

(c)   go on a strike, the mill owner

(d)   When the workers threatened to

Answer: (c)

Directions (Q. Nos. 87-90) Four alternatives are given for the idiom/phrase underlined. Choose the alternative which best expresses the meaning of the idiom/phrase.

87. To take after

(a)   to resemble

(b)   To remove

(c)   To cheat

(d)   To write down

Answer: (a)

88. She paid a flying visit to Mumbai to see her ailing father.

(a)   a surprise visit

(b)   a very short visit

(c)   a very long visit

(d)   went by aeroplane

Answer: (b)

89. Have a finger in every pie

(a)   to be greedy

(b)   to be meddlesome

(c)   to be quarrelsome

(d)   to be efficient

Answer: (b)

90. The jury is not

(a)   The entire jury decides to stay out of the proceedings

(b)   No decision has been reached.

(c)   The jury has taken a break between hearings.

(d)   A jury member is absent.

Answer: (b)

Directions (Q. Nos. 91-92) A sentence has been given in Active/Passive Voice. Out of the four alternatives suggested, select the one which best expresses the same sentence in Passive/Active Voice.

91. Take the medicine.

(a)   You are requested to take the medicine.

(b)   Let the medicine be taken.

(c)   The medicine must take you.

(d)   The medicine is to be taken by you.

Answer: (b)

92. I cannot trust him any more.

(a)   He cannot be trusted by me any more.

(b)   He could not be trusted by me any more.

(c)   He may not be trusted by me anymore.

(d)   He cannot trust by me any more.

Answer: (a)

Directions (Q. Nos. 93-96) Four words are given in each questions, out of which only one word is correctly spelt. Find the correctly spelt word.

93.

(a)   Acommodation

(b)   Accommodation

(c)   Acomodation

(d)   Accomodation

Answer: (b)

94.

(a)   Puritanical

(b)   Purritanical

(c)   Purritaniccal

(d)   Puritannical

Answer: (a)

95.

(a)   Ingenous

(b)   Ingenious

(c)   Ingenius

(d)   Ingeneous

Answer: (b)

96.

(a)   Plegiarist

(b)   Plagraist

(c)   Plagiarist

(d)   Plagearist

Answer: (c)

Directions (Q. Nos. 97-100) A sentence/a part of the sentence is underlined. Below are given alternatives to the underlined part which may improve the sentence. Choose the correct alternative. In case no improvement is needed choose ‘No improvement’.

97. He said, ‘Let the show begins’.

(a)   No improvement

(b)   ‘Let the show begin’

(c)   ‘Let’s the show begin’

(d)   ‘Let the show to begin’

Answer: (b)

98. Both of them are good, but this is the best of the two.

(a)   better

(b)   much better

(c)   No improvement

(d)   good

Answer: (a)

99. The museum’s collection includes artefacts dated back to prehistoric times.

(a)   dating back to

(b)   No improvement

(c)   date back to

(d)   date backs to

Answer: (a)

100. I’m really Sorry but I haven’t got much money myself.

(a)   I’m really sorry but I have very few money myself.

(b)   No improvement

(c)   I’m really sorry but I not have much money myself.

(d)   I’m really sorry I have lesser money myself.

Answer: (b)

Part III Quantitative Aptitude

Directions (Q. Nos. 101-105) The pie-chart shows how the school funds is spent under different heads in a certain school. Using the pie-chart answer the questions.

101. What is the ratio of expenditure on sports to that on art and craft?

(a)   1 : 1

(b)   4 : 3

(c)   1 : 4

(d)   2 : 1

Answer: (b)

102. Which head has the maximum expenditure?

(a)   Library

(b)   Art and Craft

(c)   Sports

(d)   Science

Answer: (c)

103. Which head uses 25% of the funds?

(a)   Art and Craft

(b)   Sports

(c)   Misc

(d)   Library

Answer: (a)

104. Which heads have the same amount of expenditure?

(a)   Library and Science

(b)   Science and Misc

(c)   Sports and Science

(d)   Misc and Library

Answer: (a)

105. What percentage of the total expense is spent on library?

(a)   24.3%

(b)   16.6%

(c)   20%

(d)   24%

Answer: (b)

Directions (Q. Nos. 106-109) The following table shows the world production of steel in 1920-1927. Study the table and answer the questions.

106. The number of years during which the company has its production less than the average production during 1920-1927 is approximately

(a)   4

(b)   2

(c)   3

(d)   6

Answer: (a)

107. The average production of steel is

(a)   74.07

(b)   76.09

(c)   77.10

(d)   75.13

Answer: (c)

108. The difference of the production of steel in the year 1923 and 1924 is x% of 1927. Then the value of x is approximately

(a)   .001

(b)   1

(c)   .01

(d)   .1

Answer: (b)

109. The ratio of production of steel in the year 1924 and 1925 to that of 1923 and 1927 is

(a)   2005 : 2077

(b)   2205 : 2007

(c)   2205 : 2077

(d)   2077 : 2205

Answer: (d)

110. The radius of a hemispherical bowl is 6 cm. The capacity of the bowl is (Take π = 22/7)

(a)   495.51 cm3

(b)   452.57 cm3

(c)   345.53 cm3

(d)   422 cm3

Answer: (b)

111. Simon purchased a bicycle for Rs 6810. He had paid a VAT of 13.5%. The list price of the bicycle was

(a)   Rs 6696.50

(b)   Rs 6140

(c)   Rs 5970.50

(d)   Rs 6000

Answer: (d)

112. Each side of a cube is decreased by 25%. Find the ratio of the volumes of the original cube and the resulting cube.

(a)   64 : 1

(b)   27 : 64

(c)   64 : 27

(d)   8 : 1

Answer: (c)

113. D and E are mid-points of sides AB and AC respectively of the ∆ A line drawn from A meets BC at H and DE at K.

AK : KH = ?

(a)   1 : 2

(b)   2 : 1

(c)   1 : 1

(d)   1 : 3

Answer: (c)

114. The total discount on Rs 1860 due after a certain time at 5% is Rs 60. Find the time after which it is due

(a)   9 months

(b)   10 months

(c)   7 months

(d)   8 months

Answer: (d)

115. A m an buys a TV priced at Rs 16000. He pays Rs 4000 at once and the rest after 15 months on which he is charged a simple interest at the rate of 12% per year. The total amount he pays for the TV is

(a)   Rs 18200

(b)   Rs 17800

(c)   Rs 17200

(d)   Rs 16800

Answer: (b)

116. The value of the following is 

(a)   −1

(b)   0

(c)   1/2

(d)   1

Answer: (b)

117. For real a, b, c, if a2 + b2 + c2 = ab + bc + ca, the value of  is

(a)   0

(b)   3

(c)   2

(d)   1

Answer: (d)

118. The difference between the greatest and the least four digit numbers that begins with 3 and ends with 5 is

(a)   900

(b)   909

(c)   999

(d)   990

Answer: (d)

119. AB and AC are tangents to a circle with centre O . A is the external point of the circle. The line AO intersect the chord BC at D. The measure of the ∠BDO is

(a)   45°

(b)   60°

(c)   75°

(d)   90°

Answer: (d)

120. A hemisphere and a cone have equal bases. If their heights are also equal, then the ratio of their curved surfaces will be

(a)   √2 : 1

(b)   1 : √2

(c)   2 : 1

(d)   1 : 2

Answer: (a)

121. A student goes to school at the rate of  and reaches 6 min late. If he travels at the speed of 3 km/h he is 10 min early. What is the distance to the school?

(a)   4 km

(b) 

(c)   1 km

(d) 

Answer: (a)

122. 9x2 + 25 – 30x can be expressed as the square of

(a)   3x2 – 25

(b)   3x + 5

(c)   −3x – 5

(d)   3x – 5

Answer: (d)

123. There is 10% loss if an article is sold at Rs 270. Then the cost price of the article is

(a)   Rs 300

(b)   Rs 250

(c)   Rs 270

(d)   Rs 320

Answer: (a)

124. A and B can do a piece of work in 15 days. B and C can do the same work in 10 days and A and C can do the same in 12 days. Time taken by A, B and C together to do the job is

(a)   9 days

(b)   8 days

(c)   4 days

(d)   5 days

Answer: (b)

125. The base o f a right prism is a trapezium whose lengths of two parallel sides are 10 cm and 6 cm and distance between them is 5 cm. If the height of the prism is 8 cm, its volume is

(a)   320 cm3

(b)   300 cm3

(c)   310 cm3

(d)   300.5 cm3

Answer: (a)

126. TF is a tower with F on the ground. The angle of elevation of T from A is x° such that tan x° = 2/5 and AF = 200 m. The angle of elevation of T from a nearer point B is y° with BF = 80 m. The value of y° is

(a)   75°

(b)   45°

(c)   60°

(d)   30°

Answer: (b)

127. The sum of the perfect squares between 120 and 300 is

(a)   1204

(b)   1024

(c)   1296

(d)   1400

Answer: (d)

128. The number of pair of positive integers whose sum is 99 and HCF is 9 is

(a)   5

(b)   4

(c)   3

(d)   2

Answer: (b)

129. On what sum of money will the difference between simple interest and compound interest for 2 years at 5% per annum be equal to Rs 63?

(a)   Rs 24600

(b)   Rs 24800

(c)   Rs 25200

(d)   Rs 25500

Answer: (c)

130. A pipe can fill a tank in 24 hours. Due to a leakage in the bottom. It is filled in 36 hours. If the tank is half full, how much time will the leak take to empty the tank?

(a)   24 h

(b)   48 h

(c)   72 h

(d)   36 h

Answer: (d)

131. If sin θ + cos θ = √2 sin(90° − θ), then cot θ is equal to

(a)   √2 + 1

(b)   √2 – 1

(c)   √2

(d)   0

Answer: (a)

132. The ratio of two numbers is 3 : 4 and their LCM is 120. The sum of numbers is

(a)   70

(b)   140

(c)   35

(d)   105

Answer: (a)

133. If x + y = 2a then the value of  is

(a)   2

(b)   0

(c)   −1

(d)   1

Answer: (b)

134. 50% of a number when added to 50 is equal to the number. The number is

(a)   100

(b)   75

(c)   50

(d)   150

Answer: (a)

135. AB is the diameter of a circle with centre O. P be a point on it. If ∠POA = 120°. Then, ∠PBO = ?

(a)   50°

(b)   120°

(c)   60°

(d)   45°

Answer: (c)

136. An epidemic broke out in a village in which 5% of the population died. Of the remaining, 20% fled out of panic. If the present population is 4655, then the population of the village originally was

(a)   6000

(b)   5955

(c)   6125

(d)   5995

Answer: (c)

137. If A, B, C are the angles of a ∆ ABC then following is equal to 

(a)   cosec A/2

(b)   cos A/2

(c)   sec A/2

(d)   sec B/2

Answer: (b)

138. What is the position of the circumcentre of an obtuse-angled triangle?

(a)   It lies inside the triangle.

(b)   It lies outside the triangle.

(c)   It is the vertex opposite to the largest side.

(d)   It is the mid point of the largest side.

Answer: (b)

139. A shop is electronic goods is closed on Monday. The average sales per day for remaining six days of a week is Rs 15640 and the average sale of Tuesday to Saturday is Rs 14124. The sales on Sunday is

(a)   Rs 20188

(b)   Data inadequate

(c)   Rs 23220

(d)   Rs 21704

Answer: (c)

140. The value of the following is 

(a)   4

(b)   2

(c)   3

(d)   1

Answer: (c)

141. If p3 – q3 = (p – q) {(p – q)2 – xpq} then find the value of x is

(a)   1

(b)   −3

(c)   3

(d)   −1

Answer: (d)

142. If  then the value of x3 is

(a)   27

(b)   1

(c)   0

(d)   −27

Answer: (d)

143. In a business A and C invested amounts in the ratio 2 : 1, whereas A and B invested amounts in the ratio 3 : 2. If the their annual profit be Rs 157300, then B’s share in the profit is

(a)   Rs 36300

(b)   Rs 48000

(c)   Rs 24200

(d)   Rs 48400

Answer: (d)

144. The greatest four digit number which is exactly divisible by each one of the numbers 12, 18, 21 and 28.

(a)   9288

(b)   9882

(c)   9828

(d)   9928

Answer: (c)

145. A circle touches the four sides of a quadrilateral ABCD. The value of  is equal to

(a)   1

(b)   1/4

(c)   1/3

(d)   1/2

Answer: (a)

146. Let ABC be an equilateral triangle and AD perpendicular to BC. Then, AB2 + BC2 + CA2 = ?

(a)   5 AD2

(b)   4 AD2

(c)   2 AD2

(d)   3 AD2

Answer: (d)

147. If √33 = 5.745, then the value of the following is approximately 

(a)   1

(b)   6.32

(c)   0.5223

(d)   2.035

Answer: (c)

148. The mean high temperature of the first four days of a week is 25℃ whereas the mean of the last four days is 25.5℃. If the mean of the whole week is 25.2℃, then the temperature of the 4th day is

(a)   25.2℃

(b)   25.5℃

(c)   25.6℃

(d)   25℃

Answer: (c)

149. The difference between simple interest and the true discount on Rs 2400 due 4 years hence at 5% per annum simple interest is

(a)   Rs 70

(b)   Rs 30

(c)   Rs 50

(d)   Rs 80

Answer: (d)

150. A train passes an electrical pole in 20 seconds and passes a platform 250 m long in 45 seconds. Find the length of the train.

(a)   250 m

(b)   300 m

(c)   200 m

(d)   400 m

Answer: (c)

Part IV General Awareness

151. Which one of the following is odd?

(a)   SNMP

(b)   POP

(c)   IMAP

(d)   SMTP

Answer: (a)

152. Which one of the following wood is used in making cricket bats?

(a)   Morus alba

(b)   Limum usitatissimum

(c)   Salix prupurea

(d)   Cedrus deodara

Answer: (c)

153. Which of the following memories must be refreshed many times per second?

(a)   Dynamic RAM

(b)   EPROM

(c)   ROM

(d)   Static RAM

Answer: (a)

154. Which was the first talkie film made in India?

(a)   Raja Harishchandra

(b)   Mother India

(c)   Kisan Kanya

(d)   Alam Ara

Answer: (d)

155. Which day is observed as ‘World AIDS Day’?

(a)   March 20

(b)   December 1

(c)   December 20

(d)   March 1

Answer: (b)

156. Who wrote the famous novel ‘The Guide’?

(a)   Satyajit Ray

(b)   RK Narayan

(c)   Arundhati Roy

(d)   Chetan Bhagat

Answer: (b)

157. Who was the first Indian to become member of British Parliament?

(a)   Firozshah Mehta

(b)   D. Dadadhai Naoroji

(c)   Surendranath Banerjee

(d)   DN Wacha

Answer: (b)

158. The Indian, who won the Grammy Award 2015 in the new age album category is

(a)   Kavita Krishnamurthy

(b)   A R Rehman

(c)   Neela Vaswani

(d)   Ricky Kej

Answer: (d)

159. The common name of the sodium bicarbonate is

(a)   baking powder

(b)   soda ash

(c)   baking soda

(d)   soda lime

Answer: (c)

160. World’s largest producer of coffee is

(a)   Brazil

(b)   Argentina

(c)   India

(d)   Peru

Answer: (a)

161. Eddie Redmayne, won the Oscar (2015) for Best Actor for which film?

(a)   Still Alice

(b)   Birdman

(c)   The Theory of Everything

(d)   None of the options

Answer: (c)

162. Which of the following property of sound is affected by change in air temperature?

(a)   Wavelength

(b)   Amplitude

(c)   Intensity

(d)   Frequency

Answer: (d)

163. Who was Akbar’s famous Revenue Minister?

(a)   Tansen

(b)   Rana Partap Singh

(c)   Humayun

(d)   Todarmal

Answer: (d)

164. If there is one million Mg2+ ions in MgCl2, how many chloride ions are there?

(a)   Two million

(b)   One million

(c)   Half million

(d)   Ten million

Answer: (a)

165. Which factor is necessary for the development of democratic institutions?

(a)   Respect for Individual Rights

(b)   Strong Military Forces

(c)   A One-Party System

(d)   A Agricultural Economy

Answer: (a)

166. Soilless agriculture refers to

(a)   Sericulture

(b)   Hygroponics

(c)   Hydroponics

(d)   Inter-cropping

Answer: (c)

167. Who invented the safety razor?

(a)   Lar Strauss

(b)   Gillette

(c)   Steve Job

(d)   Steve Cher

Answer: (b)

168. The non-cooperation movement was called off due to

(a)   Poona Pact

(b)   Chauri-Chaura Incident

(c)   Jallianwalla Bagh Tragedy

(d)   Gandhi-Irwin Pact

Answer: (b)

169. The idea of parliamentary form of government is adapted from

(a)   the UK

(b)   the USA

(c)   Ireland

(d)   the USSR

Answer: (a)

170. The instrument used to measure pressure

(a)   thermometer

(b)   hygrometer

(c)   aneroid barometer

(d)   anemometer

Answer: (c)

171. Who is the founder of the concept ‘Sarvodaya’?

(a)   Mahatma Gandhi

(b)   KG Mushroowala

(c)   Vinoba Bhave

(d)   Jai Prakash Narayan

Answer: (a)

172. In which year was the first ‘World Environment Day’ observed?

(a)   1972

(b)   1973

(c)   1974

(d)   1980

Answer: (a)

173. When and where did the concept of Earth Hour began?

(a)   In June, 2007 in Christchurch, New Zealand

(b)   In April, 2008 in Tokyo, Japan

(c)   In May, 2009 in Colombo, Sri Lanka

(d)   In March, 2007 in Sydney, Australia

Answer: (d)

174. The Industrial Development Bank of India was set-up in

(a)   July, 1968

(b)   July, 1966

(c)   July, 1962

(d)   July, 1964

Answer: (d)

175. The total utility from 9 units of commodity x is 20 and from 10 units is 15. Calculate the marginal utility from 10 unit.

(a)   −0.5

(b)   0.5

(c)   −5

(d)   5

Answer: (c)

176. More than 50% of the world’s coal deposits are held by

(a)   USA, Russia and China

(b)   China, India and Russia

(c)   China, India and USA

(d)   India, Russia and USA

Answer: (a)

177. Who among the following rulers abolished Jaziya?

(a)   Aurangzeb

(b)   Akber

(c)   Balban

(d)   Jahangir

Answer: (b)

178. ‘Cloud burst’ means

(a)   sowing of seeds of a crop in cloudy weather

(b)   presence of scattered flakes of cloud in the sky

(c)   abnormally heavy downpour of rain, associated with a thunderstorm

(d)   formation of artificial rain

Answer: (c)

179. RBI’s deadline to exchange pre 2005 currency notes of Rs 500 and Rs 1000 is

(a)   December 31, 2015

(b)   April 1, 2015

(c)   January 1, 2015

(d)   March 31, 2015

Answer: (a)

180. National Renewal Fund (NRF) was instituted for the purpose of

(a)   Restructuring and modernization of industries

(b)   Rural reconstruction

(c)   Providing pension for retiring employees

(d)   Social security

Answer: (a)

181. How many States are there in the Indian Union?

(a)   30

(b)   27

(c)   29

(d)   28

Answer: (c)

182. Pick out the person associated with the coining of the term ‘gene’

(a)   Waldeyer

(b)   Morgan

(c)   Mendel

(d)   Johannsen

Answer: (d)

183. The battle of Plassey was fought between

(a)   None of the options

(b)   Mir Khasim and Robert Clive

(c)   Mir Jafar and Robert Clive

(d)   Sirajudduala and Robert Clive

Answer: (d)

184. The serious environmental degradation of Maldives is considered to be essentially due to

(a)   high population density

(b)   industrial pollution of water of air

(c)   constant soil erosion

(d)   None  of the options

Answer: (d)

185. Barter transactions means

(a)   money Acts as a medium of exchange

(b)   coins are exchanged for goods

(c)   goods are exchanged with gold

(d)   goods are exchanged with goods

Answer: (d)

186. How many Nobel Prize awards are awarded each year?

(a)   6

(b)   5

(c)   8

(d)   10

Answer: (a)

187. Phycology is the study of

(a)   fungi

(b)   lichens

(c)   bacteria

(d)   algae

Answer: (d)

188. ‘Red Data Book’ provides an account of

(a)   extinct animals only

(b)   endangered plants and animals

(c)   endangered  plants only

(d)   fossil plants

Answer: (b)

189. Name the first Asian country to orbit Mars.

(a)   Japan

(b)   India

(c)   Pakistan

(d)   China

Answer: (b)

190. Which US President announced the “New Deal” for economic recovery in the aftermath of the Great Depression?

(a)   Roosevelt

(b)   Abraham Lincoln

(c)   JF Kennedy

(d)   Benjamin Franklin

Answer: (a)

191. A bullet of mass ‘m’ and velocity ‘a’ is fired in to a large block of wood of mass ‘M’. The final velocity of the system is

(a) 

(b) 

(c) 

(d) 

Answer: (d)

192. First human heart transplant was performed in

(a)   1959

(b)   1967

(c)   1972

(d)   1955

Answer: (b)

193. What is ‘Talcher’ important for?

(a)   Heavy water plant

(b)   Cable industry

(c)   Hydro-electricity generation

(d)   Atomic reactor

Answer: (a)

194. The intensity ratio of waves is 25 : 9. What is the ratio of their amplitudes?

(a)   5 : 3

(b)   50 : 18

(c)   25 : 9

(d)   3 : 5

Answer: (a)

195. Which one of the following is not coal variety?

(a)   Peat

(b)   Dolomite

(c)   Lignite

(d)   Bituminous

Answer: (b)

196. The gas that causes suffocation and death when coal or coke is burnt in a closed room is

(a)   ethane

(b)   methane

(c)   carbon dioxide

(d)   carbon monoxide

Answer: (d)

197. The directive principles incorporated in the Indian Constitution have been inspired by the Constitution of

(a)   Ireland

(b)   the USA

(c)   Australia

(d)   Canada

Answer: (a)

198. When number of turns in a coil is tripled, without any change in the length of coil, its self inductance becomes

(a)   nine times

(b)   six times

(c)   three times

(d)   one-third

Answer: (a)

199. Dry ice is the solid form of

(a)   nitrogen

(b)   water

(c)   air

(d)   carbon dioxide

Answer: (d)

200. When was the last telegram sent in India?

(a)   July 30, 2013

(b)   June 14, 2013

(c)   August 1, 2013

(d)   July 14, 2013

Answer: (d)

Staff Selection Commission (SSC) 10+2 Recruitment Examination Held on November 15, 2015 Question Paper With Answer Key

Staff Selection Commission (SSC) 10+2 Recruitment Examination Held on November 15, 2015
Staff Selection Commission (SSC) 10+2 Recruitment Examination Held on November 15, 2015 Question Paper With Answer Key

Staff Selection Commission (SSC) 10+2 Recruitment Examination Held on November 15, 2015

Part-I General Intelligence

1. Choose the similar group of numbers on the basis of certain common properties they possess :

(72, 66, 96)

(a)   (54, 57, 78)

(b)   (24, 69, 58)

(c)   (55, 66, 77)

(d)   (63, 70, 86)

Answer: (a)


Directions (Q. Nos. 2-5) In the following questions, a series is given, with one term missing. Choose the correct alternative from t he given ones that will complete the series.

2. AZ, CX, FU, ?

(a)   IV

(b)   JQ

(c)   KP

(d)   IR

Answer: (b)

3. 4, 11, 17, 22, ?, 29, 31, 32

(a)   24

(b)   26

(c)   27

(d)   23

Answer: (b)

4. 

(a) 

(b) 

(c) 

(d) 

Answer: (a)

5. Y, T, P, ?, K

(a)   M

(b)   L

(c)   O

(d)   N

Answer: (a)

Directions (Q. Nos. 6-9) In the following questions, select the missing number from the given responses.

6. 

(a)   77

(b)   79

(c)   73

(d)   75

Answer: (d)

7. 

(a)   32

(b)   45

(c)   80

(d)   30

Answer: (b)

8. 

(a)   4

(b)   6

(c)   8

(d)   2

Answer: (a)

9. 

(a)   21

(b)   20

(c)   5

(d)   4

Answer: (c)

Directions (Q. Nos. 10-17) In the following questions, select the word/letters/number from the given alternatives.

10. Carpentry : Skill : : ? : Talent

(a)   Masonry

(b)   Singing

(c)   Plumbing

(d)   Driving

Answer: (b)

11. Concord : Agreement : : Discord : ?

(a)   Blended

(b)   Comparison

(c)   Conflict

(d)   Association

Answer: (c)

12. 42 : 20 : : 64 : ?

(a)   32

(b)   33

(c)   34

(d)   31

Answer: (d)

13. Female : Feminine : : Male : ?

(a)   Manager

(b)   Man

(c)   Masculine

(d)   Macho

Answer: (c)

14. PS : VY : FI : ?

(a)   UX

(b)   VZ

(c)   WZ

(d)   LO

Answer: (d)

15. KcaC : CacK : : XgmF : ?

(a)   EgmX

(b)   Fmg X

(c)   Gmef

(d)   EmgF

Answer: (b)

16. SUMO : PRJL : : TAXI : ?

(a)   QDNF

(b)   WDXJ

(c)   QDVF

(d)   QXUF

Answer: (d)

17. 

(a)   1/3

(b)   1/2

(c)   1

(d)   1/4

Answer: (a)

Directions (Q. Nos. 18-26) In these questions, find the odd word/letters/number/number pair from the given alternatives.

18.

(a)   (45, 18)

(b)   (36,27)

(c)   (23, 14)

(d)   (82, 29)

Answer: (d)

19.

(a)   ZXVT

(b)   SRQP

(c)   LKJI

(d)   HGFE

Answer: (c)

20.

(a)   UVWX

(b)   SRQP

(c)   LKJI

(d)   HGFE

Answer: (a)

21.

(a)   NKM

(b)   DAC

(c)   UTV

(d)   IFH

Answer: (c)

22.

(a)   2518

(b)   3249

(c)   2709

(d)   8314

Answer: (d)

23.

(a)   Fast

(b)   Knock

(c)   Wrong

(d)   Psychology

Answer: (a)

24.

(a)   36

(b)   66

(c)   76

(d)   56

Answer: (b)

25.

(a)   Rice : Cereals

(b)   Tea : Beverages

(c)   Legumes : Nodules

(d)   Beans : Pulses

Answer: (c)

26.

(a)   Triangle

(b)   Cone

(c)   Rectangle

(d)   Circle

Answer: (b)

27. Which one of the given responses would be a meaningful order of the following?

1. India 2. Bengaluru

3. Asia 4. Karnataka

(a)   3, 4, 2, 1

(b)   3, 1, 4, 2

(c)   3, 1, 2, 4

(d)   1, 2, 3, 4

Answer: (b)

Directions (Q. Nos. 28-29) In the following questions, arrange the following words as per order in the dictionary.

28. 1. Necessary 2. Navigate

3. Nautical 4. Naval

(a)   4, 3, 2, 1

(b)   3, 4, 2, 1

(c)   3, 12, 4, 1

(d)   2, 4, 3, 1

Answer: (b)

29. 1. Range 2. Rain

3. Rein 4. Ranger

(a)   2, 4, 3, 1

(b)   2, 3, 4, 1

(c)   2, 1, 3, 4

(d)   2, 1, 4, 3

Answer: (d)

30. After going 80 m from his house towards East, a person turns left and goes 20 m, then turns right and moves 100 m, then turns left and goes 60 m, then turns rights and goes 120 m to reach the park. What is the distance between his hou se and the park?

(a)   80 m

(b)   120 m

(c)   20 m

(d)   100 m

Answer: (d)

31. If SUNDAY = 18, MONSOON = 21, YEAR = 12, then THURSDAY = ?

(a)   24

(b)   26

(c)   42

(d)   28

Answer: (a)

32. In following series, find 20th number.

9, 5, 1, −3, −7, − 11, …

(a)   −67

(b)   −64

(c)   −75

(d)   −70

Answer: (a)

33. If FADE is coded as 3854 then how can GAGE be coded?

(a)   1824

(b)   2834

(c)   2824

(d)   2814

Answer: (c)

34. If it is possible to form a word with the first, fourth, seventh and eleventh letters in the word SUPERFLUOUS, write the first letter of that word.

(a)   L

(b)   O

(c)   E

(d)   S

Answer: (a)

35. If ‘−’ stands for addition, ‘+’ for multiplication, ‘÷’ for subtraction and ‘×’ for division, which one of the following equations is correct?

(a)   5 + 2 – 12 ÷ 6 × 2 = 13

(b)   5 + 2 – 12 × 6 ÷ 2 = 10

(c)   5 ÷ 2 + 12 × 6 – 12 = 4

(d)   5 – 2 + 12 × 6 ÷ 2 = 27

Answer: (b)

36. If P denotes ÷, Q denotes ×, r denotes + and S denotes −, then 16Q12P6R5S4 = ?

(a)   31

(b)   32

(c)   33

(d)   30

Answer: (c)

37. Unscramble the following letters to frame a meaningful word. Then find out the correct numerical position of the letters.

O         T       Y       S       R       H       I

1          2        3        4        5        6        7

(a)   6  7  4  2  1  5  3

(b)   6  2  4  1  3  7  5

(c)   6  4  5  2  1  7  3

(d)   6  3  4  7  1  2  5

Answer: (a)

38. The sum of ages of mother, daughter and son is 87 years. What will be the sum of their ages after eight years?

(a)   101

(b)   110

(c)   111

(d)   105

Answer: (c)

39. Which one set of letters when sequentially placed at the gaps in the given letter series shall complete it?

_bbm_amb_m_a_bbm

(a)   mbabm

(b)   ambbm

(c)   mabam

(d)   abmab

Answer: (c)

40. If Ramya’s rank is 22nd out of 46 students. What is her rank from the last?

(a)   25th

(b)   29th

(c)   32nd

(d)   24th

Answer: (c)

41. Two statements are given, each followed by two conclusion/assumption, I and II. You have to consider the statement to be true even if they seem to be at variance from commonly known facts. You have to decide which of the given conclusion/assumptions, if any, follows from t he given statements.

Statements

Some papers are pens.

All the pencils are pens.

Conclusions

I. Some pens are pencils.

II. Some pens are papers.

(a)   Only conclusion II follows

(b)   Either I or II follows

(c)   Both I and II follow

(d)   Only conclusion I follows

Answer: (c)

42. Which conclusion is true with respect to the given statements?

Statements
(i) Roy studies History and Political Science.

(ii) Roy studies Oxford University.

Conclusions

(a)   Roy does not study History.

(b)   Roy does not study Political Science.

(c)   Roy studies History and Political Science in Oxford University.

(d)   Roy studies Social Science.

Answer: (c)

43. The mall is 250 m north-west of the market. The school is 250 m South-West of the market. In which direction is the school from the mall?

(a)   East

(b)   West

(c)   South

(d)   North

Answer: (c)

44. Identify the answer figure from which the pieces give in the question figure have been cut.

Answer: (a)

45. Which figure will best represent the relationship amongst the three classes?

Boy, Sportsman, Student

Answer: (d)

46. Which answer figure will complete the pattern in the question figure?

Answer: (a)

47. A piece of paper is folded and cut as shown below in the question figures. From t he given answer figures, indicate how it will appear when opened?

Answer: (b)

48. The image of a clock in a mirror is seen as 3 : 15. What is the right time?

(a)   8 : 45

(b)   10 : 45

(c)   7 : 45

(d)   9 : 45

Answer: (a)

49. A word is represented by only one set of a numbers as given in any one of the alternatives. The sets of numbers given in the alternatives are represented by two classes of alphabets as in two matrices given below. The columns and rows of Matrix-I are numbered from 0 to 4 and that of Matrix-II are numbered from 5 to 9. A letter from these matrices can be represented first by its row and next by its column, e.g.,

‘T’ can be represented by 00, 14 etc., and ‘N’ can be represented by 59, 68 etc. Similarly, you have to identify the set for the word ‘ROAD’.

(a)   67, 96, 56, 57

(b)   56, 67, 57, 96

(c)   67, 57, 96, 56

(d)   96, 67, 56, 57

Answer: (a)

50. 

The solid so formed by joining unit cubes is rotated to obtain different positions, which of these cannot b3e the shape after it has turned?

Answer: (c)

Part-II English Language

Directions (Q. Nos. 51-54) In following questions, some parts of the sentences have errors and some are correct. Find out which part of a sentence has an error. If a sentence is free from error, mark option (d) as your answer.

51. One must / obey one’s / teachers. No error

    (a)              (b)                (c)          (d)

Answer: (d)

52. She always fed / her children / before she fed her dog. No error

         (a)                           (b)                           (c)                            (d)

Answer: (b)

53. They came here in / the evening and begin making / further arrangements.

           (a)                                             (b)                                          (c)

No error

   (d)

Answer: (b)

54. Make what you write / and say more / absorbed and engrossing. No error

            (a)                              (b)                                      (c)                             (d)

Answer: (c)

Directions (Q. Nos. 55-58) In these questions sentences are given with blanks to be filled in with an appropriate word(s). Four alternatives are suggested for each question. Choose the correct alternative out of the four.

55. She failed to ……… to her name.

(a)   rise up

(b)   line up

(c)   shine up

(d)   keep up

Answer: (a)

56. All orders must ………. the rules.

(a)   conforms with

(b)   conform to

(c)   conforms to

(d)   conforming with

Answer: (b)

57. As the doctor ……….. into the room, the nurse handed him the temperature chart of the patient.

(a)   is coming

(b)   came

(c)   was coming

(d)   comes

Answer: (b)

58. The economic ……….. has affected our sales tremendously.

(a)   touchdown

(b)   showdown

(c)   slowdown

(d)   crackdown

Answer: (c)

Directions (Q. Nos. 59-62) In the following questions, out of the four alternatives, choose the one which best expresses the meaning of the given word.

59. Chastise

(a)   Praise

(b)   Upbraid

(c)   Monitor

(d)   Chase

Answer: (b)

60. Pawn

(a)   Sponge

(b)   Scrounge

(c)   Hire

(d)   Pledge

Answer: (d)

61. Feeble

(a)   Weak

(b)   Playful

(c)   Pretty

(d)   Small

Answer: (a)

62. Maestro

(a)   Genius

(b)   Admirer

(c)   Employee

(d)   Novice

Answer: (a)

Directions (Q. Nos. 63-72) In the following passage some of the words have been left out. Read the passage carefully and choose the correct answer to each question out of the four alternative and fill in the blanks.

In civilized life there is a rule (63) violence, against taking the (64) into our hands. It is a rule which (65) of us observe so often, indeed, that a great (66) of people go through life (67) orderliness and non-violence as part of the scheme of nature.

But, when (68) comes into their midst (69) refuses to observe the current rules, and (70) the simple rule that might is right, the law-abiding members (71) society do not known what to do, and look on in (72) bewildered confusion..

63.

(a)   upon

(b)   after

(c)   at

(d)   against

Answer: (d)

64.

(a)   law

(b)   police

(c)   people

(d)   rule

Answer: (a)

65.

(a)   every

(b)   most

(c)   none

(d)   many

Answer: (b)

66.

(a)   sum

(b)   amount

(c)   number

(d)   capacity

Answer: (c)

67.

(a)   accepting

(b)   not expecting

(c)   expecting

(d)   not accepting

Answer: (a)

68.

(a)   everyone

(b)   no one

(c)   any one

(d)   none

Answer: (c)

69.

(a)   whom

(b)   who

(c)   how

(d)   where

Answer: (b)

70.

(a)   follow

(b)   following

(c)   followed

(d)   follows

Answer: (d)

71.

(a)   on

(b)   of

(c)   at

(d)   in

Answer: (b)

72.

(a)   helpful

(b)   helping

(c)   helped

(d)   helpless

Answer: (d)

Directions (Q. Nos. 73-76) In the following questions, a part of the sentence is underlined. Below are given alternatives to the underlined part at (a), (b) (c) which may improve the sentence. Choose the correct alternative. In case no improvement is needed your answer is (d).

73. Hundreds of children are deaf born every

(a)   deaf are born every

(b)   every born are deaf

(c)   are born deaf every

(d)   No improvement

Answer: (c)

74. She had realized that she had seen him before.

(a)   had been realized

(b)   realized

(c)   has realized

(d)   No improvement

Answer: (b)

75. She is willing to help you.

(a)   wilful

(b)   willingly

(c)   wilfully

(d)   No improvement

Answer: (d)

76. Being ill, he came to work.

(a)   He came to work and fell ill

(b)   Despite coming to work, he was ill

(c)   Inspite of being ill, he came to work

(d)   No improvement

Answer: (c)

Directions (Q. Nos. 77-80) In the following questions, choose the word opposite in meaning to the given word.

77. Appoint

(a)   Dismiss

(b)   Reward

(c)   Yield

(d)   Disunite

Answer: (a)

78. Accusation

(a)   Encouragement

(b)   Complaint

(c)   Felicitation

(d)   Exculpation

Answer: (d)

79. Guilty

(a)   Dubious

(b)   Honest

(c)   Innocent

(d)   Sorry

Answer: (c)

80. Bizarre

(a)   Droll

(b)   Ridiculous

(c)   Ordinary

(d)   Comical

Answer: (c)

Directions (Q. Nos. 81-84) In these questions, four alternative are given for the Idiom/Phrase underlined. Choose the

81. Now-a-days it has become a fashion to take Frence Leave.

(a)   Seeking permission from French Embassy

(b)   Saying goodbye in French style

(c)   Absenting oneself without permission

(d)   Taking leave to go to France

Answer: (c)

82. International monetary affairs are governed by the gnomes of Zurich.

(a)   Witchcraft of Zurich

(b)   Foreign leaders

(c)   Big international bankers

(d)   Guardians of treasure

Answer: (c)

83. To make up one’s mind.

(a)   To decide what to do

(b)   To remember things clearly

(c)   To remind oneself of something

(d)   To think creatively

Answer: (a)

84. To put in a nut-shell.

(a)   To be long and exhaustive about something

(b)   To state something very concisely

(c)   To place something

(d)   To be blunt about something

Answer: (b)

Directions  (Q. Nos. 85-88) In the following questions, out of the four alternatives, choose the one which can be substituted the given words/sentences.

85. Art of working with metals

(a)   Meteorite

(b)   Metaphysics

(c)   Metallurgy

(d)   Metalloid

Answer: (c)

86. A gathering at a religious

(a)   Congregation

(b)   Spectators

(c)   Mob

(d)   Audience

Answer: (a)

87. One who compiles a dictionary.

(a)   Lexicographer

(b)   Cartographer

(c)   Bibliographer

(d)   Lapidist

Answer: (a)

88. A place where birds are kept.

(a)   Aquarium

(b)   Aviary

(c)   Sanctuary

(d)   Apiary

Answer: (b)

Directions (Q. Nos. 89-92) In these questions, four words are given in each question, out of which only one word is correctly spelt. Find the correctly spelt word.

89.

(a)   Equanimous

(b)   Equanamous

(c)   Ecuanemous

(d)   Ecuanimous

Answer: (a)

90.

(a)   laboratory

(b)   laboratorry

(c)   laboratery

(d)   laborratory

Answer: (a)

91.

(a)   humane

(b)   humein

(c)   humaen

(d)   humain

Answer: (a)

92.

(a)   Pedestrean

(b)   Pedestrian

(c)   Padestrian

(d)   Pedistrian

Answer: (b)

Directions (Q. Nos. 93-96) In the following questions, first and the last part of sentence are numbered 1 and 6. The rest of the sentence is split into four parts and named P, Q, R and S. These four parts are not given in their proper order. Read the sentence and find out which of the four combinations is correct.

93. 1. Falcons have sharp angular wings

P. to dive sharply

Q. and allow them

R. to chase their prey

S. that give them the speed

6. to capture their victims

(a)   SQPR

(b)   QPRS

(c)   PRSQ

(d)   SRQP

Answer: (d)

94. 1. This summer was the most

P. to believe that next

Q. and we have reason

R. scorching in living memory.

S. year and the year after

6. will be hotter still

(a)   RQPS

(b)   SRPQ

(c)   SPQR

(d)   QSPR

Answer: (a)

95. 1. The students

P. touched the

Q. arrived and

R. their teacher

S. feet of

6. with reverence

(a)   QRSP

(b)   RQSP

(c)   QPSR

(d)   QPRS

Answer: (c)

96. 1. The watchman

P. and found two thieves

Q. woke up when

R. with black masks

S. he heard the dog darking

6. trying to get in.

(a)   SPQR

(b)   QSRP

(c)   PQRS

(d)   QSPR

Answer: (d)

Directions (Q. Nos. 97-98) In these questions, a sentence has been given in Active/Passive Voice. Out of the four alternatives suggested, select the one which best expresses the same sentence in Passive/Active Voice.

97. The boys were making kites.

(a)   Kites are being made by the boys.

(b)   Kites were being made by the boys.

(c)   Kites are made by the boys.

(d)   The boys and made kites

Answer: (b)

98. He will not use the computer.

(a)   By him the computer will not be used.

(b)   The computer will not be used by him.

(c)   The use of the computer will not be by him.

(d)   The computer he will not use.

Answer: (b)

Directions (Q. Nos. 99-100) In the following questions, a sentence has been given in Direct/Indirect. Out of the four alternatives suggested, select the one which best expresses the same sentence in Indirect/Direct.

99. The Policeman said t the driver, “Do you have a license ?”

(a)   The police man asked the driver whether he have a licence.

(b)   The policeman asked the driver whether he had a licence.

(c)   The policeman asked the driver whether he had had a licence.

(d)   The policeman asked the driver whether he has a licence.

Answer: (b)

100. The teacher said to the students “March quietly to the ground.”

(a)   The teacher said to the students that they should march quietly to the ground.

(b)   The teacher instructed the students that they should march quietly to the ground.

(c)   The teacher instructed the students that they must march quietly to the ground.

(d)   The teacher instructed the students to march quietly to the ground.

Answer: (d)

Part-III Quantitative Aptitude

101. Length of the each edge of a regular tetrahedron is 1 cm. Its volume is

(a) 

(b) 

(c) 

(d) 

Answer: (d)

102. The numerical value of 

(a)   5

(b)   7

(c)   9

(d)   4

Answer: (c)

103. After allowing a discount of 20%, a radio is available for Rs 1200. Its marked price was

(a)   Rs 1550

(b)   Rs 1500

(c)   Rs 1800

(d)   Rs 1400

Answer: (b)

104. If ABCD be a rhombus, AC is its smallest diagonal and ∠ABC = 60°, find length of a side the rhombus when AC =6 cm.

(a)   6 cm

(b)   3 cm

(c)   6√2 ccm

(d)   3√3 cm

Answer: (d)

105. If a man were to sell his hand-cart for Rs 720, would lose 25%. At what price must he sell in gain 25%?

(a)   Rs 1200

(b)   Rs 960

(c)   Rs 1152

(d)   Rs 768

Answer: (a)

106. If x = a(b – c), y = b(c – a), z = c(a – b), then the value of  is

(a) 

(b) 

(c)   0

(d) 

Answer: (d)

107. Successive discounts of 20% and 10% are equivalent to a single discount of

(a)   15%

(b)   28%

(c)   25%

(d)   30%

Answer: (b)

108. The ratio of syrup and water in an mixture is 3 : 1, then the percentage of syrup in this mixture is

(a)   75%

(b)   25%

(c) 

(d) 

Answer: (a)

109. If A, B and C be the angles of a triangle, then out of the following, the incorrect relation is

(a) 

(b) 

(c) 

(d) 

Answer: (b)

110. The average of 12 numbers is 15 and the average of the first two is 14. What is the average of the rest?

(a)   15

(b) 

(c)   14

(d) 

Answer: (b)

111. If Rahim deposited the same amount of Rs x in a bank at the beginning of successive 3 years and the bank pays simple interest of 5% per annum, then the amount at his credit at the end of 3rd year will be

(a) 

(b) 

(c) 

(d) 

Answer: (d)

112. A plate on square base made of brass of length x cm and width 1 mm. The p late weighs 4725 gm. If 1 cubic cm of brass weighs 8.4 gram, then the value of x is

(a)   75

(b)   76

(c)   72

(d)   74

Answer: (a)

113. If  then the value of  is

(a) 

(b) 

(c) 

(d) 

Answer: (c)

114. In a circle with centre at O (0, 0) and radius 5 cm, AB is chord of length 8 cm. If OM is perpendicular to AB, then the length of OM is

(a)   2.5 cm

(b)   3 cm

(c)   4 cm

(d)   1 cm

Answer: (b)

115. A merchant has 1000 kg sugar, part of which he sells at 8% profit and the rest at 18% profit. He gains 14% on the whole. The quantity sold at 8% profit is

(a)   560 kg

(b)   600 kg

(c)   640 kg

(d)   400 kg

Answer: (d)

116. If 

(a)   2

(b)   1/16

(c)   1/4

(d)   4

Answer: (b)

117. Two trains start at the same time from A and B and proceed toward each other at the speed of 75 km/h and 50 km/h respectively. When both meet at a point in between, one train found to be travelled 175 km more than the other. Find the distance between A and B.

(a)   875 km

(b)   785 km

(c)   758 km

(d)   857 km

Answer: (a)

118. How much  of Rs 312 exceeds Rs 200?

(a)   Rs 96

(b)   Rs 4

(c)   Rs 8

(d)   Rs 104

Answer: (c)

119. Let ABC a triangle and AD be the perpendicular from the vertex A on the side BC such that AD2 = BD ∙ Then measure of ∠BAC is:

(a)   100°

(b)   90°

(c)   75°

(d)   120°

Answer: (b)

120. ∠A of ∆ ABC is a right angle. AD is perpendicular on BC. If BC = 14 cm and BD = 5 cm, then mean of AD is:

(a)   2√5 cm

(b)   √5 cm

(c)   3√5 cm

(d)   3.5√5 cm

Answer: (c)

121. If  then the value of tan θ is :

(a) 

(b) 

(c) 

(d) 

Answer: (*)

122. A number x is divisible by 7. When this number divided by 8, 12 and 16, it leaves a remainder each case. The least value of x is

(a)   148

(b)   149

(c)   150

(d)   147

Answer: (d)

123. If  then the value of a – b is :

(a)   1

(b)   2

(c)   −1

(d)   0

Answer: (d)

124. Measure of each interior angle of a regular hexagon is

(a)   100°

(b)   60°

(c)   45°

(d)   120°

Answer: (d)

125. The average expenditure of a man for the first five months is Rs 1200 and for the next seven months is Rs 1300. If he saves Rs 2900 in that year, h is monthly average income is

(a)   Rs 1500

(b)   Rs 1600

(c)   Rs 1700

(d)   Rs 1400

Answer: (a)

126. A, B and C can complete a piece of work in 24, 5 and 12 days respectively. Working together, they will complete the same work in

(a) 

(b) 

(c)   4 days

(d)   1/24 days

Answer: (*)

127. If tan θ + sec θ = 3, θ being acute, the value of 5 sin θ is

(a)   5/2

(b) 

(c) 

(d)   4

Answer: (d)

128. A man purchased an article for Rs 1500 and sold it at 25% above the cost price. If he has to pay Rs 75 as tax on it, his net profit percentage will be

(a)   20%

(b)   25%

(c)   30%

(d)   15%

Answer: (a)

129. If the ratio of principal and the simple interest of 5 years is 10 : 3, then the rate of interest is

(a)   5%

(b)   6%

(c)   8%

(d)   3%

Answer: (b)

130. Base of a right prism is a rectangle, the ratio of whose length and breadth is 3 : 2. If the height of the prism is 12 cm and total surface area is 288 sq cm, the volume of the prism is

(a)   291 cm3

(b)   288 cm3

(c)   290 cm3

(d)   286 cm3

Answer: (b)

131. At what rate of compound interest per annum will a sum of Rs 1200 become Rs 1348.32 in two years?

(a)   7.5%

(b)   6.5%

(c)   7%

(d)   6%

Answer: (d)

132. A man travels for 5 hours 15 minutes. If he covers the first half of the journey at 60 km/h and rest at 45 km/h,, then find the total distance travelled by him.

(a) 

(b)   189 km

(c)   378 km

(d)   270 km

Answer: (d)

133. The Vulgar fraction of is

(a)   15/33

(b)   11/39

(c)   17/39

(d)   13/33

Answer: (d)

134. If x = y = z, then  is

(a)   4

(b)   2

(c)   3

(d)   1

Answer: (c)

135. A candidate who gets 20% marks in an examination, fails by 30 marks. But if he gets 32% marks, he gets 42 marks more than the minimum pass marks. Find the pass percentage of marks.

(a)   52%

(b)   20%

(c)   25%

(d)   12%

Answer: (c)

136. The quotient when 10100 is divided by 575 is

(a)   225 × 1075

(b)   1025

(c)   2‑75

(d)   275 × 1025

Answer: (d)

137. The volume of a right circular cone which is obtained from a wooden cube of edge 4.2 dm wasting minimum amount of wood is

(a)   19404 cu dm

(b)   194.04 cu dm

(c)   19.404 cu dm

(d)   1940.4 cu dm

Answer: (c)

138. A can do in one day three times the work done by B in one day. They together finish two-fifth of the work in 9 days. The number of days by which B can do the work alone is

(a)   90 days

(b)   120 days

(c)   100 days

(d)   30 days

Answer: (a)

139. In ∆ ABC, AD ⊥ BC and AD2 = BD ∙ The measure of ∠BAC is

(a)   60°

(b)   75°

(c)   90°

(d)   45°

Answer: (a)

140. Among the following statements, the statement which is not correct, is

(a)   Every natural number is an integer.

(b)   Every natural number is a real number.

(c)   Every real number is a rational number.

(d)   Every integer is a rational number.

Answer: (c)

141. In ∆ ABC, ∠B = 90°, AB = B cm and BC = 15 cm, then sin C = ?

(a)   15/17

(b)   8/17

(c)   15/8

(d)   8/15

Answer: (b)

Directions (Q. Nos. 142-145) The total expenditure of a company for a particular month is Rs 60000. The various heads of expenditure I to IV are indicated in a pie-chart given below.

These heads are

I. Raw materials

II. Conveyance

III. Overhead expenses

142. What percentage of total expenditure is on electricity?

(a)   23%

(b)   25%

(c)   30%

(d)   20%

Answer: (d)

143. What is the amount spent on overhead expenses?

(a)   Rs 12000

(b)   Rs 15000

(c)   Rs 18000

(d)   Rs 10000

Answer: (c)

144. Total expenditure on conveyance is

(a)   Rs 12000

(b)   Rs 15000

(c)   Rs 20000

(d)   Rs 10000

Answer: (b)

145. What percentage of total expenditure is on raw materials?

(a)   25%

(b)   30%

(c)   60%

(d)   23%

Answer: (a)

Directions (Q. Nos. 146-150) The following chart represents Demand and Production for 5 companies ABCDE.

On the basis of the graph, answer the following questions :

146. The ratio of the number of companies having more demand than production to those having more production than demand is

(a)   4 : 1

(b)   2 : 2

(c)   3 : 2

(d)   2 : 3

Answer: (c)

147. If company A desires to meet the demand by purchasing surplus production of a company, then the most suitable company i s

(a)   C

(b)   D

(c)   E

(d)   B

Answer: (b)

148. If x% of demand for company C equals demand for company B, then x equals

(a)   24

(b)   20

(c)   60

(d)   4

Answer: (a)

149. The different between average demand and average production of the five companies taken together is

(a)   400

(b)   280

(c)   130

(d)   620

Answer: (b)

150. If the production of company D is h times of the production of company A. Then h equals

(a)   1.5

(b)   2.5

(c)   1.2

(d)   1.8

Answer: (d)

Part-IV General Awareness

151. The amount of matter in a ball of steel is its

(a)   weight

(b)   mass

(c)   density

(d)   volume

Answer: (b)

152. A demand curve will not shift

(a)   when only income changes

(b)   when only prices substitute products change

(c)   when there is a change in advertisement expenditure

(d)   when only price of the commodity changes

Answer: (c)

153. ‘Earth Day’ is celebrated on

(a)   April 4

(b)   April 22

(c)   September 17

(d)   February 16

Answer: (b)

154. The Lalit Kala Academy is devoted to the promotion of

(a)   fine Arts

(b)   literature

(c)   music

(d)   dance and drama

Answer: (c)

155. Where did the Black-Hole tragedy took place?

(a)   Monghyr

(b)   Calcutta

(c)   Mushidabad

(d)   Dacca

Answer: (b)

156. In which of the Round Table Conference Mahatma Gandhi participated?

(a)   First Round Table Conference 1930

(b)   Second Round Table Conference 1931

(c)   Third Round Table Conference 1932

(d)   All of the above

Answer: (b)

157. All forms of ROM are also known as

(a)   Freeware

(b)   Middleware

(c)   Firmware

(d)   Shareware

Answer: (c)

158. Which Article of the Indian Constitution deals with Election Commission?

(a)   Article-352

(b)   Article-356

(c)   Article-360

(d)   Article-324

Answer: (d)

159. India born Vijai Sheshadri won the prestigious 2014 Pulitzer Prize in which of the following categories?

(a)   Music

(b)   Poetry

(c)   Drama

(d)   Journalism

Answer: (b)

160. P. Rama Rao Committee is related to which of the following?

(a)   Taxes

(b)   Defence

(c)   Industry

(d)   Agriculture

Answer: (b)

161. What is Kyoto Protocol?

(a)   It is an agreement among countries to take steps for reducing acid rain.

(b)   It is an agreement among countries to take steps for planting trees to control pollution.

(c)   It is an agreement among countries to start using nuclear energy.

(d)   It is an agreement among countries to take steps for reducing global warming.

Answer: (d)

162. The simplest CPU-scheduling algorithm is

(a)   SJF Scheduling algorithm

(b)   Roundrobin scheduling algorithm

(c)   Multilevel scheduling algorithm

(d)   FCFS scheduling algorithm

Answer: (d)

163. The ‘King of Metals’ is

(a)   gold

(b)   silver

(c)   iron

(d)   aluminium

Answer: (a)

164. The time element in price analysis was introduced by

(a)   JM Keynes

(b)   Alfred Marshall

(c)   JS Mill

(d)   JR Hicks

Answer: (b)

165. How much of world’s surface is covered by water?

(a)   55%

(b)   70%

(c)   80%

(d)   25%

Answer: (b)

166. Christmas factor is involved in

(a)   blood coagulation

(b)   excretion

(c)   digestion

(d)   respiration

Answer: (a)

167. Amino acids are required for the synthesis of

(a)   alkaloids

(b)   lipids

(c)   proteins

(d)   carbohydrates

Answer: (c)

168. Mixed Economy means

(a)   promoting both agriculture and Industries in the economy

(b)   co-existence of public and  private sectors

(c)   co-existence of rich and poor

(d)   co-existence of small and large Industries

Answer: (b)

169. Which of the following phenomenon helps to conclude that light is a transverse wave?

(a)   Interference

(b)   Diffraction

(c)   Polarization

(d)   Refraction

Answer: (c)

170. Which was the first Linguistic State to be created?

(a)   Kerala

(b)   Tamil Nadu

(c)   Andhra Pradesh

(d)   Maharashtra

Answer: (c)

171. Plank’s constant has the dimensions of

(a)   energy

(b)   linear momentum

(c)   angular momentum

(d)   force

Answer: (c)

172. ‘Rand’ is the currency of

(a)   Namibia

(b)   Iran

(c)   Romania

(d)   Norway

Answer: (a)

173. The first Biosphere Reserve in India has been established in

(a)   Kanha

(b)   Nilgiri

(c)   Nandadevi

(d)   Hazaribagh

Answer: (b)

174. Arundhati Roy is the author of

(a)   God of Small Things

(b)   Disgrace

(c)   The Tin Drum

(d)   My Childhood Days

Answer: (a)

175. Minamata disease is caused by pollution of water by

(a)   lead

(b)   tin

(c)   methyl isocyanate

(d)   mercury

Answer: (d)

176. Kimono is a dress style of which Asian country?

(a)   Japan

(b)   Korea

(c)   Laos

(d)   China

Answer: (a)

177. If the President wants to resign, he shall address his letter of resignation to

(a)   Prime Minister of India

(b)   Vice-President of India

(c)   Speaker of Lok Sabha

(d)   Chief Justice of India

Answer: (b)

178. The largest reservoir of fresh water is

(a)   glaciers

(b)   ground water

(c)   ponds

(d)   lakes

Answer: (a)

179. By whom was the autonomous investment separated from induced investment?

(a)   Schumpeter

(b)   Malthus

(c)   Joan Robinson

(d)   Adam Smith

Answer: (a)

180. Transpiration increases in

(a)   hot, dry and windy condition

(b)   hot, damp and windy condition

(c)   cool, damp and windy condition

(d)   cool, dry and still condition

Answer: (a)

181. When price of a substitute of commodity ‘x’ falls, the demand for ‘x’

(a)   falls

(b)   remains unchanged

(c)   increases at increasing rate

(d)   rises

Answer: (c)

182. The most abundant element by number in the living system is

(a)   nitrogen

(b)   hydrogen

(c)   oxygen

(d)   carbon

Answer: (a)

183. World Wildlife Fund was founded in

(a)   1965

(b)   1969

(c)   1992

(d)   1961

Answer: (d)

184. The headquarters o the Survey of India Department is located at

(a)   Jaipur

(b)   Dehradun

(c)   Hyderabad

(d)   New Delhi

Answer: (b)

185. Who built the famous Shiva temple at Ellora?

(a)   Rashtrakuta Ruler Krishna I

(b)   Mauryan Emperor Ashoka

(c)   Gupta King Samudragupta

(d)   Chalukyan King Pulikeshi II

Answer: (a)

186. Who is the author of the book ‘Romancing with Life’?

(a)   Kapil Dev

(b)   Dev Anand

(c)   Shashi Tharoor

(d)   Bill Clinton

Answer: (b)

187. Kathakali is a dance prevalent in which state?

(a)   Kerala

(b)   Andhra Pradesh

(c)   Tamil Nadu

(d)   Odisha

Answer: (a)

188. Blowing air with open pipe is an example of

(a)   Isothermal process

(b)   Isochoric process

(c)   Isobaric process

(d)   Adiabatic process

Answer: (c)

189. The most suitable unit for expressing nuclear radius is

(a)   nanometer

(b)   fermi

(c)   angstrom

(d)   micron

Answer: (b)

190. When was the ‘League of Nations’ established?

(a)   In 1918

(b)   In 1920

(c)   In 1939

(d)   In 1914

Answer: (b)

191. Iron filings can be separated from a heterogenous mixture using the technique of

(a)   Sublimation

(b)   Magnetization

(c)   Sedimentation

(d)   Evaporation

Answer: (b)

192. Who discovered the link between electricity and magnetism?

(a)   Volta

(b)   Maxwell

(c)   Diesel

(d)   Michael Faraday

Answer: (b)

193. Natural System of Classification was proposed by ……… botanists.

(a)   Indian

(b)   German

(c)   Swedish

(d)   British

Answer: (c)

194. Who composed the Allahabad Pillar inscription?

(a)   Harisena

(b)   Mahasena

(c)   Veerasena

(d)   Vishnusena

Answer: (a)

195. What is the number of players on each side in Rugby Football?

(a)   15

(b)   16

(c)   12

(d)   11

Answer: (d)

196. Which of the following scientist proved that the path of each planet around the Sun is elliptical?

(a)   Kepler

(b)   Galileo

(c)   Newton

(d)   Copernicus

Answer: (a)

197. Todar Mal,, he brilliant revenue officer served under

(a)   Sher Shah

(b)   Bhagwan Das

(c)   Humayun

(d)   Baz Bahadur

Answer: (a)

198. If xylem and phloem are arranged in the same radius, such a vascular bundle is called

(a)   Collateral

(b)   Bicollateral

(c)   Concentric

(d)   Radial

Answer: (a)

199. Which of the following rivers originates from Trans Himalayas?

(a)   Yamuna

(b)   Sindu

(c)   Saraswathi

(d)   Ganga

Answer: (b)

200. Among the following districts of Tamil Nadu, which district is unfit for cultivation due to increased salinity?

(a)   Coimbatore

(b)   Tiruchirapalli

(c)   Nagapattinam

(d)   Ramanthapuram

Answer: (c)

Staff Selection Commission (SSC) Combined Higher Secondary (10+2) Level Recruitment Examination Held on December 20, 2015 Question Paper With Answer Key

Staff Selection Commission (SSC) Combined Higher Secondary (10+2) Level Recruitment Examination Held on December 20, 2015
Staff Selection Commission (SSC) Combined Higher Secondary (10+2) Level Recruitment Examination Held on December 20, 2015 Question Paper With Answer Key

Staff Selection Commission (SSC) Combined Higher Secondary (10+2) Level Recruitment Examination Held on December 20, 2015

Part-I General Intelligence

Directions (Q. Nos. 1-5) In the following questions, a series is given, with one term missing. Choose the correct alternative from t he given ones that will complete the series.

1. 1001, 1004, 1012, 1027, ?

(a)   1051

(b)   1050

(c)   1036

(d)   1048

Answer: (a)

2. HIJPQRDEF ? VW

(a)   H

(b)   U

(c)   N

(d)   J

Answer: (b)

3. 8, 13, 18, 23, ?, 33, 38

(a)   23

(b)   26

(c)   28

(d)   33

Answer: (c)

4. 8, 24, ?, 80, 120

(a)   72

(b)   48

(c)   40

(d)   54

Answer: (b)

5. AC, EG, IK, ?

(a)   LM

(b)   OP

(c)   ON

(d)   MO

Answer: (d)

Directions (Q. Nos. 6-7) In the following questions, two statements are given each followed by two conclusions assumptions, I and II. You have to consider the statement to be true even if they seem to be at variance from commonly known fats. You have to decide which of the given conclusions/ assumptions, if any follows from the given statements.

6. Statements

1. The principal will address the students at 10 am.

2. You are requested to take your seats before 10 am.

Assumptions

I. If the student is not on his seat before 10 am, the function will not start.

II. The function will start as scheduled.

(a)   Both assumptions are implicit.

(b)   Assumption I is implicit.

(c)   Neither assumption I nor II is implicit.

(d)   Assumption II is implicit.

Answer: (d)

7. Statements

1. Regular polygon has equal sides and equal angles.

2. Square is a regular polygon.

Conclusions

I. Square has equal sides

II. Square has equal angles.

(a)   Neither conclusion I nor II follows

(b)   Both conclusions follow

(c)   Conclusion II follows

(d)   Conclusion I follows

Answer: (b)

8. Which one set of letters when sequentially placed at the gaps in the given letter series shall complete it?

aac_bba_cc_baa_cb_

(a)   bacbc

(b)   cabcb

(c)   abaac

(d)   bcacb

Answer: (b)

Directions (Q. Nos. 9-17) In the following questions, find the odd word/number/letters/number pair from the given alternatives.

9. Find the odd word.

(a)   Googly

(b)   Swing

(c)   Yorker

(d)   Bouncer

Answer: (d)

10. Find the odd letters.

(a)   MONR

(b)   VNHK

(c)   NQMJ

(d)   NHJM

Answer: (a)

11. Find the odd letters.

(a)   CD

(b)   BD

(c)   AC

(d)   DF

Answer: (a)

12. Find the odd word.

(a)   Humiliate

(b)   Elope

(c)   Degrade

(d)   Abase

Answer: (b)

13. Find the odd word.

(a)   President

(b)   MLA

(c)   Prime Minister

(d)   Minister

Answer: (a)

14. Find the odd letters.

(a)   LNP

(b)   EGI

(c)   BDE

(d)   MOQ

Answer: (c)

15. Find the odd number

(a)   16

(b)   56

(c)   2

(d)   128

Answer: (a)

16. Find the odd number.

(a)   47632

(b)   84129

(c)   32418

(d)   67626

Answer: (a)

17. Find the odd number.

(a)   49

(b)   100

(c)   65

(d)   36

Answer: (c)

18. A was twice as old as B, two years ago. If the difference in their ages be 2 years, find A’s age.

(a)   10

(b)   6

(c)   4

(d)   8

Answer: (b)

19. Which one of the given responses would be a meaningful order of the following ?

1. Stone 2. Sand 3. Rock       4. Boulder   5. Hill

(a)   1, 4, 2, 3, 5

(b)   5, 4, 2, 1, 3

(c)   2, 1, 3, 4, 5

(d)   5, 3, 2, 1, 4

Answer: (c)

Directions (Q. Nos. 20-28) In the following questions, select the related word/letters/number from the given alternatives.

20. Bullock : Cart : : Horse : ?

(a)   Tonga

(b)   Plough

(c)   Ride

(d)   Race

Answer: (a)

21. Motor : Coil : : ?

(a)   Wheel : Bearing

(b)   Regiment : Soldier

(c)   Wheel : Car

(d)   Table : Chair

Answer: (a)

22. Blacksmith : Hammer : : ?

(a)   Surgeon : Forceps

(b)   Cloth  : Tailor

(c)   Carpenter : Wood

(d)   Pen : Writer

Answer: (a)

23. CAT : DDY : : BIG : ?

(a)   CLL

(b)   CEP

(c)   CML

(d)   CLM

Answer: (a)

24. EAGLE : FCJPJ : : CRAWL : ?

(a)   DTDAQ

(b)   DTEZQ

(c)   DTFCR

(d)   DTDBRC

Answer: (a)

25. CEGI : DHLP :: KSEA : ?

(a)   LSGF

(b)   LTGI

(c)   LUIH

(d)   LVJH

Answer: (d)

26. 4 : 19 : : 7 : ?

(a)   49

(b)   68

(c)   28

(d)   52

Answer: (d)

27. 4863 – 2436 : : 8132 – 4261 : :

(a)   8346 – 4623

(b)   5926 – 1813

(c)   6214 – 3122

(d)   9842 – 8421

Answer: (d)

28. 558 : 186 : : 951 : ?

(a)   317

(b)   176

(c)   286

(d)   217

Answer: (a)

29. Suresh walked 7 km East and turned to his left walked 4 km. He then turned to his right and walked 5 km. Finally, he again turned to his right and walked 4 km. In which direction is he now, from his starting point?

(a)   East

(b)   North

(c)   West

(d)   South

Answer: (a)

30. Which combination figure best represents the relationship between Mosquitoes, Ants and Insects ?

(a)   a

(b)   b

(c)   c

(d)   d

Answer: (c)

Directions (Q. Nos. 31-34) In the following questions, select the missing number from the given responses.

31. 

(a)   72

(b)   18

(c)   42

(d)   56

Answer: (a)

32. 

(a)   8

(b)   9

(c)   14

(d)   12

Answer: (d)

33. 

(a)   30

(b)   32

(c)   40

(d)   50

Answer: (d)

34. 

(a)   109, 111, 117

(b)   114, 111, 117

(c)   111, 114, 117

(d)   117, 109, 111

Answer: (c)

Directions (Q. Nos. 35-36) In the following questions, from the given alternative words, select the word which cannot  be formed using the letters of the given word.

35. Synthesis

(a)   Thesis

(b)   Then

(c)   Story

(d)   This

Answer: (c)

36. Manipulate

(a)   Mantle

(b)   Pint

(c)   Pollute

(d)   Plate

Answer: (c)

37. If FATHER is coded as FBTIES what should be code for the word SISTER?

(a)   SKSVET

(b)   TJTUFS

(c)   SHSSEQ

(d)   SJSUES

Answer: (d)

38. Which answer figure will complete the pattern in the question figure?

(a)   a

(b)   b

(c)   c

(d)   d

Answer: (c)

39. Eight friends A, B, C, D, E, F, G and H are sitting around a round table in the same order at equal distances. Their positions are in clockwise direction. If G, who is sitting in the North exchange seat with C, and B exchange seat with F. Now who is sitting to the right of F?

(a)   A

(b)   G

(c)   E

(d)   B

Answer: (a)

40. If 0, 1, 2, 3, ………… 9 is written as a, b, c, d, ………. j then find dc × f – (bf – d) × d

(a)   bce

(b)   abb

(c)   abe

(d)   bcf

Answer: (a)

41. A word is represented by only one set of numbers as given in any one of the alternatives. The set of numbers given in the alternatives are represented by two classes of alphabets as in two matrices given below. The columns and row of Matrix I are numbered from 0 to 4 and that Matrix II are numbered from 5 to 9. A letter from these matrices can be represented first by its row and next by its column e.g., a can be represented by 02, 11 etc, and L can be represented by 56, 67 etc. Similarly, you have to identify the set for the word BEARD.

(a)   88, 87, 43, 21, 13

(b)   88, 13, 43, 44, 21

(c)   87, 13, 43, 21, 88

(d)   87, 13, 43, 88, 21

Answer: (c)

42. If a paper is folded in a particular manner and punch is made, when unfolded this paper appears as given below in the question figure. Find out the manner in which the paper is folded and the punch is made from the answer figures given.

(a)   a

(b)   b

(c)   c

(d)   d

Answer: (a)

43. In a certain code CONVENTIONAL is written as NOCNEVOITLAN. How is ENTHRONEMENT in that code written?

(a)   TNEROHEMNNTE

(b)   TNEORMETNHNE

(c)   TNEORHMENTNE

(d)   TNEROHEMNTNE

Answer: (c)

Directions (Q. Nos. 44-45) In the following questions, arrange the following words as per order in the dictionary.

44. 1. Electrolysis 2. Electrotyping

3. Electrician 4. Electroplating

5. Electrification

(a)   5,3, 2, 4, 1

(b)   5, 3, 1, 4, 2

(c)   3, 5, 4, 1, 2

(d)   3, 5, 1, 4, 2

Answer: (d)

45. 1. Yashmak 2. Yacht

3. Yawl 4. Yawn

(a)   2, 1, 3, 4

(b)   2, 1, 4, 3

(c)   2, 4, 3, 1

(d)   2, 3, 4, 1

Answer: (a)

46. From the given answer figures, select the one in which the question figure is hidden/ embedded.

(a)   a

(b)   b

(c)   c

(d)   d

Answer: (a)

47. Identify the answer figure from which the pieces given in the question figure have been cut.

(a)   a

(b)   b

(c)   c

(d)   d

Answer: (b)

48. If a mirror is placed on the line MN, then which of the answer figures is the right image of the given figure?

(a)   a

(b)   b

(c)   c

(d)   d

Answer: (d)

49. If in a business, Along gains 75% more profit than Akash, then by what percentage profit of Akash is less than the profit of Alok?

(a)   30.8%

(b)   42.85%

(c)   12.63%

(d)   25%

Answer: (b)

50. If # means <; O means >; □ means =, then which the following follows definitely from a O b # c □ d?

(a)   b # d

(b)   b O d

(c)   a □ c

(d)   b □ d

Answer: (a)

Part-II English Language

Directions (Q. Nos. 51-52) In the following questions, a sentences has been given, in Direct/Indirect. Out of the four alternatives suggested, select the one which best expresses the same sentence in Indirect/Direct.

51. Ravi asked me, “Have you seen the Taj Mahal?”

(a)   Ravi asked me if I had seen the Taj Mahal.

(b)   Ravi asked me did I see the Taj Mahal.

(c)   Ravi asked me if I have seen the Taj Mahal.

(d)   Ravi asked me if I saw the Taj Mahal.

Answer: (a)

52. Robin said, “Hurrah ! I have topped again.”

(a)   Robin exclaimed joyfully that he had been topped again

(b)   Robin exclaimed joyfully that he has topped again.

(c)   Robin exclaimed joyfully that he would have topped again

(d)   Robin exclaimed joyfully that he had topped again.

Answer: (d)

Directions (Q. Nos. 53-62) In the following questions, in the following passage some of the words have been left out. Read the passage carefully and choose the correct answer to each question out of the four alternatives and fill in the blanks.

Smile, they say, and soon there will be miles and miles of smiles. If we keep (i) ourselves and do not mix with others, we shall be left alone to ponder (ii) the misfortunes of life. Nobody likes to come across a (iii) and self-centred person. People (iv) gregarious and outgoing souls who are  prepared to share their joys and sorrows and have the (v) to laugh (vi) their problems and miseries. Laughter brings people (vii) whereas keeping to oneself distances people. It has (viii) been rightly said that laughter is the shortest distance between two persons. Once they (ix) together, the circle of acquaintance, and consequently friendship (x), thus making the world a happy place to live in.

53. (iv)

(a)   hate

(b)   like

(c)   opt

(d)   dislike

Answer: (b)

54. (i)

(a)   to

(b)   into

(c)   with

(d)   by

Answer: (c)

55. (ix)

(a)   come

(b)   go

(c)   are

(d)   sit

Answer: (d)

56. (iii)

(a)   glum

(b)   kind

(c)   unselfish

(d)   comic

Answer: (a)

57. (x)

(a)   broadens

(b)   grows

(c)   widens

(d)   lessens

Answer: (b)

58. (vii)

(a)   home

(b)   apart

(c)   closer

(d)   far

Answer: (c)

59. (v)

(a)   intension

(b)   capacity

(c)   feeling

(d)   sense

Answer: (b)

60. (ii)

(a)   over

(b)   upon

(c)   into

(d)   at

Answer: (a)

61. (viii)

(a)   however

(b)   so

(c)   hence

(d)   therefore

Answer: (d)

62. (vi)

(a)   over

(b)   on

(c)   away

(d)   at

Answer: (d)

Directions (Q. Nos. 63-66) In the following questions, out of the four alternatives, choose the one which best expresses the meaning of the given word.

63. Filthy

(a)   Immaculate

(b)   Dirty

(c)   Clean

(d)   Spotless

Answer: (b)

64. Acquire

(a)   Lose

(b)   Renounce

(c)   Relinquish

(d)   Procure

Answer: (d)

65. Mistake

(a)   Precise

(b)   Mistook

(c)   Error

(d)   Accurate

Answer: (c)

66. Annoy

(a)   Lazy

(b)   Revoke

(c)   Offend

(d)   Refuse

Answer: (c)

Directions (Q. Nos. 67-70) In the following questions, choose the word opposite in meaning to the given word.

67. Accumulate

(a)   Dismiss

(b)   Aware

(c)   Disperse

(d)   Below

Answer: (c)

68. Resist

(a)   Remain

(b)   Decline

(c)   Yield

(d)   Adjust

Answer: (c)

69. Concur

(a)   Disagree

(b)   Praise

(c)   Weak

(d)   Agree

Answer: (a)

70. Elementary

(a)   Compound

(b)   Complex

(c)   Hard

(d)   Involved

Answer: (b)

Directions (Q. Nos. 71-74) In the following questions, some parts of the sentence have errors and some are correct. Find out which part of a sentence has an error and corresponding to the appropriate correct option. If a sentence is free from error, then given answer ‘No error’.

71. One should have their teeth checked every six months.

(a)   every six months

(b)   one should have

(c)   No error

(d)   their teeth checked

Answer: (d)

72. It is painful to saw that some youngsters are killing time without doing any thing useful.

(a)   killing time without doing anything useful

(b)   It is painful to saw that

(c)   No error

(d)   some youngsters are

Answer: (b)

73. A honest person is upright in speech, thought and deed.

(a)   in speech, thought and deed

(b)   No error

(c)   A honest person

(d)   is upright

Answer: (c)

74. We, in India can look forward to a comfortable and settle life in the twenty-first century.

(a)   to a comfortable and settle

(b)   No error

(c)   We, in India can look forward

(d)   life in the twenty-first century

Answer: (a)

Directions (Q. Nos. 75-78) In the following questions, out of the four alternatives, choose the one which can be substituted for the given words/sentences.

75. The first public performance of a musical or theatrical work or the first showing of a film

(a)   Debut

(b)   Preview

(c)   Premiere

(d)   Opener

Answer: (c)

76. An act of misappropriation of money

(a)   Debasement

(b)   Embezzlement

(c)   Corruption

(d)   Misconduct

Answer: (b)

77. One who cannot make any mistake

(a)   Illegible

(b)   Infallible

(c)   Invisible

(d)   Inaudible

Answer: (b)

78. A person is charge of a museum

(a)   Curator

(b)   Mayor

(c)   Philatelist

(d)   Architect

Answer: (a)

Directions (Q. Nos. 79-82) In the following questions, sentences are given with blanks to be filled in with an appropriate word (s). Four alternatives are suggested for each question. Choose the correct alternative out of the four.

79. The terrorists made a vain attempt to ……….. the bridge.

(a)   blow over

(b)   blow up

(c)   blow down

(d)   blow out

Answer: (b)

80. Politics is the last ……….. of scoundrels.

(a)   retort

(b)   result

(c)   resort

(d)   report

Answer: (c)

81. The speeding car …….. a man this morning.

(a)   pushed out

(b)   hit out

(c)   knocked down

(d)   hit about

Answer: (c)

82. Take care ………. your health.

(a)   about

(b)   of

(c)   over

(d)   for

Answer: (b)

Directions (Q. Nos. 83-86) In the following questions, the first and the last part of sentence are numbered 1 and 6. The rest of the sentence is split into four  parts and named P, Q, R and S. These four parts are not given in their proper order. Read the sentence and find out which of the four combinations is correct. Then find the correct answer.

83. 1. All religions show us the path

P. know that

Q. and wisemen

R. God is one

S. which leads to God

6. but the paths are different

(a)   SQPR

(b)   SRQP

(c)   QSPR

(d)   QPRS

Answer: (a)

84. 1. Television

P.  has become

Q. the most powerful medium

R. one of the marvels

S. of modern science

6. of mass communication today

(a)   RSQP

(b)   PQRS

(c)   SRQP

(d)   RSPQ

Answer: (d)

85. 1. The Narmada Dam Project

P. many of whom

Q. the most deprived sections

R. will displace some people

S. happen to belong to some of

6. of Indian society

(a)   RPSQ

(b)   PSRQ

(c)   PQRS

(d)   SRQP

Answer: (a)

86. 1. While

P. I was walking

Q. with my brother

R. I met a monk

S. in the field

6. who was from Nepal.

(a)   RQSP

(b)   RSQP

(c)   PQSR

(d)   PSQR

Answer: (d)

Directions (Q. Nos. 87-90) In the following questions, four alternatives are given for the Idiom/Phrase underlined. Choose the alternative which best expresses the meaning of the Idiom/Phrase.

87.  This argument does not hold water.

(a)   seem approvable

(b)   seem logical

(c)   seem acceptable

(d)   seem rejectable

Answer: (b)

88. To be down to Earth.

(a)   To be realistic

(b)   To be unreasonable

(c)   To be impractical

(d)   To be pretentious

Answer: (a)

89. The truant school boy told cock-and-bull stories to escape punishment.

(a)   drab and boring

(b)   authentic and realistic

(c)   interesting and thrilling

(d)   absurd and unbelievable

Answer: (d)

90. Rohini reached the examination hall in the nick of time.

(a)   just in time

(b)   just before time

(c)   just after time

(d)   very late

Answer: (a)

Directions (Q. Nos. 91-94) In the following questions, a part of the sentence is underlined. Below are given alternatives to the underlined part which may improve the sentence. Choose the correct alternative. In case no improvement is needed give the answer option ‘No improvement’.

91. The President of India administers the oath of office to the Council of Ministers.

(a)   reads out the oath

(b)   supervises the oath taking ceremony

(c)   No improvement

(d)   imposes the promise

Answer: (b)

92. Being a wet day, he stayed at home.

(a)   Since

(b)   No improvement

(c)   Though

(d)   As it was

Answer: (d)

93. I shall write to you when I reach Agra.

(a)   will write to you

(b)   should write to you

(c)   No improvement

(d)   would write to you

Answer: (c)

94. We’ve got a new big house.

(a)   big new house

(b)   No improvement

(c)   house new big

(d)   house big new

Answer: (a)

Directions (Q. Nos. 95-98) In the following questions, four words are given in each question, out of which only one word is correctly spelt. Find the correctly spelt word.

95.

(a)   Alkali

(b)   Alkale

(c)   Alkeli

(d)   Alkalie

Answer: (a)

96.

(a)   Amateur

(b)   Amateaur

(c)   Amature

(d)   Amatuer

Answer: (a)

97.

(a)   Decisive

(b)   Deciseev

(c)   Decesieve

(d)   Decesive

Answer: (a)

98.

(a)   Milennium

(b)   Millennium

(c)   Millenium

(d)   Millanium

Answer: (b)

Directions (Q. Nos. 99-100) In the following questions, a sentence has been given in Active/Passive Voice. Out of the four alternatives suggested, select the one which best expresses the same sentence in Passive/Active Voice.

99. Put up the tent.

(a)   Let the tent be put up.

(b)   The tent has been put up.

(c)   Let the tent being put up.

(d)   The tent is being put up.

Answer: (a)

100. The agreement between the management and the workers was drafted by an advisory board.

(a)   An advisory board is drafting the agreement between the management and the workers.

(b)   An advisory board had drafted the agreement between the management and the workers.

(c)   The agreement between the management and the workers is being drafted by an advisory board.

(d)   An advisory board drafted the agreement between the management and the workers.

Answer: (d)

Part-III Quantitative Aptitude

101. In a class, there are 30 boys and their average are is 17 years. On one boy aged 18 years leaving class and another joining, the average age becomes 16.9 years. The age of the new boy is

(a)   15 years

(b)   25 years

(c)   11 years

(d)   13 years

Answer: (a)

102. A man given 50% of his money to his son and 30% to his daughter. 80% of the rest is donated to a trust. If he left with Rs 16000 now, how much money did he have in the beginning ?

(a)   Rs 800000

(b)   Rs 80000

(c)   Rs 400000

(d)   Rs 40000

Answer: (c)

Directions (Q. Nos. 103-107) The bar graphs is representing marks of a student in different subjects. Study the bar graph and answer the questions.

103. In which subject the student is poor?

(a)   English

(b)   Science

(c)   Mathematics

(d)   History

Answer: (d)

104. What is the average percentage obtained by the student?

(a)   57%

(b)   80%

(c)   90%

(d)   63%

Answer: (a)

105. What is the ratio of the highest marks to the lowest marks obtained by the student?

(a)   9 : 2

(b)   2 : 9

(c)   2 : 11

(d)   11 : 2

Answer: (a)

106. At what subject the student is sharp ?

(a)   Science

(b)   History

(c)   English

(d)   Mathematics

Answer: (d)

107. What are the average marks obtained by the student?

(a)   63

(b)   80

(c)   48

(d)   57

Answer: (d)

108. If sin (90° − θ) + cos θ = √2 cos(90° −θ), then the value of cosec, θ, is

(a)   2/3

(b) 

(c)   1/√2

(d)   1/√3

Answer: (b)

109. If ABCD is a cyclic quadrilateral with ∠A = 50°, ∠B = 80°, then ∠C and ∠D are

(a)   100°, 130°

(b)   130°, 100°

(c)   110°, 120°

(d)   115°, 115°

Answer: (b)

Directions (Q. Nos. 110-113) Revenue earned by the Central government is given in pie-chart. Study the pie-chart and answer the questions.

110. The ratio of revenue earned from Excise Duty and Customs Duty to that of others, is

(a)   11 : 5

(b)   11 : 3

(c)   11 : 4

(d)   11 : 2

Answer: (d)

111. If the income of Central government from Excise Duty Rs 28618 crore, then the total revenue earned by the

(a)   Rs 69800 crore

(b)   Rs 58900 crore

(c)   Rs 45600 crore

(d)   Rs 47200 crore

Answer: (a)

112. If the percentage of revenue earned by the Central government from Corporation Tax is x times to that of the percentage of money earned from Excise Duty, then the value of x, is

(a)   41/9

(b)   9/41

(c)   14/41

(d)   41/14

Answer: (b)

113. The ratio of revenue of the Central government earned from the Customs Duty and Income Tax, is

(a)   7 : 13

(b)   9 : 26

(c)   14 : 41

(d)   10 : 9

Answer: (a)

114. 51% of a whole number is 714.25% of that number, is

(a)   450

(b)   550

(c)   350

(d)   250

Answer: (c)

115. If (2, 0) is a solution of the linear equation 2x = 3y = K, then the value of K is

(a)   4

(b)   5

(c)   2

(d)   6

Answer: (a)

116. The list price of an electric fan is Rs 300. If two successive discounts of 15% and 10% are allowed its selling price would be

(a)   Rs 227.50

(b)   Rs 229.50

(c)   Rs 225

(d)   Rs 230

Answer: (b)

117. If cos 20° = m and cos 70° = n then the value of m2 + n2, is

(a)   1

(b)   3/2

(c)   1/√2

(d)   1/2

Answer: (a)

118. The side BC of a triangle ABC is proceed to D. If ∠ACD = 112° and then the measure of ∠B, is

(a)   48°

(b)   64°

(c)   30°

(d)   45°

Answer: (a)

119. A and B invest in a business in the ratio 3 : 2. If 5% of the total profit goes to charity and A’s share in profit is Rs 8550, then total profit is

(a)   Rs 15000

(b)   Rs 15760

(c)   Rs 14250

(d)   Rs 15735

Answer: (a)

120. After allowing 15% discount, the selling price of radio becomes Rs 255. The marked price, is

(a)   Rs 400

(b)   Rs 300

(c)   Rs 600

(d)   Rs 500

Answer: (b)

121. The number between 4000 and 5000 that is divisible by each of 12, 18, 21 and 32, is

(a)   4032

(b)   4302

(c)   4023

(d)   4203

Answer: (a)

122. The graph of linear equation y = x passes through the point

(a)   (3/2, −3/2)

(b)   (0, 3/2)

(c)   (1, 1)

(d)   (−1/2, 1/2)

Answer: (c)

123. The medians CD and BE of a triangle ABC intersect each other at O. The ratio of ∆ ODE : ∆ ABC is equal to

(a)   3 : 4

(b)   12 : 1

(c)   4 : 3

(d)   1 : 12

Answer: (a)

124. The difference between the greatest and least prime numbers which are less than 100, is

(a)   95

(b)   97

(c)   96

(d)   94

Answer: (a)

125. The rate of simple interest per annum of bank being decreased from 5% to  the annual income of a person from interest was less by Rs 105. The sum deposited at the bank was

(a)   Rs 7000

(b)   Rs 7200

(c)   Rs 6800

(d)   Rs 6000

Answer: (a)

126. If  then the value of cos α, is

(a)   0

(b)   1/√2

(c)   √3/2

(d)   1/2

Answer: (d)

127. A cuboidal shaped water tank, 2.1 m long and 1.5 m broad is half filled with water. If 630 L more water is poured into that tank, the water level will rise

(a)   2 cm

(b)   0.15 cm

(c)   0.18 cm

(d)   0.20 m

Answer: (d)

128. If the sum of all interior angles of a regular polygon is 14 right angles, then its number of sides, is

(a)   7

(b)   6

(c)   9

(d)   8

Answer: (c)

129. Height of a prism shaped part of a machine is 8 cm and its base is an isosceles triangle, whose each of the equal sides is 5 cm and remaining side is 6 cm. The volume of part, is

(a)   96 cm2

(b)   120 cm2

(c)   86 cm2

(d)   90 cm2

Answer: (a)

130. Two numbers are in ratio 5 : 8. If their difference is 48, then the smallest number, is

(a)   64

(b)   80

(c)   128

(d)   96

Answer: (b)

131. The simple interest on Rs 36000 for the period from January 5 to May 31, 2013 at 9.5% per annum, is

(a)   Rs 1400

(b)   Rs 1368

(c)   Rs 1338

(d)   Rs 1425

Answer: (*)

132. The measure of each interior angle of a regular polygon with 8 sides, is

(a)   135°

(b)   100°

(c)   45°

(d)   120°

Answer: (a)

133. If  then the value of  is

(a) 

(b) 

(c) 

(d) 

Answer: (a)

134. If  then the value of sin A ∙ cos A, is

(a)   3/2

(b)   √3/4

(c)   √3/2

(d)   1/√3

Answer: (b)

135. Two trains start at the same time from Aligarh and Delhi and proceed towards each other at the rate of 14 km and 21 km per h our respectively. When they meet, it is found that one train has travelled 70 km more than the other. The distance between two stations, is

(a)   140 km

(b)   210 km

(c)   350 km

(d)   300 km

Answer: (c)

136. Janardan completes 3/5 of his work in 10 days. Time he will take to complete 3/5 of the same work, is

(a)   6 days

(b)   8 days

(c)   4 days

(d)   9 days

Answer: (d)

137. A shopkeeper sold an article at a loss 20%. But if he could sell it at Rs 200 more, he could earn a profit of 5%. The cost price of the article, is

(a)   Rs 1000

(b)   Rs 1200

(c)   Rs 800

(d)   Rs 600

Answer: (c)

138. By melting two solid metallic spheres of radii 1 cm and 6 cm, a hollow sphere of thickness 1 cm is made. The external radius of the hollow sphere will be

(a)   9 cm

(b)   7 cm

(c)   6 cm

(d)   8 cm

Answer: (a)

139. If A : B = 2 : 1 and A : C = 1 : 3, A : B : C =

(a)   1 : 3 : 2

(b)   3 : 2 : 1

(c)   2 : 1 : 6

(d)   1 : 2 : 6

Answer: (c)

140. The value of cos 1° cos 2° cos 3° …. cos 180°, is

(a)   0

(b)   1/2

(c)   1

(d) 

Answer: (a)

141. The average marks of 50 students in a class is 72. The average marks of boys and girls in that subject are 70 and 75 respectively. The number of boys in the class, is

(a)   25

(b)   20

(c)   35

(d)   30

Answer: (d)

142. The hypotenuse of a right angled triangles is 39 cm and the difference of other two sides is 21 cm. Then, the area of the triangle is

(a)   180 sq cm

(b)   540 sq cm

(c)   450 sq cm

(d)   270 sq cm

Answer: (d)

143. If the compliment of an angle is one-fourth of its supplementary angle, then the angle, is

(a)   90°

(b)   30°

(c)   60°

(d)   120°

Answer: (c)

144. A number between 1000 and 2000 which when divided by 30, 36 and 80 gives a remainder 11 in each case, is

(a)   1712

(b)   1451

(c)   1641

(d)   1523

Answer: (b)

145. The value of the expression

is

(a)   30

(b)   5

(c)   3

(d)   2

Answer: (c)

146. Two trains of length 150 m and 180 m respectively are running in opposite directions on parallel tracks. If their speeds be 50 km/h and 58 km/h respectively, in what time will they cross each other?

(a)   11 sec

(b)   30 sec

(c)   15 sec

(d)   22 sec

Answer: (a)

147. By selling 12 kg of potatoes for Rs 63, shopkeeper gains 5%. What does his gain or lose percent by selling 50 kg of the same potatoes for Rs 247.50

(a)   2.5% loss

(b)   1% loss

(c)   1% profit

(d)   No  profit no lass

Answer: (b)

148. Arrangement of the fractions  into ascending order are

(a) 

(b) 

(c) 

(d) 

Answer: (a)

149. Work done by (x + 4) men in (x + 5) day to equal to the work done by (x – 5) men in (x + 20) days. Then the value of x, is

(a)   20

(b)   30

(c)   25

(d)   15

Answer: (a)

150. If  then the value of  is

(a)   132

(b)   122

(c)   110

(d)   123

Answer: (d)

Part-IV General Awareness

151. Kathakali dance from is associated with which State?

(a)   Manipur

(b)   Kerala

(c)   Tamil Nadu

(d)   Andhra Pradesh

Answer: (b)

152. Who was the founder of the Indian National Army?

(a)   Bal Gangadhar Tilak

(b)   Gandhiji

(c)   Subhas Chandra Bose

(d)   Nehru

Answer: (c)

153. Who won Australian Open mixed doubles title 2015?

(a)   Murray (UK) and Maria Sharapova (Russia)

(b)   Djokovic (Serbia) and Serena Williams (USA)

(c)   Kristina Mladenovic (France) and Daniel Nester (Canada)

(d)   Martina Hingis (Switzerland) and Leander Paes (India)

Answer: (d)

154. The number of hydrogen bonds between guanine and cytosine in DNA, is

(a)   3

(b)   2

(c)   1

(d)   4

Answer: (a)

155. SI unit of magnetic flux, is

(a)   Weber/m

(b)   Weber-m2

(c)   Weber

(d)   Weber/m4

Answer: (c)

156. The State with highest production of wheat in year 2012-13, is

(a)   Tamil Nadu

(b)   West Bengal

(c)   Uttar Pradesh

(d)   Haryana

Answer: (c)

157. RK Laxman died on January 26, 2016. He was a leading ……. of India.

(a)   Space scientist

(b)   Cartoonist

(c)   Playback singer

(d)   Kathakali dancer

Answer: (b)

158. As per Census of India, an urban area with a population from 50000 to 99999 is named as

(a)   Class III town

(b)   Class II town

(c)   Class I town

(d)   Class IV town

Answer: (b)

159. Which gas emitted by power stations causes acid rain?

(a)   Carbon dioxide

(b)   Helium

(c)   Sulphur dioxide

(d)   Nitrogen

Answer: (c)

160. Which of the following colour of light deviates least through the prism?

(a)   Red

(b)   Green

(c)   Yellow

(d)   Violet

Answer: (a)

161. The reduce tooth decay most toothpastes contain a

(a)   chloride

(b)   iodide

(c)   fluoride

(d)   bromide

Answer: (c)

162. The term GIGO is related to

(a)   accuracy

(b)   flexibility

(c)   automatic

(d)   versatility

Answer: (a)

163. The process of arranging data in logical sequence is called

(a)   Summarising

(b)   Sorting

(c)   Reproducing

(d)   Classifying

Answer: (b)

164. The part of brain which controls emotional reactions in our body is

(a)   hypothalamus

(b)   cerebrum

(c)   meninges

(d)   thalamus

Answer: (a)

165. An ordinance issued by the Governor has to be passed by the assembly within

(a)   6 weeks

(b)   12 weeks

(c)   10 weeks

(d)   8 weeks

Answer: (a)

166. An organism that transmits disease from one individual to another is called

(a)   Hybrid

(b)   Vector

(c)   Fragment

(d)   Clone

Answer: (b)

167. Which of the following is true with reference to blood platelets?

(a)   They are also called thromobocytes.

(b)   They have prominent nuclei.

(c)   The are involved in phagocytosis.

(d)   They have a pigment called Hamoglobin.

Answer: (a)

168. Seismography is the science of

(a)   mountains

(b)   rivers

(c)   earthquakes

(d)   volcanoes

Answer: (c)

169. A demand curve, which is parallel to the horizontal axis, showing quantity, has the price elasticity equal to

(a)   one

(b)   infinity

(c)   zero

(d)   less than one

Answer: (c)

170. Which decade is called as the Era of Decolonization?

(a)   1970 s

(b)   1990 s

(c)   1980 s

(d)   1850 s

Answer: (d)

171. Who is the only second Vice-President of India to get a second consecutive term after S. Radhakrishnan?

(a)   MH Ansari

(b)   KR Narayanan

(c)   Dr. Shankar Dayal Sharma

(d)   BS Shekhawat

Answer: (a)

172. The basic unit of Biosystematics, is

(a)   genotype

(b)   ecotype

(c)   phenotype

(d)   florotype

Answer: (a)

173. A semi enclosed coastal body of water which has a free connection with the open sea is called

(a)   Estuary

(b)   Ria coast

(c)   Fjord

(d)   Cove

Answer: (d)

174. Who is the first Indian woman to win an Asian Games gold in 400 m runs?

(a)   PT Usha

(b)   Kamaljit Sandhu

(c)   Saina Mirza

(d)   Shiny Abraham

Answer: (b)

175. Name the Republic, which was a confederacy of tribes in the 6th century BC.

(a)   Nek Chand

(b)   Edward Baker

(c)   Khuswant Singh

(d)   Charles Corbusier

Answer: (c)

176. Who was the creator of the famous Rock Garden of Chandigarh?

(a)   Nek Chand

(b)   Edward Baker

(c)   Khuswant Singh

(d)   Charles Corbusier

Answer: (a)

177. Which Indian fought the Kalinga War?

(a)   Ashoka

(b)   Shivaji

(c)   Samudragupta

(d)   Chandragupta Maurya

Answer: (a)

178. Name the two research stations maintained by India in Antarctica.

(a)   Gangotri and Himadri

(b)   Sagar Nidhi and Yamunotri

(c)   Maitri and Bharti

(d)   None of the above

Answer: (c)

179. Who has authored the book ‘The Kingdom of God is Within You’?

(a)   Leo Tolstoy

(b)   Mahatma Gandhi

(c)   John Ruskin

(d)   Henry David

Answer: (a)

180. What is the major cause of ‘October Heat’?

(a)   Hot and dry weather

(b)   Very low velocity winds

(c)   Low pressure systems over the Indo-Gangetic plains

(d)   High temperature associated with high humidity

Answer: (d)

181. What is the famous Chipko movement associated with?

(a)   Saving the tigers

(b)   Saving the wetland

(c)   None of the above

(d)   Trees

Answer: (c)

182. Hydraulic brakes work on the principle of

(a)   Bernoulli’s theorem

(b)   Thomson’s law

(c)   Newton’s law

(d)   Pascal’s law

Answer: (d)

183. If density of oxygen is 16 times that of hydrogen, what will be their corresponding ratio of velocity of sound?

(a)   1 : 16

(b)   2 : 1

(c)   4 : 1

(d)   1 : 4

Answer: (a)

184. The deterioration of a metal by an electrochemical process is commonly termed as

(a)   Passivation

(b)   Erosion

(c)   Corrosion

(d)   Abrasion

Answer: (c)

185. According to 2011 Census, the State having maximum population, is

(a)   Tamil Nadu

(b)   Maharashtra

(c)   Kerala

(d)   Uttar Pradesh

Answer: (d)

186. As a result of higher rate of inflation in India, the US dollar will

(a)   constant

(b)   negligible

(c)   appreciate

(d)   depreciate

Answer: (c)

187. Who invented the mobile phone?

(a)   Tim Berner-Lee

(b)   Chuck Hull

(c)   Martin Cooper

(d)   Raymond Samuel Tomlinson

Answer: (c)

188. Which part of the Cinchona yields drug?

(a)   Bark

(b)   Pericarp

(c)   Leaf

(d)   Endosperm

Answer: (a)

189. An example of hormone, is

(a)   cytosine

(b)   renin

(c)   pepsin

(d)   oxytocin

Answer: (d)

190. Who is the author of the book ‘The Three Mistakes in My Life’?

(a)   Chetan Bhagat

(b)   Jhumpa Lahiri

(c)   Amrita Pritam

(d)   Ruskin Bond

Answer: (a)

191. How many members are there in Public Accounts Committee?

(a)   30

(b)   22

(c)   20

(d)   28

Answer: (b)

192. Expand NABARD.

(a)   National Bank for Asian Research Development

(b)   National Bank for Agriculture and Resource Development

(c)   National Bank for Agri Related Development

(d)   National Bank for Agricultural and Rural Development

Answer: (d)

193. Find the odd one

(a)   Samveda

(b)   Vishnu Purana

(c)   Yajurveda

(d)   Rigveda

Answer: (b)

194. Cuba is the largest producer of

(a)   Rice

(b)   Sugar

(c)   Wheat

(d)   Barley

Answer: (b)

195. Pawel Pawlikowski’s film ‘Ida’ won the Best Foreign Language Film Awards at the 87th Academy Awards in 2015. This film is from which country?

(a)   South Africa

(b)   Poland

(c)   Argentina

(d)   Canada

Answer: (b)

196. Which endocrine gland is found in chest cavity?

(a)   Pineal gland

(b)   Thyroid gland

(c)   Adrenal gland

(d)   Thymus gland

Answer: (d)

197. Which of the following was the founder of the house of Peshawar?

(a)   Ramachandra Pant

(b)   Parsuram Triamsuk

(c)   Balaji Vishwanath

(d)   Balaji Baji Rao

Answer: (d)

198. Which city will host the 2022 Commonwealth Games?

(a)   Colombo

(b)   Glasgow

(c)   Durban

(d)   Montreal

Answer: (c)

199. The ‘El Nino’ phenomena which sparks climatic extreme around the globe, originates in the

(a)   Sea of China

(b)   Indian Ocean

(c)   Atlantic Ocean

(d)   Pacific Ocean

Answer: (d)

200. India’s ranking on Global Corruption Index 2014 has been placed at …………. rank among 175 nations.

(a)   83rd

(b)   81st

(c)   85th

(d)   87th

Answer: (c)

SSC Combined Graduate Level 2nd Shift (Tier-1) Examination Held on 16-8-2015 General Awareness Question Paper With Answer Key

SSC Combined Graduate Level 2nd Shift (Tier-1) Examination Held on 16-8-2015 General Awareness
SSC Combined Graduate Level 2nd Shift (Tier-1) Examination Held on 16-8-2015 General Awareness Question Paper With Answer Key

SSC Combined Graduate Level 2nd Shift (Tier-1) Examination Held on 16-8-2015

General Awareness

1. The author of the book ‘Zest for Life’ is-

(A)  Virginia Wolf

(B)  Mark Twain

(C)  Emile Zola

(D)  H. G. Walls

Answer: (C)

2. What is Amnesty International?

(A)  An Environment Protection Organization

(B)  An Animal Rights Protection Organization

(C)  A Human Rights Organization

(D)  A Peace Movement

Answer: (C)

3. Which of the following country has the longest overland tunnetl ?

(A)  Germany

(B)  France

(C)  Spain

(D)  Switzerland

Answer: (D)

4. What according to Communism is the chief enemy of the society ?

(A)  Religion

(B)  Capitalist class

(C)  Private property

(D)  Surplus value

Answer: (C)

5. When income increases, consumption also increases-

(A)  None of the options

(B)  in a lower proportion

(C)  in a  higher proportion

(D)  in the same proportion

Answer: (B)

6. Afla toxins are produced by-

(A)  Bacteria

(B)  Viruses

(C)  Fungi

(D)  Algae

Answer: (C)

7. Who was the Nawab of Bengal during ‘Battle of Plassey’?

(A)  Mir Jafar

(B)  None of these

(C)  Mir Qasim

(D)  Siraj-ud-duala

Answer: (D)

8. In mountainous areas a natural cause for road blocks is often-

(A)  land slides

(B)  tree felling

(C)  traffic congestion

(D)  road repairs and construction

Answer: (A)

9. The National Police Academy is located at-

(A)  Bengaluru

(B)  Abu Road

(C)  Hyderabad

(D)  Dehradun

Answer: (C)

10. ‘Wall Street’ is the name of the

(A)  Indian Township in Washington

(B)  Super market in Mumbai

(C)  Stock Exchange of Kolkata

(D)  Stock Exchange of New York

Answer: (D)

11. The substrate of Photorespiration is-

(A)  Pyruvic acid

(B)  Fructose

(C)  Glycolate

(D)  Glucose

Answer: (D)

12. The Indian Constitution provides the country with three categories of civil services, namely-

(A)  Administrative, police and foreign services

(B)  All India, central and state services

(C)  Administrative, railway and police services

(D)  Administrative, police and revenue services

Answer: (B)

13. Rotation of crops is essential-

(A)  For increasing the quantity of minerals

(B)  For getting different kinds of crops

(C)  For increasing fertility of the soil

(D)  For decreasing the quantity of proteins

Answer: (B)

14. Match the following-

List-I

(Crops)

(a) Rice                   (b) Maize

(c) Rubber     (d) Sugarcane

List-II

(First in the world)

1. Brazil 2. Malaysia

3. China 4. U. S. A.

(A)  (a) – 3; (b) – 4; (c) – 2; (d) – 1

(B)  (a) – 2; (b) – 1; (c) – 3; (d) – 4

(C)  (a) – 4; (b) – 2; (c) – 1; (d) – 3

(D)  (a) – 1; (b) – 3; (c) – 4; (d) – 2

Answer: (A)

15. In the case of deadlock between the two houses of the Parliament, the joint sitting is presided over by the-

(A)  President

(B)  Speaker of Lok Sabha

(C)  Vice President

(D)  Senior most member of Lok Sabha

Answer: (B)

16. Which one of the following is the infective stage of the Malarial Parasite ?

(A)  Sporoblast

(B)  Sporozoite

(C)  Schigozoite

(D)  Trophozoite

Answer: (B)

17. Headquarter of European Union (EU) is situated in-

(A)  Munich (Germany)

(B)  Brussels (Belgium)

(C)  Milam (Italy)

(D)  Paris (France)

Answer: (B)

18. Who of the following became a member of the “Din-i-Ilahi” ?

(A)  Tansen

(B)  Raja Man Singh

(C)  Raja Birbal

(D)  Todarmal

Answer: (C)

19. Competition for food, light and space is most severe in-

(A)  Distantly related species growing in the same habitats

(B)  Closely related species growing in the same area or niche

(C)  Distantly related species growing in different habitats

(D)  Closely related species growing in different habitats

Answer: (B)

20. The Thirteenth Pravasi Bhartiya Divas was held on 8th and 9th January, 2015 at-

(A)  New Delhi

(B)  Indore

(C)  Gandhinagar

(D)  Bengaluru

Answer: (C)

21. In a Database Management System (DBMS), the content and the location of the data is defined by the-

(A)  Multi Dimensional data

(B)  Meta data

(C)  Subdata

(D)  Sequence data

Answer: (B)

22. The apparent weight of a man in a lift is less than the real weight when-

(A)  the lift is going up with uniform speed

(B)  the lift is going up with an acceleration

(C)  the lift is going down with uniform speed

(D)  the lift is going down with an acceleration

Answer: (D)

23. The green color seen in firework displays is due to the chloride salt of-

(A)  Sodium

(B)  Barium

(C)  Strontium

(D)  Calcium

Answer: (B)

24. The first Sultan of Delhi to issue regular currency and declare Delhi as the capital of his empire was-

(A)  Balban

(B)  Qutubuddin Aibak

(C)  Alam Shah

(D)  Iltutmish

Answer: (D)

25. Which of the following refers to the white salt which covers the land in some areas during dry season ?

(A)  Usar

(B)  Reh

(C)  Reg

(D)  Erg

Answer: (A)

26. Match the following-

List-I

(a) Chlorophyll

(b) Nostoc

(c) Transpiration

(d) Rust of wheat

List-II

1. Plant Disease

2. Photosynthetic pigment

3. Diazotroph

4. Loss of water from plant surface

(A)  (a) – 2; (b) – 3; (c) – 4; (d) – 1

(B)  (a) – 4; (b) – 3; (c) – 1; (d) – 2

(C)  (a) – 2; (b) – 4; (c) – 3; (d) – 1

(D)  (a) – 3; (b) – 4; (c) – 2; (d) – 1

Answer: (A)

27. Rainbows are produced when sunlight-

(A)  Incident on rain drops hanging in the atmosphere is dispersed after suffering refraction

(B)  Incident on rain drops hanging in the atmosphere is dispersed after suffering refraction and internal reflection

(C)  Incident or raindrops hanging in the atmosphere is dispersed after suffering reflection

(D)  None of the given statements is correct

Answer: (B)

28. Which of the following comes under the jurisdiction of both the High Court and the Supreme Court ?

(A)  Protection against the violation of the Constitution

(B)  Protection of Fundamental Rights

(C)  Disputes between Centre and States

(D)  Disputes between the States inter se

Answer: (B)

29. One of the regions that receives rainfall from the north-easterly monsoon is-

(A)  Assam

(B)  Kerala

(C)  West Bengal

(D)  Tamil Nadu

Answer: (C)

30. The mercury and sodium street lamps light up because of-

(A)  Atomic emission

(B)  Electron absorption

(C)  Atomic absorption

(D)  Electron emission

Answer: (A)

31. Which one of the following is an indigenously built light combat aircraft of India ?

(A)  Vikrant

(B)  Tejas

(C)  Arjun

(D)  Akash

Answer: (B)

32. Addition of excessive amounts of heat to a lake is referred to as-

(A)  Green House effect

(B)  Heat Bloom

(C)  Thermal pollution

(D)  Refrigeration effect

Answer: (C)

33. Movement of people from a village to a small town and later to a city is known as-

(A)  inter-state  migration

(B)  step-wise migration

(C)  forced migration

(D)  intra-state migration

Answer: (B)

34. Which of the following is not correct about Mahatma Gandhi?

(A)  Gandhi advocated complete separation of politics from religion

(B)  Gandhi supported close relation between religion and politics

(C)  Gandhi believed in the sanctity of means

(D)  Gandhi believed in non-violence

Answer: (A)

35. The difference in the value of visible exports and visible imports is called-

(A)  Balance of Trade

(B)  Balance of Account

(C)  Balance Sheet of items

(D)  Balance of Payments

Answer: (A)

36. Man Booker Prize for 2014 is awarded to-

(A)  Shashi Tharoor

(B)  Lech Walesa

(C)  Richard Flanagan

(D)  Amrita Pritam

Answer: (C)

37. The purpose of choke in a fluorescent lamp is-

(A)  to decrease the current flow

(B)  to decrease the voltage momentarily

(C)  to decrease the resistance

(D)  to increase the current flow

Answer: (C)

38. Catalytic converters are generally made from-

(A)  Alkaline metals

(B)  Transition metals

(C)  Carbon

(D)  Hydrogen

Answer: (B)

39. A person can jump higher on the moon’s surface than on the earth because-

(A)  the acceleration due to gravity in moon is smaller than that on the earth

(B)  the Moon is cooler than earth

(C)  the moon’s surface is rough

(D)  the moon has no atmosphere

Answer: (A)

40. From the national point of view which of the following indicates Micro Approach ?

(A)  Study of sales of TISCO

(B)  Inflation in India

(C)  Per capita income in India

(D)  Educated Unemployment in India

Answer: (C)

41. The only AICC session Gandhiji presided was held at-

(A)  Lahore

(B)  Calcutta

(C)  Madras

(D)  Belgaum

Answer: (D)

42. Bio-diversity is also known as-

(A)  Within habitat diversity

(B)  Between habitat diversity

(C)  Ecosystem diversity

(D)  Global diversity

Answer: (D)

43. The soft silvery metallic element which ionizes easily when heated or exposed to light and is present in atomic clocks is-

(A)  Californium

(B)  Cesium

(C)  Calcium

(D)  Cerium

Answer: (D)

44. Which one of the following is not a feature of Indifference Curve ?

(A)  They do not interest each other

(B)  They slope downwards to the right

(C)  They are concave to each other

(D)  They are parallel to each other

Answer: (C)

45. Which one of these animals is jawless ?

(A)  Shark

(B)  Sphyrna

(C)  Trygon

(D)  Myxine

Answer: (D)

46. The remedies to soil erosion are-

1. Contour bunding

2. Regulation of land use

3. Cutting of trees

4. Allowing grazing of animals

(A)  1, 2

(B)  1, 3

(C)  2, 3

(D)  3, 4

Answer: (A)

47. Kautilya’s Arthashastra’ mainly deals with-

(A)  Political stat craft

(B)  Economic doctrines

(C)  Social aspects

(D)  Military aspects

Answer: (A)

48. Jaspal Rana is the name associated with which of the following games-

(A)  Weight lifting

(B)  Boxing

(C)  Shooting

(D)  Archery

Answer: (C)

49. What is the specialty of WINDOWS-NT ?

(A)  Supports multi-processing

(B)  Supports LAN and WAN

(C)  Support Batch processing

(D)  Supports real-time processing

Answer: (B)

50. Ecosystem consist of-

(A)  A population and its non-living elements

(B)  A biotic community

(C)  Population

(D)  A biotic community and its non-living elements

Answer: (D)

SSC Combined Graduate Level (Tier-I) 1st Shift Examination Held on 9-8-2015 General Awarness Question Paper With Answer Key

SSC Combined Graduate Level (Tier-I) 1st Shift Examination Held on 9-8-2015 General Awarness
SSC Combined Graduate Level (Tier-I) 1st Shift Examination Held on 9-8-2015 General Awarness Question Paper With Answer Key

SSC Combined Graduate Level (Tier-I) 1st Shift Examination Held on 9-8-2015

General Awareness

 

1. Allantois of Embryo helps in-

(A)  Respiration

(B)  Excretion

(C)  Protection

(D)  Digestion

Answer: (B)

2. Which one of the following animals belongs to mollusca ?

(A)  Hare

(B)  Hydra

(C)  Hyla

(D)  Haliotis

Answer: (D)

3. Outside the nucleus DNA is found in-

(A)  Mitochondria

(B)  Ribosome

(C)  Endoplasmic reticulum

(D)  Golgi bodies

Answer: (A)

4. Animal protein is called first class protein because it is-

(A)  Delicious in taste

(B)  Cheaper in the Market

(C)  Rich in essential amino acids

(D)  Easily digestible

Answer: (C)

5. It is easy to burst a gas-filled balloon with a needle than with a nail. It is because-

(A)  Nail exerts more pressure than needle on the balloon

(B)  Needle exerts more pressure than nail on the balloon

(C)  Gas is reactive with the needle

(D)  Nail is more longer than needle

Answer: (B)

6. The velocity of sound in moist air is more than in dry air because the moist air has-

(A)  Less pressure than dry air

(B)  More pressure than dry air

(C)  More density than dry air

(D)  Less density than dry air

Answer: (D)

7. X-rays can be used-

(A)  To detect heart diseases

(B)  To detect defects in precious stones and diamonds

(C)  To detect gold under the earth

(D)  For cutting and welding of metals

Answer: (B)

8. Ice is packed in saw dust because-

(A)  Saw dust is poor conductor of heat

(B)  Saw dust is a good conductor of heat

(C)  Saw dust does not stick to the ice

(D)  Saw dust will not get melted easily

Answer: (A)

9. What is used to identify whether a data word has an odd or even number of I’s ?

(A)  Sign bit

(B)  Zero bit

(C)  Parity bit

(D)  Carry bit

Answer: (C)

10. Rearranging and allocating space in memory

(A)  Multiprogramming

(B)  Multitasking

(C)  Memory Management

(D)  Networking

Answer: (C)

11. What happens when a drop of glycerol is added to crushed KMnO4 spread on a paper ?

(A)  There is a violent explosion

(B)  There is no reaction

(C)  The paper ignites

(D)  There is a crackling sound

Answer: (C)

12. Most commonly used bleaching agent is-

(A)  Alcohol

(B)  Carbon dioxide

(C)  Chlorine

(D)  Sodium chloride

Answer: (C)

13. The least penetrating power ray is-

(A)  α-Ray

(B)  β-Ray

(C)  γ-Ray

(D)  X-Ray

Answer: (A)

14. Hydrogen peroxide is an effective sterilizing agent. Which one of the following product results when it readily loses active oxygen?

(A)  Water

(B)  Hydrogen

(C)  Ozone

(D)  Nasant Hydrogen

Answer: (A)

15. The maximum fixation of solar energy is done by-

(A)  Bacteria

(B)  Fungi

(C)  Green plants

(D)  Protozoa

Answer: (C)

16. The term ‘brown air’ is used for-

(A)  Photochemical smog

(B)  Sulfurous smog

(C)  Industrial smog

(D)  Acid fumes

Answer: (C)

17. Which of the following is FALSE with respect to rain water harvesting ?

(A)  It helps raising water table

(B)  It helps meet rising water demand

(C)  It increases run-off losses

(D)  It is a device of water conservation

Answer: (C)

18. Peroxyacetyl nitrate is a-

(A)  Plant hormone

(B)  Vitamin

(C)  Secondary pollutant

(D)  Acidic dye

Answer: (C)

19. Which of the following river does not originate in Indian territory ?

(A)  Mahanadi

(B)  Brahmaputra

(C)  Satluj

(D)  Ganga

Answer: (B)

20. To whom the line ‘A thing of beauty is a joy for ever’ is attributed ?

(A)  John Keats

(B)  Dr. Charles Dickens

(C)  Dr. Jonathan Swift

(D)  William Wordsworth

Answer: (A)

21. The birthday of which of the following leaders is celebrated as ‘Teachers Day’ in India?

(A)  Dr. Rajendra Prasad

(B)  S. Radhakrishnan

(C)  C. Rajgopalachari

(D)  Lala Lajpat Rai

Answer: (B)

22. The award given for outstanding performance in sports is-

(A)  Bharat Ratna

(B)  Padma Shri Award

(C)  Arjuna Award

(D)  Dronacharya Award

Answer: (C)

23. Which will station’s name mean ‘place of the thunderbolt’?

(A)  Shillong

(B)  Oottacamand

(C)  Darjeeling

(D)  Gangtok

Answer: (C)

24. The ship building yard-Mazgaon Dock is located at-

(A)  Kochi

(B)  Kolkata

(C)  Mumbai

(D)  Vishakhapatnam

Answer: (C)

25. Electric current is measured using which of the following instrument ?

(A)  Voltmeter

(B)  Anemometer

(C)  Wattmeter

(D)  Ammeter

Answer: (D)

26. ‘Agha Khan Cup’ is related with which of the following sport event ?

(A)  Cricket

(B)  Hockey

(C)  Table Tennis

(D)  Football

Answer: (B)

27. Where was the first conference of SAARC (South Asian Association for Regional Cooperation held ?

(A)  Dhaka

(B)  New Delhi

(C)  Colombo

(D)  Kathmandu

Answer: (A)

28. Which among the following is not a Bretton Woods Institution?

(A)  International Monetary Fund (IMF)

(B)  World Bank

(C)  Organization of Economic Cooperation and Development (O.E.C.D.)

(D)  None of these

Answer: (C)

29. Equilibrium price in the market is determined by the-

(A)  equality between total cost and total revenue

(B)  equality between average cost and average revenue

(C)  equality between marginal cost and marginal revenue

(D)  equality between marginal cost and average cost

Answer: (C)

30. In the national context which of the following indicates Macro Approach?

(A)  Sales of Bata Shoe Company

(B)  Exports of Mangoes to U.K.

(C)  Income from Railways

(D)  Inflation in India

Answer: (D)

31. Internal economies-

(A)  arise in an economy as it makes progress

(B)  accrue to a firm when it expands its output

(C)  arise when there is expansion in internal trade

(D)  arise when there is expansion in an industry

Answer: (D)

32. One of the features of a free market economy is-

(A)  public ownership of factors of production

(B)  rationing and price control

(C)  consumer’s sovereignty

(D)  active state intervention

Answer: (C)

33. Gross National Product-Depreciation Allowance = ?

(A)  Gross Domestic Product

(B)  Personal Income

(C)  Net National Product

(D)  Per Capita Income

Answer: (C)

34. The Panchayat Samiti remains accountable for its functions to-

(A)  The Gram Panchayats and Gram Sabhas

(B)  Zilla Parishads

(C)  Anchal Panchayats

(D)  Janpad Panchayats

Answer: (B)

35. The legislature gains a priority over the executive in-

(A)  A Federal Government

(B)  An Authoritarian Government

(C)  A Parliamentary Government

(D)  A Presidential Government

Answer: (C)

36. The legislature in a democratic country can influence public opinion by-

(A)  Granting rights

(B)  Enacting non-controversial laws

(C)  Defining the duties of the citizens

(D)  Focusing attention on public issues

Answer: (D)

37. If the President wants to resign from his office, he may do so by writing to the-

(A)  Vice President

(B)  Chief Justice of India

(C)  Prime Minister

(D)  Speaker of Lok Sabha

Answer: (A)

38. Which of the following is not a Union Territory?

(A)  Lakshadweep

(B)  Puducherry

(C)  Nagaland

(D)  Dadra and Nagar Haveli

Answer: (C)

39. The greatest king of Partihara dynasty was-

(A)  Bhoj (Mihir-Bhoj)

(B)  Dantidurga

(C)  Nagbhatta II

(D)  Vatsaraj

Answer: (A)

40. In 1939 Subhash Chandra Bose was elected as President of the Congress Party defeating-

(A)  Jawaharlal Nehru

(B)  Maulana Abul Kalam Azad

(C)  V. B. Patel

(D)  Pattabhi Sitharamayya

Answer: (D)

41. Jallianwala incident took place at-

(A)  Lucknow

(B)  Surat

(C)  Amritsar

(D)  Allahabad

Answer: (C)

42. Who was the founder of Lodhi dynasty?

(A)  Sikandar Lodhi

(B)  Bahlol Lodhi

(C)  Ibrahim Lodhi

(D)  Daulat Khan Lodhi

Answer: (B)

43. Which one of the following pair is not correctly matched ?

(A)  Akbar – Todarmal

(B)  Chanakya – Chandragupta

(C)  Vikramaditya – Chaitanya

(D)  Harshvardhan – Hiuen Tsang

Answer: (C)

44. The South-East trade winds are attracted towards the Indian subcontinent in the rainy season due to-

(A)  the effect of easterlies

(B)  the effect of Northern-East trade winds

(C)  the presence of low atmospheric pressure over North-West India

(D)  the development of cyclone over the equator

Answer: (C)

45. The ‘graded profile’ of a river course is a-

(A)  smooth curve in the upper course

(B)  smooth curve in the middle course

(C)  smooth curve in the lower course

(D)  smooth curve from source to mouth

Answer: (D)

46. Sink hole is a phenomenon of ……….. topography.

(A)  Desert

(B)  Tundra

(C)  Karst

(D)  Plain

Answer: (C)

47. Kerala famous for the cultivation of-

1. Coconut

2. Black-pepper

3. Rubber

4. Rice

(A)  1, 2 and 4

(B)  2, 3 and 4

(C)  1 and 4

(D)  1, 2 and 3

Answer: (A)

48. The longest continental Railway in the world is-

(A)  Trans Siberian Railway

(B)  Canadian Pacific Railway

(C)  Canadian National Railway

(D)  Trans Atlantic Railway

Answer: (A)

49. Photoperiodism affects-

(A)  Flowering

(B)  Vegetative growth

(C)  Fruiting

(D)  All of these

Answer: (D)

50. Math the following –

List-I

(a) Ascorbic acid

(b) Chlorophyll

(c) Carotenoid

(d) Superoxide dismutase

List-II

1. Photosynthetic pigment

2. Quencher

3. Enzyme

4. Vitamin-C

(A)  (a) – 4; (b) – 2; (c) – 1; (d) – 3

(B)  (a) – 2; (b) – 4; (c) – 1; (d) – 3

(C)  (a) – 4; (b) – 1; (c) – 3; (d) – 2

(D)  (a) – 4; (b) – 1; (c) – 2; (d) – 3

Answer: (C)

SSC Combined Graduate Level (Tier-I) Examination 1st Shift Held on 9-8-2015 General Intelligence and Reasoning Question Paper With Answer Key

SSC Combined Graduate Level (Tier-I) Examination 1st Shift Held on 9-8-2015 General Intelligence and Reasoning
SSC Combined Graduate Level (Tier-I) Examination 1st Shift Held on 9-8-2015 General Intelligence and Reasoning Question Paper With Answer Key

SSC Combined Graduate Level (Tier-I) Examination 1st Shift Held on 9-8-2015

General Intelligence and Reasoning

 

Directions – (Q. 1-6) In the following Six Questions, find the odd word/number/letters/number pair from t he given alternatives

1.

(A)  Morning

(B)  Noon

(C)  Evening

(D)  Night

Answer: (B)

2.

(A)  Liberty

(B)  Society

(C)  Equality

(D)  Fraternity

Answer: (B)

3.

(A)  DWFU

(B)  EVHS

(C)  HSKP

(D)  KQNN

Answer: (D)

4.

(A)  CBEF

(B)  EDGH

(C)  IHKL

(D)  GFHJ

Answer: (D)

5.

(A)  4025

(B)  7202

(C)  6023

(D)  5061

Answer: (D)

6.

(A)  96 : 80

(B)  64 : 48

(C)  80 : 60

(D)  104 : 78

Answer: (A)

7. Choose the correct alternative to complete the series.

Lily, Daisy, Datura, ?

(A)  Sun Flower

(B)  Hibiscus

(C)  Marigold

(D)  Jasmine

Answer: (D)

8. Which one of the given responses would be a meaningful order of the following?

1. Elephant 2. Cat

3. Mosquito 4. Tiger

5. Whale

(A)  5, 3, 1, 2, 4

(B)  1, 3, 5, 4, 2

(C)  3, 2, 4, 1, 5

(D)  2, 5, 1, 4, 3

Answer: (C)

Directions- (Q. 9-10) In the following Two Questions, which one set of letters when sequentially placed at the gaps in the given letter series shall complete it?

9. ccbab_caa_bccc_a_

(A)  babb

(B)  bbba

(C)  baab

(D)  babc

Answer: (A)

10. a_ _ dba_ _ bacad _ _ da _ _ cd

(A)  bccdbcab

(B)  abcddcba

(C)  cbcddcba

(D)  aabbccdd

Answer: (A)

Directions – (Q. 11-12) In the following Two Questions, a series is given, with one term missing. Choose the correct alternative from the given ones that will complete the series.

11. 4, 6, 10, 16, 24, ?

(A)  28

(B)  30

(C)  34

(D)  40

Answer: (C)

12. 3, 5, 9, 17, ?

(A)  26

(B)  65

(C)  33

(D)  42

Answer: (C)

13. A train starts from station A and reaches B 15 minutes late when it moves with 40 km/hr and 24 minutes late when it goes 30 km/hr. The distance between the two stations is-

(A)  16 km

(B)  18 km

(C)  21 km

(D)  24 km

Answer: (B)

14. In a row men, Manoj is 30th from the right and Kiran is 20th from the left. When they interchange their position, Manoj becomes 35th from the right. What is the total number of men in the row?

(A)  45

(B)  44

(C)  54

(D)  34

Answer: (C)

15. Seven persons A, B, C, D, E, F and G are standing in a straight line.

D is to the right of G.

C is between A and B.

E is between F and D.

There are three persons between G and B. Who is on the extreme left?

(A)  A

(B)  B

(C)  D

(D)  G

Answer: (D)

16. From the given alternative words, select the word which cannot be formed using the letters of the given word :

CUMBERSOME

(A)  MOUSE

(B)  SOBER

(C)  ROME

(D)  MERCY

Answer: (D)

17. Name a single letter, which can be prefixed to the following words in order to obtain entirely new words?

TILL TABLE PILE TAB PRING

(A)  S

(B)  B

(C)  H

(D)  C

Answer: (A)

18. Unscramble the following letters to frame a meaningful word. Then find out the correct numerical position of the letters :

B  C  U  S  M  E  L  R  N  A

1   2  3   4   5   6   7  8   9  10

(A)  6 1 4 3 2 5 8 7 9 10

(B)  3 1 5 7 10 4 2 6 9 8

(C)  3 9 4 2 8 10 5 1 7 6

(D)  2 1 3 4 6 8 9 7 5 10

Answer: (C)

19. Using the following code and key decode the given coded word :

Code : L  X  P  Z  J  Y   Q   M   N   B

Key :   b   a   e   s   p  r   h    i     g    t

Code word : ZBYXMNQB

(A)  height

(B)  struggle

(C)  straight

(D)  strength

Answer: (C)

20. In a certain code ‘MOUSE’ is written as ‘PRUQC’. How is ‘SHIFT’ written in that code?

(A)  VJIDR

(B)  VIKRD

(C)  RKIVD

(D)  VKIDR

Answer: (D)

21. In a certain code, 253’ means ‘books are old’ ; ‘546’ means ‘mean is old’ and ‘378’ means ‘buy good books’. What stands for “are “ in that code ?

(A)  2

(B)  4

(C)  5

(D)  6

Answer: (A)

22. If, + stands for division; x stands for addition; − stands for multiplication; ÷ stands for subtraction, which of the following is correct ?

(1)   46 × 6 ÷ 4 – 5 + 3 = 74

(2)   46 – 6 + 4 × 5 ÷ 3 = 71

(3)   46 ÷ 6 × 4 – 5 + 3 = 75.5

(4)   46 × 6 – 4 + 5 ÷ 3 = 70.1

(A)  4

(B)  2

(C)  1

(D)  3

Answer: (B)

23. If + = ×, − = ÷, × = +, ÷ = −, then which is the correct equation out of the following ?

(A)  18 ÷ 6 + 4 – 2 ÷ 3 = 22

(B)  18 + 6 – 4 × 2 ÷ 3 = 26

(C)  18 × 6 – 4 + 7 × 8 = 47

(D)  18 – 6 × 7 ÷ 2 + 8 = 63

Answer: (B)

Directions – (Q. 24-28) In the following Five Questions, select the missing number from the given responses.

24. 

(A)  97

(B)  907

(C)  1097

(D)  9107

Answer: (C)

25. 

(A)  12

(B)  17

(C)  18

(D)  16

Answer: (C)

26. 9  11   13

     13   15    17

    10    12    14

    14    16    18

    11    13     ?

(A)  22

(B)  14

(C)  156

(D)  21

Answer: (C)

27. 

(A)  30

(B)  11

(C)  0

(D)  1

Answer: (C)

28. 7     5                 3

      8      4                 9

    2          8               ?

    112      60               162

(A)  4

(B)  6

(C)  8

(D)  12

Answer: (B)

29. Pinky walks a distance of 600 m towards east, turns left and moves 500 m then turns left and walks 600 m and then turns left again and moves 500 m and halts At what distance in metres is she from the starting point ?

(A)  2200

(B)  500

(C)  0

(D)  600

Answer: (C)

30. Sunita rode her scooty northwards, then turned left and then again rode to her left 4 km. She found herself exactly 2 km west of her starting point. How far did she ride northwards initially?

(A)  2 km

(B)  4 km

(C)  5 km

(D)  6 km

Answer: (B)

Directions – (Q. 31-32) In the following Two Questions, one statement is given followed by two Conclusions, I and II. You have to consider the statement to be true, even if it seems to be at variance from commonly known facts. You are to decide which of the given conclusions can definitely by drawn from the give statement. Indicate your answer.

31. Statement : Every school should promote partnerships that will increase parental involvement and participation for promoting the growth of children.

Conclusion :

I. For the growth of the children, parents should be involved in various school activities.

II. Involvement of parents in school activities has no influence on the growth of the children.

Codes :

(A)  Only I follows.

(B)  Only II follows.

(C)  Neither I nor II follows.

(D)  Both I and II follow.

Answer: (A)

32. Statement : Aggressive animals can be trained with care and affection to behave as the occasion demands.

Conclusions :

I. Trained dogs cannot be aggressive.

II. Animals are always aggressive unless care and affection is given to them.

Codes :

(A)  Only I follows.

(B)  Only II follows.

(C)  Neither I nor II follows.

(D)  Both I and II follow

Answer: (C)

Directions – (Q. 33-38) In the following Six Questions, select the related word / letters / number from the given alternatives

33. Haematology : Blood : : Phycology : ?

(A)  Fungi

(B)  Fishes

(C)  Algae

(D)  Diseases

Answer: (C)

34. Pri9de of Lions : : ………. of cats.

(A)  Herd

(B)  School

(C)  Clowder

(D)  Bunch

Answer: (C)

35. MAN : PDQ : : WAN : ?

(A)  ZDQ

(B)  NAW

(C)  YQD

(D)  YDQ

Answer: (A)

36. AEFJ : KOPT : : ? : QUVZ

(A)  GLKP

(B)  GKLP

(C)  HLKP

(D)  HKQL

Answer: (B)

37. 2 : 32 : : 3 : ?

(A)  243

(B)  293

(C)  183

(D)  143

Answer: (A)

38. D × H : 4 × 8 as M × Q : ?

(A)  12 × 17

(B)  12 × 16

(C)  13 × 17

(D)  14 × 18

Answer: (C)

39. How many triangles are there in the figure ?

(A)  24

(B)  14

(C)  28

(D)  20

Answer: (C)

40. The figure given on the left hand side is folded to form a box. Choose from the alternatives (1), (2), (3) and (4) the boxes that is similar to the box formed.

(A)  (2) and (3) only

(B)  (1), (3) and (4) only

(C)  (2) and (4) only

(D)  (1) and (4) only

Answer: (B)

41. Find out the number of circles in the given figure :

(A)  14

(B)  16

(C)  17

(D)  18

Answer: (C)

42. Identify the diagram that best represents the relationship among the classes given below : Animals, land animals, sea animals

Answer: (D)

43. 

Which number indicates doctors who are not married ?

(A)  6

(B)  4

(C)  2

(D)  1

Answer: (D)

44. 

In the fig :

Q represents all quadrilaterals

K represents all Kites

R represents all Rhombus

P represents all Parallelogram

The statement ‘Rhombus is also a Kite’ can be described as-

(A)  P and K is nothing but R

(B)  P or K is nothing but R

(C)  P and R is nothing but K

(D)  P or R is nothing but K

Answer: (A)

Directions – (Q. 45-46) In the following Two Questions, which answer figure will complete the pattern in the question figure ?

45. 

Answer: (A)

46.

Answer: (C)

47. From the given answer figures, select the one which is hidden/ embedded in the question figure :

Answer: (B)

48. A piece of paper is folded and cut as shown below in the question figures. From the given answer figures, indicate how it will appear when opened.

Answer: (C)

49. If a mirror is places on the line MN, then which of the answer figures is the right image of the given figure?

Answer: (A)

50. Directions- A word is represented by only one set of numbers as given in any one of the alternatives. The sets of numbers given in the alternatives are represented by two classes of alphabets as in two matrices a given below. The columns and rows of Matrix I are numbered from 0 to 4 and that of numbered from 0 to 4 and that of Matrix II are numbered from 5 to 9. A letter from these matrices can be represented first by its row and next by its column, e.g., ‘A’ can be represented by 01, 14 etc., and ‘O’ can be represented by 59 , 67 etc. similarly, you have to identify the set for the word ‘PEARL’.

(A)  00, 55, 22, 11, 96

(B)  00, 66, 14, 32, 56

(C)  13, 77, 30, 14, 88

(D)  12, 88, 43, 89

Answer: (A)

SSC Combined Graduate Level (Tier-I) Examination 2nd Shift Held on 9-8-2015 General Awareness Question Paper With Answer Key

SSC Combined Graduate Level (Tier-I) Examination 2nd Shift Held on 9-8-2015 General Awareness
SSC Combined Graduate Level (Tier-I) Examination 2nd Shift Held on 9-8-2015 General Awareness Question Paper With Answer Key

SSC Combined Graduate Level (Tier-I) Examination 2nd Shift Held on 9-8-2015

General Awareness

1. In the glass lands, trees do not replace the grasses as a part of an ecological succession because of-

(A)  Water limits and fire

(B)  Limited sun light and  paucity of nutrients

(C)  Insect and fungi

(D)  None of the above

Answer: (A)

2. The business in Stock Markets and other securities markets is regulated by-

(A)  Stock and Exchange Bank of Board of India

(B)  Securities and Exchange Board of India

(C)  Sole Trade and Exchange Bank of India

(D)  State and exchange Bank of India

Answer: (B)

3. The Women’s Reservation Bill seeks how much reservation for women in the State assemblies and Lok Sabha?

(A)  30%

(B)  36%

(C)  25%

(D)  33%

Answer: (D)

4. The oldest form of composition of Hindustani vocal music is-

(A)  Ghazal

(B)  Thumri

(C)  Dhrupad

(D)  None of these

Answer: (C)

5. G E F, an international aid – giving agency has the full form-

(A)  Global Environment Fund

(B)  Global Energy Fund

(C)  Global Economic Fund

(D)  Global Educational Fund

Answer: (A)

6. On 2nd June, 2015 a Conference on make in India-Indigenization of currency was organized by the department of Economic Affairs, Ministry of Finance, govt. of India. Who inaugurated the conference?

(A)  Finance Secretary

(B)  Finance Minister

(C)  Prime Minister

(D)  Governor R.B.I

Answer: (B)

7. A Fuse wire is characterized by-

(A)  Low resistance and high melting point

(B)  High  resistance and low melting point

(C)  Low resistance and low melting point

(D)  High resistance and high melting point

Answer: (B)

8. The ore of Aluminium is-

(A)  Hematite

(B)  Bauxite

(C)  Fluorspar

(D)  Chalco pyrites

Answer: (B)

9. Heavy metals got their name because compared to other atoms they have-

(A)  Higher atomic masses

(B)  Higher densities

(C)  Higher atomic numbers

(D)  Higher atomic radii

Answer: (B)

10. Fascism believes in the application of the principle of-

(A)  Dictatorship

(B)  Utilitarianism

(C)  Democracy

(D)  Totalitarianism

Answer: (D)

11. Raga Kameshwari was composed by-

(A)  Pandit Ravi Shankar

(B)  Ustad Amzad Ali Khan

(C) None of these

(D)  Uday Shankar

Answer: (A)

12. The Dynamo converts-

(A)  Electrical energy into Mechanical Energy

(B)  Mechanical energy into Electrical Energy

(C)  Mechanical energy into Magnetic Energy

(D)  None of these

Answer: (B)

13. What is ordinary Law?

(A)  Laws made by the common people

(B)  Laws made and enforced by the Government

(C)  Laws made by the High Court

(D)  Laws made by the Supreme Court

Answer: (B)

14. Photo chemical smog is resultant of the reaction among-

(A)  High concentration of NO2, O3 and CO in the evening

(B)  NO2, O3 and peroxy acetyl nitrate in the presence of Sunlight

(C)  CO, CO2 and NO2 at low temperature

(D)  CO, O2 and peroxy acetyl nitrate in the presence of Sunlight

Answer: (B)

15. The father of computer is-

(A)  Charles Babbage

(B)  Oliver twist

(C)  Love lice

(D)  Charles Dickens

Answer: (A)

16. Which of the following costs is related to marginal cost?

(A)  Fixed Cost

(B)  Implicit Cost

(C)  Prime Cost

(D)  Variable Cost

Answer: (D)

17. Laberalism stands for-

(A)  A movement & an attitude

(B)  Self-emancipation

(C)  Freedom in social, political and economic aspects

(D)  Religious orthodoxy

Answer: (C)

18. Mahatma Gandhi began his political activities in India first from-

(A)  Kheda

(B)  Champaran

(C)  Dandi

(D)  Sabarmati

Answer: (B)

19. When a helium atom loses an electron it becomes-

(A)  A proton

(B)  A positive helium ion

(C)  A negative helium ion

(D)  An alpha particle

Answer: (B)

20. The Environment (Protection) Bill was passed by the Parliament of India in-

(A)  1986

(B)  1981

(C)  1984

(D)  1972

Answer: (A)

21. The Battle of Plassey was fought in year-

(A)  1775

(B)  1757

(C)  1576

(D)  1761

Answer: (B)

22. Xenobiotics which are inherently resistant to microbial attack are called as-

(A)  Biodegradable

(B)  Persistent

(C)  Recalcitrant

(D)  All of the given options

Answer: (C)

23. PSW stands for-

(A)  Program status word

(B)  Process status word

(C)  Primitive status word

(D)  Processor status word

Answer: (A)

24. Earthquakes are caused by-

(A)  Denudation

(B)  Earth revolution

(C)  Tectonism

(D)  Earth rotation

Answer: (C)

25. The Liquidity Preference Theory of Interest was propounded by-

(A)  Adam Smith

(B)  David Ricardo

(C)  Alfred Marshall

(D)  J. M. Keynes

Answer: (D)

26. When is International ‘Yoga Divas’ is celebrated?

(A)  21st June

(B)  25th April

(C)  21st May

(D)  21st July

Answer: (A)

27. Cyanide poisoning causes death in seconds because-

(A)  It  breaks the electron transport chain

(B)  It denatures enzymes of the heart/muscle

(C)  It causes cardiac arrest

(D)  It causes Lysis of red blood

Answer: (C)

28. Who directed the film on the dacoit queen Phoolan Devi?

(A)  Shashi Kapoor

(B)  Shekhar Kapoor

(C)  Abbas Mastan

(D)  Anil Kapoor

Answer: (B)

29. In which year did Dada Saheb Phalke produce the first Feature fim?

(A)  1911

(B)  1910

(C)  1912

(D)  1913

Answer: (D)

30. The earth completes one rotation on its axis in-

(A)  23 hrs. 56 min. 4.9 sec.

(B)  23 hrs. 10 min. 2 sec

(C)  24 hrs.

(D)  23 hrs. 30 min.

Answer: (A)

31. The technology used in the electronic printer is called-

(A)  Micro encapsulation

(B)  Micro technology

(C)  Micro array

(D)  Micro millimetric

Answer: (B)

32. For a missile launched with a velocity less than the earth’s escape velocity, the total energy is-

(A)  Positive

(B)  Negative

(C)  Either positive or negative

(D)  Zero

Answer: (A)

33. Which one of the following is the correct sequence of ecosystem in the order of decreasing productivity?

(A)  Mangroves, oceans, grassland, lakes

(B)  Oceans, lakes, grassland, mangroves

(C)  Mangroves, grasslands, lakes, oceans

(D)  Oceans, mangroves, lakes, grassland

Answer: (C)

34. The classical Dance of Andhra Pradesh is-

(A)  Kathakali

(B)  Odissi

(C)  Bharatanatyam

(D)  Kuchipudi

Answer: (D)

35. The Mediterranean region are characterized by heavy rain in-

(A)  Spring

(B)  Autumn

(C)  Winter

(D)  Summer

Answer: (C)

36. Which was the first hydel power project in India?

(A)  Siva Samudram in Karnataka

(B)  Nizamnagar in Andhra Pradesh

(C)  Paikara in Tamilnadu

(D)  Palli vassal in Kerala

Answer: (A)

37. Molybdenum deficiency affects the activity of-

(A)  Nitrogenase

(B)  Chlorate reductase

(C)  Nitrate reductase

(D)  All of the given options

Answer: (C)

38. Autocracy means-

(A)  Absolute rule by one

(B)  Rule by few

(C)  Rule by the representatives of the People

(D)  Rule by King

Answer: (A)

39. Waksman got the Nobel Prize of the discovery of-

(A)  Streptomycin

(B)  Chloromycetin

(C)  Penicillin

(D)  Neomycin

Answer: (A)

40. Spring tides occur on-

(A)  New moon day only

(B)  Full moon day only

(C)  The day when the moon’s position is in its first

(D)  Full moon day as well as on new moon day

Answer: (D)

41. What is the fascist view of state ?

(A)  Nation state is unquestionably sovereign

(B)  State is the instrument of exploitation in the hands of few

(C)  State is under the control of a king

(D)  State enhances the ideal of individualism

Answer: (B)

42. Which of the following can be found as pollutants in the drinking water in some parts of India ?

Select the correct answer using the code given below :

1. Arsenic

2. Sorbitol

3. Fluoride

4. Formaldehyde

5. Uranium

(A)  1 and 3

(B)  1, 2, 3, 4 and 5

(C)  2, 4 and 5

(D)  1, 3 and 5

Answer: (A)

43. Who is considered founder of the Gupta Empire?

(A)  Chandra Gupta

(B)  Srigupta

(C)  Samudra Gupta

(D)  Chandra Gupta II

Answer: (B)

44. When one gene pair hides the effect of the other unit, the phenomenon is referred to as-

(A)  Mutation

(B)  Epistasis

(C)  Dominance

(D)  None of the options

Answer: (C)

45. State Bank of India was previously known as-

(A)  Canara Bank

(B)  Imperial Bank of India

(C)  Syndicate Bank

(D)  Cooperative Bank of India

Answer: (B)

46. “Don’t Laugh: We are Police”, This book was compiled by-

(A)  K. P. S. Gill D. G. P.

(B)  Bishan Lal Vohra I.G.P.

(C)  Shanti Swaroop I.G. Police

(D)  None of the options

Answer: (B)

47. Indian first talkie film produced in 1931 was-

(A)  Almara

(B)  Indra Sabha

(C)  Shakuntala

(D)  Neel Kamal

Answer: (A)

48. The name of the upper house of the Indian Parliament is-

(A)  Legislative assembly

(B)  Rajya Sabha

(C)  House of Lords

(D)  Senate

Answer: (B)

49. Microbial Type Culture Collection Centre is situated at-

(A)  Chandigarh

(B)  New Delhi

(C)  Hyderabad

(D)  Bangalore

Answer: (A)

50. Who discovered the first antibiotic?

(A)  Louis Pastour

(B)  A. Fleming

(C)  W. Fleming

(D)  C. Waksman

Answer: (B)

SSC CGL (Tier-1) Preliminary Examination Held on August 23, 2017 Question Paper With Answer Key

SSC CGL (Tier-1) Preliminary Examination Held on August 23, 2017
SSC CGL (Tier-1) Preliminary Examination Held on August 23, 2017 Question Paper With Answer Key

SSC CGL (Tier-1) Preliminary Examination Held on August 23, 2017

1. Select the related word from the given alternatives.

River : Stream : : Ocean : ?

(a)   Current

(b)   Pond

(c)   Dam

(d)   Sea

Answer: (a)

2. Select the related letters from t he given alternatives.

WUS : DFH : : MKI : ?

(a)   LJH

(b)   GEC

(c)   OQS

(d)   NPR

Answer: (d)

3. Select the related number from the given alternatives.

(a)   2/9

(b)   −9/2

(c)   7/2

(d)   −2/9

Answer: (d)

4. Select the odd word from the given alternatives.

(a)   Mother-in-Law

(b)   Nephew

(c)   Grandson

(d)   Great grandfather

Answer: (a)

5. Select the odd letters from the given alternatives.

(a)   GHI

(b)   XYZ

(c)   VUT

(d)   CDE

Answer: (c)

6. Select the odd number from the given alternatives.

(a)   43

(b)   22

(c)   13

(d)   41

Answer: (b)

7. A series is given, with one word missing. Choose the correct alternative from the given ones that will complete the series.

Shy, Food, Plate, Recess, ?

(a)   Monsoon

(b)   Soon

(c)   Eat

(d)   Lunch

Answer: (a)

8. A series is given, with one term missing. Choose the correct alternative from the given ones that will complete the series.

XIIIII, IXIIII, IIXIII, IIIXII, IIIIXI, ?

(a)   IIIIXII

(b)   IIIIIXI

(c)   IIIIIX

(d)   XIIIIX

Answer: (c)

9. A series is given, with one number missing. Choose the correct alternative from the given ones that will complete the series.

49, 64, ?, 100, 121

(a)   74

(b)   80

(c)   75

(d)   81

Answer: (d)

10. Nasir’s birthday is on Thursday May 18. On what day of the week will be Rehan’s Birthday in the same year if Rehan was born on August?

(a)   Wednesday

(b)   Saturday

(c)   Friday

(d)   Thursday

Answer: (b)

11. The weights of 4 boxes are 30, 40, 50 and 100 kg respectively. Which of the following cannot be the total weight, in kilograms, of any combination of these boxes and in a combination a box can be used only once?

(a)   190

(b)   180

(c)   160

(d)   140

Answer: (c)

12. From the given alternative words, select the word which cannot be formed using the letters of the given word :

ECCENTRICITY

(a)   NIECE

(b)   CREATE

(c)   TRINITY

(d)   RETICENT

Answer: (b)

13. If ETHANOL is coded as HWKDQRO, the how will MIX be coded as?

(a)   PLA

(b)   RFV

(c)   BGT

(d)   NHY

Answer: (a)

14. In a certain code language, ‘+’ represents ‘×’, ‘−‘ represents ‘+’ ‘×’ represents ‘÷’ and ‘÷’ represents ‘−‘. What is the answer to the following question?

24 × 6 – 8 + 2 = ?

(a)   25

(b)   50

(c)   40

(d)   20

Answer: (d)

15. If 10$25 = 8, 12$25 = 10 then what is the value of 14$53 = ?

(a)   13

(b)   15

(c)   11

(d)   9

Answer: (a)

16. Select the missing number from the given responses :

(a)   0

(b)   68

(c)   2

(d)   10

Answer: (c)

17. X and Y start from the same point. X walls 40 m North, then turns West and walks 80 m, then turns to his right and walks 50 m. At the same time, Y walks 90 m North. Where is Y now with respect to the position of X?

(a)   Y is 30 m to the East of X.

(b)   Y is 80 m to the West of X.

(c)   Y is 30 m to the West of X.

(d)   Y is 80 m to the East of X.

Answer: (d)

18. In the question a statement is given, followed by two arguments, I and II. You have to consider the statement to be true even if it seems to be at variance from commonly known facts. You have to decide which of the given arguments, if any, is a strong argument.

Statement : Should one year of army training be compulsory for all Indian citizens?

Argument I : No, the costs of training will be prohibitive and one year of labour will  be lost.

Argument II : Yes, army training helps make better citizens.

(a)   if only argument I is strong.

(b)   if only argument II is strong.

(c)   if both I and II are strong.

(d)   if neither I nor II is strong.

Answer: (d)

19. Find the number of triangles in the figure.

(a)   8

(b)   9

(c)   11

(d)   12

Answer: (d)

20. In the following figure, square represents lawyers, triangle represents cyclists, circle represents men and rectangle represents post-graduates. Which set of letters represents men who are not cyclists?

(a)   B, C, D

(b)   E

(c)   G, F

(d)   G, F, H, A

Answer: (a)

21. Which answer figure will complete the pattern in the question figure?

Answer: (c)

22. From the given answer figures, select the one in which the question figure is hidden/embedded

Answer: (d)

23. A piece of paper is folded and cut as shown below in the question figure. From the given answer figures, indicate how it will appear when opened?

Answer: (c)

24. If a mirror is placed on the line MN, then which of the answer figures is the right image of the given figure?

Answer: (b)

25. In this question, the sets of numbers given in the alternatives are represented. The columns and rows of Matrix I are numbered from 0 to 4 and that of Matrix II are numbered from 5 to 9. A letter from these matrices can be represented first by its row and next by its column, e.g., O can be represented by 03, 14 etc and ‘K’ can be represented by 56, 65, etc. Similarly you have to identify the set for the word ‘EASE’

(a)   55, 85, 44, 42

(b)   77, 85, 88, 44

(c)   77, 66, 31, 44

(d)   00, 98, 23, 98

Answer: (b)

Part 2 General Knowledge and General Awareness

26. India’s first official Census operation was undertaken in which year?

(a)   1841

(b)   1881

(c)   1921

(d)   1961

Answer: (b)

27. The change in the optimal quantity of a goods when its price changes and the consumer’s income is adjusted so that she can just buy the bundle that she was buying before the price change is called?

(a)   Law of demand

(b)   Substitution effect

(c)   Problem of choice

(d)   Optimal choice

Answer: (b)

28. ‘Relief of the disabled and unemployable’ is listed in the ……… list given in the Seventh Schedule in the Constitution of India.

(a)   Union

(b)   State

(c)   Global

(d)   Concurrent

Answer: (b)

29. ……………. amends the Constitution.

(a)   Ministry of Defence

(b)   Prime Minister’s Office

(c)   Parliament

(d)   Securities and Exchange Board of India

Answer: (c)

30.  According to the categories of land mentioned in the Chola inscriptions ……… was known as the land gifted to temples

(a)   Vellanvagai

(b)   Brahmadeya

(c)   Shalabhoga

(d)   Devadana

Answer: (d)

31. Rajendra I was the son of?

(a)   Bindusara I

(b)   Devabhuit I

(c)   Skanda Gupta I

(d)   Rajaraja I

Answer: (d)

32. ……………… is a cold desert lying the Great Himalayas.

(a)   Ladakh

(b)   Satpura

(c)   Aravalli

(d)   Vindhya

Answer: (a)

33. The method of soil conservation in which different crops are grown in alternate rows and are sown at different times to protect the soil from rain wah is called

(a)   Mulching

(b)   Intercropping

(c)   Rock dam

(d)   Terrace farming

Answer: (b)

34. In each testes in a male reproductive system, there are …………. compartments called testicular lobules.

(a)   150

(b)   200

(c)   250

(d)   300

Answer: (c)

35. The predominant stage of the life cycle of moss is the gametophyte which consists of two stages. The second stage is the ……….. stage.

(a)   Agar

(b)   Leafy

(c)   Chlorella

(d)   protonema

Answer: (b)

36. In Animal Kingdom classification, which of the following is not a Phylum?

(a)   Mollusca

(b)   Chordata

(c)   Coelomates

(d)   Annelida

Answer: (c)

37. Friction is caused by the ……. on the two surfaces in contact.

(a)   irregularities

(b)   smoothness

(c)   densities

(d)   gaps

Answer: (a)

38. The motion of a freely falling body is an example of ……….. motion.

(a)   uniformly accelerated

(b)   non-uniformly accelerated

(c)   constant velocity

(d)   constant speed

Answer: (a)

39. …………….. (HHDD) is a technology where the conventional disk drive is combined with non-volatile flash memory of typically 128 MB or more to cache data during normal use.

(a)   Hyper Hard Disk Drive

(b)   Hybrid Hard Disk Drive

(c)   Hybrid Helium Disk Drive

(d)   Hyper Helium Disk Drive

Answer: (b)

40. Chemical formula for sulphurous acid is

(a)   H2SO4

(b)   H2SO3

(c)   H3SO3

(d)   H3SO4

Answer: (b)

41. The temperature at which a solid melts to become a liquid at the atmospheric pressure is called its

(a)   Crystallization

(b)   Melting point

(c)   Evaporation

(d)   Galvanization

Answer: (b)

42. …………… is the interaction in which one species benefits and the other is neither harmed nor benefited.

(a)   Predation

(b)   Commensalism

(c)   Competition

(d)   Parasitism

Answer: (b)

43. …………… scheme by the Central government aims at providing affordable aviation by making domestic flight charges economically accessible for middle-class citizens of India.

(a)   Uday Desh Ka Aam Nagrik

(b)   Urja Ganga

(c)   Ek Bharat Shrestha Bharat

(d)   Namami Ganga Yojana

Answer: (a)

44. Who invented the hot air balloon?

(a)   Montgolfier brothers

(b)   Wright brothers

(c)   Lisitsyn brothers

(d)   Walton brothers

Answer: (a)

45. Which nation won the highest number of medals at 2016 Rio Olympics?

(a)   Great Britain

(b)   The USA

(c)   China

(d)   Russia

Answer: (b)

46. Group of monuments at Hampi is located in

(a)   West Bengal

(b)   Gujarat

(c)   Karnataka

(d)   Tamil Nadu

Answer: (d)

47. Which of the following is India’s highest civilian award?

(a)   Padma Bhushan

(b)   Param Vir Chakra

(c)   Padma Vibhushan

(d)   Bharat Ratna

Answer: (d)

48. Which of the statements given below are correct?

1. The author of the novel ‘An Affair Downstairs’ is Sherri Browning.

2. The author of the novel ‘White Teeth’ is Zadie Smith.

3. The author of the novel ‘Atonement’ is Ian McEwan.

(a)   1 and 2

(b)   2 and 3

(c)   1 and 3

(d)   All of the above

Answer: (d)

49. Recently, scientists have found second Great Spot 24000 km across and 12000 km wide on which planet?

(a)   Saturn

(b)   Mars

(c)   Jupeter

(d)   Mercury

Answer: (c)

50. The Sundarbans Reserve Forest (SRF) which is adjacent to India’s Sundarbans National Park is located in which neighbouring country?

(a)   Bhutan

(b)   Pakistan

(c)   Bangladesh

(d)   Nepal

Answer: (c)

Part 3 Quantitative Aptitude

51. Convert binary 10110 to decimal

(a)   22

(b)   20

(c)   18

(d)   16

Answer: (a)

52. A, B and C can do a job working alone in 12, 16 and 24 days respectively. In how many days they can do the job if they worked together?

(a)   16/3

(b)   15/4

(c)   17/3

(d)   19/4

Answer: (a)

53. The diagonal of a square is 10 cm. What is the length of it side?

(a)   5 cm

(b)   5√2 cm

(c)   10√2 cm

(d)   5/√2 cm

Answer: (b)

54. If on a Sale there is 40% discount on the marked price of Rs 1000, but the sale is done at Rs 510 only, then what additional discount did the customer get?

(a)   25%

(b)   15%

(c)   10%

(d)   30%

Answer: (b)

55. What is the third proportional to 9 and 45?

(a)   405

(b)   225

(c)   5

(d)   81

Answer: (b)

56. In a class of 39 students there are 26 girls. The average weight of these girls is 42 kg and average weight of the full class is 48 kg. What is the average weight of the boys of the class?

(a)   54 kg

(b)   66 kg

(c)   60 kg

(d)   62 kg

Answer: (c)

57. If a saree is sold for Rs 2880 the seller will face 10% loss, at what price should he sell to gain 20% profit?

(a)   Rs 4830

(b)   Rs 3840

(c)   Rs 3480

(d)   Rs 4380

Answer: (b)

58. Priya got 9 marks more in History than what she got in Geography. Her History marks are 56% of the sum of her History and Geography marks. What are her Geography marks?

(a)   42

(b)   65

(c)   53

(d)   33

Answer: (d)

59. To travel 720 m, a Express train takes 6 hours more than Duronto. If however, the speed of the Express train is doubled, it takes 2 hours less than Duronto. The speed of Duronto is?

(a)   60 km/h

(b)   72 km/h

(c)   66 km/h

(d)   78 km/h

Answer: (b)

60. What is the difference between the compound interests on Rs 1000 for 1 year at 10% per annum compounded yearly and half-yearly?

(a)   Rs 1.5

(b)   Rs 0.5

(c)   Rs 2.5

(d)   Rs 3.5   

Answer: (c)

61. If  then the value of x is

(a)   7

(b)   20

(c)   −7

(d)   −20

Answer: (c)

62. If a + b = 8 and ab = 15, then what is the value of a3 + b3?

(a)   98

(b)   152

(c)   124

(d)   260

Answer: (b)

63. The sum of a non-zero number and 4 times its reciprocal is 17/2. What is the number?

(a)   8

(b)   12

(c)   16

(d)   4

Answer: (a)

64. The 5th and 9th term of an arithmetic progression are 7 and 13 respectively. What is the 15th term?

(a)   22

(b)   21

(c)   55

(d)   59

Answer: (a)

65. What is the reflection of the point (6, −3) in the line y = 2?

(a)   (−2, −3)

(b)   (6, 7)

(c)   (−6, 7)

(d)   (−2, 3)

Answer: (b)

66. Point A (4, 2) divides segments BC in the ratio 2 : 5. Co-ordinates of B are (2, 6) and C are (7, y). What is the value of y?

(a)   8

(b)   −8

(c)   6

(d)   −6

Answer: (b)

67. At what point does the line 2x – 3x = 6 cuts the X axis?

(a)   (−3, 0)

(b)   (0, 3)

(c)   (0, −3)

(d)   (3, 0)

Answer: (d)

68. ∆ABC is right angled at B. BD is an altitude. AD =4 cm and DC = 9 cm. What is the value of BD?

(a)   5 cm

(b)   4.5 cm

(c)   5.5 cm

(d)   6 cm

Answer: (d)

69. tan 45° + cosec 60° = tan 45° + cosec 60° =

(a) 

(b) 

(c) 

(d)  

Answer: (d)

70. ∆XYZ is right angled at Y. If m ∠Z = 60°, then cosec X =

(a)   2

(b)   1/√2

(c)   1/√2

(d)   1/√3

Answer: (a)

71. If  then cot θ = ?

(a)   17/8

(b)   15/8

(c)   8/15

(d)   17/15

Answer: (b)

Directions (Q. Nos. 72-75) The bar graph shows the number of males and females (in crores) in India during 1951-91. Read the graph and answer the following questions.

72. What was the approximate percentage of female population in India during 1991?

(a)   48.23%

(b)   48.02%

(c)   48.03%

(d)   48.33%

Answer: (a)

73. What was the approximate number of males in India in 1971 per thousands females?

(a)   913

(b)   1075

(c)   1077

(d)   1175

Answer: (c)

74. What is the ratio of the numbers of females in India in 1961 per thousands males in India in 1991 per thousand females?

(a)   943 : 10777

(b)   1077 : 943

(c)   1073 : 913

(d)   913 : 1073

Answer: (d)

75. Assuming that the rate of increase in the total population in India during 1991-2001 remains the same as that was during the period 1981-91, estimate the total population in India in 2001.

(a)   105.62 crore

(b)   106.25 crore

(c)   106.52 crore

(d)   105.26 crore

Answer: (b)

Part 4 English Comprehension

Directions (Q. Nos. 76-77) In the following questions, some part of the sentence may have errors. Find out which part of the sentence has an error and select the appropriate option. If a sentence is free from error, select ‘No Error’.

76. While he went beside (a)/ with our decisions, we were (b) / never sure he totally agreed. (c) No error (d)

(a)   a

(b)   b

(c)   c

(d)   d

Answer: (a)

77. The after morning (a) / she woke to the first ray of (b) / light through the window. (c) No error (d)

(a)   a

(b)   b

(c)   c

(d)   d

Answer: (a)

Directions (Q. Nos. 78-79) In the following questions, the sentence given with blank to be filled in with an appropriate word. Select the correct alternative out of the four and indicate it by selecting the appropriate option.

78. The psychopath get a ……. sense of enjoyment from torturing animals.

(a)   perverse

(b)   agreeable

(c)   happy

(d)   resonable

Answer: (a)

79. When Jaspal got angry, he started to ……….. his feelings. with his hands.

(a)   participate

(b)   immolate

(c)   intoxicate

(d)   gesticulate

Answer: (d)

Directions (Q. Nos. 80-83) In the following questions, out of the four alternatives, select the word similar in meaning to the word given.

80. Fossilise

(a)   Amalgamate

(b)   Dissolve

(c)   Flex

(d)   Liquefy

Answer: (a)

81. Depict

(a)   Conceal

(b)   Distort

(c)   Characterize

(d)   Suppress

Answer: (c)

82. Differ

(a)   Alter

(b)   Concur

(c)   Digress

(d)   Vary

Answer: (b)

83. Salve

(a)   Cerate

(b)   Liniment

(c)   Remedy

(d)   Blockage

Answer: (d)

Directions (Q. Nos. 84-85) In the following questions, out of the four alternatives, select the alternative which best expresses the meaning of the idiom/phrase.

84. Gnash your teeth

(a)   Express rage

(b)   Brush your teeth

(c)   Laugh hysterically

(d)   Take a big bite

Answer: (a)

85. Let something slip through one’s fingers

(a)   Lose a wedding ring

(b)   Not be able to understand a difficult concept

(c)   Let go of certain unpleasant things

(d)   to waste an opportunity to achieve something

Answer: (d)

Directions (Q. Nos. 86-87) In the following questions, out of the four alternatives, select the alternative which will improve the bracketed part of the sentence. In case no improvements is needed, select ‘no improvement’.

86. What would you like (to do it) for your birthday?

(a)   to do

(b)   doing

(c)   does

(d)   No improvement

Answer: (a)

87. You must (be mistake).

(a)   be mistakenly

(b)   be mistaken

(c)   be  mistook

(d)   No improvement

Answer: (b)

Directions (Q. Nos. 88-89) In the following questions, out of the four alternatives, select the alternative which is the best substitute of the words/sentence.

88. Complete with regard to every detail

(a)   Thorough

(b)   Lackadaisical

(c)   Lax

(d)   Cursory

Answer: (a)

89. The faculty or power of using one’s will

(a)   Antagonism

(b)   Aversion

(c)   Rejection

(d)   Volition

Answer: (d)

Directions (Q. Nos. 90-91) In the following questions, four words are given out of which one word is correctly spelt, select the correctly spelt word.

90.

(a)   Jukstapose

(b)   Jaxtapuse

(c)   Juxtapose

(d)   Jaxtapose

Answer: (c)

91.

(a)   Amusemant

(b)   Ammusement

(c)   Ammusemant

(d)   Amusement

Answer: (d)

Directions (Q. Nos. 92-93) Each of these questions below consists of a set of labeled sentences. Out of the four options given, select the most  logical order of the sentences to form a coherent paragraph.

92. Evidently it was too

X-of a deadly-looking cobra or

Y-two we saw no reptiles

Z-dry for game, and with the exception

(a)   YZX

(b)   YXZ

(c)   XZY

(d)   ZXY

Answer: (d)

93. Some of these

X-height of more than

Y=2000 feet above the sea

Z=rocks rise to the

(a)   YZX

(b)   ZXY

(c)   YXZ

(d)   XZY

Answer: (b)

94. In the following question, a sentence has been given in Active/Passive voice. Out of the four alternatives suggested, select the one which best expresses the same sentence in Passive/Active voice.

I will wash the car every Sunday.

(a)   The car is washed by me every Sunday.

(b)   Every Sunday by me the car is to be washed.

(c)   Every Sunday by myself the car is to be washed.

(d)   The car will be washed by me every Sunday.

Answer: (d)

95. In the following question, a sentence has been given in Direct/Indirect speech. Out of the four alternatives suggested, select the one which best expresses the same sentence in Indirect/Direct speech.

The watchman said, thief ! thief ! catch him!?

(a)   The watchman shouted to t he crowd to catch the thief.

(b)   The watchman shouted to the crowd, thief, thief, catch him.

(c)   The watchman shouted to the crowd, catch the thief.

(d)   The watchman shouted to the crowd, catch him.

Answer: (a)

Directions (Q. Nos. 96-100) A passage is given with five questions following it. Read the passage carefully and choose the best answer to each question out of the four alternatives.

There’s more than the 101-seater dining table at the Falaknuma palace that is awe inspring. The library done in rosewood and mahogany is a p lace that will force anyone to pause, if only to admire the fine original polish, the furniture and the walnut carved high roof. The last is supposedly and imitation of the one at Windsor Castle.

The library is almost the size of a large banquet hall and acts an the passage to other parts o the palace including the dining area. For years the palace had no librarian because they couldn’t find the right person to do justice to the treasure trove of books collected by the Nizam’s family they found Asif Husain Arastu who belongs to the family that owned the famous. AA Husain book store in Abids. So when his fried and historian Anurdadha Naik pointed out the palace’s requirement at the library, Husain was delighted. On his very first day as the libranina at the magnificent Taj Falaknuma place, Asif Husain found a reference book on ‘How to paint grass’ by Alfred East.

96. Which of the following is false about Arastu?

(a)   He was a descendent of the Nizam’s family.

(b)   He was the librarian at the Falaknuma Palace.

(c)   He owned a book store.

(d)   He was a friend of Anuradha Naik.

Answer: (a)

97. The roof the library has carvings made of which wood?

(a)   Rosewood

(b)   Mahogany

(c)   Sandalowood

(d)   Walnut

Answer: (d)

98. Name the author of the book that Hussain found on his first day as the librarian?

(a)   Alfred East

(b)   Anuradha Naik

(c)   Mark Twain

(d)   Charles Dickens

Answer: (a)

99. The books at the library were collected by

(a)   Hyderabad University

(b)   Anuradha Naik

(c)   the Nizam’s Family

(d)   Asif Hussain Arastu

Answer: (c)

100. Which part of the palace is an imitation of the Windsor Palace?

(a)   101-seater dining table

(b)   The library’s roof

(c)   The banquet hall

(d)   The library’s furniture

Answer: (b)

SSC Sub-Inspector (Delhi Police-BSF-SSB-CRPF-ITBP) and Assistant Sub-Inspector (CISF) Preliminary Examination Held on July 7, 2017 Question Paper With Answer Key

SSC Sub-Inspector (Delhi Police-BSF-SSB-CRPF-ITBP) and Assistant Sub-Inspector (CISF) Preliminary Examination Held on July 7, 2017
SSC Sub-Inspector (Delhi Police-BSF-SSB-CRPF-ITBP) and Assistant Sub-Inspector (CISF) Preliminary Examination Held on July 7, 2017 Question Paper With Answer Key

SSC Sub-Inspector (Delhi Police/BSF/SSB/CRPF/ITBP) and Assistant Sub-Inspector (CISF) Preliminary Examination Held on July 7, 2017

Part A General Intelligence and Reasoning

 

Directions (Q. Nos. 1-6) In the following questions, select the related word/letters/number from the given alternatives.

1. Pen : Write : : ? : ?

(a)   Erase : Rubber

(b)   Cup : Liquid

(c)   Pencil : Wood

(d)   Knife : Cut

Answer: (a)

2. Create : Destroy : : ? : ?

(a)   Thin : Small

(b)   Long : Short

(c)   Fat : Huge

(d)   Long : Topmost

Answer: (b)

3. SNOP : ONSP : : CLAY : ?

(a)   ALCY

(b)   LCYA

(c)   LYCA

(d)   ACLY

Answer: (a)

4. PAN : TDM : : ? : ?

(a)   GIL : KMG

(b)   HOT : KLG

(c)   MAN : PAM

(d)   SIP : WLO

Answer: (d)

5. 23: 72 : : 38 : ?

(a)   110

(b)   117

(c)   123

(d)   112

Answer: (b)

6. 62 : 145 : : ? : ?

(a)   79 : 168

(b)   119 : 226

(c)   167 : 291

(d)   34 : 122

Answer: (b)

Directions (Q. Nos. 7-11) In the following questions, select the odd word/letters/number from the given alternative.

7.

(a)   sea

(b)   River

(c)   Ocean

(d)   Swimming Pool

Answer: (d)

8.

(a)   Red

(b)   White

(c)   Blue

(d)   Yellow

Answer: (b)

9.

(a)   NQTW

(b)   PSVZ

(c)   WZCF

(d)   BEHK

Answer: (b)

10.

(a)   XTCG

(b)   NJMQ

(c)   EAUZ

(d)   SOHL

Answer: (c)

11.

(a)   61

(b)   51

(c)   97

(d)   89

Answer: (b)

12. In the following question, select the odd number pair from the given alternatives

(a)   13-2028

(b)   11-12110

(c)   7-336

(d)   9-648

Answer: (c)

Directions (Q. Nos. 13-14) Arrange the given words in the sentence in which they occur in the dictionary.

13. Mobile 2. Mandate 3. Mandarin 4. Monkey 5. Master

(a)   3, 2, 5, 1, 4

(b)   3, 1, 2, 5, 4

(c)   3, 5, 2, 4, 1

(d)   3, 5, 2, 1, 4

Answer: (a)

14. 1. Drink 2. Drinking 3. Drive 4. Dictionary 5. Dracula

(a)   4, 5, 1, 2, 3

(b)   4, 1, 5, 3, 2

(c)   4, 2, 5, 1, 3

(d)   4, 5, 2, 1, 3

Answer: (a)

15. In the following questions, which one set of letters when sequentially placed at the gaps in the given letter series shall complete it?

………ml…..nl……….mlm……. I …. mlmn ……..

(a)   nnmmll

(b)   nmlnml

(c)   mnlnlm

(d)   nmnnnl

Answer: (d)

16. A series is given with one term missing. Select the correct alternative from the given ones that will complete the series.

FGC, HHA, JIY, ?

(a)   KKW

(b)   LJW

(c)   LKX

(d)   KJX

Answer: (b)

Directions (Q. Nos. 17-18) In the following questions, select the missing number from the given series.

17. 2 6 4 9 8 13 16 18 32 ?

(a)   24

(b)   26

(c)   22

(d)   28

Answer: (a)

18. 534 543  559  584  620  ?

(a)   648

(b)   676

(c)   669

(d)   671

Answer: (c)

19. The ratio of present ages of Sumit and Amit is 3 : 4. If the age of Sumit 20 years hence will be 62 years, then what is the present age of Amit?

(a)   56 yr

(b)   64 yr

(c)   60 yr

(d)   52 yr

Answer: (a)

20. In al line of boys Aman is 12th from top and Baman is 18th from bottom. If there are 6 boys between Aman and Baman, then how many maximum boys are there in the row?

(a)   34

(b)   36

(c)   35

(d)   37

Answer: (b)

21. 6 boys, P, Q, R, S, T and U are standing in a row facing North. P and Q cannot be either at 1st or 2nd place. R and S will always be together and R must be at one of the ends. R doesn’t have S to his right. Who is standing on the West end?

(a)   S

(b)   S or U

(c)   T

(d)   T or U

Answer: (d)

Directions (22-23) In the following questions, select the word which cannot be formed using the letters of the given word.

22. BLENDING

(a)   BLEND

(b)   BALD

(c)   LEND

(d)   BINGE

Answer: (b)

23. TRADITIONAL

(a)   NATIONAL

(b)   TRAIN

(c)   LAND

(d)   RATION

Answer: (a)

24. In a certain code language, UPDATE is written a FRWCRY and GREATS is written as GTIQRY. How is BLENDY written in that code language?

(a)   DMGWBL

(b)   CJZAFP

(c)   ZJCPFA

(d)   GNDWBL

Answer: (d)

25. In a certain code language, FRAME is written as OUGVH and STONY is written as IVUXU. How is RESTS written in that code language?

(a)   CBYIT

(b)   ABDHS

(c)   EAZHS

(d)   EAZIT

Answer: (a)

26. If + denotes ‘multiplied by’, − denotes ‘added to’, × denotes ‘divided by’ and ÷ denotes ‘subtracted from’, then which of the following equation is true?

(a)   30 ÷ 8 × 4 – 6 + 1 = 32

(b)   8 + 5 – 20 × 4 ÷ 6 = 41

(c)   12 – 12 ÷ 6 × 6 + 3 = 21

(d)   6 + 7 ×3 – 4 ÷ 6 = 26

Answer: (c)

27. In the following question, by using which mathematical operators will the expression become correct?

35 ? 5 ? 10 ? 15 ? 4

(a)   ÷,  +, = and ×

(b)   ×, ÷, > and ×

(c)   ÷, ×, > and ×

(d)   ÷, ×, < and +

Answer: (c)

28. If 34 × 15 = 495 and 43 × 12 = 504, then 98 × 17 = ?

(a)   1649

(b)   1683

(c)   1763

(d)   1751

Answer: (a)

29. If 26(52) 8 and 48(192) 16, then what is the value of ‘A’ in A (175) 14?

(a)   50

(b)   25

(c)   35

(d)   40

Answer: (a)

30. If 64 + 7 = 460 and 25 +8 = 212, then 43 + 8 = ?

(a)   360

(b)   376

(c)   332

(d)   356

Answer: (d)

31. In the following question, select the number which can be placed at the sign of question mark from the given alternatives.

(a)   7

(b)   10

(c)   9

(d)   8

Answer: (d)

32. Find the missing number from the given options :

(a)   13

(b)   15

(c)   17

(d)   19

Answer: (b)

33. How many triangles are there in the given figure?

(a)   13

(b)   14

(c)   12

(d)   15

Answer: (d)

Directions (Q. Nos. 34-35) In each of the following questions below are given some statements followed by some conclusion. Taking the given statements to  be true even if they seem to be at variance from commonly known facts, read all the conclusions and then decide which of the given conclusion logically follows the given statements.

34. Statements I.  All gapes are green.

II. Some green are hard.

Conclusions

I. Some green are grapes.

II. Some hard are grapes.

III. No grape is hard.

(a)   Only conclusion I follows

(b)   conclusions I and II follow

(c)   conclusions I and III follow

(d)   All conclusions follow

Answer: (a)

35. Statements

I. All cars are black.

II. No black is shining.

Conclusions

I. Some cars are shining.

II. No car is shining.

III. No shining is car.

(a)   conclusions I and II Follow

(b)   conclusions II and III follow

(c)   conclusions I and III follow

(d)   All conclusions follow

Answer: (b)

36. Two position of a cube are shown below. What will come opposite to face containing ‘9’?

(a)   7

(b)   4

(c)   6

(d)   4 or 6

Answer: (b)

37. Which number is wrong in the give series?

7, 56, 447, 3584, 28672

(a)   3584

(b)   56

(c)   7

(d)   447

Answer: (d)

38. Find the missing number of series from the given alternatives.

1, 4, 2, 3, 2, ?

(a)   2

(b)   5

(c)   3

(d)   4

Answer: (d)

Directions (39-40) Identify the diagram that best represents the relationship among the given classes.

39. Tree, Branches, Root

Answer: (b)

40. Shirt, Trouser, Clothes

Answer: (a)

Directions (Q. Nos. 41-42) Which answer figure will complete will complete the pattern in the question figure?

41. Question Figure

Answer Figure

Answer: (b)

42. Question Figure

Answer Figure

Answer: (a)

Directions (Q. Nos. 43-44) From the given answer figure, select the one in which the question figure is hidden/embedded.

43. Question Figure

Answer Figure

Answer: (a)

44. Question Figure

Answer Figure

Answer: (b)

Directions (45-46) A piece of paper is folded and punched as shown be answer figures, indicate how it willow in the question figures. From the given appear when opened?

45. Question Figures

Answer Figures

Answer: (d)

46. Question Figures

Answer Figures

Answer: (c)

47. If a mirror is placed on the line MN, then which of the answer figure is the right image of the given figured.

Question Figure

Answer: (b)

48. If a mirror is placed on the line AB, then which of the answer figures is the right image of the given figure?

Answer: (c)

49. A word is represented by only one set of numbers as given in any one of the alternatives. The sets of numbers given in the alternatives are represented by two classes of alphabets as shown in the given two matrices. The columns and rows of Matrix-I are numbered from 0 to 4 and that of Matrix-II are numbered from 5 to 9. A letter from these matrices can be represented first by its row and next by its column, for example, ‘E’ can be represented by 10, 22, etc., and ‘O’ can be represented by 56, 78 etc. Similarly, you have to identify the set for the word ‘HALT’.

(a)   40, 03, 76, 24

(b)   21, 41, 68, 13

(c)   12, 14, 69, 00

(d)   34, 41, 87, 31

Answer: (b)

50. A word is represented by only one set of numbers as given in any one of the alternatives. The sets of numbers given in the alternatives are represented by two classes of alphabets as shown in the given two matrices. The columns and rows of Matrix-I are numbered from 0 to 4 and that of Matrix-II are numbered from 5 to 9. A letter from these matrices can be represented first by its row and next by its column, for example, ‘V’ can be represented by 31, 44, etc., and ‘D’ can be represented by 67, 86 etc. Similarly, you have to identify the set for the word ‘GLOW’.

(a)   56, 89, 01, 04

(b)   68, 78, 21, 42

(c)   87, 65, 22, 41

(d)   95, 57, 13, 33

Answer: (d)

Part B General Knowledge and General Awareness

51. Which among the following comes under tertiary sector of Indian Economy?

(a)   Cloth Industry

(b)   Transport of goods

(c)   Diary

(d)   Sugar Industry

Answer: (b)

52. ……….. publishes Economic Survey in India.

(a)   Government of India

(b)   Ministry of Finance

(c)   NITI Aayog

(d)   Prime Minister of India

Answer: (b)

53. In which economic system the government decides what goods are to be produced in accordance with the needs of society?

(a)   Socialist

(b)   Mixed

(c)   Capitalist

(d)   Traditional

Answer: (a)

54. 1 Rupee note bears the signature of whom?

(a)   Government of India

(b)   Chief Justice of India

(c)   Finance Secretary of India

(d)   Prime Minister of India

Answer: (c)

55. Which five years plan recognized human development as the core of all developmental efforts?

(a)   First Five Year Plan

(b)   Second Five Year Plan

(c)   Eight Five Year Plan

(d)   Ninth Five Year Plan

Answer: (c)

56. A system of rules that takes effect when a military authority takes control of the normal administration justice is called as

(a)   Coup

(b)   Strike

(c)   Martial law

(d)   Political prisoner

Answer: (c)

57. Who among the following is the most important political institution in a democratic country?

(a)   President

(b)   Parliament

(c)   Prime Minister

(d)   Cabinet Ministers

Answer: (c)

58. Which kind of decisions are usually based on careful calculation of gains and losses?

(a)   Moral

(b)   Socialist

(c)   Prudential

(d)   Ethnic

Answer: (c)

59. Emergency provisions in Indian Constitution has been taken from

(a)   British Constitution

(b)   Government of India Act 1935

(c)   Irish Constitution

(d)   Japanese Constitution

Answer: (b)

60. Match the following

(a)   A – 2; B – 3; C – 1

(b)   A – 2; B – 1; C – 3

(c)   A – 3; B – 2; C – 1

(d)   A – 3; B – 1; C – 2

Answer: (a)

61. Which of the following can be amended by special majority?

(a)   Directive Principles of State Policy

(b)   Rules of Procedure in Parliament

(c)   Admission of new State

(d)   Use of English language in Parliament

Answer: (a)

62. For how much time Financial Emergency can be proclaimed?

(a)   6 months

(b)   12 months

(c)   24 months

(d)   No maximum period

Answer: (d)

63. Which among the following do not participate in election of the President?

(a)   Elected members of both the houses

(b)   Elected members of Legislative Assemblies of State

(c)   Elected members of Legislative Assemblies of Delhi and Puducherry

(d)   Nominated members of State Legislative Assemblies

Answer: (d)

64. In which year (in AD) did Babar invaded India?

(a)   1530

(b)   1520

(c)   1526

(d)   1550

Answer: ()

65. Which among the following metal was not found in Harappan civilization?

(a)   Gold

(b)   Copper

(c)   Silver

(d)   Iron

Answer: (d)

66. Who built Red Fort?

(a)   Humayun

(b)   Akbar

(c)   Shah Jahan

(d)   Aurangzeb

Answer: (c)

67. In whose reign Hiuen-Tsang visited India?

(a)   Jalauddin

(b)   Harshvardhana

(c)   Jahangir

(d)   Pandayas

Answer: (b)

68. Set events in choronological order in which they happened.

1. Sepoy mutiny

2. Vasco-da-Gama Reached India

3. Babur started mughal dynasty

(a)   1, 3, 2

(b)   3, 1, 2

(c)   2, 3, 1

(d)   3, 2, 1

Answer: (c)

69. Which of the following planet is farthest from Sun?

(a)   Saturn

(b)   Jupiter

(c)   Neptune

(d)   Uranus

Answer: (c)

70. In ocean, where is ‘Benthos’ found?

(a)   at bottom of ocean

(b)   at upper surface of ocean

(c)   in an enclosed area of ocean

(d)   along the shore

Answer: (a)

71. In which continent ‘Great Dividing Range’ is located?

(a)   North America

(b)   South America

(c)   Asia

(d)   Australia

Answer: (d)

72. ‘Jog falls’ is located on which river?

(a)   Narmada

(b)   Krishna

(c)   Godavari

(d)   Sharavati

Answer: (d)

73. For a time difference of two hours, the longitudinal distance will be equal to

(a)   15°

(b)   30°

(c)   45°

(d)   60°

Answer: (b)

74. Where does Amoeba digests its food?

(a)   Pseudopodia

(b)   Nucleus

(c)   Food Vacuole

(d)   Cell membrane

Answer: (c)

75. What are female gametes called as?

(a)   Zygote

(b)   Ova

(c)   Sperms

(d)   Embryo

Answer: (b)

76. Which of the following cell organells are present only in plant cell?

(a)   Lysosomes

(b)   Plastids

(c)   Cell membrane

(d)   Mitochondria

Answer: (b)

77. Muscles are connected to bones by connective tissue called as

(a)   Tendon

(b)   Ligament

(c)   Neuron

(d)   Adipose

Answer: (d)

78. India’s first digitalized Panchayat ward situated in which state?

(a)   Mizoram

(b)   Gujarat

(c)   Maharashtra

(d)   Kerala

Answer: (d)

79. Neurons are part of which system of human body?

(a)   Circulatory system

(b)   Excretory system

(c)   Reproductive system

(d)   Nervous system

Answer: (d)

80. Which contact force is responsible for changing the State of motion of an object?

(a)   Magnetic force

(b)   Frictional force

(c)   Muscular force

(d)   Electrostatic force

Answer: (b)

81. According to law of reflection

(a)   Angle of incidence is greater than angle of reflection

(b)   Angle of incidence is smaller than angle of reflection

(c)   Angle of incidence is always equal to angle of reflection

(d)   Both angles are always unequal

Answer: (c)

82. What is the unit of relative density?

(a)   kg/m3

(b)   g/cm3

(c)   mg/mm3

(d)   No unit

Answer: (d)

83. Which law/principle States that when a body is immersed fully or partially in a fluid, it experiences an upward force that is equal to the weight of the fluid displaced by it?

(a)   Boyle’s law

(b)   Charles law

(c)   Archimedes principle

(d)   Pascal’s law

Answer: (c)

84. Which among the following is/are not an input devices?

1. Scanner    2. Joystic

3. Plotter

(a)   Only 1

(b)   Only 2

(c)   Only 3

(d)   Both 1 and 2

Answer: (c)

85. Protocol used for receiving an email is

(a)   HTTP

(b)   FTP

(c)   POP-3

(d)   SSH

Answer: (c)

86. Tartaric Acid is not found in

(a)   Tamarind

(b)   Grapes

(c)   Unripe mangoes

(d)   Spinach

Answer: (d)

87. What is the chemical name of quick lime?

(a)   Calcium carbonate

(b)   Sodium hydroxide

(c)   Calcium oxide

(d)   Calcium sulphate

Answer: (c)

88. What is the common name of CaOCl2?

(a)   Washing soda

(b)   Bleaching powder

(c)   Baking powder

(d)   Baking soda

Answer: (b)

89. In a periodic table, while moving from left to right in a period, number of ……… remains same.

(a)   electrons

(b)   protons

(c)   shells

(d)   neutrons

Answer: (c)

90. Which among the following is an Abiotic component of environment?

(a)   Green plants

(b)   Non-Green plants

(c)   Water

(d)   Parasites

Answer: (c)

91. Cellulose fibers are organic pollutant mainly produced by which Industry/Industries?

1. Mining Industry

2. Soap and detergent Industry

3. Paper and Pulp Industry

(a)   Only1

(b)   Only 2

(c)   Only 3

(d)   Both 2 and 3

Answer: (c)

92. Which among the following is not a Rabi crop?

(a)   Barley

(b)   Wheat

(c)   Groundnut

(d)   Mustard

Answer: (c)

93. For which disease, Ministry of Health and Family Welfare has launched ‘Test and Treat’ policy?

(a)   Chicken pox

(b)   HIV

(c)   Polio

(d)   Tuberculosis

Answer: (b)

94. Laser was invented by

(a)   AH Taylor

(b)   TH Maiman

(c)   Lee De Forest

(d)   Thomas Edison

Answer: (b)

95. Twiddle is associated with

(a)   Table Tennis

(b)   Lawn Tennis

(c)   Golf

(d)   Swimming

Answer: (a)

96. ‘Madhubani’ is a famous folk painting form of which Indian State?

(a)   Madhya Pradesh

(b)   Jharkhand

(c)   Bihar

(d)   Uttar Pradesh

Answer: (c)

97. Who among the following has been designated as UN messenger of peace?

(a)   Gururaja Bhat

(b)   Shrinivas Khulkarni

(c)   Malala Yousafzai

(d)   Emma Watson

Answer: (c)

98. ‘The Red Sari’ is written by

(a)   Mamta Banerjee

(b)   Javier Moro

(c)   Malala Yousafzai

(d)   Neel Mukherjee

Answer: (b)

99. Which country will host 9th BRICS summit?

(a)   Brazil

(b)   China

(c)   Russia

(d)   India

Answer: (b)

100. Which of the following is not a member of Indian Ocean Rim Association (IORA)?

(a)   Bangladesh

(b)   Mauritius

(c)   Afghanistan

(d)   India

Answer: (c)

Part C Quantitative Aptitude

101. How many factors of 108 is/are prime number?

(a)   2

(b)   3

(c)   5

(d)   6

Answer: (c)

102. Which of t he following expression (s) is/are true?

(a)   Only 1

(b)   1 and 3

(c)   Only 2

(d)   All expressions are true

Answer: (d)

103. Which of the following expression(s) is/are true?

1. 337 is a prime number

2. The number 12 has 6 positive factors.

3. 32724 is completely divisible by 9.

(a)   Only 1

(b)   1 and 2

(c)   2 and 3

(d)   All expressions are true.

Answer: (d)

104. If the product of two positive numbers be 1575 and their ratio is 7 : 9, then the greater number is

(a)   35

(b)   63

(c)   45

(d)   135

Answer: (c)

105. What is the value of (1004)2 – (998)2?

(a)   11012

(b)   12012

(c)   120012

(d)   1212

Answer: (b)

106. A and B together can complete a work in 15 days. They started together but after 5 days A left the work. If the remaining work is completed by B in 15 more days, then A alone can complete the entire work in how many days?

(a)   25

(b)   50

(c)   35

(d)   45

Answer: (d)

107. P is 20% more efficient than Q. If Q alone can finish a work in 10 days, then in how many days P alone will finish the work?

(a)   7 days

(b)   8.5 days

(c)   9 days

(d)   12 days

Answer: (b)

108. The ratio of marked price and cost price of an article is 6 : 5. If 15% discount is given, then how much is the profit?

(a)   4%

(b)   0.2%

(c)   2%

(d)   1%

Answer: (c)

109. Out of the following, which is the best discount for a customer?

1. 50% + 50% off

2. 60% + 40% off

3. 70% + 30% off

Buy 1 get 4

(a)   1 and 4

(b)   Only 2

(c)   1, 2 and 3

(d)   Only 3

Answer: (d)

110. If the average of eight consecutive even number be 93, then the greatest number among them is

(a)   86

(b)   98

(c)   100

(d)   102

Answer: (c)

111. In a mixture, milk and water are in ratio of 2 : 3. Some milk is added to the mixture because of which ratio of milk and water becomes 2 : 1. How much milk was added as a percentage of initial mixture?

(a)   75%

(b)   60%

(c)   80%

(d)   50%

Answer: (c)

112. The average of marks obtained by A and B is 15 less than that of average marks obtained by B and C. If the marks obtained by C is 65, what is the marks obtained by A?

(a)   35

(b)   50

(c)   65

(d)   80

Answer: (*)

113. What is the average of squares of first 10 natural numbers?

(a)   55.5

(b)   45.6

(c)   38.5

(d)   40.5

Answer: (c)

114. Cost price and selling price of an article are in ratio 13 : 9. If the loss incurred on article is Rs 320, what is the sum of cost and selling prices?

(a)   Rs 1480

(b)   Rs 1620

(c)   Rs 1500

(d)   Rs 1760

Answer: (d)

115. By selling 64 apples for Rs 60 a person gains 25%. In order to have 40% loss, how many apples shall he sell for Rs 36?

(a)   80

(b)   70

(c)   60

(d)   50

Answer: (a)

116. A mixture contains 18% copper by weight. How much mixture is required to obtain 81 kg of copper?

(a)   350 kg

(b)   300 kg

(c)   450 kg

(d)   250 kg

Answer: (c)

117. A person spends 30% of his income on food, 20% on children’s education and 60% of remaining on house rent. What percent of income is left with him?

(a)   20

(b)   30

(c)   35

(d)   25

Answer: (a)

118. A man rides his vehicle at the rate of 36 km/h but stops for 12 minutes, to change parts at the end of every 14th km. What will be the timed taken to cover a distance of 90 km?

(a)   6 h 12 min

(b)   2 h 30 min

(c)   3 h 42 miin

(d)   5 h 42 min

Answer: (c)

119. A walks at a uniform speed of 8 km/h and 8 hours after his start, B starts on his cycle after him at speed of 24 km/h. How far from the starting point will B catch A?

(a)   72 km

(b)   96 km

(c)   120 km

(d)   144 km

Answer: (b)

120. What is the rate (in percent) of simple interest at which a sum of money becomes three times of itself in 50 years?

(a)   2%

(b)   4%

(c)   5%

(d)   8%

Answer: (b)

121. A sum of Rs 3100 is lent out at simple interest in two parts. One at 8% per annum and another at 6% per annum. If the total annual interest is Rs 212, then what is the money lent at rate of 8%?

(a)   Rs 1000

(b)   Rs 1250

(c)   Rs 1300

(d)   Rs 1400

Answer: (c)

Directions (Q. Nos. 122-125) The bar chart representing the number of first year B.com students of st. Xavier’s college using different companies smartphones. Study bar chart and answer the questions given below.

122. The ratio of number of boys to the number of girls using the smartphones of Samsung and Sony together is

(a)   12 : 13

(b)   13 : 12

(c)   14 : 11

(d)   11 : 14

Answer: (b)

123. What percentage of boys are using the smartphones of Samsung

(a)   16.52%

(b)   17.52%

(c)   18.52%

(d)   15.52%

Answer: (c)

124. What percentage of girls are using the smartphones of Nokia

(a)   33.58%

(b)   32.58%

(c)   30.58%

(d)   31.58%

Answer: (d)

125. The difference between the total number of students using smart phones of Samsung combined together and the total number of students using smart phones of Sony taken together is

(a)   20

(b)   60

(c)   80

(d)   40

Answer: (b)

126. If length and breadth of a cuboid is increased by 20%, then by how much percent the height should be reduced to keep the volume same?

(a)   40%

(b)   44%

(c)   36%

(d)   38%

Answer: (c)

127. The volume of a conical tent is 154 cm3 and the area of its base is 38.5 cm2. What is the length of canvas required to build the tent, if the canvas is 2 cm in width?

(a)   71.35 cm

(b)   68.75 cm

(c)   73.25 cm

(d)   75.75 cm

Answer: (b)

128. A metallic sphere is melted and moulded to form conical shaped bullets. If radius of the bullet is a twice of its height and radius of bullet is half of the radius of the metallic sphere, then what is the numbers of bullet formed?

(a)   32

(b)   16

(c)   128

(d)   64

Answer: (d)

129. A chord of length 24 cm is drawn in a circle of diameter 40 cm. Another chord of length 32 cm is drawn in the same circle parallel to 24 cm long chord. What is the minimum distance between them?

(a)   4 cm

(b)   2 cm

(c)   8 cm

(d)   3 cm

Answer: (a)

130. If the length of a rectangle is increased by 50%, then by how much percent its breadth should be reduced to keep the area the same?

(a)   50%

(b)   33.33%

(c)   35%

(d)   40%

Answer: (b)

131. If  then x is equal to

(a)   5

(b)   3

(c)   2

(d)   4

Answer: (b)

132. If  then  is equal to

(a)   4/3

(b)   3/2

(c)   5/2

(d)   5/3

Answer: (c)

133. If x3 + 6x2 + 12x = 19, then what is the value of x3?

(a)   8

(b)   27

(c)   −1

(d)   1

Answer: (d)

134. One root of the quadratic equation 5x2 – 6x + 1 = 0 is equal to

(a)   2/5

(b)   3/5

(c)   4/5

(d)   1/5

Answer: (d)

135. If x2 + 16 = −4x, then what is the value of x3 – 64?

(a)   128

(b)   0

(c)   64

(d)   256

Answer: (b)

136. PQRS is a rectangle, A , B, C and D are the mid points of sides PQ, QR, RS and PS respectively. If area of ∆PQR is 48 cm2, then what is the area of ∆BCD?

(a)   24 cm2

(b)   6 cm2

(c)   16 cm2

(d)   12 cm2

Answer: (a)

137. If PA is the median of the triangle PQR and G be the centroid, then what is the ratio of (PA + GA) : (PG – GA)?

(a)   4 : 1

(b)   3 : 1

(c)   2 : 1

(d)   3 : 2

Answer: (a)

138. In a ∆PQR, PD is the median and G is centroid, If PG = 24, then what is the length of PD?

(a)   48 cm

(b)   36 cm

(c)   60 cm

(d)   72 cm

Answer: (b)

139. The tangents drawn at the point P and Q of a circle centered at O meet at A. If ∠POQ = 120°, then what is the ratio of ∠PAQ : ∠PAO?

(a)   2 : 3

(b)   4 : 1

(c)   2 : 1

(d)   5 : 2

Answer: (c)

140. ABCD is a trapezium, such that AB = CD and AD | | BC. AD = 10 cm. If the area of ABCD is 70 cm2, then what is the value of CD?

(a)   5 cm

(b)   √29 cm

(c)   √41 cm

(d)   6 cm

Answer: ()

141. What is the value of sin (30 + x) – cos(60 – x)?

(a)   1

(b)   2

(c)   0

(d)   1/2

Answer: (c)

142. What is the least value of 15 cos2θ + 17sin2θ?

(a)   14

(b)   15

(c)   2

(d)   18

Answer: (b)

143. If cos x + cos2 x = 1, (sin12 x + 3 sin10 x + 3 sin8 x + sin6 x – 1)

(a)   −1

(b)   2

(c)   0

(d)   1

Answer: (c)

144. sinθ cos θ = 1/2, (sin θ = cos θ)2

(a)   0

(b)   1

(c)   2

(d)   3

Answer: (a)

145. α + β = 90° and α : β = 2 : 1, tan α : tan β =

(a)   1 : 3

(b)   2 : 3

(c)   3 : 2

(d)   3 : 1

Answer: (d)

Directions (146-150) The given pie chart shows the monthly expenditure on various items and monthly saving of a household. The same distribution is followed for all the months of the year.

146. If monthly income is Rs 50000, then how much is spent on fuel?

(a)   Rs 12000

(b)   Rs 10000

(c)   Rs 13500

(d)   Rs 15000

Answer: (b)

147. If monthly income is Rs 65000, then what is the difference between expenditure on household items and Clothing?

(a)   Rs 5200

(b)   Rs 6500

(c)   Rs 7200

(d)   Rs 4500

Answer: (a)

148. If Rs 2400 are saved per month, then what is the monthly salary of the household?

(a)   Rs 24000

(b)   Rs 20000

(c)   Rs 30000

(d)   Rs 18000

Answer: (a)

149. If the difference in monthly expenditure on Fuel and Bills is Rs 4800, then what is the annual income of household?

(a)   Rs 660000

(b)   Rs 600000

(c)   Rs 540000

(d)   Rs 480000

Answer: (d)

150. If the difference in monthly expenditure on clothing and education is Rs 9000, then what is the difference in yearly saving and yearly expenditure on Bills?

(a)   Rs 64000

(b)   Rs 76000

(c)   Rs 36000

(d)   Rs 48000

Answer: (c)

Part D English Comprehension

Directions (Q. Nos. 151-155) Read each sentence to find out whether there is any grammatical error in it. The error if any, will be in one part of the sentence, the letter of that part will be the answer. If there is no error the answer is d i.e., No error (Ignore the error of punctuation if any)

151. Ritu was explain the (a)/ benefits of doing yoga at (b) / least four times a week. (c) No error (d)

Answer: (a)

152. After 70 years of independent many (a) / socio-economic problem is still (b) /remaining to be solved in India. (c) No error (d)

Answer: (b)

153. Gurgaon is full of tall buildings which is (a) / its strength as well as weakness (b) / depending upon how one looks at it. (c)/ No error (d)

Answer: (a)

154. Unless you do not (a) / give up bad habits (b) / you will suffer. (c) No error (d)

Answer: (a)

155. Vegetables as well as fruits (a) / have fallen considerably (b) / in prices in recent days. (c) No error (d)

Answer: (d)

Directions (Q. Nos. 156-160) In the following questions, a sentence is given with blank to be filled in with an appropriate word. Select the correct alternative out of the four and indicate it by selecting the appropriate option.

156. Everyone was greatly amazed ……… Rsoshni’s awkward behaviour during the wedding ceremony.

(a)   on

(b)   at

(c)   of

(d)   for

Answer: (b)

157. Deepak agreed …………. all my views but his parents did not.

(a)   of

(b)   with

(c)   on

(d)   to

Answer: (d)

158. It is very hot, you ……… your jackets.

(a)   need not bring

(b)   need not have brought

(c)   need not brought

(d)   need not had been brought

Answer: (b)

159. I do not know where Akansha lives because I have been ……… of touch for quite sometime.

(a)   out

(b)   hardly

(c)   scarcely

(d)   her

Answer: (a)

160. Rahul was ………. with a natural talent for painting.

(a)   given

(b)   entrusted

(c)   showered

(d)   endowed

Answer: (d)

Directions (Q. Nos. 161-165) In the following questions out of the four alternatives, select the word similar in meaning to the given word in bold.

161. Fatuous

(a)   Prudent

(b)   Irrational

(c)   Senile

(d)   Renewal

Answer: (b)

162. Bustle

(a)   Quiet

(b)   Fragile

(c)   Haste

(d)   Gloomy

Answer: (c)

163. Sauciness

(a)   Assistance

(b)   Impudence

(c)   Meekness

(d)   Venerate

Answer: (b)

164. Congruous

(a)   Absurd

(b)   Balanced

(c)   Hidden

(d)   Diligent

Answer: (b)

165. Contradiction

(a)   Sanction

(b)   Repose

(c)   Despicable

(d)   Confrontation

Answer: (d)

Directions (Q. Nos. 166-170) In the following questions out of the four alternatives select the word opposite in meaning to the word given in bold.

166. Trivial

(a)   Shallow

(b)   Profound

(c)   Immature

(d)   Reckless

Answer: (b)

167. Flagitious

(a)   Immoral

(b)   Virtuous

(c)   Profligate

(d)   Displease

Answer: (b)

168. Abridge

(a)   Stretch

(b)   Curtail

(c)   Pacify

(d)   Violate

Answer: (a)

169. Solicit

(a)   Sanctity

(b)   Scorn

(c)   Prohibit

(d)   Disguise

Answer: (c)

170. Substantial

(a)   Doubtful

(b)   Reliable

(c)   Florid

(d)   Enormous

Answer: (a)

Directions (Q. Nos. 171-175) In the following questions, out of the four alternatives, select the alternative which best expresses the meaning of the idiom/phrase.

171. A green horn

(a)   An inexperienced man

(b)   A dangerous man

(c)   Suspicious man

(d)   A wise person

Answer: (a)

172. To paddle once own canoe

(a)   Thoughtful

(b)   Manage independently

(c)   To praise oneself

(d)   Working vigorously

Answer: (b)

173. To keep the pot

(a)   To sharper my wits

(b)   To be cautious

(c)   To keep controversy alive

(d)   To repent

Answer: (c)

174. To beat the air

(a)   To settle a dispute

(b)   To gamble

(c)   To make efforts that are useless

(d)   To try to do the impossible

Answer: (c)

175. Threw a spanner

(a)   Reverie

(b)   Disinterested

(c)   Sabotage

(d)   Boasting

Answer: ()

Directions (Q. Nos. 176-180) Improve the bracketed part of the sentence.

176. Sunil didn’t pay any heed to his superior’s instruction, (I did neither).

(a)   nor did I

(b)   neither didn’t I

(c)   I either did

(d)   No improvement

Answer: (a)

177. The Arabian Nights (have been written) by Edward William Lane.

(a)   are being written

(b)   has been written

(c)   have written

(d)   No improvement

Answer: (b)

178. Ashina (hesitated to listen to) what her elder sister was saying.

(a)   hesitating to listen to

(b)   hesitates in listening to

(c)   hesitate listened to

(d)   No improvement

Answer: (d)

179. Nitin assured Ankit that he (would look at) his work while he was on leave.

(a)   would over look

(b)   would look after

(c)   will look

(d)   No improvement

Answer: (b)

180. (When the weather was) cold, I wear monkey cap.

(a)   When weather is

(b)   Why the weather is

(c)   While the weather was

(d)   No improvement

Answer: (a)

Directions (Q. Nos. 181-185) In the following questions, out of the four alternatives, select the alternative which is the best substitute for the phrase.

181. Life less objects

(a)   Amateur

(b)   Inanimate

(c)   Alien

(d)   Amnesty

Answer: (b)

182. A court or open space usually rectangular and enclosed by a building

(a)   Quadragenarian

(b)   Quadrilateral

(c)   Quadrangle

(d)   Quadruped

Answer: (c)

183. Bringing about gentle and painless death from incurable disease

(a)   Euthanasia

(b)   Sty

(c)   Canicide

(d)   Suicide

Answer: (a)

184. One who lives on fish

(a)   Carnivorous

(b)   Baccivorous

(c)   Aurivorous

(d)   Piscivorous

Answer: (d)

185. Line at which the Earth or sea and sky seem to meet

(a)   Nadir

(b)   Zenith

(c)   Horizon

(d)   Trivet

Answer: (c)

Directions (Q. Nos. 186-190) In the following questions, four words are given out which one word is incorrectly spelt. Select the incorrectly spelt word.

186.

(a)   Stratagem

(b)   Repentence

(c)   Corrigendum

(d)   Alliteration

Answer: (b)

187.

(a)   Napthalene

(b)   Impeccable

(c)   Stupefy

(d)   Psychiatrist

Answer: (a)

188.

(a)   Fulfil

(b)   Wilful

(c)   Skelful

(d)   Shameful

Answer: (c)

189.

(a)   Sergant

(b)   Pedestal

(c)   Scavenger

(d)   Luminescent

Answer: (a)

190.

(a)   Mellifluence

(b)   Centenarian

(c)   Plebisite

(d)   Analysable

Answer: (c)

Directions (Q. Nos. 191-195) In the following passage some of the words have been left out. Read the passage carefully and select the correct answer for the given blank out of the four alternatives.

A …………… is made, not born. The process begins when someone hits upon a bright new way of stating a ………. experience. At what point the remark is an ………….. But it is particularly ……….. as well as catchy, the saying receives wide circulation as verbal coin. Soon it is likely to be ………. from overwork.

191. A ………… is made, not born.

(a)   nuance

(b)   coinage

(c)   cliche

(d)   current

Answer: (c)

192. New way of starting a ………. experience.

(a)   common

(b)   authentic

(c)   mental

(d)   rational

Answer: (a)

193. The remark is an ………….

(a)   epigram

(b)   epitaph

(c)   epilogue

(d)   epicenter

Answer: (a)

194. It is particularly ……….. as well as catchy

(a)   vague

(b)   apt

(c)   ingenuine

(d)   emotional

Answer: (b)

195. Likely to be ……… from overwork.

(a)   judged

(b)   suffering

(c)   overlooked

(d)   inducing

Answer: (b)

Directions (Q. Nos. 196-200) A passage is given with five questions following it. Read the passage carefully and select the best answer to each question out of the given four alternatives.

The rural economy is an important segment of the ecosystem and accounts for around 70% of employment and 50% of GDP with agriculture being the main driver followed by services and manufacturing. It is largely unorganized and hence those working in rural India or consuming in this economy are a different category of economic agents driven by different factors. The economy is quite complex in so far as the fact that cash dominates in terms of transactions and while there has been some intrusion of the  use of credit (Kishan) and debit cards and ATMs, dependence on technology driven payments system is limited.

This has hence also become a haven four routing black money both in terms of seeking tax exemptions by channeling funds, to convert to legitimate funds. But a lot of black money gets into land and ‘apparent rural activity’.

Now consider some aspects of this economy and the cash conundrum. Almost all transactions in the mandis (there are above 7000 organized ones and over double the number that is unorganized), are based on cash as it is easy to use. The farmers prefer to receive cash and while some do take in cheques there is a sense of doubt if the counter party in unknown. Hence one reason why electronic mandis is a good idea is that payments can also be made through the electronic mode as all transactions would be e-enabled. The recent demonetization has caused significant distortions as farmer are unable to sell their goods. This has happened just when we are in middle of the kharif harvest which involved rice, soybean, cotton, maize, sugarcane, bajra, besides fruits and vegetables which are all year through. The second issue Indian agriculture is the rabi season which has begun where farmers start sowing their seeds. The issue here is less serious as a large part is backed by credit where the prevalent cash crunch may not matter. It would only be at the margin that farmers may be impacted, and hence the pain here would be secondary.

196. According to the passage, which among the following is the major contributor to India’s GDP growth?

(a)   Services

(b)   Agriculture

(c)   Manufacturing

(d)   FDIs

Answer: (b)

197. What has become a haven for routing black money in India?

(a)   Cash transactions over digital cash transactions

(b)   Intrusion of credit cards

(c)   Prevailing debit cards

(d)   Technology driven payment

Answer: (a)

198. Which among the following is not a kharif crop?

(a)   Bajra

(b)   Maize

(c)   Rice

(d)   Wheat

Answer: (d)

199. Why does demonetization has lesser impact onto the rabi season?

(a)   Due to tax exemptions

(b)   Due to digital payments

(c)   Due to credit payments

(d)   Huge margins

Answer: (c)

200. Which of the following is not true in regards to rural economy?

(a)   It account for major percent in employmenting the people of the country.

(b)   It has a technology driven mandis.

(c)   Major transactions are based on cash.

(d)   Demonetization has caused significant distortions in rural economy especially to farmers.

Answer: (b)

SSC Stenographers (Grade ‘C’ and ‘D’) Examination Held on January-2016 Question Paper With Answer Key

SSC Stenographers (Grade 'C' and 'D') Examination Held on January-2016
SSC Stenographers (Grade ‘C’ and ‘D’) Examination Held on January-2016 Question Paper With Answer Key

SSC Stenographers (Grade ‘C’ and ‘D’) Examination Held on January-2016

PART-I : GENERAL INTELLIGENCE & REASONING

1. Rajan started from his house and went 2 km east, then he turned right and went 3 km, again turned right and went 3 km, again he turned right and covered 1 km and finally after turning right and going 1 km he reached point ‘K’. In which direction was Rajan walking when he reached there?

(A) North

(B) East

(C) West

(D) South

Answer: (B)

2. 4 friends, A, B, C, D are sitting in a coffee shop. A and B are sitting face to face. D is not sitting next to A but can see the facial expression of A clearly. B is talking to C who is sitting opposite to him. Who are sitting together?

(A) A and D

(B) A and C

(C) D and C

(D) A and B

Answer: (B)

Directions (Qs. 3 to 11) : In these questions, select the related word/letters/number from the given alternatives.

3. Yard : Inch : : Quart : ?

(A) Milk

(B) Ounce

(C) Liquid

(D) Gallon

Answer: (B)

4. CAFD : IGLJ : : OMRP : ?

(A) USXV

(B) PONM

(C) ZYXW

(D) UTSR

Answer: (A)

5. Optimist : Cheerful : : Pessimist : ?

(A) Gloomy

(B) Helpful

(C) Petty

(D) Mean

Answer: (A)

6. 6 : 64 : : 27 : ?

(A) 64

(B) 125

(C) 81

(D) 212

Answer: (B)

7. 27 : 9 : : 64 : ?

(A) 16

(B) 20

(C) 12

(D) 8

Answer: (A)

8. 4 : 20 : : 6 : ?

(A) 8

(B) 36

(C) 12

(D) 42

Answer: (D)

9. Pesticide : Crop : : Antiseptic : ?

(A) Bleeding

(B) Wound

(C) Clothing

(D) Bandage

Answer: (B)

10. CIQY : DJRZ : : AGOW : ?

(A) BHPU

(B) BJPX

(C) BHPX

(D) BJRZ

Answer: (C)

11. GHJM : QRTW : : CDFI : ?

(A) NOPR

(B) GHIJ

(C) MNPS

(D) XYZQ

Answer: (C)

12. X is elder than Z, Y is younger than Z, Z is elder than W, W is younger than X, who is the eldest?

(A) W

(B) Y

(C) Z

(D) X

Answer: (D)

13. The following figure is folded to form a block. Which symbol will appear on the opposite of ▲?

(A) *

(B) ●

(C) ○

(D) ■

Answer: (C)

Directions (Qs. 14 to 17) : In these question, select the missing number from the given responses.

14. 

(A) 11

(B) 33

(C) 22

(D) 12

Answer: (C)

15. 2     7        14

     3     4         5

    75    165     ?

(A) 185

(B) 165

(C) 285

(D) 425

Answer: (C)

16. 

(A) 10

(B) 20

(C) 15

(D) 40

Answer: (B)

17. 

(A) 11

(B) 15

(C) 10

(D) 20

Answer: (A)

18. From the given alternative words, select the word which can be formed using the letters of the given word.

Measurement

(A) Assure

(B) Mantle

(C) Summit

(D) Master

Answer: (D)

Directions (Qs. 19 to 22) : In these questions, a series is given with one (or more) term missing. Choose the correct alternative from the given ones that will complete the series :

19. 720, 180, 176, 44, 40, 10, ?, ?

(A) 6, 4

(B) 4, 2

(C) 6, 1.5

(D) 8, 6

Answer: (C)

20. 5, 10, 20, 40, 80, ?

(A) 150

(B) 120

(C) 140

(D) 160

Answer: (D)

21. C4X, F9U, I16R, ?

(A) L27P

(B) K25P

(C) L25O

(D) L25U

Answer: (C)

22. 15, 21, 57, ?, 221

(A) 96

(B) 126

(C) 121

(D) 108

Answer: (C)

23. What do you infer from the following set of statements?

I have observed many plant roots.

All these plant roots were found to grow downward.

(A) I am a botanist.

(B) A plant root grows downward.

(C) Most of the plant roots grow downward.

(D) Some plant roots grow downward.

Answer: (D)

Directions (Qs. 24 to 30): In these questions, find the odd word/number/letters/number pair from the given alternatives.

24. Find the odd number pair from the given alternatives.

(A) 112, 95

(B) 167, 150

(C) 84, 67

(D) 79, 63

Answer: (D)

25. Find the odd word from the given alternatives.

(A) Tent

(B) Lodge

(C) Inn

(D) Mansion

Answer: (A)

26. Find the odd letters from the given alternatives.

(A) AKWE

(B) LUCF

(C) JHMX

(D) PNTO

Answer: (C)

27. Find the odd number pair from the given alternatives.

(A) 24, 44

(B) 28, 82

(C) 36, 63

(D) 99, 99

Answer: (B)

28. Find the odd letters from the given alternatives.

(A) IJSO

(B) CBUV

(C) RQOP

(D) YXTU

Answer: (A)

29. Find the odd word from the given alternatives.

(A) Triangle

(B) Sphere

(C) Square

(D) Circle

Answer: (B)

30. Find the odd word from the given alternatives.

(A) Beak

(B) Wing

(C) Fin

(D) Rudder

Answer: (D)

31. Unscramble the following letters to frame a meaningful word, then find out the correct numerical position of the letters:

I           N       R       D       T       P       E       E       S

1          2        3        4        5        6        7        8        9

(A) 756432189

(B) 637914825

(C) 639185251

(D) 735124389

Answer: (B)

32. A piece of paper is folded and punched as shown below in the question figure. From the given answer figures, indicate how it will appear when opened.

Answer: (D)

33. If DANGER is coded as

11 – 8 – 21 – 14 – 12 – 25,

then how will be coded the word MACHINE?

(A) 20 – 8 – 10 – 16 – 17 – 22 – 13

(B) 20 – 10 – 8 – 12 – 15 – 16 – 7

(C) 10 – 21 – 15 – 14 – 26 – 17 – 18

(D) 20 – 8 – 10 – 15 – 16 – 21 – 12

Answer: (D)

34. Which number is wrong in the given series?

225, 169, 144, 100, 64, 48

(A) 144

(B) 64

(C) 48

(D) 225

Answer: (C)

Directions (Qs. 35 & 36) : In these questions, which one set of letters when sequentially placed at the gaps in the given letter series shall complete it?

35. bb_aab_caab_ca_

(A) bcab

(B) cbba

(C) abbc

(D) acab

Answer: (B)

36. _cdb_ddb_db_c_d

(A) cccbc

(B) bbbcc

(C) ccbcc

(D) bbcbb

Answer: (A)

37. A’s mother is sister of B and has a daughter C.

How can A be related to B from among the following?

(A) Father

(B) Uncle

(C) Daughter

(D) Niece

Answer: (D)

38. Choose the correct figure to complete the matrix:

Answer: (B)

39. If the following words are arranged in a dictionary order, which will be 3rd word?

Amphibian, Amorphous, Ambiguous, Ambidextrous, Ambivalent

(A) Ambiguous

(B) Amphibian

(C) Amorphous

(D) Ambivalent

Answer: (A)

40. If the number indicates the number of persons, then how many youth graduates are there?

(A) 20

(B) 30

(C) 40

(D) 50

Answer: (B)

41. What will be the correct mathematical signs that can be inserted in the following?

4_6_2_4_8 = 16

(A) ÷ + × −

(B) × ÷ − +

(C) + ÷ − ×

(D) − × + ÷

Answer: (B)

42. In the following questions, number of letters skipped in between adjacent letters of the series starting from behind increased by one. Which of the following observes the rule?

(A) OMKIG

(B) OIGDC

(C) OMJFA

(D) ONLKJ

Answer: (C)

43. A word is represented by only one set of numbers is given in any one of the alternatives. The sets of numbers given in the alternatives are represented by two classes of alphabets as in two matrices given The columns and rows of Matrix I are numbered from 0 to 3 and that of Matrix II are numbered from 4 to 7. A letter from these matrices can be represented first by its row and next  by its column, e.g., D can be represented by 01 and R can be represented by 44. Similarly, you have to identify the set for the word ‘TALE’.

(A) 00, 31, 64, 32

(B) 64, 00, 31, 32

(C) 46, 13, 00, 23

(D) 30, 76, 23, 32

Answer: (B)

44. Three statements are given followed by three conclusions I, II and III. You have to consider the three statements to be true even if they seem to be at variance from commonly known facts. You have to decide which o f the given conclusions, if any, follow from the given statements.

Statements:

1. Mary said, “Ann and I both have cats.”

2. Ann said, “I don’t have a cat.”

3. Mary always tells the truth, but Ann sometimes lies.

Conclusions :

I. Ann has a cat.

II. Mary has a cat.

III. Ann is lying.

(A) II only

(B) I only

(C) I, II and III

(D) I and II only

Answer: (C)

45. If a mirror is placed on the line MN, then which of the answer figures is the right image of the given figure?

Answer: (A)

46. From the given answer figures, select the one in which the question figure is hidden/embedded.

Answer: (A)

47. Arrange the following words according to the dictionary order:

a. extol b. extinct

c. extra d. extort

(A) a, d, b, c

(B) a, b, c, d

(C) b, a, d, c

(D) d, c, a, b

Answer: (C)

48. Which will appear fourth in the dictionary?

(A) deterioration

(B) degrade

(C) determination

(D) density

Answer: (C)

49. If – denotes +

+ denotes ×

÷ denotes –

× denotes ÷

(A) 15

(B) 35

(C) 14.5

(D) 3.5

Answer: (*)

50. If Blue means Pink, Pink means Green, Green means Yellow, Yellow means Red and Red means White, then what is the colour of turmeric?

(A) Yellow

(B) Green

(C) Red

(D) Pink

Answer: (C)

PART – II : GENERAL AWARENESS

51. Which one among the following books is centred around “environment”?

(A) And Then One Day

(B) Here I Stand

(C) Silent Spring

(D) The Late, Great Planet Earth

Answer: (C)

52. The first person ever to reach the South Pole was

(A) Magellan

(B) Peary

(C) Amundsen

(D) Amerigo Vespucci

Answer: (C)

53. The island of Honshu in Japan is famous for

(A) Iron ore

(B) Diamonds

(C) Coal

(D) Oil

Answer: (D)

54. 40°N latitude acts as demarcation line between

(A) Egypt and Sudan

(B) North and South Vietnam

(C) USA and Canada

(D) North and South Korea

Answer: (B)

55. ‘Intervening opportunities model’ was proposed by

(A) Davis

(B) S. A. Stouffer

(C) E. S. Lee

(D) Revenstein

Answer: (B)

56. DHCP is mainly used in

(A) Routing

(B) Converting IP address to domain name

(C) Providing IP address automatically to the devices

(D) Multicasting

Answer: (C)

57. Who first gave the concept of ‘Distributive Justice’?

(A) Machiavelli

(B) Aristotle

(C) Plato

(D) Locke

Answer: (A)

58. Which blood vessels carry pure blood from the lungs to the heart?

(A) Pulmonary veins

(B) Cardiac artery

(C) Cardiac vein

(D) Pulmonary arteries

Answer: (A)

59. The antacid marketed as milk of magnesia has as main ingredient:

(A) MgSO4

(B) MgCl2

(C) Mg(OH)2

(D) MgCO3

Answer: (C)

60. ‘National Science Day’ is observed on

(A) 28th February

(B) 5th January

(C) 14th March

(D) 2nd June

Answer: (A)

61. Who fixes the REPO rate in India?

(A) IMF – International Monetary Fund

(B) RBI – Reserve Bank of India

(C) WTO – World Trade Organization

(D) SEBI – Securities and Exchange Board of India

Answer: (B)

62. August Weismann proposed

(A) Inheritance of acquired character

(B) Natural selection theory

(C) Germ plasm theory

(D) Modern synthetic theory

Answer: (C)

63. The book ‘Gokhale, My Political Guru’ was written by

(A) C. R. Das

(B) M. A. Jinnah

(C) Shaukat Ali

(D) M. K. Gandhi

Answer: (D)

64. Philadelphia is famous for

(A) Ship-building

(B) Silk textiles

(C) Locomotives

(D) Diary industry

Answer: (C)

65. The market system in which there are only two buyers facing a large number of sellers is called

(A) monopsony

(B) duopoly

(C) oligopoly

(D) duopsony

Answer: (B)

66. Bodo and Dogri were added in the 8th Schedule by the following Amendment:

(A) 81st Amendment

(B) 92nd Amendment

(C) 85th Amendment

(D) 91st Amendment

Answer: (B)

67. Sir Eyre Coote was associated with which of the following?

(A) Battle of Ambur

(B) Seize of Arcot

(C) Battle of Adyar

(D) Battle of Wandiwash

Answer: (D)

68. The largest proven oil reserve of the word lies in

(A) Saudi Arabia

(B) Iraq

(C) Iran

(D) Venezuela

Answer: (D)

69. Which among the following is related Sadr-us-Sadr?

(A) Ecclesiastical matters

(B) Military administration

(C) Land revenue

(D) Judicial administration

Answer: (D)

70. One of the following is not a component of foreign exchange reserves in India:

(A) Gold stock of RBI

(B) Foreign exchange assets of RBI

(C) SDR holdings of government

(D) Foreign exchange assets of government

Answer: (B)

71. Who amongst following is the author of the book ‘Namesake’?

(A) Shobha De

(B) Kiran Desai

(C) Vikram Seth

(D) Jhumpa Lahiri

Answer: (D)

72. The original name of Mahabharata is

(A) Kathasaritsagar

(B) Jai Samhita

(C) Rajtarangini

(D) Bharat Katha

Answer: (B)

73. A bag is dropped from an aeroplane flying horizontally at a constant speed. Neglecting air resistance, where will the aeroplane be when the bag reaches the ground?

(A) Data is not sufficient

(B) Behind the bag

(C) Ahead of the bag

(D) Directly above the bag

Answer: (D)

74. Which among the following inscription is known as ‘Prayaga Prashasti”?

(A) Mehroli Inscription

(B) Aihole Inscription

(C) Allahabad Pillar Inscription

(D) Hathigumpha Inscription

Answer: (C)

75. When the velocity of a body is doubled

(A) its K.E. is doubled.

(B) its acceleration is doubled.

(C) its P.E. is doubled.

(D) its momentum is doubled.

Answer: (D)

76. Among the following quantities which one has dimensions different from the remaining three?

(A) Angular momentum per unit mass

(B) Force per unit area

(C) Energy per unit volume

(D) Product of voltage and charge per unit volume

Answer: (A)

77. Which of the following units is used to measure the speed of a computer?

(A) BAUD

(B) MIPS

(C) SYPS

(D) Byte

Answer: (B)

78. An indirect instrument of monetary policy is

(A) Statutory Liquidity Ratio

(B) Cash Reserve Ratio

(C) Bank Rate

(D) Open market operations

Answer: (D)

79. The book “Problem of Human Geography” was written by

(A) Jean Brunches

(B) De Mortonne

(C) Albert Demangeon

(D) None of these

Answer: (C)

80. Kudankulam Project is located in which State?

(A) Tamil Nadu

(B) Karnataka

(C) Telangana

(D) Kerala

Answer: (A)

81. Which was the first National News Agency of free India?

(A) The Indian Review

(B) The Associated Press of India

(C) The free Press of India

(D) None of these

Answer: (C)

82. Who was the first ‘black’ actor to win Oscar?

(A) Morgan Freeman

(B) Wesely Snipes

(C) Sidney Politiers

(D) Eddie Murphy

Answer: (C)

83. Arrange the names of the Presidents in the order they served.

(A) N. S. Reddy, Giani Zial Singh, R. Venkataraman, Dr. Shankar Dayal Sharma

(B) N. S. Reddy, R. Venkataraman, Giani Zail Singh, Dr. Shankar Dayal Sharma

(C) N. S. Reddy, Dr. Shankar Dayal Sharma R. Venkataraman, Giani Zail Singh

(D) R. Venkataraman, Dr. Shankar Dayal Sharma, Giani Zial Singh, N. S. Reddy

Answer: (A)

84. The reorganization of States on linguistic basis was done in

(A) 1952

(B) 1951

(C) 1956

(D) 1950

Answer: (C)

85. Kala-azar is transmitted by

(A) Sand fly

(B) Mites

(C) Black flies

(D) Tsetse fly

Answer: (A)

86. The largest tea growing country in the world is

(A) India

(B) China

(C) Brazil

(D) Sri Lanka

Answer: (B)

87. Presence of excess fluorine in water causes

(A) Respiratory disease

(B) Tooth decay

(C) Dental cavity

(D) Fluorosis

Answer: (B)

88. Who among the following was responsible for the founding of the Anglo-Muhammadan Oriental College?

(A) Altaf Husain

(B) Sir Sayyid Ahmad Khan

(C) Muhammad Iqbal

(D) Yusuf Ali

Answer: (B)

89. Which of the following are used to prepare the main storage (starch) form of food in plants?

(A) Carbon dioxide and oxygen

(B) Carbon dioxide and nitrogen

(C) Carbon dioxide and water

(D) Water and oxygen

Answer: (C)

90. The chemical name of quick lime is

(A) Calcium chloride

(B) Calcium oxide

(C) Calcium carbonate

(D) Calcium hydroxide

Answer: (B)

91. Which of the following breeds of chicken belongs to Mediterranean class?

(A) Sussex

(B) Brahma

(C) Leghorn

(D) Sustra lorp

Answer: (C)

92. The term ‘politics’ was first used by

(A) Plato

(B) Aristotle

(C) John Locke

(D) Socrates

Answer: (B)

93. Which of the following pair is not correctly matched?

(A) Nikitin – Samarkand

(B) Marco Polo – Italy

(C) Alberuni – Uzbekistan

(D) Ibn Batuta – Morocco

Answer: (A)

94. Bernoullis theorem is a statement of the conservation of

(A) Linear momentum

(B) Mass

(C) Energy

(D) Pressure

Answer: (C)

95. An international movement with its motto to save the world by involving itself with environmental problems is

(A) Clean En

(B) Green-field

(C) Eco-friend

(D) Green-peace

Answer: (D)

96. Fibrinogen is converted into fibrin by

(A) thrombin

(B) prothrombin

(C) thromboplastim

(D) thrombokinase

Answer: (A)

97. Match each Element (List-2) given below with its Property (List-1) given above.

Property (List-1)

(a) A dark red liquid

(b) A metal used in jewellery

(c) A totally inert gas

(d) A reactive metal that explodes in water

Element (List-2)

(i) Neon         (ii) Bromine

(iii) Sodium   (iv) Gold

(v) Nitrogen

(A) a-(ii), b-(iv), c-(i), d-(iii)

(B) a-(i), b-(ii), c-(iii), d-(iv)

(C) a-(iii), b-(ii), c-(v), d-(iv)

(D) a-(i), b-(iii), c-(iv), d-(v)

Answer: (A)

98. It is reported that there is an ongoing decrease in the pH value of ocean water because of global warming. It happens due to

(A) Lesser uptake of atmospheric nitrogen by ocean water.

(B) Larger uptake of CO2 by ocean water.

(C) Larger uptake of atmospheric nitrogen by ocean water.

(D) Lesser uptake of CO2 by ocean water.

Answer: (B)

99. Which of the following is related to horticulture?

(A) Operation flood

(B) Golden revolution

(C) White revolution

(D) Green revolution

Answer: (D)

100. Lakes which contain high concentration of humic acid in water is

(A) Dystrophic lakes

(B) Desert salt lakes

(C) Deep ancient lakes

(D) Volcanic lakes

Answer: (A)

PART-III : ENGLISH LANGUAGE

Directions (Qs. 101 to 110) : In these question, some parts of the sentences have errors and some are correct. Find out which part of a sentence has an error and mark that part as your answer. If a sentence is free from error, then your answer is that  part which represents “No Error”.

101. The children left the playground one after one.

(A) The children left

(B) one after one

(C) the playground

(D) No error

Answer: (B)

102. I knew the town well so I was able to advice him where to go.

(A) No error

(B) to advice him where to go.

(C) so I was able

(D) I knew the town well

Answer: (B)

103. You will not succeed unless you don’t work hard.

(A) You will not

(B) succeed unless you

(C) don’t work hard.

(D) No error

Answer: (C)

104. She has been complaining about headache from morning.

(A) She has been

(B) from morning

(C) complaining about headache

(D) No error

Answer: (C)

105. He stated that he prefers tea than coffee.

(A) He stated that

(B) No error

(C) tea than coffee.

(D) he prefers

Answer: (D)

106. Not much people realize his sincerity.

(A) No much

(B) his sincerity.

(C) No error

(D) people realize

Answer: (A)

107. The fight for liberation brings out the best and a noblest quality in mankind.

(A) No error

(B) a noblest quality in mankind.

(C) brings out the best and

(D) The fight for liberation

Answer: (B)

108. He runs more faster than I.

(A) than I.

(B) No error

(C) He runs

(D) more faster

Answer: (D)

109. The baby was clinging with her mother in fear.

(A) mother in fear.

(B) The baby was

(C) clinging with her

(D) No error

Answer: (C)

110. The government must provide facilities for the upbringing of women.

(A) The government

(B) upbringing of women.

(C) must provide facilities for the

(D) No error

Answer: (B)

Directions (Qs. 111 to 115) : In these questions, out of the four alternatives, choose the one which best expresses the meaning of the given word.

111. Brutal

(A) Humane

(B) Savage

(C) Compassionate

(D) Sympathetic

Answer: (B)

112. Stroll

(A) Trat

(B) Walk

(C) Jog

(D) Gallop

Answer: (B)

113. Revel

(A) Glory

(B) Make Merry

(C) Revert

(D) Reveal

Answer: (B)

114. Eternal

(A) Forever

(B) Short term

(C) Temporary

(D) Time being

Answer: (A)

115. Adversity

(A) Spirituality

(B) Seniority

(C) Misery

(D) Familiarity

Answer: (C)

Directions (Qs. 116 to 120) : In these questions, choose the word opposite to the meaning to the given word.

116. Exaggerate

(A) Abundance

(B) Extravagant

(C) Understate

(D) Excerpt

Answer: (C)

117. Abrupt

(A) Sudden

(B) Spongy

(C) Smooth

(D) Crisp

Answer: (C)

118. Formal

(A) Pitiable

(B) Plain

(C) Heedful

(D) Informal

Answer: (D)

119. Thorough

(A) Detailed

(B) Utter

(C) Cursory

(D) Intensive

Answer: (C)

120. Nasty

(A) Unpleasant

(B) Pleasant

(C) Beautiful

(D) Ugly

Answer: (B)

Directions (Qs. 121 to 125) : Read the passage carefully and choose the best answer to each question out of the four alternatives.

    Like watering a plant, we grow our friendships (and all our relationships) by nurturing them. Friendships need the same attention as other relationships, if they are to continue. These relationships can  be delightfully non-judgemental, supportive, understanding and fun.

Sometimes a friendship can bring out the positive side that you never show in any other relationship. This may be because the pressure of playing a ‘role’ (daughter, partner or child) is removed. With a friend you are to be yourself and free to change. Of course, you are free to do this in all other relationships as well, but in friendships you get to have lots of rehearsals and discussion about changes as you experience them. It is an unconditional experience where you receive as much as you give. You can explain yourself to a friend openly without the fear of hurting a family member. How do friendships grow ? The answer is simple. By revealing yourself; being attentive; remembering what is most showing empathy, seeing the world through the eyes of your friend, you will understand the value of friendship. All this means learning to accept a person from a completely different family from your own or perhaps someone from a completely different cultural background. This is the way we learn tolerance. In turn we gain tolerance and acceptance for our own differences.

121. Empathy means

(A) someone else’s misfortunes.

(B) ability to do something

(C) the ability to share and understand another’s feelings.

(D) skill and efficiency.

Answer: (C)

122. When we are with a good friend, we tend

(A) to be ourselves.

(B) to shut ourselves.

(C) not to  be ourselves.

(D) to be someone else.

Answer: (A)

123. In good friendships, we

(A) only receive.

(B) only give.

(C) give and receive.

(D) neither give nor receive.

Answer: (C)

124. Friendships and relationships grow when they are __.

(A) favoured

(B) divided

(C) nurtured

(D) compared

Answer: (C)

125. Through strong friendships, we gain

(A) only acceptance.

(B) acceptance and tolerance.

(C) only attention.

(D) only tolerance.

Answer: (B)

Directions (Qs. 126 to 135) : In these questions, a sentence has been given  in Direct/Indirect. Out of the four alternatives suggested, select the one which best expresses the same sentence in Indirect/Direct.

126. My mother said, “You have been sleeping for twelve hours.”

(A) My mother said that I have been sleeping for twelve hours.

(B) My m other said that I had been sleeping for twelve hours.

(C) My mother said that she had been sleeping for twelve hours.

(D) My mother said that I was sleeping for twelve hours.

Answer: (B)

127. Dhanya complained, “I am losing weight steadily.”

(A) Dhanya complained that she is losing weight steadily.

(B) Dhanya complains that she was losing weight steadily.

(C) Dhanya complained that she was losing weight steadily.

(D) Dhanya complains that she is losing weight steadily.

Answer: (C)

128. I said to my friend, “Can you pick me up after work ?”

(A) I asked my friend if he can pick me up after work.

(B) I told my friend if I could pick him up after work.

(C) I told my friend to pick me up after work.

(D) I asked my friend if he could pick me up after work.

Answer: (D)

129. Rajan said to Urvashi, “I’am going to Kashmir tomorrow.”

(A) Rajan told to Urvashi he was going to Kashmir the next day.

(B) Rajan said to Urvashi that the was going to Kashmir tomorrow.

(C) Rajan told Urvashi that he was going to Kashmir the next day.

(D) Rajan said Urvashi I am going to Kashmir tomorrow.

Answer: (C)

130. Mary said to Simon, “Sharon and Peter are getting engaged next month.”

(A) Mary told Simon that Sharon and Peter will  be getting engaged next month.

(B) Mary told Simon that Sharon and Peter are getting engaged next month.

(C) Mary told Simon that Sharon and Peter were getting engaged next month.

(D) Mary told Simon that Sharon and Peter was getting engaged next month.

Answer: (C)

131. She said to me, “Have you finished your work?”

(A) She asked me if I had finished my work.

(B) She asked me if I have finished my work.

(C) She asked me if she had finished her work.

(D) She asked me if she had finished my work.

Answer: (A)

132. My friend told me, “This is not a good book to read.”

(A) My friend told me that is not a good book to read.

(B) My friend told me that will not be a good book to read.

(C) My friend told me that this was not a good book to read.

(D) My friend told me that was not a good book to read.

Answer: (D)

133. Suman said to me, “Did you enjoy the Olympic Games in London?”

(A) Suman asked me if I was enjoyed the Olympic Games in London.

(B) Suman asked me if I enjoyed the Olympic Games in London.

(C) Suman asked me did I enjoy the Olympic Games in London.

(D) Suman asked me if I had enjoyed the Olympic Games in London.

Answer: (D)

134. “Turn around,” he told her.

(A) He told her turn around.

(B) He asked turn around.

(C) He asked her to turn around.

(D) He asked to her to turn around.

Answer: (C)

135. He said, “I have been studying in this college for two years.”

(A) He said that he had been studying in that college for two years.

(B) He said he studied in that college for two years.

(C) He said for two years he studied in that college.

(D) He said he had studied in that college for two years.

Answer: (A)

Directions (Qs. 136 to 145) : In these questions, a  part of the sentence is underlined. Below are given alternatives to the underlined part which may improve the sentence. Choose the correct alternative. In case no improvement is required, choose “No Improvement” option.

136. Tom was standing besides the school-house poster, when the ball rolled towards him.

(A) stood beside the school-house poster

(B) standing beside the school-house poster

(C) No Improvement

(D) stand besides the school house poster

Answer: (B)

137. She stood by him under all conditions and undaunted by anything.

(A) No Improvement

(B) through thick and him

(C) without hesitation

(D) without reserve

Answer: (B)

138. The boat was drowned.

(A) was sink

(B) No Improvement

(C) was drown

(D) was sunk

Answer: (B)

139. No sane person or government can tolerate terrorists of any degree or kind.

(A) terrorism of any degree or kind

(B) terrorise of any degree or kind

(C) torture of any degree or kind

(D) No Improvement

Answer: (A)

140. Saibal has got an extensive to finish writing his thesis.

(A) an extension

(B) No Improvement

(C) an extension

(D) an extended

Answer: (A)

141. Either Kiran or Mala is sure to be chosen for the school debate team.

(A) Either Kiran nor Mala are sure to

(B) Either Kiran or Mala are sure to

(C) Either Kiran or Mala will

(D) No Improvement

Answer: (D)

142. Sheela was reprimanded by the school Marshall for coming lately to school.

(A) No Improvement

(B) to school lately

(C) late to school

(D) to school later

Answer: (C)

143. “I would like to tell you about this my friend,” said John.

(A) this friend of mine

(B) No Improvement

(C) the friend of mine

(D) this friend

Answer: (A)

144. Kuna looked very manliness in his police uniform.

(A) manly

(B) menliness

(C) man-like

(D) No Improvement

Answer: (A)

145. The school was very co-operation when we made a documentary film there.

(A) very co-operative

(B) very co-operator

(C) very co-operated

(D) No Improvement

Answer: (A)

Directions (Qs. 146 to 150) : In these questions, sentences are given with blanks to be filled in with the most appropriate word (s). Four alternatives are suggested for each question. Choose the most correct alternative out of the four.

146. He ______ to help me and called me a fool.

(A) denied

(B) offered

(C) thought

(D) refused

Answer: (D)

147. We all have ________ the widowed girl.

(A) sympathy of

(B) sympathy for

(C) sympathy at

(D) sympathy with

Answer: (B)

148. They organized an event last month in _______ with another company.

(A) competition

(B) collaboration

(C) regulation

(D) connection

Answer: (B)

149. The lady ______ to be a close relative of ours.

(A) turned out

(B) turned about

(C) turned up

(D) turned in

Answer: (A)

150. One who tries may fail but one who does not try never _____.

(A) succeeds

(B) success

(C) gain

(D) prosper

Answer: (A)

Directions (Qs. 151 to 155) : Read the passage carefully and choose the best answer to each questions out of the four alternatives.

    Chameleons can make their skin change, but not because they decide to. The colour changes to help the chameleon avoid its enemies. It is a form of camouflage, a disguise that lets it blend in with its surroundings. The change is actually determined by environmental factors, such as light and temperature.

Bright sunlight causes the  skin to darken. On cool nights,, the colour fades to a creamy colour. The colour also changes when chameleons are excited, angry or afraid. The colour change is rapid and increased when the chameleon is handled, injured, or approached by another chameleons. Almost half of them are found on the African island of Madagascar. The others mostly occur in the Shara Desert, with few in Western Asia and Southern Europe. Chameleons live in trees, where they usually eat insects. Very large chameleons may even use their sticky tongues to catch birds.

151. A chameleon’s colour changes to help it

(A) avoid its enemies

(B) fly away.

(C) attract prey.

(D) look beautiful

Answer: (A)

152. Half of the world’s chameleons are found

(A) on the African island of Madagascar.

(B) on the Asian island of Madagascar.

(C) in the Sahara Desert

(D) in the continent of Asia.

Answer: (A)

153. The colour changing ability of a chameleon is a form of camouflage which is a

(A) dance done by chameleons.

(B) disease which affects chameleons.

(C) disguise that lets it blend in with its surroundings

(D) colour that fades.

Answer: (C)

154. Chameleons change colour when they are

(A) excited,, angry or hungry.

(B) afraid, excited or angry.

(C) afraid, angry or hungry.

(D) angry, excited or happy.

Answer: (B)

155. The colour change is determined by

(A) light and temperature

(B) light and wind.

(C) light and pressure.

(D) pressure and temperature.

Answer: (A)

Directions (Qs. 156 to 165) : In the following passage some of the word have been left out. Read the passage carefully and choose the correct answer to each question out of the four alternatives and fill in the blanks:

    The Red Indians …(i)…. for the arrival of t he bison. The  beasts, which come only once a year, are a good source of meat and hide. The Red Indians ….(ii)…. to survive the cold, bitter winter.

Tikki, the young boy …(iii)…. to look out for the herd’s arrival Everyday, he …(iv)… to the top of the hill to see if he could spot them on the vast horizon.

One night, while he …(v)…, the ground shook …(vi)… and his bag of arrows fell of from its hook on the wall. Tikki …(vii)… of his wigwam and felt a strong gust of wind and dust gush past him. Shouting excitedly, he ran to the other wigwams and woke everyone up. A big fire…(viii)…. built and the Red Indians danced around it, giving praises to their God who had at last, ….(ix)… their food and clothing of animal hide. The next morning, all the warriors …(x)… to hunt down the beasts which were grazing near the river.

156. (ii)

(A) needing this

(B) needed their

(C) needing these

(D) needed these

Answer: (D)

157. (v)

(A) was sleeping

(B) is sleep

(C) were slept

(D) was sleep

Answer: (A)

158. (i)

(A) are waited

(B) is waiting

(C) were waited

(D) were waiting

Answer: (D)

159. (iii)

(A) was chose

(B) is choose

(C) was choose

(D) was chosen

Answer: (D)

160. (vii)

(A) were immediate

(B) was immediately

(C) was immediate

(D) were immediately

Answer: (C)

161. (viii)

(A) were immediate

(B) was immediately

(C) was immediate

(D) were immediately

Answer: (B)

162. (x)

(A) is assigned

(B) was assigning

(C) were assigned

(D) were assigning

Answer: (C)

163. (iv)

(A) will be running

(B) will running

(C) would running

(D) would run

Answer: (D)

164. (ix)

(A) sent them

(B) send their

(C) send them

(D) sent their

Answer: (A)

165. (vi)

(A) violent

(B) violence

(C) violet

(D) violently

Answer: (D)

Directions (Qs. 166 to 175) : In these questions, a sentence has been given in Active/Passive voice. Out of the four alternatives suggested, select the one which best expresses the same sentence in Passive/Active voice and mark your answer accordingly.

166. Please give me your pen and take your seat.

(A) You are warned to give me your pen and take your seat.

(B) You are requested to give me your pen and take your seat.

(C) Let your pen give me and take your seat.

(D) You are ordered to give me your pen and take your seat.

Answer: (B)

167. The prisoner is known to have assaulted the warden earlier too.

(A) It is known that the warden has been assaulted by the prisoner earlier too.

(B) The warden was assaulted by the prisoner earlier too.

(C) It is known that the earlier too.

(D) It is known that the prisoner has assaulted the warden earlier too.

Answer: (D)

168. Can she write an interesting story?

(A) Can an interesting story be written by her?

(B) Could an interesting story be written by her?

(C) Can an interesting story be written for her?

(D) Can an interesting story be written to her?

Answer: (A)

169. The poet, Blake, wrote many poems for children.

(A) Children wrote many poems by the poet Blake.

(B) Many are the poems written by children for the poet Blake.

(C) Many poems were written for children by the poet, Blake.

(D) Many poems were written by children for the poet, Blake.

Answer: (C)

170. Each person exhibited various facial expressions.

(A) Various facial expressions were being exhibited by each person.

(B) Various facial expressions have been used by each person.

(C) Various facial expressions were exhibited by each person.

(D) Various facial expressions are exhibited by each person.

Answer: (C)

171. The girls ate a mango yesterday.

(A) A mango was eaten by the girls yesterday.

(B) A mango is eaten by the girls yesterday.

(C) A mango has been eaten by the girls yesterday.

(D) A mango is being eaten by the girls yesterday.

Answer: (A)

172. Your manners irritate me.

(A) I am irritated by your manners.

(B) I am irritated by my manners.

(C) I am being irritated by your manners.

(D) Manners are irritating me.

Answer: (A)

173. The boys laughed at the old man.

(A) The old man was being laughed by the boys.

(B) The old man was being laughed at by the boys.

(C) The old man was laughed by the boys.

(D) The old man was laughed at by the boys.

Answer: (D)

174. Ideas are generated by Group Discussions.

(A) Group discussions generates ideas.

(B) Group discussions have generated ideas.

(C) Group discussions generate ideas.

(D) Group discussions generated idea.

Answer: (C)

175. Who is singing such a sweet song?

(A) By whom is such a sweet song sung?

(B) By whom has such a sweet song sung?

(C) By whom is such a sweet song being sung?

(D) By whom is such a sweet song sang?

Answer: (C)

Directions (Qs. 176 to 180) : Read the passage carefully and choose the best answer to each question out of the four alternatives.

In the history o f Britain, the period from 1837 to 1901, is known as the Victorian Age.

The period saw the long and prosperous reign of Queen Victoria in England. Charles Dickens was the most popular novelist of this period. he became famous for his depiction of the life of the working class, intricate plots and sense of humour. However, it was the vast galaxy of unusual characters created by him that made him more popular than any of his contemporaries. Drawn from everyday life and the world around him, these characters were such that readers could relate to them. Beginning with The Pickwick Papers in 1836, Dickens wrote numerous novels, each uniquely filled with believable personalities and vivid physical, descriptions. According to Dickens’ friend and biographer, John Forster, Dickens made “characters real existences, not by describing them but letting them describe themselves.”

176. The word ‘popular’ in the passage means

(A) propelling

(B) poor

(C) problematic

(D) successful

Answer: (D)

177. John Forster was Dickens’

(A) friend

(B) friend and editor biographer

(C) friend and doctor

(D) best friend and philosopher

Answer: (A)

178. Dickens’ characters were drawn from

(A) royal families.

(B) everyday life and world around him.

(C) everyday life and the world beyond him.

(D) unbelievable personalities.

Answer: (B)

179. The period between 1837-1901 was known as

(A) the Elizabethan Age

(B) the Dark Age

(C) the Victorian Age

(D) the Shakespearian Age

Answer: (C)

180. Dickens became famous for depicting the life of

(A) the working class, dull plots and sense of humour

(B) the working class, dull plots and sense of humour

(C) the working class, intricate plots and sense of humour

(D) the business class, intricate plots and sense of homour

Answer: (C)

Directions (Qs. 181 to 190) : In the following passage some of the words have been left out. Read the passage carefully and choose the correct answer to each question out of the four alternatives and fill in the blanks:

   I will always …(i)… the trip I made to the zoo in 1988. It was then that I …(ii)… measles from one of my friends, Peter, who had …(iii)… along too. Before he met us at the zoo, he had gone to visit his cousin who was …(iv)… from measles. The next day, Peter was …(v)… of a sore throat, a bad cold and high fever. When he was diagnosed by a doctor as having measles, his parents rang me up to warn me that I had been …(vi)… to measles too. By the next day, I was also showing/having the same symptoms. My doctor …(vii)… me to stay at home for the next two weeks.

I was quite pleased with the doctor’s instruction. I spent the time reading story-books, …(viii)… to music and watching television. When I got bored, I would call up Peter, who also had to spend two weeks at home, for a chat. Unfortunately, the two weeks passed too/by quickly. When we …(ix)… to school, we had to work twice as hard to …(x)… with our classmates. It was definitely not worth the ‘holiday’.

181. (v)

(A) explained

(B) complained

(C) complaining

(D) complain

Answer: (C)

182. (viii)

(A) looking

(B) listening

(C) listen

(D) listened

Answer: (B)

183. (ii)

(A) caught

(B) catch

(C) catching

(D) will catch

Answer: (A)

184. (vii)

(A) advised

(B) advise

(C) advising

(D) is advised

Answer: (A)

185. (iii)

(A) gone

(B) went

(C) going

(D) go

Answer: (A)

186. (iv)

(A) recovering

(B) removed

(C) revealed

(D) recover

Answer: (A)

187. (vi)

(A) expose

(B) exposable

(C) exposed

(D) exposing

Answer: (C)

188. (i)

(A) be remember

(B) remembered

(C) remember

(D) remembering

Answer: (C)

189. (x)

(A) catch

(B) caught up

(C) catch up

(D) catching up

Answer: (C)

190. (ix)

(A) returned

(B) returning

(C) return

(D) is returned

Answer: (A)

Directions (Qs. 191 to 195) : In these questions, four alternatives are given for the Idiom/Phrase underlined in the sentence. Choose the alternative which best expresses the meaning of the Idiom/Phrase.

191. Sit on the fence.

(A) to be defeated and dejected

(B) halting between two opinions

(C) to be relaxed and comfortable

(D) to be in a tricky situation

Answer: (C)

192. Who will believe, your cock and bull story?

(A) Common story

(B) Ambiguous story

(C) Authentic story

(D) Absurd story

Answer: (D)

193. The manager is not dismissed, but he is definitely under a cloud.

(A) warned severely

(B) under suspension

(C) under suspicion

(D) under scrutiny

Answer: (C)

194. Do not pull a long face.

(A) look happy

(B) look tired

(C) look ugly

(D) look dejected

Answer: (D)

195. For years 1 could not shake off the trauma of that day.

(A) forget

(B) imagine

(C) remember

(D) None of these

Answer: (A)

Directions (Qs. 196 to 200) : In these questions, the 1st and the last part of the sentence / passage are numbered 1 and 6. The rest of the sentence / passage is split into four parts and named P, Q, R and S. These four parts are not given in their proper order. Read the sentence/passage and find out which of the four combinations is correct.

196. 

1. Today

P. by the hunters for their

Q. are trapped or killed

R. millions of wild animals

S. each year

6. skin.

(A) SPRQ

(B) RQSP

(C) PSQR

(D) QPRS

Answer: (B)

197.

1. The President

P. from Tokyo

Q. where he

R. had been meeting

S. came back

6. other world leaders.

(A) QSPR

(B) PSQR

(C) RPQS

(D) SPQR

Answer: (D)

198.

1. Over 67 years

P. but the problems

Q. have remained

R. have passed

S. of the common man

6. as daunting as ever.

(A) RPSQ

(B) QPRS

(C) SRQP

(D) QSPR

Answer: (A)

199.

1. Man’s

P. in the modern

Q. insatiable thirst or knowledge

R. the wonderful achievements of

S. is at the root of

6. world.

(A) RSPQ

(B) QSRP

(C) PQRS

(D) SQPR

Answer: (B)

200.

1. These

P. about the heavenly

Q. experiments by the scientists

R. with amazing knowledge

S. supply us

6. bodies.

(A) PSRQ

(B) PSQR

(C) QSRP

(D) RQPS

Answer: (C)

SSC Combined Graduate Level (Tier-I) Exam-2015 Question Paper With Answer Key

SSC Combined Graduate Level (Tier-I) Exam-2015
SSC Combined Graduate Level (Tier-I) Exam-2015 Question Paper With Answer Key

SSC Combined Graduate Level (Tier-I) Exam-2015

PART-A : GENERAL INTELLIGENCE AND REASONING

1. P, Q, R, S, T, U are 6 members of a family in which there are two married couples. T, a teacher is married to a doctor who is mother of R and U. Q, the lawyer is marred to P – P has one son and one grandson. Of the two married ladies on is a house wife. There is also one student and one male engineer in the family. Which of the following is true about the grand-daughter of the family?

(A) She is a student

(B) She is a an engineer

(C) She is a doctor

(D) She is a lawyer

Answer: (A)

Directions (Qs. 2 & 3) : In these questions, which answer figure will complete the pattern in the question figure?

2. 

Answer: (D)

3. 

Answer: (C)

Directions (Qs. 4 & 5) : In these questions, which one set of letters when sequentially placed at the gaps in the given letter series shall complete it?

4. a_b_ca_b_c_a_cc

(A) a b a b a c

(B) a c b c a b

(C) a c a c a b

(D) a b a b c a

Answer: (C)

5. adb_ac_da_cddcb_dbc_cbda

(A) b c c b a

(B) b b c a d

(C) c b b a a

(D) c c b b a

Answer: (C)

6. If a mirror is placed on the line MN, then which of the answer figures is the right image of the given figure?

Answer: (C)

7. In certain code FARMER is written as MAFMRE, in that code which word will be written a GIVALEL.

(A) VIALEGL

(B) VELAIGL

(C) VAGIELL

(D) AIGALE

Answer: (D)

Directions (Qs. 8 to 12) : In these questions, select the missing number from the given responses.

8. 

(A) 3564

(B) 5634

(C) 6543

(D) 5364

Answer: (D)

9. 

(A) 30

(B) 10

(C) 20

(D) 40

Answer: (A)

10. 18   21      24

       3      9        3

      6       4        8

      21      26      ?

(A) 27

(B) 29

(C) 24

(D) 22

Answer: (B)

11. 2       4       2

    3         9       3

    4        16      4

    8         64      ?

(A) 8

(B) 24

(C) 16

(D) 9

Answer: (A)

12. I          25      15      40      8

     II          65      25      90      [?]

     III         45      15      60      12

(A) 6

(B) 18

(C) 24

(D) 12

Answer: (B)

Directions (Qs. 13 to 17) : In these questions, select the related word/letters/number from the given alternatives.

13. PALAEONTOLOGY : FOSSIL : : PHRENOLOGY : ?

(A) PANCREAS

(B) SKULL

(C) LUNGS

(D) THYROID

Answer: (B)

14. MIKE : OGMC : : CIAD : ?

(A) AJCF

(B) ENCF

(C) EGCB

(D) EICB

Answer: (C)

15. 9 : 50 : : ?

(A) 18 : 190

(B) 22 : 110

(C) 15 : 225

(D) 20 : 105

Answer: (D)

16. 16 : 22 : : 36 : ?

(A) 46

(B) 26

(C) 24

(D) 44

Answer: (A)

17. 

(A) O

(B) N

(C) K

(D) I

Answer: (C)

18. From the given alternative words, select the word which cannot be formed using the letters of the given word:

ENVIRONMENT

(A) EMINENT

(B) ENTRANCE

(C) MOVEMENT

(D) ENTER

Answer: (B)

19. Identify the pair which ‘DOES NOT’ exhibit the same relationship as the capitalized pair :

ETYMOLOGY : WORDS

(A) ANATOMY : BODY

(B) PSYCHOLOGY : MIND

(C) ARCHEOLOGY : ANTIQUES

(D) PHILOSOPHY : LANGUAGES

Answer: (D)

Directions (Qs. 20 to 23) : In these questions, find the odd number/letters/number pair from the given alternatives.

20.

(A) (56, 19)

(B) (42, 14)

(C) (108, 36)

(D) (69, 23)

Answer: (A)

21. 32, 13, 51, 24, 46, 20, 72, 45

(A) 72

(B) 13

(C) 20

(D) 46

Answer: (B)

22.

(A) EARS

(B) NOSE

(C) EYES

(D) VESTIBULAR

Answer: (D)

23.

(A) RNJF

(B) YUQM

(C) SOKG

(D) TPLI

Answer: (D)

Directions (Qs. 24 & 25) : In these questions, which one of the given responses would be a meaningful order of the following?

24. Crop 2. Root 3. Stem 4. Seed 5. Flower

(A) 4 2 3 5 1

(B) 2 4 5 1 3

(C) 2 3 4 1 5

(D) 2 3 5 1 4

Answer: (A)

25. Frog 2. Eagle 3. Grasshopper 4. Snake 5. Grass

(A) 5, 3, 1, 4, 2

(B) 5, 3, 4, 2, 1

(C) 3, 4, 2, 5, 1

(D) 1, 3, 5, 2, 4

Answer: (A)

26. Identify the diagram that best represents the relationship among the classes given below:

Soda Water, Mineral Water, Liquid

Answer: (D)

27. Some equations are solved on the basis of a certain system. On the same basis find out the correct answer for the unsolved equation.

4 – 5 – 1 = 514, 3 – 5 – 6 = 563, 0 – 6 – 8 = ?

(A) 068

(B) 806

(C) 680

(D) 860

Answer: (C)

Directions (Qs. 28 & 29) : In these questions, two statements are given followed by two conclusions / assumptions, I and II. You have to consider the statement to be true, even if it seems to be at variance from commonly known facts. You are to decide which of the given conclusions/assumptions can definitely be drawn from the given statement. Mark your answer accordingly.

28. Statements :

I. Some men are good.

II. Some men are wise.

Conclusions :

I. Some wise men are good

II. Some good men are wise.

(A) Only II follows

(B) Both I and II follow

(C) Only I follows

(D) Neither I nor II follows

Answer: (D)

29. Statements :

I. All poets are day dreamers.

II. All painters are day dreamers.

Conclusions :

I. All painters are poets.

II. Some day dreamers are not painters.

(A) Both I and II follows

(B) Only I follows

(C) Only II follows

(D) Neither I nor II follows

Answer: (D)

30. Identify the answer figure from which the pieces given in the question figure have been cut.

Answer: (A)

Directions (Q. 31) : A word is represented by only one set of numbers as given in any one of the alternatives. The sets of numbers given in the alternatives are represented by two classes of alphabets as in two matrices given below. The columns and rows of matrix I are numbered from 0 to 4 and that of matrix II are numbered from 5 to 9. A letter from these matrices can be represented first by its row and next by its column, e.g. ‘A’ can be represented by 01, 14 etc. and ‘E’ can be represented by 55, 66 etc. Similarly, you have to identify the set for the word ‘ORGAN’.

31.

(A) 98, 03, 44, 22, 58

(B) 67, 22, 31, 58, 22

(C) 75, 03, 11, 22, 76

(D) 86, 40, 23, 14, 96

Answer: (A)

Directions (Q. 32):

If you start running from a point towards North and after covering 4 kms. you turn to your left and run 5 kms. and then again turn to your left and run 5 kms. and then turn to left again and run another 6 kms. and before finishing you take another left turn and run 1 km, then answer the following question  based on this information.

32. From the finishing point if you have to reach the point from where you started, in which direction will you have to run?

(A) West

(B) North

(C) South

(D) East

Answer: (A)

33. A man takes 6 km. distance to go around the rectangular area. If the area of the rectangle is 2 sq km, find the difference between length and breadth.

(A) 0.75 km.

(B) 1 km.

(C) 2 km.

(D) 0.5 km.

Answer: (B)

Directions (Qs. 34 & 35) : In these questions, a series is given, with one term missing. Choose the correct alternative from the given ones that will complete the series.

34. 0, 7, 26, 63, ?

(A) 124

(B) 96

(C) 87

(D) 123

Answer: (A)

35. 1, 2, 2, 4, 3, 8, 7, 10, ?

(A) 11

(B) 13

(C) 9

(D) 8

Answer: (A)

36. How many circles are there in this figure.

(A) 16

(B) 17

(C) 13

(D) 22

Answer: (B)

37. Identify which one of the given alternatives will be another member o f the group of that class.

Lucknow, Patna, Bhopal, Jaipur, ?

(A) Mysore

(B) Shimla

(C) Pune

(D) Indore

Answer: (B)

38. Sohan drove 15 kms. to the west from his house, then turned left and walked 20 kms. He then turned East and walked 25 kms. and finally turning left covered 20 kms. How far he is from his house.

(A) 10 kms

(B) 5 kms

(C) 40 kms

(D) 80 kms

Answer: (A)

39. A piece of paper is folded and cut as shown below in the given question figure. From the given answer figures, indicate how it will appear when opened.

Answer: (C)

40. In the given figure 10% are students and parents, and the 10% are students, teachers and parents, 15% are teachers and parents. 35% are students and teachers. How many percentage are only teachers, parents and students.

(A) 65, 40, 45

(B) 45, 40, 65

(C) 40, 45, 65

(D) 40, 65, 45

Answer: (D)

41. Name a single letter that can be suffixed to the following words to form new words?

HAT BAR BAT PIN BATH

(A) B

(B) D

(C) A

(D) E

Answer: (D)

42. From the answer figures, select the cut piece from which the question figure is formed/made.

Answer: (B)

43. Which one of the areas marked I-VII represents the urban educated who are not hardworking?

(A) I

(B) II

(C) III

(D) IV

Answer: (D)

44. In a certain code, LUTE is written as MUTE and GATE is written as HATE, then how BLUE will be written in that code?

(A) GLUE

(B) FLUD

(C) FLUE

(D) CLUE

Answer: (D)

45. Six students, A, B, C, D, E, F are sitting on the ground. A and B belong to Ruby House, while the rest belong to Emerald House. D and F are tall, while others are short. C and D are wearing glasses while other are not wearing. Which girl of Emerald House is tall and is wearing glasses?

(A) C

(B) B

(C) A

(D) D

Answer: (D)

46. Which single letter can be prefixed to the following words in order to  obtain entirely new words? (Same letter has to be prefixed in all the five words of each.)

EAT OUR IS AS AT

(A) C

(B) S

(C) H

(D) B

Answer: (C)

47. From the given answer figures, select the one which is hidden/embedded in the question figure.

Answer: (D)

48. In the question given below, the symbols used are as

∆ = equal to

⊗ = not equal to

+ = greater than

∅ = less than

− = not less than

O = not greater than

Read the question, mark the correct response.

IF X – Y – Z then it does not.

(A) X ⊗ Y∆Z

(B) X ∅ Y + Z

(C) X O Y + Z

(D) X ∅ Y – Z

Answer: (*)

49. In a certain code language ‘481’ means ‘sky is blue’, ‘246’ means ‘sea is deep’ and ‘698’ means ‘sea looks blue’. What number is the code for ‘blue’.

(A) 1

(B) 9

(C) 8

(D) 6

Answer: (C)

50. Arrange these letters of each group to make a meaningful word and then find the odd one out.

(A) VEENS

(B) GHIET

(C) VIDEID

(D) ORFU

Answer: (C)

PART – B : GENERAL AWARENESS

51. Heavy metals got their name because compared to other atoms they have

(A) Higher atomic numbers

(B) Higher atomic radii

(C) Higher densities

(D) Higher atomic masses

Answer: (C)

52. What is ordinary Law ?

(A) Laws made and enforced by the Government

(B) Laws made by the Supreme Court

(C) Laws made by the High Court

(D) Laws made by the common people.

Answer: (A)

53. Who discovered the first antibiotic?

(A) A. Fleming

(B) W. Fleming

(C) C. Waksman

(D) Louis Pasteur

Answer: (A)

54. Cyanide poisoning causes death in seconds because

(A) It causes cardiac arrest

(B) It breaks the electron transport chain

(C) It denatures enzymes of the heart/muscle

(D) It causes Lysis of red blood cells.

Answer: (B)

55. Fascism believes in the application of the principle of

(A) Dictatorship

(B) Utilitarianism

(C) Totalitarianism

(D) Democracy

Answer: (C)

56. Microbial Type Culture Collection Centre is situated at

(A) Chandigarh

(B) New Delhi

(C) Hyderabad

(D) Bengaluru

Answer: (A)

57. When a helium atom loses an electron it becomes

(A) A positive helium ion

(B) A proton

(C) A negative helium ion

(D) An alpha particle

Answer: (A)

58. When is International ‘Yoga Divas’ celebrated?

(A) 21st May

(B) 21st July

(C) 21st June

(D) 25th April

Answer: (C)

59. Photo chemical smog is a resultant of the reaction among

(A) CO, CO2 and peroxy acetyl nitrate in the presence of Sunlight.

(B) CO, CO2 and NO2 at low temperature.

(C) NO2, O3 and peroxy acetyl nitrate in the presence of Sunlight.

(D) High concentration of NO2, O3, and CO in the evening.

Answer: (C)

60. The Dynamo converts

(A) Mechanical energy into Electrical Energy

(B) Electrical energy into Mechanical Energy

(C) Mechanical energy into Magnetic Energy

(D) None of these

Answer: (A)

61. The name of the upper house of the Indian Parliament is :

(A) Legislative Assembly

(B) Rajya Sabha

(C) Senate

(D) House of Lords

Answer: (B)

62. The father of computer is

(A) Charles Babbage

(B) Love Lice

(C) Charles Dickens

(D) Oliver Twist

Answer: (A)

63. The ore of Aluminium is

(A) Fluorspar

(B) Chalco pyrites

(C) Bauxite

(D) Hematite

Answer: (C)

64. When one gene pair hides the effect the other unit, the phenomenon is referred to as

(A) None of the options

(B) Epistasis

(C) Mutation

(D) Dominance

Answer: (B)

65. Which of the following can be found as pollutants in the drinking water in some parts of India?

Select the answer using the code given below.

1 – Arsenic

2 – Sorbitol

3 – Fluoride

4 – Formaldehyde

5 – Uranium

(A) 1, 2, 3, 4 and 5

(B) 1, 3 and 5

(C) 1 and 3

(D) 2, 4, and 5

Answer: (B)

66. “Don’t Laugh : We are Police”. This books was compiled by :

(A) Shanti Swaroop, I.G. Police

(B) K. P. S. Gill, D. G. P.

(C) Bishan Lal Vohra, I.G.P.

(D) None of the options

Answer: (C)

67. The Liquidity Preference Theory of Interest was propounded by :

(A) J. M. Keynes

(B) Alfred Marshall

(C) David Ricardo

(D) Adam Smith

Answer: (A)

68. Earthquakes are caused by

(A) Earth revolution

(B) Denudation

(C) Earth rotation

(D) Tectonism

Answer: (D)

69. A Fuse wire is characterized by

(A) Low resistance and high melting point

(B) High resistance and high melting point

(C) Low resistance and low melting point

(D) High resistance and low melting point

Answer: (D)

70. PSW stands for

(A) Program status word

(B) Primitive status word

(C) Process status word

(D) Processor status word

Answer: (A)

71. In the grasslands, trees do not replace the grasses as a part of an ecological succession because of

(A) Insect and fungi

(B) Water limits and fire

(C) None of the options

(D) Limited sunlight and paucity of nutrients

Answer: (B)

72. Which of the following costs is related to marginal cost ?

(A) Implicit cost

(B) Fixed cost

(C) Variable cost

(D) Prime cost

Answer: (C)

73. Liberalism stands for

(A) Religious orthodoxy

(B) Freedom in social, political and economic aspects

(C) Self-emancipation

(D) A movement & an attitude

Answer: (B)

74. Mahatma Gandhi began his political activities in India first from :

(A) Dandi

(B) Champaran

(C) Kheda

(D) Sabarmati

Answer: (B)

75. Waksman got the Nobel Prize for the discovery of

(A) Penicillin

(B) Chloromycetin

(C) Neomycin

(D) Streptomycin

Answer: (D)

76. Who is considered founder of the Gupta Empire?

(A) Samudra Gupta

(B) Chandra Gupta I

(C) Srigupta

(D) Chandra Gupta II

Answer: (C)

77. On 2nd June 2015 a Conference on Make in India Indigenization of Currency was organized by the Department of Economic Affairs, Ministry of Finance, Govt. of India. Who inaugurated the Conference?

(A) Prime Minister

(B) Finance Minister

(C) Governor, R.B.I

(D) Finance Secretary

Answer: (B)

78. For a missile launched with a velocity less than the earth’s escape velocity, the total energy is

(A) Positive

(B) Zero

(C) Negative

(D) Either positive or negative

Answer: (C)

79. The Battle of Plassey was fought in the year

(A) 1775

(B) 1757

(C) 1761

(D) 1576

Answer: (B)

80. Which was the first hydel power project in India?

(A) Paikara in Tamil Nadu

(B) Palli Vasal in Kerala

(C) Siva Samudram in Karnataka

(D) Nizamnagar in Andhra Pradesh

Answer: (C)

81. In which year did Dada Saheb Phalke produce the first Feature film ?

(A) 1910

(B) 1911

(C) 1912

(D) 1913

Answer: (D)

82. Spring tides occur on

(A) The day when the moon’s position is in its first quarter

(B) Full  moon day as well as on new moon day

(C) Full moon day only

(D) New moon day only

Answer: (B)

83. The oldest form of composition of Hindustani vocal music is

(A) None of the options

(B) Ghazal

(C) Dhrupad

(D) Thumri

Answer: (C)

84. Molybdenum deficiency affects the activity of

(A) Nitrogenase

(B) Nitrate reductase

(C) Chlorate reductase

(D) All of the given options

Answer: (D)

85. The Classical Dance of Andhra Pradesh is

(A) Kuchipudi

(B) Kathakali

(C) Odissi

(D) Bharatnatyam

Answer: (A)

86. The Environment (Protection) Bill was passed by the Parliament of India in

(A) 1984

(B) 1986

(C) 1972

(D) 1981

Answer: (B)

87. The Mediterranean regions are characterized by heavy rain in

(A) Autumn

(B) Spring

(C) Winter

(D) Summer

Answer: (C)

88. Xenobiotics which are inherently resistant to microbial attack are called as :

(A) Biodegradable

(B) Persistent

(C) Recalcitrant

(D) All of the given options

Answer: (C)

89. Autocracy means

(A) Rule by few

(B) Absolute rule by one

(C) Rule by the representatives

(D) Rule by the king

Answer: (B)

90. G E F, an international aid-giving agency, has the full form

(A) Global Educational Fund

(B) Global Environment Fund

(C) Global Economic Fund

(D) Global Energy Fund

Answer: (B)

91. Who directed the film on the dacoit queen Phoolan Devi

(A) Anil Kapoor

(B) Shashi Kapoor

(C) Abbas Mastan

(D) Shekhar Kapoor

Answer: (D)

92. The Women’s Reservation Bill seeks how much reservation for women in the State Assemblies and Lok Sabha?

(A) 36%

(B) 25%

(C) 30%

(D) 33%

Answer: (D)

93. Raga Kameshwari was composed by

(A) Uday Shankar

(B) Ustad Amzad Ali Khan

(C) None of the options

(D) Pandit Ravi Shankar

Answer: (D)

94. The business in Stock Markets and other securities markets is regulated by :

(A) State and Exchange Bank

(B) Stock and Exchange Bank

(C) Securities and Exchange Board of India

(D) Sole  Trade and Exchange Bank of India

Answer: (C)

95. Indian first talkie film produced in 1931 was

(A) Alamara

(B) Neel Kamal

(C) Indra Sabha

(D) Shakuntala

Answer: (A)

96. Which one of the following is the correct sequence of ecosystem in the order of decreasing productivity

(A) Mangroves, grasslands, lakes, oceans

(B) Oceans, lakes, grasslands, mangroves

(C) Oceans, mangroves, lakes grasslands

(D) Mangroves, oceans, grasslands, lakes

Answer: (A)

97. What is the fascist view of state ?

(A) State is under the control of a king

(B) Nation state is unquestionably sovereign

(C) State enhances the ideal of individualism

(D) State is the instrument of exploitation in the hands of few.

Answer: (B)

98. State Bank of India was previously known as :

(A) Syndicate Bank

(B) Co-operative Bank of India

(C) Imperial Bank of India

(D) Canara Bank

Answer: (C)

99. The technology used in the electronic printer is called

(A) Micro array

(B) Micro technology

(C) Micro millimetric

(D) Micro encapsulation

Answer: (D)

100. The earth completes one rotation on its axis in :

(A) 23 hrs./30 min

(B) 24 hrs.

(C) 23 hrs./56 min./4.9 sec

(D) 23 hrs./10 min./2 sec

Answer: (C)

PART – C : QUANTITATIVE APTITUDE

101. If PQRS is a rhombus and ∠SPQ = 50°, then ∠RSQ is

(A) 55°

(B) 45°

(C) 75°

(D) 65°

Answer: (D)

Directions (Qs. 102 to 105) : In these questions, the Graph shows the demand and production of different companies. Study the graph and answer these questions.

102. What is the ratio of the companies having more demand than production to those having more production than demand?

(A) 2 : 1

(B) 2 : 3

(C) 3 : 2

(D) 1 : 2

Answer: (D)

103. What is the difference between the average demand and the average production of the companies (in lakh tones)? [Approximately]

(A) 325

(B) 275

(C) 250

(D) 200

Answer: (B)

104. The production of company A is approximately what percent of the demand of company C ?

(A) 65%

(B) 60%

(C) 50%

(D) 55%

Answer: (D)

105. The demand of company B is what percentage of the production of company F?

(A) 60%

(B) 70%

(C) 50%

(D) 80%

Answer: (B)

106. If  then the value of cos2β is

(A) 0

(B) 

(C) 

(D) 

Answer: (D)

Directions (Qs. 107 to 109) : In these questions, study the following frequency polygon and answer the questions, Given a line graph showing the number of students passed in Higher Secondary Examination in a school over the years 2008 to 2013

107. The increase in percentage of passed students from 2008 to 2011 approximately is :

(A) 555%

(B) 54.5%

(C) 53.05%

(D) 50.5%

Answer: (B)

108. The average of passed students in the years 2008, 2009, 2012 approximately is :

(A) 134.56

(B) 134.32

(C) 134.67

(D) 134.41

Answer: (C)

109. The decrease is percentage of passed students from 2011 to 2012 approximately is :

(A) 8.25%

(B) 8.27%

(C) 8.22%

(D) 8.24%

Answer: (D)

110. A man spends 75% of his income. His income is increased by 20% and he increased his expenditure by 10%. His savings are increased by :

(A) 37(1/2)%

(B) 10%

(C) 25%

(D) 50%

Answer: (D)

111. If the sum of two numbers, one of which is 2/3 times the other, is 50, then the number are :

(A) 

(B) 

(C) 

(D) 

Answer: (B)

112. Out of four numbers the average of the first three is 16 and that of the last three is 15. If the last number is 20 then the first number is

(A) 21

(B) 25

(C) 23

(D) 28

Answer: (C)

113. A shopkeeper gains 17% after allowing a discount of 10% on the marked price of an article. Find his profit percent if the articles are sold at marked price allowing no discount.

(A) 23%

(B) 37%

(C) 30%

(D) 27%

Answer: (C)

114. Two circles touch externally. The sum of their areas is 130 π sq cm and the distance between their centres is 14 cm. The radius of the smaller circle is :

(A) 3 cm

(B) 2 cm

(C) 4 cm

(D) 5 cm

Answer: (A)

115. If  , then the value of  is

(A) 2

(B) 3/7

(C) 1/2

(D) 1

Answer: (C)

116. A number exceeds its two fifth by 75. The number is

(A) 125

(B) 150

(C) 112

(D) 100

Answer: (A)

117. The length of canvas, 75 cm wide required to build a conical tent of height 14 m and the floor area 346.5 m2 is :

(A) 770 m

(B) 665 m

(C) 490 m

(D) 860 m

Answer: (A)

118. If two supplementary angles differ by 44°, then one of the angles is

(A) 102°

(B) 65°

(C) 68°

(D) 72°

Answer: (C)

119. The ratio of each interior angle to each exterior angle of a regular polygon is :

(A) 6

(B) 9

(C) 7

(D) 8

Answer: (D)

120. If  , then the value of  is

(A) 0

(B) −4

(C) 4

(D) 2

Answer: (A)

121. A can do a work in 10 days and B in 20 days. If they together work on it for 5 days, then the fraction of the work that is left is :

(A) 3/4

(B) 3/20

(C) 4/3

(D) 1/4

Answer: (D)

122. XY and XZ are tangents to a circle, ST is another tangent to the circle at the point R on the circle, which intersects XY and XZ at S and T, respectively. If XY = 15 cm and TX = 9 cm then RT is

(A) 3 cm

(B) 7.5 cm

(C) 4.5 cm

(D) 6 cm

Answer: (D)

123. If θ is an acute angle and tan2θ + 1/tan2θ = 2, then the value of θ is :

(A) 15°

(B) 30°

(C) 60°

(D) 45°

Answer: (D)

124. The percentage increase in the surface area of a cube when each side is doubled is :

(A) 200%

(B) 300%

(C) 50%

(D) 150%

Answer: (B)

125. Given that x3 + y3 = 72 and xy = 6 with x > y, then the value of x – y is

(A) −4

(B) −2

(C) 4

(D) 2

Answer: (D)

126. In a rhombus ABCD, ∠A = 60° and AB = 12 cm, then the diagonal BD is :

(A) 6 cm

(B) 2√3 cm

(C) 10 cm

(D) 12 cm

Answer: (D)

127. If the cube root of 79507 is 43, then the value of

(A) 47.73

(B) 0.4773

(C) 4.773

(D) 477.3

Answer: (C)

128. The measures of two angles of a triangle is in the ratio 4 : 5. If the sum of these two measures is equal to the measure of third angle, find the smallest angle.

(A) 40°

(B) 90°

(C) 50°

(D) 10°

Answer: (A)

129. The angle of elevation of a tower from a distance of 100 mt. from its foot is 30°. Then the height of the tower is :

(A) 100√3 mt

(B) 

(C) 

(D) 50√3 mt

Answer: (C)

130. Cost price of 100 books is equal to the selling price of 60 books. The gain percentage/loss percentage is –

(A) 66%

(B) 66(3/2)%

(C) 67%

(D) 66(2/3)%

Answer: (D)

131. A, B and C can do a piece of work in 24, 30 and 40 days, respectively. They began the work together but C left 4 days before completion of the work. In how many days was the work done ?

(A) 14

(B) 13

(C) 11

(D) 12

Answer: (C)

132. If m – 5n = 2, then the value of (m3 – 125n3 – 30 mn) is

(A) 6

(B) 8

(C) 7

(D) 9

Answer: (B)

133. A boat can travel with a speech of 13 km/hr in still water. If the speed of stream is 4 km/hr in the same direction, time taken by boat to go 60 km in opposite direction is

(A) 7 hrs

(B) 9 hrs

(C) 3(9/17) hrs

(D) 4 hrs

Answer: (A)

134. A and B entered into a partnership investing Rs. 16000 and Rs. 12000, respectively. After 3 months A withdrew Rs. 5000 while B invested Rs. 5000 more. After 3 more months C joins the business with a capital of Rs. 21000. The share of B exceeds that of C, out of a total profit of Rs. 26400 after one year by

(A) Rs. 4800

(B) Rs. 1200

(C) Rs. 3600

(D) Rs. 2400

Answer: (C)

135. Pipe A can fill a tank in 4 hours and pipe B can fill it in 6 hours. If they are opened on alternate hours and if pipe A is opened first, in how many hours, the tank shall be full ?

(A) 4(2/3)

(B) 3(1/4)

(C) 4(1/2)

(D) 3(1/2)

Answer: (A)

136. The average of 7,11, 15, x, 14, 21, 25 is 15, then the value of x is

(A) 3

(B) 14.5

(C) 13.3

(D) 12

Answer: (D)

137. If two medians BE and CF of a triangle ABC, intersect each other at G and if BG = CG, ∠BGC = 60° and BC = 8 cm, then area of the triangle ABC is

(A) 48√3 cm2

(B) 96√3 cm2

(C) 48 cm2

(D) 64√3 cm2

Answer: (A)

138. If 0° ≤ A ≤ 90°, the simplified form of the given expression sin A cos A(tan A – cot A) is.

(A) 1

(B) 1- cos2A

(C) 2sin2A – 1

(D) 1 – 2sin2A

Answer: (C)

139. Find the square root of 

(A) 2/3

(B) 1/3

(C) 3/2

(D) 3

Answer: (B)

140. The printed price of an article is 40% higher than its cost price. Then the rate of discount such that he gains 12% profit is :

(A) 18%

(B) 21%

(C) 15%

(D) 20%

Answer: (D)

141. A person of height 6ft. wants to pluck a fruit which is on a  high tree. If the person is standing  away from t he base of the tree, then at what angle should be throw a stone so that, it hits the fruit.

(A) 60°

(B) 45°

(C) 30°

(D) 75°

Answer: (C)

142. A sum of money lent out at simple interest amounts to Rs. 720 after 2 years and Rs. 1020 after a further period of 5 years. Find the principal.

(A) Rs. 120

(B) Rs. 6000

(C) Rs. 1740

(D) Rs. 600

Answer: (D)

143. If tan θ + cot θ = 5, then tan2θ + cot2θ is

(A) 26

(B) 23

(C) 25

(D) 24

Answer: (B)

144. List price of a book is Rs. 100. A dealers sells three such books for Rs. 274.50 after allowing discount at a certain rate. Find the rate of discount.

(A) 8.33%

(B) 8.34%

(C) 8.16%

(D) 8.5%

Answer: (D)

145. If  then value of x is

(A) 

(B) 

(C) 

(D) 

Answer: (C)

146. If  the ratio of (2x + 3y) and (3y – 2x) is

(A) 1 : 1

(B) 2 : 1

(C) 3 : 2

(D) 3 : 1

Answer: (D)

147. The value of tan 1° tan 2° tan 3° ……… tam 89° is

(A) −1

(B) 1

(C) None of the options

(D) 0

Answer: (B)

148. ABC is a cyclic triangle and the bisector of ∠BAC, ∠ABC and ∠BCA meet the circle at P, Q and R, respectively. Then the angle ∠RQP is

(A) 

(B) 

(C) 

(D) 

Answer: (B)

149. ABC is a triangle and the sides AB, BC and CA are produced to E, F and G, respectively. If ∠CBE = ∠ACF = 130°, then the value of ∠GAB

(A) 100°

(B) 130°

(C) 80°

(D) 90°

Answer: (A)

150. Raj and Prem walk in opposite directions at the rate of 3 km and 2 km per hour, respectively. How far will they be from each other after 2 hrs. ?

(A) 8 km

(B) 10 km

(C) 2 km

(D) 6 km

Answer: (B)

PART – D : ENGLISH COMPREHENSION

Directions (Qs. 151 to 160) : In these questions, a sentence / a part of the sentence is underlined. Below are given alternatives to the underlined part which may improve the sentence. Choose the correct alternative. In case no improvement is needed choose “No Improvement”. Mark your answer accordingly.

151. We do not believe in a dual policy of the company.

(A) in these dual

(B) No improvement

(C) in this dual

(D) on these dual

Answer: (C)

152. The glass figurine that was being shown in the store window appealed to me.

(A) that was exhibited

(B) No improvement

(C) that was on display

(D) that was advertised

Answer: (C)

153. The notorious criminal went to the police to go to prison.

(A) gave himself up for the police

(B) No improvement

(C) submitted to the police

(D) surrendered himself before the police

Answer: (A)

154. He met European lady at the conference who works for an NGO.

(A) No improvement

(B) a European lady who works for an NGO, at the conference

(C) an European lady at the conference who works for an NGO

(D) European lady who works for an NGO at the conference

Answer: (B)

155. The practice of starving the children in order to cure diarrhoea also aggravates the situation.

(A) No improvement

(B) starve children

(C) starving children

(D) starving child

Answer: (C)

156. She said that she was glad to be here that evening.

(A) to come here

(B) No improvement

(C) to have been here

(D) to be there

Answer: (D)

157. I did not see you for a long time.

(A) No improvement

(B) saw

(C) have not seen

(D) have not been seeing

Answer: (C)

158. This news is too good to be true.

(A) cannot be true

(B) so good that it should be true

(C) No improvement

(D) so good that it cannot be true

Answer: (D)

159. If he has time he will telephone

(A) might

(B) could

(C) No improvement

(D) would

Answer: (C)

160. He is somehow tall for his age.

(A) many

(B) rather

(C) No improvement

(D) much

Answer: (B)

Directions (Qs. 161 to 165) : In these questions, some parts of the sentences have errors and some are correct. Find out which part of a sentence has an error and m ark that part as your answer. If a sentence is free from error, then mark the letter corresponding to “No Error” as your answer.

161. I watched how the pianoist used her left hand

(A) used her left hand

(B) the pianoist

(C) I watched how

(D) No error

Answer: (D)

162. You are required to give an explanation for your conduct within two days of receipt of this letter.

(A) within two days of the receipt of this letter.

(B) for your conduct

(C) No error

(D) You are required to

Answer: (A)

163. The ruins of the spillway are a vital clue to the epic struggle that unfold as generations of Khmer engineers coped with a water system that grew complex and unruly.

(A) The ruins of the spillway are a vital clue

(B) No error

(C) coped with a water system that grew complex and unruly.

(D) to the epic struggle that unfold as generations of Khmer engineers

Answer: (D)

164. Bobby learnt the alphabets at the age of two.

(A) No error

(B) Bobby learnt the

(C) alphabets at the

(D) age of two

Answer: (C)

165. The sum and substance of this poem is as follows.

(A) No error

(B) of this poem

(C) is as follows

(D) The sum and substance

Answer: (C)

Directions (166 & 167) : In these questions, four words are given in each question, out of which only one word is correctly spelt. Find the correctly spelt word.

166.

(A) commendation

(B) advicable

(C) tommorow

(D) coherent

Answer: (A)

167.

(A) annonymous

(B) disipate

(C) carcas

(D) aneursym

Answer: (D)

Directions (Qs. 168 to 174) : In these questions, out of four alternatives, choose the one which can be substituted for the given words/sentences.

168. To free someone from all blames

(A) Fling

(B) Consolidate

(C) Exonerate

(D) Forbid

Answer: (C)

169. The burial of a corpse

(A) Interment

(B) Inernment

(C) Interrogate

(D) Interpose

Answer: (A)

170. One who makes an official examination of accounts

(A) Registrar

(B) Creditor

(C) Chartered Accountant

(D) Auditor

Answer: (D)

171. A person who deliberately sets fire to a building

(A) Hijacker

(B) Extortionist

(C) Arsonist

(D) Assassin

Answer: (C)

172. Conferred ass an honor

(A) Honorific

(B) Honorary

(C) Honorarium

(D) Honorable

Answer: (B)

173. That which cannot be effaced

(A) Illegible

(B) Indelible

(C) Affable

(D) Invincible

Answer: (B)

174.  One who does not believe in the existence of God

(A) Cynic

(B) Atheist

(C) Mystic

(D) Theist

Answer: (B)

Directions (Qs. 175 to 179) : In these questions, four alternatives are given for the idiom/phrase underlined in the sentence. Choose the alternative which best expresses the meaning of the idiom/phrase.

175. I and my friend always go Dutch when we eat out.

(A) go together

(B) pay for each other’s meal

(C) divide the costs

(D) drive together

Answer: (C)

176. He’s as daft as a brush. Don’t believe a word of what he says.

(A) unreliable

(B) very funny

(C) really silly

(D) a liar

Answer: (C)

177. A closefisted man does not know the significance of human life.

(A) a miser

(B) a poor man

(C) an ill-mannered man

(D) a rich man

Answer: (A)

178. I hope you will back me at t he meeting.

(A) criticise

(B) support

(C) follow

(D) speak after I do

Answer: (B)

179. I shall always remember my alma mater with gratitude.

(A) teacher who inspired me

(B) kindergarten days

(C) institutions where I got education

(D) mother’s loving care

Answer: (C)

Directions (Qs. 180 to 182) : In these questions, out of the four alternatives, choose the one which best expresses the meaning of the given word.

180. FRUGAL

(A) Simple

(B) Miserly

(C) Plain

(D) Economical

Answer: (D)

181. PLEBISCITE

(A) Representation

(B) Renunciation

(C) Referendum

(D) Reservation

Answer: (C)

182. DIMINISH

(A) Worsen

(B) Shorten

(C) Reduce

(D) Prohibit

Answer: (C)

Directions (Qs. 183 to 187) : In these questions, sentences are given with blanks to be filled in with an appropriate word(S). Four alternatives are suggested for each question. Choose the correct alternative out of the four.

183. Ships are ________ by giant engines.

(A) sailed

(B) driven

(C) pulled

(D) dragged

Answer: (C)

184. Krisda’s stories ________ me very strangely.

(A) afflicted

(B) affected

(C) effected

(D) changed

Answer: (B)

185. Work should be guided by some _______ desire of altruism and ______.

(A) shallow ; ability

(B) genuine ; philanthropy

(C) good ; liberty

(D) false ; honesty

Answer: (B)

186. She pipped herrival _______the gold medal.

(A) to

(B) for

(C) near

(D) with

Answer: (B)

187. By morning, the fury of the floods _______.

(A) abated

(B) retired

(C) ebbed

(D) receded

Answer: (A)

Directions (Qs. 188 to 192) : In these questions, you have a passage with 5 questions following, Read the passage carefully and choose the best answer to each question.

PASSAGE

The desert floras shame us with their cheerful adaptations to the seasonal limitations. Their whole duty is to flower and fruit, and they do it hardly, or with tropical luxuriance, as the rain admits. It is recorded in the report of the Death Valley expedition that after a year of abundant rains, on the Colorado desert was found a specimen of Amaranthus ten feet high. A year later the same species in the same place matured in the drought at four inches. Seldom does the desert herb attain the full stature of the type. Extreme aridity and extreme altitude have the same dwarfing effect, so that we find in the high Sierras and in Death Valley related species in miniature that reach a comely growth in mean temperatures. Very fertile are the desert plants in expedients to prevent evaporation, turning their foliage edgewise toward the sun, growing silky hairs, exuding thick gum. The wind, which has a long sweep, harries and helps them. It rolls up dunes about the stocky stems, encompassing and protective, and above the dunes, which may be, as with the mesquite, three times as high as a man, the blossoming twigs flourish and bear fruit.

188. The mesquite is a ______

(A) a tribe of people

(B) a type of desert animal

(C) a desert flora

(D) a sand dune

Answer: (C)

189. How does the wind keep the desert floras to grow?

(A) by blowing the clouds away

(B) by blowing the heat away

(C) by rolling up protective sand dunes.

(D) by blowing gently.

Answer: (C)

190. What stops the desert floras performing their duty well?

(A) the desert animals

(B) the people who pluck them

(C) the desert sand

(D) the rain

Answer: (D)

191. What lesson do the desert floras have to teach us?

(A) How to grow in dry places.

(B) How to adapt to limitations.

(C) How to live a long time.

(D) How to grow with grace.

Answer: (B)

192. The desert plants face the danger of _______ from extreme aridity and extreme altitude.

(A) loss of reproduction

(B) early death

(C) painful growth

(D) dwarfism

Answer: (D)

Directions (Qs. 193 to 197) : In these questions, you have a passage with 5 questions following. Read the passage carefully and choose the best answer to each question out of the four alternatives.

PASSAGE

The  public distribution system, which provides food at low prices, is a subject of vital concern. There is growing realization that through India has enough food to feed its masses two square meals a day, the monster of starvation and food insecurity continues to haunt the poor in our country.

Increasing the purchasing power of the poor through providing productive employment leading to rising income, and thus good standard of living the ultimate objective of public policy. However, till then, there is a need to provide assured supply of food through a restructured, more efficient and decentralized public distribution system (PDS). Although the PDS is extensive-it is one of the largest such systems in the world-it hasn’t reached the rural poor and the remote places. It remains an urban phenomoneon, with the majority of the rural poor still out of its reach due to lack of economic and physical access. The poorest in the cities and the migrants are left out, for they generally do not possess ration cards. The allocation of PDS supplies in big cities is larger than in rural areas. In view of such deficiencies in the system, the PDS urgently needs to be streamlined. Also, considering the large foodgrain production combined with food subsidy on one hand and the continuing slow starvation and dismal poverty of rural population on the other, there is a strong case for making PDS target-group oriented. By making PDS target-group oriented. By making PDS target-group oriented, not only the poorest and the neediest would be reached without additional cost but we can also reduce the overall costs incurred.

193. What should be an appropriate step to make the PDS effective?

(A) To reduce administrative cost

(B) To increase the amount of foodgrain per ration card.

(C) To streamline the PDS.

(D) To decrease the allotment of food grains.

Answer: (C)

194. What according to the passage, would be the rationale of making the PDS target-group oriented?

(A) It will remove poverty

(B) It will give food to the poorest section without additional cost.

(C) It will motivate the target-group population to work more.

(D) It will abolish the imbalance of urban and rural sectors.

Answer: (B)

195. The public distribution system, which provides food at _______ is a subject of vital concern.

(A) low prices

(B) as per capita income

(C) high prices

(D) fair prices.

Answer: (A)

196. The full form a PDS is

(A) Partial Distribution System

(B) Private Distribution System

(C) Party Distribution System

(D) Public Distribution System

Answer: (D)

197. Which of the following is true of Public Distribution System?

(A) It has reached the remotest corner of the country

(B) It has remained effective only in the cities

(C) It has improved its effectiveness over the years

(D) It is unique in the world because of its effectiveness

Answer: (B)

Directions (Qs. 198 to 200) : In these questions, choose the word opposite in meaning to the given word

198. URBANE

(A) Crude

(B) Loud

(C) Native

(D) Rural

Answer: (A)

199. BOLD

(A) Fearful

(B) Timid

(C) Coy

(D) Nervous

Answer: (B)

200. AUTHENTIC

(A) Real

(B) False

(C) Genuine

(D) Factual

Answer: (B)

SSC Stenographers (Grade ‘C’ and ‘D’) Examination Held on 2016 General Intelligence and Reasoning Question Paper With Answer Key

SSC Stenographers (Grade 'C' and 'D') Examination Held on 2016
SSC Stenographers (Grade ‘C’ and ‘D’) Examination Held on 2016 General Intelligence and Reasoning Question Paper With Answer Key

Staff Selection Commission (SSC) Stenographers (Grade ‘C’ and ‘D’) Examination Held on 2016

GENERAL INTELLIGENCE AND REASONING

Directions (Qs. 1 to 9) : In these questions, select the related word/letters/number from the given alternatives.

1. OPMN : JKHI : : TURS : ?

(A) NMOP

(B) OPMN

(C) POMN

(D) OPNM

Answer: (B)

2. DEIJ : KLPQ : : FGJK : ?

(A) MNQR

(B) LNQR

(C) OPQR

(D) MNPQ

Answer: (A)

3. 5 : 35 : : ?

(A) 9 : 45

(B) 11 : 55

(C) 6 : 66

(D) 7 : 77

Answer: (D)

4. 3 : 4 : : 5 : ?

(A) 27

(B) 20

(C) 22

(D) 15

Answer: (B)

5. 36 : 16 : : 81 : ?

(A) 46

(B) 44

(C) 49

(D) 48

Answer: (C)

6. Tadpole : ? : : Caterpillar : Butterfly

(A) Frog

(B) Spider

(C) Owl

(D) Wasp

Answer: (B)

7. Muslims : Mosque : : Sikhs : ?

(A) Medina

(B) Gurudwara

(C) Sun Temple

(D) Golden Temple

Answer: (B)

8. Elated : Despondent : : Enlightened : ?

(A) Ignorant

(B) Miserable

(C) Tolerant

(D) Aware

Answer: (A)

9. EFKL : UVOP : : CDGH : ?

(A) WXTS

(B) WXST

(C) KLOP

(D) XWTS

Answer: (B)

Directions (Qs. 10 to 17) In these questions, find the odd word/letters/number pair/numbers from the given alternatives.

10. 

(A) DEOP

(B) QUVR

(C) LMYZ

(D) JKRS

Answer: (B)

11.

(A) 19-33

(B) 24-45

(C) 35-62

(D) 16-23

Answer: (C)

12.

(A) 17, 163, 9

(B) 19, 76, 4

(C) 14, 112, 8

(D) 23, 161, 7

Answer: (A)

13.

(A) 65, 13

(B) 130, 26

(C) 75, 16

(D) 35, 7

Answer: (C)

14.

(A) Potato

(B) Ginger

(C) Turmeric

(D) Carrot

Answer: (D)

15.

(A) Pond

(B) River

(C) Sea

(D) Lake

Answer: (C)

16.

(A) Helmet

(B) Hamlet

(C) Cap

(D) Wig

Answer: (B)

17.

(A) EFGI

(B) IJKO

(C) UVWZ

(D) ABCE

Answer: (C)

18. Number of letters skipped in between adjacent letters in the series increases by one. Which of the following series observes the rule given above?

(A) EGJMPS

(B) BDGKPV

(C) CEHKOU

(D) ADGKNR

Answer: (B)

19. Arrange the given words in alphabetical order and choose the second word.

Exhilarate, Ephemeral, Entrench, Enterprise, Enthusiasm

(A) Enthusiasm

(B) Enterprise

(C) Ephermeral

(D) Entrench

Answer: (A)

Directions (Qs. 20 & 21) : In these questions, arrange the following words as per order in the dictionary :

20. 

1. Negative 2. Negotiable

3. Negligence 4. Negotiate

5. Negligent

(A) 1, 3, 5, 4, 2

(B) 1, 5, 3, 2, 4

(C) 1, 3, 5, 2, 4

(D) 1, 3, 4, 5, 2

Answer: (C)

21. 

1. Cholesterol

2. Choreography

3. Chocolatier

4. Chrestomathy

(A) 3, 1, 2, 4

(B) 1, 3, 2, 4

(C) 3, 1, 4, 2

(D) 1, 2, 3, 4

Answer: (A)

Directions (Qs. 22 & 23) : In these questions, which one set of letters when sequentially placed at the gaps in the given letter series shall complete it?

22. w_uww_ _ w_ xuw_x_w

(A) xwuuxw

(B) xxuwwu

(C) xxwwuu

(D) xuwuwx

Answer: (B)

Directions (Qs. 24 to 27) : In these questions, a series is given, with one/two term(s) missing. Choose the correct alternative from the given ones that will complete the series.

23. 36, 34, 30, 28, 24, ?

(A) 22

(B) 23

(C) 26

(D) 20

Answer: (A)

24. 36, 34, 30, 28, 24, ?

(A) 22

(B) 23

(C) 26

(D) 20

Answer: (A)

25. 4, 7, 26, 10, 13, 20, 16, ?, ?

(A) 19, 17

(B) 14, 17

(C) 19, 14

(D) 14, 4

Answer: (C)

26. AYD, BVF, DRH, ?, KGL

(A) GMJ

(B) GLJ

(C) HLK

(D) FMI

Answer: (A)

27. 2, 3, 5, 7, ?

(A) 9

(B) 10

(C) 11

(D) 8

Answer: (C)

28. In a certain code, ‘MEMORANDUM’ is written as ‘ADEMMMNORU’. How can ‘MEMNDELEVIUM’ be written ?

(A) DEEEILMMNUV

(B) EDEEILMNVU

(C) EEEDILMNVU

(D) DEEILMEMUV

Answer: (A)

29. If BALLOON is coded as 05-04-15-15-18-18-17, then LAGOON is coded as

(A) 15-04-10-18-18-17

(B) 15-07-03-18-18-17

(C) 12-01-10-18-18-17

(D) 12-01-07-18-18-17

Answer: (A)

30. Introducing Geeta, Sita said, “She is the only daughter of my father’s only daughter.” How is Sita related to Geeta?

(A) Niece

(B) Cousin

(C) Aunt

(D) None of these

Answer: (D)

31. The boy in the photograph is the only son on the only son of Suresh’s mother. How is Suresh related to that boy ?

(A) Uncle

(B) Cousin

(C) Father

(D) Brother

Answer: (A)

32. From the given alternatives, select the longest word (with maximum number of letters) that can be made from the following letters.

TENOHS

(A) HONEST

(B) HONESTY

(C) SHORTEN

(D) TENSE

Answer: (A)

33. From the given alternative words, select the word which can be formed using the letters of the given word.

JOURNALIST

(A) JOURNEY

(B) TOURIST

(C) RATION

(D) NATION

Answer: (C)

Directions (Qs. 34 to 37) : In these questions, select the missing number from the given responses.

34.

(A) 26

(B) 24

(C) 22

(D) 28

Answer: (A)

.

35.

(A) 98

(B) 92

(C) 87

(D) 152

Answer: (B)

36.

(A) 9

(B) 12

(C) 19

(D) 28

Answer: (A)

37.

Answer: (B)

38. Lata walks 7 km towards North, turns to her left and walks 9 km, again she turns to her left and walks 7 km. How far is the she from the starting point ?

(A) 7 k m

(B) 9 km

(C) 5 km

(D) 8 km

Answer: (B)

39. A and B are sitting face-to-face in a tea shop at about 7 a.m. Due to sun rays falling on this face B shifts his place and sits next to A. Which direction are both facing now ?

(A) West

(B) North

(C) South

(D) East

Answer: (A)

40. If ‘−‘ stands for multiplication, ‘+’ stands for division, ‘÷’ stands for addition, ‘×’ stands for subtraction, then which of the following equations is correct?

(A) 30 ÷ 6 + 20 = 36

(B) 40 + 8 – 7 = 35

(C) 15 ÷ 9 – 2 = 39

(D) 24 + 8 × 7 = 49

Answer: (B)

41. If ‘×’ means ‘−’, ‘÷’ means ‘+’, ‘+’ means ‘×’, ‘−‘ means ‘÷’, then what is the value of

200 × (180 – 90) ÷ (12 + 6) ?

(A) 410

(B) 250

(C) 270

(D) 370

Answer: (C)

Directions (Qs. 42 & 43) : In these questions, two or three statements are given followed by four conclusions, I, II, III and IV. You have to consider the statements to be true, even if they seem to be at variance from commonly known facts. You have to decide which of the given conclusions, if any, follow from the given statements.

42. Statements:

1. All C’s are D’s.

2. All D’s are E’s.

Conclusions:

I. All C’s are E’s.

II. All E’s are C’s.

III. Some D’s are C’s.

IV. Some D’s are not C’s.

(A) Only conclusions III and IV follow

(B) Only conclusions I and III follow

(C) All conclusions follow

(D) Only conclusions I and IV follow

Answer: (B)

43. Statements :

1. All bottles are boxes.

2. All boxes are bags.

3. Some bags are trays.

Conclusions :

I. Some bottles are trays.

II. Some trays are boxes.

III. All bottles are bags.

IV. Some trays are bags.

(A) Only conclusions II and III follow

(B) Only conclusion IV follows

(C) Only conclusions III and IV follow

(D) Only conclusions I, III and IV follow

Answer: (C)

44. All the adjacent faces of a cube are red, green and white in colour. If they are cut into 64 small cubes, how many small cubes are there whose 3 faces are coloured?

(A) 16

(B) 8

(C) 24

(D) 4

Answer: (B)

45. Which one of the following best depicts the relationship among INSAT, EDUSAT, ARYABHATTA and ISRO ?

Answer: (A)

46. If a mirror is placed on the line MN, then which of the answer figures is the right image of the question figure ?

Answer: (C)

47. A word is represented by only one set of numbers as given in any one of the alternatives. The sets of numbers given in the alternatives are represented by two classes of alphabets as in the two matrices given below. The columns and rows of Matrix I are numbered from 0 to 4 and that of Matrix II are numbered from 5 to 9. A letter from these matrices can be represented first by its row and next by its column, e.g., ‘A’ can be represented by 01, 14, etc., and ‘M’ can be represented by 56, 68, etc. Similarly, you have to identify the set for the word ‘AMPLE’.

(A) 14, 68, 21, 97, 99

(B) 22, 95, 00, 57, 88

(C) 31, 86, 33, 69, 77

(D) 01, 56, 34, 78, 89

Answer: (B)

48. Which answer figure will complete the pattern in the question figure?

Answer: (A)

49. From the given answer figures, select the one in which the question figure is hidden/embedded.

Answer: (C)

50. A piece of paper is folded and cut as shown below in the question figures. From the given answer figures, indicate how it will appear when opened.

Answer: (C)

GENERAL AWARENESS

51. The demand curve for ‘Veblen goods’

(A) has a negative slope

(B) is horizontal

(C) is vertical

(D) has a positive slope

Answer: (D)

52. If an economy’s production possibility frontier shifts to the right, it would mean

(A) decreasing opportunity cost

(B) a fall in resource utilization

(C) economic growth

(D) increasing opportunity cost

Answer: (C)

53. How many types of writs are there ?

(A) 4

(B) 5

(C) 6

(D) 3

Answer: (B)

54. Which age group of children are covered under the 93rd Amendment Act?

(A) 6-12 years

(B) 6-14 years

(C) 6-15 years

(D) 0-10 years

Answer: (B)

55. The Fundamental Rights are guaranteed to the citizens of India under

(A) Part III of the Constitution

(B) Part IV of the Constitution

(C) Part V of the Constitution

(D) Part II of the Constitution

Answer: (A)

56. Land Ceiling means

(A) putting fence for a plot

(B) not allowing others to enter agricultural land

(C) fixing the maximum size of land which could be owned by an individual

(D) covering the land with a sheet

Answer: (C)

57. Which among the following theories is called the Iron Law of Wages?

(A) Marginal Productivity Theory

(B) Residual Claimant Theory

(C) Subsistence Theory

(D) Wage Fund Theory

Answer: (D)

58. Crop Insurance comes under

(A) General Insurance Corporation

(B) National Insurance Company

(C) Food Corporation of India

(D) Life Insurance Corporation

Answer: (A)

59. The United States of America declared their War of Independence in the year

(A) 1776

(B) 1789

(C) 1790

(D) 1775

Answer: (A)

60. Who was the founder of the ‘Red Army’?

(A) Vladimir Lenin

(B) Joseph Stalin

(C) Max Weber

(D) Leon Trotsky

Answer: (D)

61. When was the capital of India transferred from Calcutta to Delhi?

(A) 1911

(B) 1947

(C) 1950

(D) 1910

Answer: (A)

62.  The philosophy of ‘Vishishtadvaita’ was preached by

(A) Ramanujacharya

(B) Shankaracharya

(C) Madhavacharya

(D) Kapila

Answer: (A)

63. The ‘Treaty of Aix-la-Chapelle’ in 1748 was concluded at the end of

(A) The Seven Years War

(B) The War of Austrian Succession

(C) The Battle of Plassey

(D) The First Karnataka War

Answer: (B)

64. The unit of Beaufort Scale is

(A) Knots

(B) Horsepower

(C) Hertz

(D) Joules

Answer: (A)

65. Which of the following Acts established dyarchy in the provinces?

(A) Regulating Act, 1773

(B) Government of India Act, 1919

(C) Pitt’s India Act, 1784

(D) Government of India Act, 1935

Answer: (B)

66. Which President of the United States of America coined the term “United Nations”?

(A) Jimmy Carter

(B) John F. Kennedy

(C) Garfield

(D) F. D. Roosevelt

Answer: (D)

67. Which one of the following viruses was discovered first?

(A) TMV

(B) HTLV

(C) Polio

(D) HIV

Answer: (A)

68. ‘Mulching’ helps in

(A) Increasing soil fertility

(B) Moisture conservation

(C) Improving soil structure

(D) Weed control

Answer: (B)

69. The numerous circular pits found in the Karst region are called

(A) Stalactites

(B) Stalagmites

(C) Sinkholes

(D) Caves

Answer: (C)

70. Which of the following is correctly matched?

       Rivers              Origin

(A) Damodar – Amarkantak plateau

(B) Ramganga – Uttarakhand

(C) Brahmaputra – Mansarovar

(D) Son – Chota Nagpur plateau

Answer: (B)

71. The instrument ‘gnomon’ was first used by

(A) Anaximander

(B) Thales

(C) Ptolemy

(D) Homer

Answer: (A)

72. Enzymes consist of chains of

(A) Nucleotides

(B) Amino acids

(C) Carbohydrates

(D) Fatty acids

Answer: (B)

73. Embryo formation from egg without fertilization is called

(A) Parthenogenesis

(B) Allogamy

(C) Apospory

(D) Parthenocarpy

Answer: (A)

74. DNA is associated with highly basic proteins called

(A) Non-histones

(B) Albumins

(C) Non-albumins

(D) Histones

Answer: (D)

75. Blind spot is a point

(A) where the optic nerve emerges

(B) where specialized photoreceptors are tightly packed

(C) where the vision is sharpest

(D) None of the above

Answer: (A)

76. Who first introduced the electrochemical cell?

(A) Volta

(B) James Watt

(C) Thomas Alva Edison

(D) Michael Faraday

Answer: (A)

77. Cosmic rays are coming from

(A) Radioactive elements

(B) Interstellar space

(C) Cyclotrons

(D) Nuclear reactors

Answer: (B)

78. Which one of the following exhibits paramagnetic property?

(A) Cobalt

(B) Aluminium

(C) Nickel

(D) Iron

Answer: (B)

79. An n-type and p-type semiconductor can be obtained by doping pure silicon with

(A) Phosphorus and Boron

(B) Boron and Phosphorus

(C) Indium and Sodium

(D) Sodium and Magnesium

Answer: (A)

80. ENIAC uses the concept of

(A) Storage program

(B) Parallel programming

(C) Multithreading

(D) Multiprogramming

Answer: (A)

81. High capacity computers with hundreds of thousands of processors that can perform more than one trillion calculations per second are called

(A) Laptop computers

(B) Workstations

(C) Supercomputers

(D) Desktop computers

Answer: (C)

82. The application of laws of genetics for the improvement of the human race deals with

(A) Euthenics

(B) Euphenics

(C) Eugenics

(D) All of the above

Answer: (C)

83. In which year did India win its first Olympic Hockey Gold Medal?

(A) 1925

(B) 1928

(C) 1930

(D) 1935

Answer: (B)

84. Tommy Eman Gold Cup is associated with

(A) Football

(B) Basketball

(C) Cricket

(D) Hockey

Answer: (D)

85. International Day for Biological Diversity is observed on

(A) 10th May

(B) 22nd May

(C) 25th May

(D) 5th May

Answer: (B)

86. Who invented e-mail?

(A) R. S. Tomlinson

(B) Jacck Dorsey

(C) Larry Page

(D) Tim Berners-Lee

Answer: (A)

87. Who among the following is the author of the book “The Heart of India”?

(A) Ruskin Bond

(B) Meera Syal

(C) Gita Mehra

(D) Mark Tully      

Answer: (D)

88. Which country was declared least corrupt in the Corruption Perception Index, 2015?

(A) Portugal

(B) China

(C) Denmark

(D) Spain

Answer: (C)

89. Which of the following is present in DNA but not in RNA?

(A) Guanine

(B) Cytosine

(C) Thymine

(D) Adenine

Answer: (C)

90. Antiperspirants generally contain a/an

(A) Calcium compound

(B) Magnesium compound

(C) Aluminium compound

(D) Iron compound

Answer: (C)

91. Which one of the following is a triatomic molecule?

(A) Ammonia

(B) Ozone

(C) Chlorine

(D) Oxygen

Answer: (B)

92. Marble is used as a building material and also for making statues. Its chemical name is

(A) Calcium hydroxide

(B) Calcium bicarbonate

(C) Calcium carbonate

(D) Calcium chloride

Answer: (C)

93. Eustatic movement is related to

(A) Formation of a system of mountains

(B) Rise and fall of landmasses as a result of faulting

(C) Large scale rise and fall of sea-level

(D) Movement of ocean currents

Answer: (C)

94. Every year, a month long ecologically important campaign/festival is held, during which certain communities/tribes plant saplings of fruit-bearing trees. Which of the following are such communities/tribes?

(A) Gond and Korku

(B) Irula and Toda

(C) Sahariya and Agariya

(D) Bhutia and Lepcha

Answer: (A)

95. Which is the only industrialized country of the word that did not ratify the ‘Kyoto Protocol’?

(A) Russia

(B) United States of America

(C) France

(D) Japan

Answer: (B)

96. Kuttanad (or Kuttanadu) in Kerala is famous for

(A) The region with lowest altitude in India

(B) A coral island

(C) The westernmost point of India

(D) A fresh water lake

Answer: (A)

97. Name the two-faced drum made of mud.

(A) Dhol

(B) Tabla

(C) Maddalam

(D) Khol

Answer: (D)

98. Which newspaper in India brought out the first on-line edition?

(A) The Hindustan Times

(B) The Times of India

(C) The Hindu

(D) The Indian Express

Answer: (C)

99. The oldest church (St. Thomas Church) in India is located in the State of

(A) Kerala

(B) West Bengal

(C) Tamil Nadu

(D) Goa

Answer: (A)

100. Shri Palghat Mani Iyer is associated with which of the following instruments?

(A) Kanjira

(B) Dholak

(C) Mridangam

(D) Ghatam

Answer: (C)

SSC Stenographers (Grade ‘C’ and ‘D’) Examination Held on 2016 English Language and Comprehension Question Paper With Answer Key

SSC Stenographers (Grade 'C' and 'D') Examination Held on 2016
SSC Stenographers (Grade ‘C’ and ‘D’) Examination Held on 2016 English Language and Comprehension Question Paper With Answer Key

Staff Selection Commission (SSC) Stenographers (Grade ‘C’ and ‘D’) Examination Held on 2016

ENGLISH LANGUAGE AND COMPREHENSION

Directions (Qs. 1 to 10) : In these questions, some parts of the sentences have errors and some are correct. Find out which part of a sentence has an error and choose that part (A), (B) or (C) as your answer. IF a sentence is free from error, choose the ‘No error’ option as your answer.

1. Had Mukesh work hard (A) / he would have got a (B) / distinction in the exam. (C) / No error. (D)

Answer: (A)

2. The beggar thanks (A) /him a lot (B) / for the help. (C) / No error. (D)

Answer: (A)

3. The little girl was playing (A) /with her brother (B) /over the lawn. (C) / No error. (D)

Answer: (C)

4. North-East India (A) /has a beautiful places (B) / to visit. (C) / No error. (D)

Answer: (B)

5. There were no furniture (A) / in the flat (B) / except for a couple of beds. (C) / No error. (D)

Answer: (A)

6. Maintaining a healthy body and engaging (A) / in activities according to a time-tables will give you (B) / the feeling that you have enough time to do everything. (C) / No error. (D)

Answer: (B)

7. In the valley of Kadisha (A) / where the mighty river flow, (B) / two little streams met. (C) / No error. (D)

Answer: (C)

8. The aeroplane (A) / not only crashes (B) / but also caught fire. (C) / No error. (D)

Answer: (B)

9. They not only came (A) /late but also (B) / go away early. (C) / No error. (D)

Answer: (C)

10. All query will (A) / be answered (B) / by the chief instructor. (C) / No error. (D)

Answer: (A)

Directions (Qs. 11 to 15) : In these questions, sentences are given with blanks to be filled in with an appropriate word(s). Four alternatives are suggested for each question. Choose the most appropriate alternative out of the four.

11. I ________ you to be home by 8 o’clock.

(A) expect

(B) accept

(C) aspect

(D) except

Answer: (A)

12. Admission was _______ ladies.

(A) limited of

(B) restricted to

(C) restricted by

(D) limited by

Answer: (B)

13. The palace gardens ________ to the public on Saturdays and Sundays.

(A) is opened

(B) is open

(C) are open

(D) is opening

Answer: (C)

14. Two boys ________ the prize for General Proficiency.

(A) decided

(B) deputed

(C) shared

(D) give

Answer: (C)

15. You will come, _______?

(A) won’t you

(B) will you

(C) shouldn’t you

(D) aren’t you

Answer: (A)

Directions (Qs. 16 to 20) : In these questions, out of the four alternatives, choose the one which best expresses the meaning of the given word.

16. Supple

(A) Hard

(B) Round

(C) Rigid

(D) Flexible

Answer: (D)

17. Robust

(A) Weak

(B) Sturdy

(C) Heavy

(D) Thin

Answer: (B)

18. Submissive

(A) Obstinate

(B) Unyielding

(C) Stubborn

(D) Docile

Answer: (D)

19. Affluent

(A) Wealthy

(B) Precious

(C) Benevolent

(D) Friendly

Answer: (A)

20. Bargain

(A) Surplus

(B) Bonus

(C) Negotiate

(D) Dispute

Answer: (C)

Directions (Qs. 21 to 25) : In these questions, choose the word opposite in meaning to the given word.

21. Juxtaposition

(A) Remoteness

(B) Separation

(C) Termination

(D) Proximity

Answer: (B)

22. Authentic

(A) Fictitious

(B) Daring

(C) Enticing

(D) Legitimate        

Answer: (A)

23. Choose

(A) Reject

(B) Want

(C) Dislike

(D) Desire

Answer: (A)

24. Hasten

(A) Lack

(B) Obstruct

(C) Instruct

(D) Slow

Answer: (D)

25. Bold

(A) Wealthy

(B) Timid

(C) Brave

(D) Beautiful

Answer: (B)

Directions (Qs. 26 to 30): In these questions, four alternatives are given for the Idiom/Phrase underlined. Choose the alternative which best expresses the meaning of the Idiom/Phrase.

26. After much contemplation the two rivals decided to bury the hatchet and work in unison.

(A) defeat their enemies

(B) get reconciled

(C) go their own way

(D) hide their secrets

Answer: (B)

27. An axe to grind

(A) Personal end to serve

(B) No sense

(C) No work to do

(D) No property

Answer: (A)

28. My colleague made his mark as a lawyer at an early age.

(A) secured good marks

(B) distinguished himself

(C) created a vacancy

(D) desired to be

Answer: (B)

29. I am all ears to listen to your stories of chivalry.

(A) curious

(B) suspicious

(C) worried

(D) eager

Answer: (D)

30. A burning problem

(A) A problem that is obvious

(B) An urgent, important problem

(C) A dangerous problem

(D) A  problem on fire

Answer: (B)

Directions (Qs. 31 to 35) : In these questions, the first and the last p arts of the sentence/passage are numbered 1 and 6. The rest of the sentence/passage is split into four parts and named P, Q, R and S. These four parts are not given in their proper order. Read the sentence/passage and find out which of the four combinations is correct.

31. 

1. Mr. Hawking

P. can writ

Q. at the University of Cambridge

R. on his computer

S. who teaches

6. using a cheek sensor.

(A) RSQP

(B) SPQR

(C) SQPR

(D) PRSQ

Answer: (C)

32. 

1. I would urge you

P. and take no further action

Q. sympathetic view

R. to take a very

S. of the matter

6. against her.

(A) RPSQ

(B) RSQP

(C) SPRQ

(D) RQSP

Answer: (D)

33. 

1. Accumulation

P. aim of

Q. is not

R. the sole

S. of wealth

6. human life.

(A) RSPQ

(B) SQRP

(C) PRQS

(D) QRSP

Answer: (B)

34. 

1. A large

P. been felled by

Q. number of sandalwood

R. a notorious

S. trees have

6. gangster.

(A) QSPR

(B) SPQR

(C) RPSQ

(D) RQPS

Answer: (A)

35. 

1. Housing facilities

P. to keep up

Q. with the increasing

R. influx of

S. are struggling

6. population.

(A) QSPR

(B) SPQR

(C) RPSQ

(D) RQPS

Answer: (B)

Directions (Qs. 36 to 45) : In these questions, a sentence has been given in Active/Passive Voice. Out of the four alternatives suggested, select the one which best expresses the same sentence in Passive/Active Voice and mark your answer accordingly.

36. The invigilator was reading out the instructions.

(A) The instructions were being read out by invigilator.

(B) The instructions had been read out by the invigilator.

(C) The instructions had been read by the invigilator.

(D) The instructions were read by the invigilator.

Answer: (A)

37. You need to clean your shoes properly.

(A) You are needed to clean your shoes properly.

(B) Your shoes are needed by you to clean properly.

(C) Your shoes need to be cleaned properly.

(D) Your shoes are needed to clean properly.

Answer: (C)

38. He teaches us English.

(A) We are taught English by him.

(B) English will be taught to us by him.

(C) We were taught English by him.

(D) English is taught to us by him.

Answer: (D)

39. Four languages were studied by him at school.

(A) He had studied four languages at school.

(B) Four languages he studied at school.

(C) He studied for languages at school.

(D) At school, he studied four languages

Answer: (C)

40. We do not advise him these days.

(A) He was not advised these days by us.

(B) He will not be advised these days by us.

(C) He has not been advised these days by us.

(D) He is not advised these days by us.

Answer: (D)

41. The crowd gave a standing ovation to Maradona.

(A) Maradona is being given a standing ovation by the crowd.

(B) Maradona had been given a standing ovation by the crowd.

(C) Maradona gave a standing ovation to the crowd.

(D) Maradona was given a standing ovation by the crowd.

Answer: (D)

42. People avoid crowded buses.

(A) Crowded buses are avoided by people.

(B) People avoid the buses that are crowded.

(C) Crowded buses have been avoided by people.

(D) People are avoided by crowded buses.

Answer: (A)

43. We must reduce pollution in big cities.

(A) Pollution would be reduced by us in big cities.

(B) Pollution will be reduced by us in big cities.

(C) Pollution must  be reduced by us in big cities.

(D) Pollution should be reduced by us in big cities.

Answer: (C)

44. My mother has cooked dinner for me.

(A) Dinner has been cooked for me by my mother.

(B) My dinner was cooked by my mother.

(C) Mother cooked my dinner for me.

(D) Dinner was cooked by my mother.

Answer: (A)

45. Body language is used by all good communicators.

(A) All good communicators are using body language.

(B) Good communicators use the body languages.

(C) Good communicators were using body language.

(D) All good communicators use body language.

Answer: (D)

Directions (Qs. 46 to 55) : In these questions, a part of the sentence is underlined. Below are given alternatives to the underlined part which may improve the sentence. Choose the correct alternative. In case no improvement is required, choose “No Improvement” option. Choose your answer accordingly.

46. Her work a great improvement showed last year.

(A) a great showed improvement

(B) improvement a great showed

(C) showed a great improvement

(D) No improvement

Answer: (C)

47. Will you me visit and come in the hospital?

(A) come and me visit

(B) visit me and come

(C) come and visit me

(D) No improvement

Answer: (C)

48. Your hair needs cutting badly.

(A) Your hair  badly needs cutting.

(B) Your hairs needs cutting real bad.

(C) Badly your hair needs cutting.

(D) No improvement

Answer: (A)

49. They spent many ideally hours just sitting in the Sun.

(A) idly hours

(B) idle hours

(C) ideally hour     

(D) No improvement

Answer: (B)

50. There are many communicable diseases and almost all communicable diseases could be prevented.

(A) communicable diseases can

(B) communicable diseases should

(C) communicable disease can

(D) No improvement

Answer: (A)

51. Please examine my homework.

(A) investigate

(B) check

(C) see

(D) No improvement

Answer: (B)

52. The sheriff arrested the hoodlums for playing with the law.

(A) breaking

(B) cracking

(C) tossing

(D) No improvement

Answer: (A)

53. To tell you the truth, I am not there last night.

(A) were

(B) was

(C) is

(D) No improvement

Answer: (B)

54. If only he can swim!

(A) will

(B) could

(C) would

(D) No improvement

Answer: (B)

55. Strict laws were passed so that this cruel custom was no more.

(A) done away with

(B) wiped off

(C) eradicated

(D) No improvement

Answer: (A)

Directions (Qs. 56 to 65) : In these questions, a sentence has been given in Direct/Indirect. Out of the four alternatives suggested, select the one which best expresses the same sentence in Indirect/Direct and choose your answer accordingly.

56. I said to him, “Where have you lost the new pen?”

(A) I asked him where did he lose the new pen.

(B) I asked him where he has lost the new pen.

(C) I asked him where he had lost the new pen.

(D) I ask him where had he lost the new pen.

Answer: (C)

57. His uncle said, “I am always punctual.”

(A) His uncle said that I am always punctual.

(B) His uncle said that he was always punctual.

(C) His uncle said that I was always punctual.

(D) His uncle said that he is always punctual.

Answer: (D)

58. “Wait outside,” the receptionist told me.

(A) The receptionist told me wait outside.

(B) The receptionist asked me to wait outside.

(C) The receptionist told to me wait outside.

(D) The receptionist asked me wait outside.

Answer: (B)

59. The examiner said, “Candidates at the exam hall were nervous.”

(A) The examiner said that candidates at the exam hall was nervous.

(B) The examiner said that candidates at the exam hall had been nervous.

(C) The examiner said that candidates at the exam hall have been nervous.

(D) The examiner said that candidates were nervous at the exam hall.

Answer: (B)

60. The teacher said, “The Earth goes round the Sun.”

(A) The teacher said that the Earth went round the Sun.

(B) The teacher said the Earth went round the Sun.

(C) The teacher said the Earth goes round the Sun.

(D) The teacher said that the Earth goes round the Sun.

Answer: (D)

61. Raghu said, “I am not coming tomorrow.”

(A) Raghu said that he is coming the next day.

(B) Raghu said that he was coming the next day.

(C) Raghu said that he was not coming the next day.

(D) Raghu said that he was coming tomorrow.

Answer: (C)

62. He said, “All the players must report in time.”

(A) He said that all the players must be reported in time.

(B) He said that all the players must report in time.

(C) He said that all the players had to report in time.

(D) He said that all the players had reported in time.

Answer: (C)

63. My daughters said to me, “I am thinking of getting married.”

(A) My daughter told me that she is thinking of getting married.

(B) My daughter told me that she was thinking of getting married.

(C) My daughter told me that she will be thinking of getting married.

(D) My daughter thinks of getting married.

Answer: (B)

64. Rama said to the passer-by, “Where is the ticket counter?”

(A) Rama asked the passer-by where the ticket counter is.

(B) Rama asked the passer-by if there was a ticket counter.

(C) Rama asked the passer-by where was the ticket counter.

(D) Rama asked the passer-by where the ticket counter was.

Answer: (D)

65. Ragini said to the shopkeeper, “How much do you want me to pay for this dress?”

(A) Ragini asked the shopkeeper how much he wanted her to pay for that dress.

(B) Ragini asked the shopkeeper how he wanted her to pay for that dress.

(C) Ragini asked the shopkeeper if he wanted her to  pay for that dress.

(D) Raginig asked the shopkeeper how much he wants her to pay for that dress.

Answer: (A)

Directions (Qs. 66 to 75) : In these questions, in the following two passages some of the words have  been left out. Read the passages carefully and choose the correct answer to each question out of the four alternatives and fill in the blanks.

Passage-I

     The every foreign ………(66)………. India has something to ….(67) …. Those fascinated with our past can feast their eyes on the …(68)… of formerly princely states –the carefully restored palaces, forts and antique collections. Those taken in by our ….(69)… will find in a number of …(70)… waiting to have fresh recruits, preferably from the …(71)… west. Here one can buy …(72)… that the best traditional craftsmanship can produce. …(73)… too can take their pick for India has warm water …(74)…, vast open spaces, snowy mountain peaks and luxurious tropical …(75)… .

66.

(A) tourist

(B) person

(C) man

(D) human

Answer: (A)

67.

(A) see

(B) offer

(C) take

(D) give

Answer: (B)

68.

(A) leftovers

(B) ruins

(C) monuments

(D) remains

Answer: (C)

69.

(A) traditions

(B) culture

(C) religions

(D) spirituality

Answer: (D)

70.

(A) gurus

(B) godmen

(C) leaders

(D) teachers

Answer: (C)

71.

(A) lenient

(B) affluent

(C) intelligent

(D) prominent

Answer: (B)

72.

(A) gifts

(B) articles

(C) things

(D) handicrafts

Answer: (D)

73.

(A) Divers

(B) Nature lovers

(C) Wildlife enthusiasts

(D) Mountaineers

Answer: (B)

74.

(A) rivers

(B) seas

(C) beaches

(D) sands

Answer: (A)

75.

(A) trees

(B) forests

(C) shrubs

(D) bushes

Answer: (B)

Passage-II

There ….(76)… a large colony of monkeys in Bareilly jail. One day a baby monkey …(77)… to come down near our rooms and he …(78)… not get up on the wall again. Some prisoners and warders …(79)… hold of him and tied a bit of string …(80)… his neck. The parents of the little one saw all this ….(81)… the top of the high wall, and their anger …(82)… . Suddenly the father, a huge monkey, jumped down, and charged almost right ….(83)… the crowd which was around the baby monkey. It was a brave thing to do, for the  men …(84)… many in number. But the monkey’s courage won and the crowd of men fled, terrified, leaving their sticks behind them. The little monkey was taken back to its …(85)… .

76.

(A) is

(B) are

(C) happens

(D) was

Answer: (A)

77.

(A) happened

(B) happening

(C) happen

(D) happens

Answer: (A)

78.

(A) will

(B) could

(C) would

(D) can

Answer: (B)

79.

(A) catching

(B) catch

(C) caught

(D) was caught

Answer: (C)

80.

(A) on

(B) at

(C) over

(D) around

Answer: (D)

81.

(A) above

(B) form

(C) from

(D) over

Answer: (B)

82.

(A) boil

(B) grew

(C) growing

(D) brew

Answer: (B)

83.

(A) in

(B) upon

(C) under

(D) into

Answer: (B)

84.

(A) is

(B) were

(C) be

(D) was

Answer: (B)

85.

(A) brother

(B) tree

(C) cave

(D) mother

Answer: (D)

Directions (Qs. 86 to 100) : In these questions, you have three brief  passages with 5 questions following each passage. Read the passages carefully and choose the best answer to ach question out of the four alternatives.

Passage-I

Clouds are an essential part of the weather conditions which exist around us. Warm air rises upwards in the atmosphere carrying with it dust and moisture from evaporated surface water. At higher altitudes the air is cooler and the vapor in the warm air reaches its ‘dew point’ where it changes either to small particles of ice or into water, thus forming clouds. Since cloud formation takes place a different heights and temperatures, clouds are of different types.

   There are basically three main types of clouds. UP to about 1.5 k from the ground we generally see the first type called the ‘stratus’ cloud; on occasions it is the grey stratus clouds which bear light rain. In colder areas the water in these clouds, freezes, thus resulting in snowfall. Slowly rising warm air produces these clouds. Above these and up to about 4.5 km lie the ‘cumulus’ clouds; the fluffy shapes are a result of quickly rising warm air. Above the cumulus clouds, at an altitude of about 14 km, float the ‘cirrus’ types of clouds. Cirrus clouds are wispy clouds formed at such a high altitude that they contain only ice crystals.

   Lying between these three main types of clouds are other types like the ‘stratocumulus’, ‘altocumulus’, ‘cirrocumulus’, ‘altostratus’ and ‘cumulonimbus’ clouds.

86. “What does quickly rising warm air produce ?

(A) Altostratus clouds

(B) Cumulus clouds

(C) Altocumulus clouds

(D) Stratus clouds

Answer: (B)

87. What do cirrus clouds contain?

(A) Ice crystals

(B) Water

(C) Particles of ice

(D) Light rain

Answer: (A)

88. What are the three main types of clouds?

(A) Status, cumulus, cirrus

(B) Cirrus, altostratus, cirrocumulus

(C) Cumulus, cirrus, altocumulus

(D) Stratocumulus, stratus, cumulonimbus

Answer: (A)

89. Why are there different types of clouds?

(A) Because warm air rises upwards in the atmosphere.

(B) Because the atmosphere carries with it dust and moisture from evaporated surface water.

(C) Because cloud formation takes p lace in different heights and temperatures.

(D) Because they are the essential part of the weather conditions.

Answer: (C)

90. How are stratus clouds produced?

(A) By quickly rising warm air

(B) By quickly rising cold air

(C) By slowly rising cold air

(D) By slowly rising warm air

Answer: (D)

Passage-II

  Sariska National Park is a wildlife sanctuary, located at a distance of 107 km from Jaipur. The park possesses historical monuments and temples, which depict the legacy of the Maharajas of Alwar. The park is bigger than Ranthambore but has a similar topography. Though this sanctuary doesn’t have many tigers yet, many wild animals dwell on the grounds of this park. Apart from other carnivores and herbivores, you can also trace Rhesus Monkeys, which can usually be seen playing tricks upon each other around the Tad Vriksh (palm tree). The park also shelters many different species of birds including Bush Quails, Sand Grouses, Tree Pies, Golden Backed Woodpeckers, crested Serpent Eagles and Great Indian Horned Owls. Animals lovers would definitely love this place, as it will offer them some of the best scenes to remember. The best place to spot wildlife is at the waterholes where animals come to satisfy their thirst. You can trace hundreds of birds at the Kalighati Waterhole and Nilgai can be spotted at Salupka Waterhole.

91. The park possesses

(A) monuments and temples

(B) monuments and ramparts

(C) monuments and forts

(D) temples and ramparts

Answer: (A)

92. In the Sariska National Park, there are

(A) only herbivores

(B) both carnivores and herbivores

(C) no carnivores or herbivores

(D) only carnivores

Answer: (B)

93. The Sand Grouse is a species of

(A) birds

(B) plants

(C) trees

(D) animals

Answer: (A)

94. Animals come to satisfy their thirst at a

(A) borehole

(B) deep well

(C) bore-well

(D) waterhole

Answer: (D)

95. Sariska National Park is not far from

(A) Jodhpur

(B) Raipur

(C) Mewar

(D) Jaipur

Answer: (D)

Passage-III

  Using modern computer technology scientists have restored part of a fossil of a rare dinosaur that lived more than 90 million years ago during the Cretaceous Period in what is now Mongolia. The focus of the study was the skull of Erlikosaurus andrewsi, a 10-13 foot herbivorous dinosaur called Therizinosaur. Using a digital model of the fossil, the team virtually disassembled the skull of Erlikosaurus into its individual elements. They, then digitally filled in any breaks and cracks in the bones, duplicated missing elements and removed deformation by applying retro-deformation techniques, digitally reversing the steps of deformation. In the final step, the reconstructed elements were re-assembled. This approach not only allowed the restoration of the complete skull of Erlikosaurus but also the study of its individual elements. “With modern computer technology such as CT scanning and digital visualization, we now have powerful tools at our disposal with which we can get a step closer to restore fossil animals to their life-like condition” said author Stephan Lautenschlager from the University of Bristol in Britain.

96. Therizinosaur ate only

(A) plants

(B) both meat and plants

(C) fruits

(D) meat

Answer: (A)

97. The team applied

(A) retro-deformation techniques

(B) deformation techniques

(C) retro-formation techniques

(D) formation techniques

Answer: (A)

98. The main intention of Stephan Lautenschalger was

(A) the projection of computer technology

(B) restoration of fossilized animals to life

(C) to study dinosaurs

(D) the development of powerful tools

Answer: (B)

99. The restoration of a part of the fossil of the dinosaur was possible by a process of

(A) destruction

(B) disassembling  

(C) formation

(D) assembling

Answer: (B)

100. The scientists focused on the study of

(A) the  bones of the dinosaur

(B) the tail of the dinosaur

(C) the skull of the dinosaur

(D) the leg of the dinosaur

Answer: (C)

Staff Selection Commission (SSC) Multi Tasking Staff Examination Held On May 2017 Question Paper With Answer Key

Staff Selection Commission (SSC) Multi Tasking Staff Examination Held On May 2017
Staff Selection Commission (SSC) Multi Tasking Staff Examination Held On May 2017 Question Paper With Answer Key

Staff Selection Commission (SSC) Multi Tasking Staff Examination

Held On May 2017

PART – A : GENERAL INTELLIGENCE AND REASONING

Directions (Qs. 1 to 4) Select the related word/letters/number from the given alternatives.

1. AUTHOR : PEN : : DOCTOR : _____

(A) DISPENSARY

(B) WARD

(C) HOSPITAL

(D) STETHOSCOPE

Answer: (D)

2. ADIP : DGLS : : BEJQ : ?

(A) EJQU

(B) EHMT

(C) FINU

(D) CGLS

Answer: (B)

3. POLITE : ETILOP : : _______ : _______

(A) SINGLE : ELGGNIS

(B) DRAOB : BROAD

(C) WOMEN : WOMAN

(D) CHART : TRACH

Answer: (A)

4. 8 : 512 : : 9 : ?

(A) 728

(B) 729

(C) 781

(D) 792

Answer: (B)

Directions (Qs. 5 & 6) A series is given with one term missing. Out of the four alternatives, choose the alternative that will complete the series.

5. ACE, GIK, ?, SUW, YAC

(A) MPQ

(B) MOQ

(C) MNP

(D) MOP

Answer: (B)

6. 2, 5, 10, 17, 26, ?

(A) 38

(B) 35

(C) 37

(D) 33

Answer: (C)

7. Which answer figure will complete the pattern in the question figure?

Answer: (B)

8. If ‘+’ means ‘×’, ‘−‘ means ‘÷’, ‘×’ means, ‘−‘ and ‘÷’ means ‘+’, then the value of the given equation is 9 + 8 ÷ 8 – 4 × 6 = ?

(A) 68

(B) 36

(C) 11

(D) 65

Answer: (A)

9. Find the number of triangles in the adjoining figure.

(A) 14

(B) 18

(C) 16

(D) 20

Answer: (C)

10. Some equations are solved on the basis of certain system. Find out the correct answer for the unsolved equation on that basis.

9 × 6 × 2 = 269, 8 × 6 × 5 = 568, 5 × 4 × 1= ?

(A) 201

(B) 145

(C) 415

(D) 451

Answer: (B)

11. From the given alternative words, select the word which cannot be formed using the letters of the given word.

(A) TRUE

(B) TURN

(C) RENT

(D) RATE

Answer: (D)

12. A word is represented by only one set of numbers as given in any one of the alternatives. The sets of numbers given in the alternatives are represented by two classes of alphabets as in two matrics given The columns and rows of Matrix I are numbered from 0 to 4 and that of Matrix II are numbered from 5 to 9. A letter from these matrices can be represented first by its row and next by its column, e.g. ‘A’ can be represented by 00, 44 etc. and ‘P’ can be represented by 56, 79 etc. Similarly, you have to identify the set for the word ‘ZEST’

(A) 89, 31, 30, 01

(B) 89, 13, 03, 01

(C) 98, 13, 33, 04

(D) 98, 13, 30, 10

Answer: (D)

Directions (Qs. 13 to 15) : Find the odd number/letters/words from the given alternatives.

13. 

(A) 154

(B) 119

(C) 51

(D) 85

Answer: (A)

14. 

(A) DEGJ

(B) QRTW

(C) YZBE

(D) JKNQ

Answer: (D)

15.

(A) QUINTALS

(B) TONNES

(C) KILOMETRES

(D) KILOGRAMS

Answer: (C)

16. The age of father is twice that of the elder son. After ten years, the age of father will be three times th at of the younger son. If the difference of ages of the two sons is 15 years, the age of the father is?

(A) 50 years

(B) 60 years

(C) 70 years

(D) 55 years

Answer: (A)

17. If NAME is coded as MZLD, how will CLAIM be coded?

(A) BKZII

(B) BKZHL

(C) BKZHI

(D) BKYHL

Answer: (B)

18. One or two statements are given followed by two Conclusions/Assumptions I and II. You have to consider the statement to be true, even if it seems to be at variance from commonly known facts. You are to decide which of the given conclusions/assumptions can definitely be drawn from the given statement.

Statement : No man is donkey. Rahul is a man.

Conclusions :

I. Rahul is not a donkey.

II. All men are not Rahul.

(A) Only conclusions I follows.

(B) Neither Conclusion I nor Conclusion II follows.

(C) Only Conclusion II follows.

(D) Either Conclusion I or Conclusion II follows.

Answer: (A)

19. If a mirror is placed on the line MN, then which of the answer figures is the right image of the given figure?

Answer: (B)

20. Which one of the given responses would be a meaningful order of the following :

(1) Family     (2) Community

(3) Member   (4) Locality

(5) Country

(A) 3, 1, 4, 2, 5

(B) 3, 1, 2, 4, 5

(C) 3, 1, 2, 5, 4

(D) 3, 1, 4, 5, 2

Answer: (A)

21. Find the missing number from the given responses.

4        8        2

3        2        2

5        8        ?

60      128    68

(A) 19

(B) 17

(C) 13

(D) 15

Answer: (B)

22. From the given answer figures, select the one in which the question figure is hidden/embedded.

Answer: (B)

23. A man facing south turns to his left and walks 10 m, then he turns to his right and walks 15 m, again he turns to his left and walks 5 m and then he turns to his left and walks 15 m. In which direction he is facing now?

(A) West

(B) South

(C) East

(D) North

Answer: (C)

24. A piece of paper is folded and cut as shown below in the question figure. From the given answer figures, indicate how it will appear when opened.

Answer: (B)

25. Which one of the following diagrams represents the relationship among

Answer: (C)

PART B : NUMERICAL APTITUDE

26. A contractor has the target of completing a work in 40 days. He employed 20 persons who completed (1/4) of the work in 10 days and left. The number persons he has to employ to finish the remaining part as per target is

(A) 30

(B) 20

(C) 10

(D) 40

Answer: (B)

27. Each side of a square is increased by 10%. The percentage increase of its area is

(A) 25

(B) 12.5

(C) 20

(D) 21

Answer: (D)

28. The average of three numbers of which the greatest is 16 is 12. If the smallest is half of the greatest, the remaining number is

(A) 14

(B) 12

(C) 10

(D) 8

Answer: (B)

29. The ratio of the areas of two squares one having double its diagonal than the other is

(A) 2 : 1

(B) 3 : 2

(C) 4 : 1

(D) 3 : 1

Answer: (C)

30. A man purchased 120 reams of paper at Rs 80 per ram. He spent Rs 280 on transportation, paid octroi at the rate of 40 paise per ream and paid Rs 72 to porter. In order to gain 8%, he must sell each ream of paper for

(A) Rs 89

(B) Rs 87.5

(C) Rs 85

(D) Rs 90

Answer: (D)

31. In a college, 1/5th of the girls and 1/8th of the boys took part in a social camp. The total number of students in the college took part in the camp is

(A) 13/80

(B) 13/40

(C) 2/13

(D) 4/15

Answer: (B)

32. The price of electricity has been increased by 25. If a person wants to keep the expenditure same, then the percentage reduction in u se of electricity should be

(A) 21

(B) 18

(C) 19

(D) 20

Answer: (D)

33. A box contains one rupee, fifty paise and twenty-five paise coins. The total number of coins is 378. The ratio of values of the above coins is 13 : 11 : 7. The number of twenty-five paise coins was

(A) 132

(B) 168

(C) 210

(D) 78

Answer: (B)

34. The unit’s digit of the number 6256 – 4256 is

(A) 4

(B) 1

(C) 0

(D) 7

Answer: (B)

35. The selling price of a radio was Rs 255 when 15% discount was allowed. Then the marked price of the radio was

(A) Rs 350

(B) Rs 275

(C) Rs 300

(D) Rs 400

Answer: (C)

36. P invests Rs 9100 for 3 months, Q invests Rs 6825 for 2 months and R Rs 8190 for 5 months in a business. If the total profit amounts to Rs 4158, how much profit should Q get?

(A) Rs 346.50

(B) Rs 682.50

(C) Rs 1386

(D) Rs 693

Answer: (D)

37. The radius of a sphere is doubled. The percentage of increase in its surface area is

(A) 75

(B) 300

(C) 400

(D) 100

Answer: (B)

38. A man retired from his service at the age of 60. He served for 3/5th years of his retirement age. He joined his job at the age of

(A) 36 years

(B) 24 years

(C) 20 years

(D) 18 years

Answer: (B)

39. A dealer marks his goods 20% above cost price. He then allows some discount on it and makes a profit of 8%. The rate of discount is

(A) 4%

(B) 10%

(C) 6%

(D) 12%

Answer: (B)

Directions (Qs. 40 & 41) The pie-diagram shows the expenditure incurred on the printing of a book by a publisher, under various heads. Study the pie-diagram and answer the questions.

40. The two expenditures, which together will form an angle of 108° at the centre are

(A) A and D

(B) A and E

(C) B and E

(D) D and E

Answer: (A)

41. If the expenditure incurred on the printing under had A is Rs 5000, then the sum of expenditure incurred under head B and D is

(A) Rs 5780

(B) Rs 8750

(C) Rs 7850

(D) Rs 8570

Answer: (D)

42. Two varieties of sugar are mixed together in a certain ratio. The cost of the mixture per kg is Rs 0.50 less than that of the superior and Rs 0.75 more than the inferior variety. The ratio in which the superior and inferior varieties of sugar have been mixed is

(A) 5 : 2

(B) 3 : 2

(C) 5 : 1

(D) 2 : 3

Answer: (B)

43. A man travels 600 km by train at 80 km/hr, 600 km by ship at 30 km/hr, 500 km by aeroplane at 400 km/hr and 300 km by car at 60 km/hr. What is the average speed (km/hr) for the entire distance?

(A)  

(B)  

(C)  

(D) 63

Answer: (A)

44. P, Q and R can do a piece of work in 60 days, 100 days and 80 days respectively. They together work to finish the work and receive Rs 2256. Then P will get

(A) Rs 576

(B) Rs 960

(C) Rs 752

(D) Rs 564

Answer: (B)

45. The price of a bicycle is marked by a trader at Rs 1000. He sold the bicycle allowing successive discounts of 20%, 10% and 5%. Thus the trader gained 14%, then the cost price of bicycle (in rupees) is

(A) 600

(B) 560

(C) 510

(D) 790

Answer: (A)

46. A person covers a certain distance in 6 hours, if travels at 40 km/hr. If he has to cover the same distance in 4 hours, then his speed must be

(A) 80 km/hr

(B) 70 km/hr

(C) 60 km/hr

(D) 50 km/hr

Answer: (C)

47. A two-wheeler depreciates at 20% of its value every year. If present value of the same be Rs 90000, its depreciation value would be Rs 36864 after

(A) 4 years

(B) 3 years

(C) 5 years

(D) 6 years

Answer: (A)

48. By selling a coat for Rs 630, a shopkeeper gains 5%. Find the cost price of the coat?

(A) Rs 700

(B) Rs 600

(C) Rs 650

(D) Rs 625

Answer: (B)

49. A sum was doubled with  rate of simple interest, per annum. Then time taken for that sum is

(A)  

(B) 8 years

(C)  

(D) 10 years

Answer: (B)

50. Ram’s income is Rs 100 more than that of Shyam. If the average income of Ram and Shyam is Rs 850, then Ram’s income is

(A) Rs 900

(B) Rs 475

(C) Rs 850

(D) Rs 800

Answer: (A)

PART-C : GENERAL ENGLISH

Directions (Qs. 51 to 55) : Out of the four alternatives, choose the one which can most appropriately substitute the given word(s) or sentence.

51. All in a mess.

(A) disarranged

(B) disturbing

(C) distracting

(D) negligence

Answer: (A)

52. Happening every year.

(A) routine

(B) season

(C) annually

(D) timely

Answer: (C)

53. A number of people listening to a concert or lecture.

(A) audience

(B) spectator

(C) group

(D) tribe

Answer: (A)

54. In a primitive or uncivilized state.

(A) savage

(B) rural

(C) village

(D) olden

Answer: (A)

55. One who believes in God.

(A) theist

(B) worshipper

(C) believer

(D) pilgrim

Answer: (A)

Directions (Qs. 56 to 60) : Read the following passage carefully and choose the most appropriate answer to each question out of the four alternatives.

Passage

At times there was something inhuman about Mr. Rogers, the Headmaster. His formidable chin fitted o ut in the most formidable way, and he seemed to be always frowning at th world in general. At the assembly he was like an inspecting officer in the army; he surveyed the gathered innocents with his crinkled eyes, and then began his list of morning rebukes. Any hapless offender, for the most venial offence, would be commanded to appear before his presence in front of the gathered ranks, and would then be subjected to a tirade of abuse that would leave him trembling or even in tears. Mr. Rogers was ruthless and cruel in public. “A dehydrated old sadist” was how Mr. Jones, the chemistry master described him.

    The extraordinary thing was that in private he could be magnanimous and gentle. Any boy who was really in trouble would receive a sympathetic hearing, and went out feeling that the burden had been lightened.

56. The staff considered Mr. Rogers to be

(A) a person without any scruples

(B) a gentle and understanding person

(C) a person to be idolised

(D) an intimidating person

Answer: (D)

57. Another word for ‘magnanimous’ is

(A) high handed

(B) generous

(C) magnificent

(D) arrogant

Answer: (B)

58. What happened to boys who visited him in private?

(A) They came out trembling and in tears

(B) They seemed relieved of their troubles

(C) They looked even more worried than before

(D) They echoed the sentiments of the staff members

Answer: (B)

59. What best describes Mr. Rogers at the school assembly?

(A) a sensitive army officer

(B) a genial person

(C) a formidable teacher

(D) ruthless and cruel

Answer: (D)

60. “In a most intimidating way” means

(A) in an extremely frightening manner

(B) in an extremely intimate manner

(C) in a very fowl way

(D) in a very helpless way

Answer: (A)

Directions (Qs. 61 to 66) : Four words are given, out of which only one word is correctly spelt. Find the correct spelt word.

61.

(A) harass

(B) haraas

(C) harras

(D) harrass

Answer: (A)

62.

(A) license

(B) liscense

(C) licanse

(D) lisense

Answer: (A)

63.

(A) exhilarate

(B) exhelerate

(C) exelerate

(D) exhilerate

Answer: (A)

64.

(A) separate

(B) saperate

(C) separete

(D) seperate

Answer: (A)

65.

(A) jornale

(B) journal

(C) jornal

(D) jurnale

Answer: (B)

66.

(A) muneciple

(B) municipal

(C) munecipal

(D) municiple

Answer: (B)

Directions (Qs. 67 to 69) : Out of the four alternatives, choose the one which best expresses the meaning opposite to the given word.

67. Authentic

(A) futile

(B) fake

(C) honest

(D) frank

Answer: (B)

68. Methodical

(A) random

(B) crazy

(C) flurry

(D) orderly

Answer: (C)

69. Polite

(A) rough

(B) rash

(C) tough

(D) nude

Answer: (A)

Directions (Qs. 70 to 72): Out of the four alternatives, choose the one which best expresses the meaning of the given word.

70. Abhorrent

(A) Uncommon

(B) Irregularity

(C) Repugnant

(D) Admirable

Answer: (C)

71. Stationery

(A) Computer accessories

(B) Writing accessories

(C) Fixed

(D) Moving

Answer: (B)

72. Innocent

(A) Guiltless

(B) Guilt

(C) Bias

(D) Offence

Answer: (A)

Directions (Qs. 73 to 82) : Some part of the sentences may have errors and some are correct. Find out which part of the sentence has an error and select that part as your answer. If a sentence is free from error, then select the “No error” option as your Answer.

73. Have you been doing what has been asks of you?

(A) been doing

(B) what has been asks of you?

(C) Have  you

(D) No error

Answer: (B)

74. This sponge is not able to absorbing water.

(A) No error

(B) not able to

(C) absorbing water

(D) This sponge is

Answer: (C)

75. I am too much pleased to know that you have topped the list.

(A) No error

(B) to know that

(C) you have topped the list

(D) I am too much pleased

Answer: (D)

76. We rejoiced at his being promoted.

(A) No error

(B) We rejoiced

(C) at his

(D) being promoted

Answer: (A)

77. Kindly submit your documents to the clerk.

(A) No error

(B) your documents

(C) Kindly submit

(D) to the clerk

Answer: (A)

78. He is not what you would call an honest man, doesn’t he?

(A) doesn’t he

(B) you would call an honest

(C) He is not what

(D) No error

Answer: (A)

79. I am thinking I will go to the market later this afternoon.

(A) I will go to the market

(B) No error

(C) I am thinking

(D) later this afternoon

Answer: (D)

80. It is a most beautiful painting in the gallery.

(A) beautiful painting

(B) in the gallery

(C) it is a most

(D) No error

Answer: (C)

81. Love for ours environment is enough to protect nature.

(A) is enough to protect nature

(B) Love for

(C) ours environment

(D) No error

Answer: (C)

82. He has not been attending English classes for one week.

(A) for one week

(B) English classes

(C) He has not been attending

(D) No error

Answer: (D)

Directions (Qs. 83 to 92) : Sentences given with blank to be filled in with appropriate word(s). Four alternatives are suggested for each question. Choose the most appropriate alternative out of the four.

83. The actor died when his car turned _____

(A) across

(B) tortoise

(C) turtle

(D) tiger

Answer: (C)

84. They were worried about a new cult which _______ many followers.

(A) is gaining

(B) must be gaining

(C) were gaining

(D) was gaining

Answer: (D)

85. Anna is not popular. She has _________ friends.

(A) little

(B) a little

(C) few

(D) a few

Answer: (C)

86. He must _______ the loans.

(A) receipt

(B) repeat

(C) repeal

(D) repay

Answer: (D)

87. _______ she can’t drive, Anita has bought a car.

(A) Whether or not

(B) Even though

(C) Since

(D) Even if

Answer: (B)

88. Sally parked and got ________ the car quickly.

(A) over

(B) out of

(C) on

(D) in

Answer: (B)

89. Since she is a teacher of language, one would not expect her to be guilty of a __________.

(A) schism

(B) solecism

(C) stanchion

(D) bombast

Answer: (B)

90. A man is known ________ the company he keeps.

(A) by

(B) for

(C) in

(D) of

Answer: (A)

91. You will have to face some practical problems when you start________ this plan.

(A) executing

(B) prosecuting

(C) proscribing

(D) prescribing

Answer: (A)

92. The convict was ______ on a Monday.

(A) hanging

(B) hanged

(C) hang

(D) hung

Answer: (B)

Directions (Qs. 93 to 97) : A sentence or a part of the sentence is underlined. Below are given alternatives to the underlined part which may improve the sentence. Choose the correct alternative. In case on improvement is needed, choose “No improvement”.

93. Even at the peak of her success, Kalpana Chawla did not forget her home or her duty towards other human, this is evident from the fact that she distributed education of girls in India.

(A) she contribute to education of girls

(B) No improvement

(C) she contributed girls for education

(D) she contributed to the education of girls

Answer: (D)

94. How long I shall stay is doubtful.

(A) I want to stay

(B) I will stay

(C) I can stay

(D) No improvement

Answer: (B)

95. The doctor scribbled his prescription which I could not make out.

(A) make for

(B) make over

(C) No improvement

(D) make up

Answer: (C)

96. Neither parties deserves our help.

(A) No improvement

(B) parties deserve

(C) party deserves

(D) parties deserved

Answer: (C)

97. Every person was not fitted for

(A) No improvement

(B) was not fit at

(C) is not fit at

(D) is not fit for

Answer: (D)

Directions (Qs. 98 to 100) : Four alternatives are given for the Idiom/Phrase underlined in the sentence. Choose the alternative which best expresses the meaning of the Idiom/Phrases.

98. All and sundry were asked to leave the village immediately.

(A) all the adults

(B) all the people

(C) all the adults and elderly

(D) some people

Answer: (B)

99. Joe can be a loose cannon

(A) one who speaks without thinking.

(B) one who acts crazy.

(C) one who acts without thinking.

(D) one who is unpredictable

Answer: (D)

100. The presentation by the group at the meeting passes muster.

(A) achieves to impress.

(B) fails to impress.

(C) meets required standards.

(D) does not meet the required standards.

Answer: (C)

© Copyright Entrance India - Engineering and Medical Entrance Exams in India | Website Maintained by Firewall Firm - IT Monteur